Está en la página 1de 206

 

CICLO INTENSIVO ESCOLAR NACIONAL Material de Estudio Nº 1 

1ER MATERIAL DE
ESTUDIO

 ADMISIÓN 2020 - 1

CEPRE-UNI HUMANIDADES 1
 

CICLO INTENSIVO ESCOLAR NACIONAL Material de Estudio Nº 1 

Lenguaje
TEMA I. USO DE
DE LAS L ETRAS MAYÚSC
MAYÚSCULAS
ULAS I: MAYÚSCULA
MAYÚSCULA EXIGIDA POR LA
PUNTUACIÓN E INDEPENDIENTE DE LA PUNTUACIÓN. USO DE MINÚSCULA INICIAL
1. USO DE
DE LAS L
LETRAS
ETRAS MAYÚSCU
MAYÚSCUL
L AS
1.
1.1
1UUso
so de la mayúscula
mayúscu la inici
inicial
al e
exigi
xigida
da por la puntuaci
puntuación
ón
Después
Después de
d e los …  Ejemplos
. puntos suspensivos (…) cuando   ̶  Escuché en el baño que... L uego te cuento.
finalizan una oración. En caso   ̶  Comimos ceviche, jalea... Fue un gran almuerzo.
contrario, la palabra que sigue se  A veces pienso
pienso que... d ebemos dormir más.
escribirá con minúscula.   ̶  Compra lo que necesites, pero... n o te excedas.
. dos puntos (:) al empezar una cita La vendedora dijo: “ L as ofertas serán solo hoy”.  
textual y ante fórmulas de saludo.   ̶  Estimado director:/R
director:/Reciba un cálido saludo… 
En otros casos, se usará Él es tímido e introvertido: n o socializa fácilmente.
minúscula.   ̶ Estudié todos los cursos del Cepre-UNI: s í ingresaré.
. signos de cierre de inter rogación   ̶ ¡Me alegra verte por fin! ¿Q
interrogación ¿Qué ha sido de tu vida?
(? ) y exclamación (!
(!), salvo que se ¿Cómo te fue en la fiesta? Debes haberte divertido mucho.
interponga otro signo diferente.   ̶  ¡Qué hermosa mañana!, q ueridos alumnos.
  ̶  ¿Hablas tres idiomas?: ¿e¿eres políglota?

3.
3.2
2UUso
so de la mayúscula ini cial ind
independiente
ependiente de la puntuación
puntu ación
Se escribe con mayúscula inicial en los siguientes casos
casos::

cosas 
Nombres propios de personas, animales y cosas 
1

Ejemplos:
  ̶ Mi hermana Carmen
Carmen sabe
 sabe preparar jugos deliciosos.
  ̶ La oveja Dolly
Dolly fue
 fue clonada por aquellos científicos.
  ̶ El lujoso barco Titanic
Titanic se
 se hundió con tanta lentitud.

Si los apellidos tienen preposición, esta se escribe con minúscula, salvo que el apellido
aparezca al inicio o sin el nombre de la persona.

Ejemplos:
  ̶ Lucía de la Cruz interpretó
Cruz interpretó canciones de Chabuca Granda.
  ̶ El renacentista Garcilaso de la Vega escribió
Vega escribió Églogas.
  ̶ La señora De la Praga cocinó
Praga cocinó guiso de brócoli con pollo.

Si los apellidos tienen solo artículo, este se escribirá siempre con mayúscula.  
Ejemplos:
  ̶ El ingeniero La Paz explicó
Paz explicó cómo realizó su tesis.
t esis.
  ̶ La española Olivia La Madrid viajó
Madrid  viajó a Cajamarca.
  ̶  La familia La Piedra llegó
Piedra llegó temprano al matrimonio.

Cuando los nombres propios son usados para expresar una cualidad o característica, se
escriben con minúscula. 
Ejemplos:
  ̶ Esa modelo era una afrodita
afrodita en
 en la pasarela. (guapa)

CEPRE-UNI HUMANIDADES 2
 

CICLO INTENSIVO ESCOLAR NACIONAL Material de Estudio Nº 1 

  ̶ Tu compañero es un einstein


einstein en
 en las clases. (genio)
  ̶ Mi exenamorado parece todo un donjuán
donjuán.. (mujeriego)

2 Los sustantivos y adjetivos de los sobrenombres, apodos, seudónimos; excepto


los artículos que los anteceden y las preposiciones. Se emplea la mayúscula en
las denomin
denominacio
aciones
nes al
altern
ternativ
ativas
as de lu
lu ares. 

  ̶Ejemplos:
Sabía que ellos viajaron juntos a Cusco, el Ombligo del Mundo.
Mundo .
  ̶ Bjorn, Brazo de Hierro,
Hierro , fue el hijo mayor del rey vikingo Ragnar.
  ̶ Quijote, el Caballero
Caballero de la
la T
Tris
riste
te Figura
Figura,, amaba mucho a Dulcinea.

2. USO DE LAS
LA S LETRAS MAYÚSCULAS INDEP INDEPENDIE
ENDIENTE
NTE DE LA PUNT
PUNTUACIÓN
UACIÓN
Se escribe con mayúscula inicial en los siguientes casos
casos::

Nombres propios de continentes, países, ciudades, áreas geopolíticas, regiones


1 naturales

Ejemplos:
  ̶ En el Cono Sur registraron varios accidentes.
  ̶ En el Perú
  ̶ CoreaPerú,
del ,Nort
del muchos
Norte viajan durante
e y EE.UU. Semana
siguen en Santa.
rencillas.

Los nombres propios de los accidentes geográficos pero no los sustantivos


2 comunes o genéricos que los acompañan (mar, cabo, volcán, etc.), los cuales
deben escribirse con minúscula 

Ejemplos:
  ̶ El estudio se realizará en el distrito de La Moli
Molina
na..
  ̶ Dicen que el
el  Pacífico
Pacífico posee
 posee una diversidad ictiológica.
  ̶ Encontramos ofertas para viajar este mes a La Paz. Paz.

En cuanto a los accidentes geográficos (río, pico, mar, océano, valle, lago, volcán, etc.) se
escriben con mayúscula solo si forman parte de un nombre propio. Ejemplos:
  ̶ El mar  Rojo
Rojo fue
 fue muy conocido gracias a Moisés.
  ̶ Viajará a Ma
Marr del Plata
Plata dentro
 dentro de cinco días.
  ̶ El cabo
cabo de Esperanza se ubica en África. 
 de Buena Esperanza se

Las siglas se escriben enteramente


enteramente en may
mayúsculas.
úsculas. Los acróni
acrónimos,
mos, con cinco
3 o más letras, referidos a nombres propios mantienen la mayúscula inicial
(Unesco, Sunarp, Mercosur, etc.). Si tienen menos de cinco letras se escriben
enteramente con mayúsculas. Otros referidos a nombres comunes se escriben
).  
enteramente en minúscula (ovni, láser, radar, pyme, uci, etc.).

Ejemplos:
  ̶ El JNE tiene 86 años de fundado, según su portal.
  ̶ Pagó en Sedapal
Sedapal todos
 todos los recibos que debía.

CEPRE-UNI HUMANIDADES 3
 

CICLO INTENSIVO ESCOLAR NACIONAL Material de Estudio Nº 1 

  ̶ El sida
sida se
 se manifiesta a través de diversos síntomas.
Sustantivo y adjetivos de congresos, simposios, coloquios, ferias, festivales,
4 torneo
torneoss ddee ort
ortiv
ivos,
os, etc.
etc. 
Ejemplos:
  ̶ Organizaremos el III Encuentro de Estudiantes de de F
Física
ísica..
  ̶ Se acerca la FeFeria
ria del Libro
Libr o , un gran evento en Lima.
  ̶  Asistiremos al Primer Concurso
Concu rso de Da
Danza
nza M
Moderna
oderna..

Las denominaciones oficiales de los premios se escriben en mayúscula, excepto


si aluden al ganador o al objeto que lo representa. 
Ejemplos:
  ̶ El Premio Nobel se otorga a los investigadores o descubridores.
  ̶  Aún no organizan
organizan los Premios Grammy Latino 2018.
2018.  
  ̶ El grupo mostró su gaviota
gaviota a
 a todos los peruanos aquel día.

Los sustantivos y adjetivos de nombres de festividades cívicas y religiosas,


5 periodos geológicos e históricos y acontecimientos históricos 

1. Ac
 Acon
ontec
la P tecim
imien
rimera Giento
to s h
uerra his
Mistó
tó ri
rico
undial coss 1. EdadPeriodos
Pe riodos
M edia históricos
2. la Guerra Fría 2. Edad de los Metales
3. la Reconquista la Alta
3. la A lta Edad Media
4. la Primavera de Praga 4. el Siglo de las L uces

Ejemplos:
  ̶ Siempre viajamos durante en Añ
en Añoo n uevo
uev o  y Navidad
Navidad..
  ̶ En el Día del
del Padre
Padr e, prepararán una deliciosa cena.
  ̶ Leímos sobre la Segunda Guerra Mundial este
Mundial  este ciclo.

Se escriben con letra minúscula en los siguientes casos:

a) Cargos, títulos y nombres de dignidad Ejemplos


presidente, ministro, alcalde, congresista,
secretaria, ministro, papa
b) Puntos cardinales, líneas imaginarias y sur, ecuador, hemisferio norte, latitud sur,
nombres de las divisiones de la Tierra el meridiano, noreste
c) Días de la semana, meses, estaciones del lunes, enero, invierno, primavera, febrero,
año domingo
d) Lenguas, culturas, religiones, monedas, quechua, la cultura paracas, judaísmo, yen,
enfermedades, gentilicios, comidas gastritis, síndrome de Down, limeño, causa
e) Los sustantivos o adjetivos que usan para ibérico, siamés, persa, pequinés,
designar razas o variedades de animales. labradores, caniches, etc.
  NOTA: Cuando el nombre está formado por
Tienen un angora pequeño.
un topónimo, este irá con mayúscula. Crían a un chihuahua.

CEPRE-UNI HUMANIDADES 4
 

CICLO INTENSIVO ESCOLAR NACIONAL Material de Estudio Nº 1 

EVALUACIÓN

1. Marque el enunciado que presente uso A) ¿Creen que Del Solar se comportará como
correcto de las mayúsculas y las minúsculas. un Judas en algún momento?
A) Pérez De Cuéllar fue embajador de Perú en B) Juan del Valle y Caviedes, conocido como El
Francia. Poeta de la Ribera, nació en España.
B) Mi amigo Carlos la Paz parece un Adonis. C) El escritor César Vallejo perdió la vida en...
C) El modelo Luis Maximiliano La Madrid es un la Ciudad de la Luz, París.
Adonis. D) ¿Vendrás a visitarme mañana con... mejor
D) La goleta Covadonga se hundió en Chancay ánimo?... aquí te esperaré.
en 1980. E) Hay muchos inventos del siglo XX: Televisor,
E) La perra laika fue al espacio en la nave aerosol, radar, nailon, etc.
soviética Sputnik 2.
6. ¿Qué oración evidencia el uso adecuado de
2. Indique qué enunciado(s) presenta(n) uso las mayúsculas o minúsculas?
correcto de las letras mayúsculas y minúsculas. A) El Jefe de Estado inspeccionó arduamente la
I. Stephen Hawking fue un físico nacido bajo Amazonía del Vrae.
Capricornio. B) El rector apertura el IV congreso de
II. Algunos gases nobles son... Helio, Neón, egresados de Mecatrónica.
argón, xenón. C) El ministro de Salud recomendó a no
III. René Descartes aseveró: «pienso, luego exponerse al sol sin bloqueador.
existo». D) Nuestro Sistema Solar forma parte de la
IV. ¡Qué calor que siento acá! ¡que ya llegue el Láctea, 
galaxia Vía Láctea, 
invierno! E) Eurasia, zona geográfica, comprende europa
A) II y III B) I y IV C) II y IV D) I E) Solo IV y asia unidas.

3. ¿Qué serie completa adecuadamente las 7. ¿En qué alternativa se evidencia uso
siguientes oraciones con respecto al uso de adecuado de las mayúsculas o minúsculas?
mayúsculas y minúsculas? A) Los trillizos recién nacidos están internados
I. El parlamentario Jorge ____________ en la Uci.
Castillo es aprista. B) Fui al concierto de Dina Páucar, la diosa
II. Según la novela de Cervantes, Sancho era Hermosa del Amor.
amigo del ____________. C) En mes de Junio, en Islandia, el Sol nunca se
III. La que era mi mejor amiga resultó ser una _. oculta totalmente.
A) del-Quijote -celestina D) Viajaré con mi familia al Callejón de Huaylas
B) Del-Quijote -Celestina para celebrar la Navidad y el Año Nuevo.
E) Los chancas, de naturaleza guerrera, fueron
C)
D) del-quijo
del-quijote
te -Celestina
Del-quijote -celestina derrotados por el último inca Pachacútec.
E) del-Quijote -Celestina 8. Elija la opción que presenta uso adecuado de
4. ¿Cuántas mayúsculas faltan el siguiente las mayúsculas y minúsculas.
texto? A) Los Premios Nobeles de Literatura practican
Margaret Tacher, conocida como la dama de golf y tenis.
hierro, fue una política del siglo XX nacida en el B) El ganador del balón de oro 2018 fue
viejo continente. Debido a que era libra, se entrevistado por un corresponsa
corresponsal.l.
caracterizó por su firmeza para dirigir los asuntos C) Los Alcaldes del Cono Sur propondrán
de su país. Además, sorprendió a muchos grandes cambios viales.
cuando dijo: «si quieres algo dicho, pídeselo a D) La SUNAT implementó, desde este año, la
un hombre; si quieres algo hecho, pídeselo a una visita virtual desde el celular.
mujer. E) El pastor Alemán fue diagnosticado por el
A) Dos B) Tres C) Cuatro D) Cinco E) Seis médico internista de Distémper.

5. Marque el enunciado que presenta uso 9. Señale la alternativa que presenta uso
correcto de las letras mayúsculas y minúsculas. adecuado de las mayúsculas y minúsculas.

CEPRE-UNI HUMANIDADES 5
 

CICLO INTENSIVO ESCOLAR NACIONAL Material de Estudio Nº 1 

A) Tiro es un Dóberman curioso; Bala, un gato A) La estrella Polar es la más próxima del Polo
Siamés. Norte.
B) Los líderes de Fuerza Popular debatieron, con B) Roma, la Ciudad Eterna, es la ciudad más
el Vice Primer Ministro. poblada de Italia y de la UE.
C) Desea conocer Países bajos de la Unión C) Marco De las Casas siempre es un buen
Europea. amigo…no lo dudes.
D) Los directores de la Unesco y de la ONU se D) Francisco Bolognesi aseveró: «tengo deberes
reunirán con el presidente de Bolivia. sagrados que cumplir…».
E) En Cabo de Hornos, en Enero, celebraron un E) Ernesto es todo un Quijote: se preocupa
festival. mucho por todos sus amigos.
10. ¿Qué enunciado presenta uso adecuado de
las letras mayúsculas y minúsculas?

TEMA II. LA SÍLABA: CLASES. SILABEO ORTOGRÁFICO. SECUENCIAS VOCÁLICAS:


DIPTONGO, TRIPTONGO Y HIATO. EL ACENTO: REGLAS GENERALES DE
 ACENTUACIÓN
 ACENTUA CIÓN

1. LA SÍLAB
SÍLABA A
Es el sonido o conjunto de sonidos pronunciados en un solo golpe de voz.
Ejemplos:
  Pan = Pan (1 sílaba) = palabra monosilábica

  Hogar = Ho – gar (2 sílabas) = palabra bisilábica
1.1 Clases
Criterios De
Definic
finic ión Eje
Ejemplos
mplos
Tónica. Presenta la mayor fuerza de voz. Es me- sa
me-sa car-pe
car- pe-ta
-ta  
Por el palabra. 
una sola sílaba en una palabra.  car-te-
car-te-ra
ra   te-o-do-lili--to 
te-o-do- to 
acento  Áton
 Át ona.
a. Presenta la menor fuerza de voz. me-sa
me -sa car-pe-ta
car-pe -ta
Pueden ser una o más silabas. car-te
car- te-ra
-ra te-o-do-li-to
te-o-do-li -to
au-las te-ma
Libre. Finaliza en sonido vocálico. 
vocálico.  cau-sa car -ta
-ta
Por la
terminación au-la te-ma
Trabada. Finaliza en sonido consonántico. 
consonántico.  cau-sa car -ta
-ta

1.2 Secuencia vocálica

Es la concurrencia de dos o más sonidos vocálicos consecutivo


consecutivoss dentro de una misma
palabra. Hay tres tipos de secuencias vocálicas: diptongo, triptong
triptongoo y hiato.
NOTA: En el español, las vocales se pueden clasificar como abiertas (VA): a, e,
o y cerradas (VC): i, u, según el ángulo de apertura bucal.

VA  + VC deu-da
deu -da , con-voyy  
con-vo

DIPTONGO 
Dos vocales forman una VC + VA   ci
ciee-lo , mu
muee-la 
misma sílaba.
VC + VC (≠) ciu-dad
ciu-dad , f ui
ui  

CEPRE-UNI HUMANIDADES 6
 

CICLO INTENSIVO ESCOLAR NACIONAL Material de Estudio Nº 1 

En las secuencias VC+VA  y VA +VC del diptongo, la vocal abierta es la que siempre lleva la
mayor fuerza de voz. En el caso de la secuencia VC+VC, las vocales deben ser diferentes
(iu, ui). 
TRIPTONGO 
VC + VA + VC  An-da-huay-las
 An-da-huay -las
Tres vocales forman una
misma sílaba. lim-piéis
lim-piéis  

En los triptongos, las vocales abiertas (a,e,o) llevan siempre la mayor fuerza de voz; la vocal
del medio siempre es abierta y la
l a de los extremos, cerradas.

VA - VA Co-o -pe-rar
Co-o-pe-rar
Hiato simple po--e-ma-rio 
po
VC + VC o-dr
o-drii -is -ta
less  
I uale
HIATO  du--un
du un-vi-ra-to
-vi-ra-to 
Dos vocales forman Ca-í-da
Ca-í
sílabas diferentes. VA - VC’
VC’  
o-í
o-í-do 
Hiato
VC’ - VA bo-hí-o
bo-hí -o
ac-tú
ac-tú -a 
En el hiato acentual, la vocal que lleva la
l a mayor fuerza de voz es la vocal cerrada (i,u), la
cual llevará tilde obligatoriamente.

1.3 Silabeo ortográfico

Reglas   Ejemplos   Silabeo 


1. La “h” entre vocales no impide la Cohibido Cohi-bi-do  
Cohi-bi-do 
formación de diptongo ni de hiato. Si hay Sahumar   Sahu-mar  
hiato, la “h” forma una sílaba con la Cohecho  
Cohecho Co-he-cho  
Co-he-cho
segunda vocal. La “h” entre una Prohíben Pro-hí-ben
consonante y una vocal forman una sola Inherente I- nhe-ren-te
sílaba:  Alhaja  A-lha-ja
2. La “y” en posición final de la sílaba Espray Es-pray  
Es-pray
puede
su formar
sonido diptongo
equivale a la ovocal “i”. cuando Corduroy 
triptongo Corduroy
Buey    
Buey Cor-du-roy
Cor-du-roy 
Buey    
Buey
Paraguay  
Paraguay Pa-ra-guay  
Pa-ra-guay
3. Las sílabas con dígrafos gue, gui, que, Barquillo Bar-qui-llo
qui no
qui  no presentan diptongo porque la vocal Manguera Man-gue-ra
“u” no representa sonido. En cambio, sí Desagüe De-sa-güe
forman diptongo cuando llevan diéresis Bilingüismo Bi-lin-güis-mo
(güe, güi).
güi). Degüello De-güe-llo
4. La “x” entre dos vocales forma una  Axila  A-xi-la
sílaba con la segunda vocal. La “h”, al no Crucifixión Cru-ci-fi-xión
ello. 
presentar sonido, no impide ello.  Exhibir E-xhi-bir
Exhortar E-xhor-tar
5. Si “cc” concurren en una palabra, cada Coacción
5.  Co-ac-ción
diferentes. 
letra se va a sílabas diferentes.  Occiso Oc-ci-so
Cocción Coc-ción
6. 
6.  Las palabras con prefijos se Suboficial Su-bo-fi-cial
segmentarán aplicando las reglas del De-sho-nes-to

CEPRE-UNI HUMANIDADES 7
 

CICLO INTENSIVO ESCOLAR NACIONAL Material de Estudio Nº 1 

silabeo: cuando una consonante se ubica Deshonesto Tra-sa-tlán-ti-co


entre dos vocales, esta forma inicio de Trasatlántico Po-so-pe-ra-to-rio
sigue. 
sílaba con la vocal que le sigue.  Posoperatorio De-she-cho
Deshecho

2. EL ACENTO
Es la mayor intensidad de voz con que se pronuncia la sílaba de una palabra. Sirve, en
algunos casos, para diferenciar significados. Ejemplos: pa so (modo o manera de andar),
paso
pa só   (tiempo
pasó
(pasado pasado
del verbo del verbo), há bito (costumbre), habi to (del verbo habitar) y habitó  
habitar).

2.
2.1
1CClases
lases de pala
palabras
bras según la
l a posición
posic ión d
del
el acento
acento o sílaba tónic
tónicaa

POSICIÓN DEL ACENTO


CLASES
CLASES Trasantepe
Trasantepenúl
núltima
tima Antepenúlt
Antepenúltima
ima Pe
Penúlt
núlt ima Última
Agudas u ti li dad
Graves com pren de
Esdrújulas brú  ju la
Sobresdrújulas llé va te lo

2.2 Reglas de acentuación general

PALABRAS LLEVAN TILDE…  EJEMPLOS


(a, e, i, o, u)
si terminan en vocal (a, u) o en acción
acción,, traspi
traspiéé, radar 
radar ,
Agudas consonante n o s. Si terminan en nitidez
niti dez,, corrió
corrió,, segú
según n,
grupo consonántico
consonántico,, no se tildan. zigzag
zig zag,, esnobs
esnobs,, tuaregs
tuaregs..
solo si acaban en consonante, obl i cuo, náu
náuseas,
seas, poe
poeta
Graves excepto en n o s. Si terminan en carác
carácter,
ter, err
errá
átil, néc
néctar
tar
grupo consonántico
consonántico,, sí se tildan. cómics,
có mics, sóviets
só viets,, trémens.
trémens.
ánfora,
án fora, náu
náutica,
tica, índice
ín dice
Esdrújulas
método,
mé todo, fascí
fascículo,
culo, pictó
pictó rico.
siempre.
Ex
Expl
pl ícaselo, ll évatelo, imagí
imagínatela,
natela,
Sobresdrújulas
recí
recítaselo,
taselo, devuél
devuélvesela.
vesela.

Por regla general, las palabras de una sola sílaba no llevan tilde. Ejemplos: Hui, guion, vio, fe, ti . 
EVALUACIÓN

1. Con respecto a la verdad (V) o falsedad (F) de 2. Señale la alternativa cuyas palabras se han
los siguientes enunciados
enunciado s acerca de las palabras silabeado correctamente. Se omitieron las tildes.
según su acentuación, señale la opción que I. Pre-in-ca, sahu-me-rio, gen-ti-o
contiene la secuencia correcta. II. Re-cons-tru-i-do, mo-hin, flu-i-do
I. Las palabras sobresdrújulas tienen dos III. Pa-ra-no-ia, re-hu-se-se, huay-las
acentos. IV. Bo-hi-o, ca-us-ti-co, a-cep-ta-ri-a
II. Todas las palabras esdrújulas son A) I, II, III B) I, III C) I, IV D) II, IV E) II, III
compuestas.
III. La palabra “avaro” se tilda por ser esdrújula
esdrújula.. 3. Acerca de la acentuación general, señale si el
IV. “Llévatelo” tiene el acento en la enunciado es verdadero (V) o falso (F) según
trasantepenúltima
trasantepenúl
A) tima sílaba.
FVFV B) VFVF C) VVFF D) FFFV E) FVVV I.corresponda.
Las palabras “cohibe” y “ahinco” no requieren
tilde en “i”. 

CEPRE-UNI HUMANIDADES 8
 

CICLO INTENSIVO ESCOLAR NACIONAL Material de Estudio Nº 1 

II. “Récords” y “superávits” presentan correcta 7. Elija la alternativa que presenta la secuencia
tildación. de palabras grave, aguda y esdrújula. Se han
III. En “exámen” y “resúmen”, se ha tildado omitido las tildes.
adecuadamente. A) Cancer, canjear, cuentalo
A) VFF B) FVV C) FVF D) VFV E) FFF B) Monitor, futil, poetico
C) Increible, exiguo, impudico
4. ¿Qué enunciado evidencia error de D) Incesto, mamut, dejamelo
acentuación? E) Espontaneo, campeon, brujula
A) Wálter licuó ayer las frutas para hacer el
 jugo. 8. ¿Qué alternativ
alternativaa evidencia correcta
B) Planteó un argumento futil en su exposición. segmentación silábica?
C) Algunos regímenes son antidemocrá
antidemocráticos.
ticos. A) In-hós-pi-to, cau-ción, li-te-ra-ria
D) El joven ingeniero presentó su monografía. B) Pro-hi-bi-ción, te-nue, coe-tá-neo
E) Encontraron un raro espécimen en el mar. C) Ca-rro-ce-rí-a, e-xha-la-ción, có-mics
D) Hui-rí-a, te-ra-pé-u-ti-co
te-ra-pé-u-ti-co,, cá-us-ti-co
5. ¿Qué alternativa requiere mayor cantidad de E) Ahín-co, o-bli-cuo, vein-ti-séis
tildes?
A) El joven sufrio un vahido en el omnibus 9. ¿Qué alternativ
alternativaa presenta triptongo, diptongo
anoche. y hiato, respectivamen
respectivamente? te?
B) Se quedo livido al ver la huida del fiero A) El poema del Mio Cid fue leído por Eleuterio
mastin. García.
C) Mi tio tiene un lindisimo bonsay en su patio. B) No desconfiéis de quienes son sinceros y
D) El ultimo campeon de boxeo es un novel honestos.
pugilista. C) El callejón de Huaylas tiene vías para el
E) Raul Leon estudia Ingenieria Metalurgica en turismo.
la UNI. D) Huaura fue la primera ciudad libertaria del
norte.
6. Señale la alternativa cuyas palabras se han E) Una almohada nueva debe ser bastante
acentuado correctamente
correctamente.. suave.
A) Caracteres, diluído, boína
B) Frenesí, apoteósico, truhan 10. Señale la alternativa que presenta hiato,
C) vió mohín, teniaís triptongo y diptongo, respectivamente.
D) tí, vehículo, tictác A) Estuvieron en Paraguay durante dos días.
E) Fluído, nubil, oidor B) El zoológico es muy elegante en Huaura.
C) Veo que los guiais con mucha atención.
D) El búho produjo un sonido biaural real.
E) Incluí la biografía de los autores uruguayos.

TEMA
 ACENTUAIII. ACENTUACIÓN
 ACENTUACIÓNENFÁTICA
CIÓNENFÁTICA, DIACRÍTICA
CIÓN DE PAL(MONOSÍLABOS,
, ACENTUACIÓN
ACENTUA AB
ABRAS POLISÍLABOS),
RAS COMPUESTAS.

1. ACENTUACIÓN DIACRÍTICA (TILDE DISTINTIVA) 


La tilde diacrítica diferencia palabras que, a pesar de contener las mismas grafías,
distintas.  
pertenecen a categorías gramaticales distintas.

Ejemplo:
  De nada sirve que le dé
dé algunas
 algunas explicaciones. 
explicaciones. 

(verbo) (preposición)

  Si llegas
Si  llegas temprano, sí
sí veremos
 veremos el atardecer.

(conjunción) (adverbio)

CEPRE-UNI HUMANIDADES 9
 

CICLO INTENSIVO ESCOLAR NACIONAL Material de Estudio Nº 1 

2.
2.1
1 En monosílabos
Las palabras monosilábicas, por regla general, no se tildan, por ejemplo, fui, Sion, fie. Sin
embargo, los siguientes monosílabos pueden tildarse, porque es necesario diferenciarlos
entre sí.
Monosílabos
Monosí labos con tilde Monosí labos sin tilde  
Monosílabos
Dé Verbo “dar":  De Preposición:
Ojalá que le dé la
dé la vacante. La encuesta es lo de menos.
de  menos.  
Más Adverbio de cantidad:
Más Mas Conj. adversati
adversativa
va  “pero”: 
 Ahora conocen más
más la
 la teoría. Parece que desea participar, mas
mas   su
Sustantivo (signo matemático): engreimiento la domina. 
domina. 
Colocó el más
más adecuadamente.
 adecuadamente.
Él Pronombre personal: El Artículo masculino:
No creo que él
él note
 note sus ambiciones.
ambiciones. Reparó el
Reparó  el motor
 motor de corriente continua.
continua.  
Mí Pronombre personal: Mi De
Determinante
terminante posesivo:
 A mímí   no me va a engañar con Mi amortiguador
Mi  amortiguador está dañado.
hipocresías. Sustantivo (nota musical):
musical):
Escuchó la melodía en mi menor.
mi  menor.
Tú Pronombre personal: Tu De
Determinante
terminante posesivo:
Es cierto que tú   no eres de esta Parece que tu experiencia
tu  experiencia es mínima.
espacialidad.  
espacialidad.
Sé Verbo “ser” o “saber”:  Se Pronombre personal:
 más honesto. (“ser”) 
Sé más
Sé Es posible que se
que se desmaye
desmaye.. 
No sé  si es limeño. (“saber”) 
sé si
Sí Adverbio de afirmación: Si Con
Conjunción
junción condicional:
Martín sí
sí entendió
 entendió el mensaje. Si
Si está
 está en Lima, lo invitaré.
Pronombre personal:
Estaba orgullosa de sí misma.
sí  misma. Sustantivo (nota musical):
Sustantivo:  
Sustantivo: Compuso la melodía en si  bemol.
 bemol.  
Dudo que le dé el sí.sí . 
Té Sustantivo: Te Pronombre personal:
Prefiere tomar un té verde con limón. No te informó
te  informó sobre sus faltas.
Los tés
tés ya enfriaron. 
 ya se enfriaron. 

2.2 En el polisílabo “aún” 


 Aún
 Aú n Aun
Au n 
Se acentúa cuando puede reemplazarse
No se tilda cuando puede reemplazarse por las palabras
por ‘todavía’ (adverbio de tiempo). 
tiempo). 
‘hasta’, ‘incluso’ o la locución conjuntiva ‘aun cuando’. 
  Au
 Aunn  Maricarmen resolvió la pregunta. (Hasta)
  Los laboratorios no están listos aún.
aún.  
  Au
 Aunn  en invierno, usa polos cortos. (Incluso)
  Aú
 Aún n no revisa la ley de Newton.

2.3 Tilde enfática


Es usada cuando las palabras qué, quién, cuál, cómo, cuándo, cuánto, dónde y adónde 
adónde  son
pronunciadas con énfasis, y forman estructuras de sentido interrogativo o exclamativo ya sean
directas o indirectas.
Ejemplos:  
Oraciones interrogativas o exclamativas directas
  ¿Dónde queda la Facultad de Petróleo?   * ¿A  quién le pertenece este carné? 
¿A quién
  ¡Qué
Qué  calor tan intenso hubo este verano! * ¡Cuán  inteligente eres, estimado Víctor! 
Cuán inteligente

CEPRE-UNI HUMANIDADES 10
 

CICLO INTENSIVO ESCOLAR NACIONAL Material de Estudio Nº 1 

Oraciones interrogativas o exclamativas indirectas


   Avísale a qu
quéé hora será el primer simulacro.
  Te enteraste en dónde  estuvieron regalando libros. 
dónde estuvieron
  Fue sorprendente cómo pudo controlar sus nervios. 
  Observa cuán sencillo es elaborar las preguntas.

 En las oraciones exclamativas que expresan deseo, la palabra “que” no se tilda.
NOTA: En
NOTA:
E em lo: Que
Que  sea todo ara bien! / Que
Que  te me ores
ores ro
ron
nto ami
ami a! / Que
Que aumenten
 aumenten el sueldo! 
sueldo! 

3. ACENTUACIÓN DE
DE PALAB
PAL ABRAS
RAS COMPUESTAS
COMPUESTAS
Reglas Ejemplos
 Cortacésped

Unidas sin Se tildan según las reglas de  Veintidós

guion acentuación.   Abrefácil


Abrefácil
 Vigesimocuarto

  Analítico-sintético
Analítico-sintético
Unidas con Cada palabra del compuesto conserva  Germano-soviético

guion la tilde si la necesitase.  Económico-social


 Sánchez-Cano

 Dócil+ -mente = dócilmente


 física+ -mente = físicamente


Terminadas Conservan la acentuación original del  legal + -mente = legalmente



en -mente adjetivo del que derivan.
 sarcástico+-mente =

sarcásticamente

Principales
Princi pales modificacio
modif icaciones
nes de la RA
RAE
E en
en las reglas de acentuación
acentuación diacríti
diacrítica
ca

la conjunción “o”  entre cifras.


- No sé si son 10 o 14 actos.
No se
tilda el adverbio “solo”   aun cuando exista ambigüedad.
- Estará solo
solo este
 este sábado.
incluso cuando haya riesgo de
los pronombres ambigüedad.
demostrativos
- Dile que esta
esta es
 es más confortable. 

EVALUACIÓN

1. Determine las oraciones que presentan uso


1. 
tilde. 
correcto de la tilde.  2. Señale qué afirmaciones son verdaderas en
I. Tiene que estudiar mas aún para este último relación a las reglas de acentuación
acentuación..
examen. I. La palabra “huida” se tilda por presentar hiato
II. Fácilmente, Raúl logró freír el ají, incluida la acentual.
mayonesa. II. La palab
palabra
ra “brutálme
“brutálmente”nte” está cor
correctamente
rectamente
III. No sabe el porqué de esta situación por falta tildada.
de fé y paz. III. En la oración “Cómo lograste aprobar el
IV. Si tuviera tiempo, lo visitaría y él prepararía curso”, hay correcto empleo de la tilde. 
A) I, II y III B) II y III C) I y III
un Ibuen
A) y III té. B) II y IV C) III y IV D) Solo III E) Solo I
D) I y IV E) II y III

CEPRE-UNI HUMANIDADES 11
 

CICLO INTENSIVO ESCOLAR NACIONAL Material de Estudio Nº 1 

3.  Señale la oración que presenta más errores


3.  E) Debe corregir la te de la última palabra
palabr a de este
de acentuación
acentuación.. texto.
A) El ión es una partícula cargada eléctricamente
eléctricament e
constituida por un átomo o molécula. 7. Determine la oración que requiere de mayor
B) Despues del mítin, el premiér dió varias número de tildes.
declaraciones a distintos medios de A) Saul Alvarez cria animales de corral y vive
comunicación. cerca de un rio.
C) El manager de Oscar de León perdió varios B) Aun no encuentra los veintitres soles con
miles de dólares en un conocido hotel. cinco cents.
D) Existen varias anecdotas acerca del tíovivo C) Este escaner tenia muy buena resolución de
imagenes.
por la música y asientos desplazados
mecánicamente. D) Amigo mio, sabes que opina el; el arcoiris esta
es ta
E) No sólo la curcuma ayuda a reducir la acidéz bellisimo.
del estómago, sino también otros alimentos. E) El pixel es la resolución o el numero de puntos
del toner.
4. ¿Cuál es la oración que presenta correcta 8. Marque la oración que presenta correcta
acentuación ortográfica? acentuación.
A) El hispano-marroquí cuestionaba la tradición A) Timidamente, se presentó con un paraguas
etnico-religiosa.  
etnico-religiosa. ante el portugués.
B) Él vió cómo caían las púas de aquel animal B) El niño reventó los buscapies bajo el frio
tras la pelea. invierno de París.
C) Pídele que te dé el solucionario de Física C) El físicoculturista la mayoria de veces entrena
Cuántica. mucho con pesas.
D) La máquina producía un fuerte ruído
r uído cerca al
salón principal. D) Para preparar
únicamente las legumbres, necesita
el pasapurés.
E) Mónica de los Santos se sorprendió por todo E) Recibió el tentempié en el décimocuarto piso
lo que se de ella. de la Vía Expresa.
5. ¿En qué oración es necesaria la tilde enfática? 9. Determine la oración en la que hay error de
A) Newton propuso que qu e  había una atracción acentuación enfática.
gravitatoria. A) ¡Que seas feliz
f eliz por siempre con todos ellos!
B) ¡Que
Que   sean muy felices porque realmente se B) Averigua si dónde vive es casa propia o
lo merecen! rentada.
C) Nadie entiende lo que
qu e a ella le pasa cuando C) Nadie sabe cuándo terminara todo esto.
la evalúan. D) ¡Qué jóvenes tan inteligentes y audaces!
D) Será interesan te aclarar cual
interesante cual es
 es la meta para E) Me gusta cómo estudian todos mis alumnos.
este ciclo.
E) No encontró la hoja en la cual
cual firmó
 firmó y colocó 10. Marque la alternativa cuyo enunciado
su huella.
evidenci
evidencia
a adecuada
A) Cuándo oimos el tildación.
ruído pensamos que era un
6. Indique la oración en la cual se haya omitido terremoto.
la tilde diacrítica. B) Aún cuando tengas la razón, se prudente en
A) Se fue con él esperando que responda un si tu conversación
conversación..
absoluto. C) Querida amiga, el té que te pedí ayer aún no
B) Si no es lo mismo ir allá o estar acá, se lo sé qué tenía.
comentará. D) ¿Por qué no vino Oscar si tenía prácticas de
C) Seguirá a tu lado aun cuando le des pocas laboratorio?
esperanzas. E) Desarrollamos los ejercicio
ejercicioss teorico-práctic
t eorico-prácticos
os
D) ¿A ti te agradan las notas musicales en mi de Química.
menor?

TEMA IV. SIGNOS DE PUNTUACIÓN. USO DEL PUNTO, COMA, PUNTO Y COMA, DOS
PUNTOS Y PUNTOS SUSPENSIVOS

CEPRE-UNI HUMANIDADES 12
 

CICLO INTENSIVO ESCOLAR NACIONAL Material de Estudio Nº 1 

Los signos de puntuación son signos ortográficos cuya función primordial es organizar los
enunciados de forma óptima para facilitar su comprensión y evitar posibles ambigüedades.
Estos son el punto, la coma, el punto y coma, los dos puntos, lo
loss paréntesis, los corchetes, la
raya, las comillas, los signos de interrogación y exclamación, los paréntesis y los puntos
suspensivos.

1. LA COMA ( , ) 
Indica una pausa breve
breve en la oración. Se escribe pegada a la pal
palabra
abra que la precede y
separada porestablece
presenta, se un espacio
espacio
su de la palabra que la sigue. De acuerdo con las funciones qu
clasificación. quee

1.1 Clases de coma


co ma
El ingeniero metalúrgico desarrolla sus actividades
dentro del campo de la minería,
minería, la metalurgia física
Separa elementos ferrosa y la metalurgia química
química..
similares (palabras, La ingeniería ambiental estudia los problemas
Enumerativa frases, proposiciones) de ambientales de forma integrada considerando
enumeración. 
una enumeración.  dimensiones ecológicas,
ecológicas,  sociales,
sociales,  económicas y
tecnológicas.
Observaba, meditaba, deducía y resolvía los casos.
Niels Bohr, ganador del Premio Nobel de Física,
Encierra un adicional.
explicación comentarioSe
o formuló
Los el principio
ashánincas de correspondencia.
y aguarunas, los grupos étnicos
inserta, por lo general, amazónicos con mayor población, fueron los que
Explicativa dentro del enunciado impulsaron la organización de sus comunidades.
principal. El Chino, como cariñosamente lo llaman sus
colegas, narra que en el año 1987 quedó herido y
casi pierde la vista cuando una granada explotó en
manos de un terrorista.
terrorista. 
¿Podría repetir su explicación
explicación,, profesor?
Vocativa  
Vocativa Separa al vocativo de la Queridos alumnos, nos vemos pronto.
oración.  
oración. Permiso, Daniel, necesito salir.
Se utiliza cuando la  Al estudiar la evaporaci
evaporación,ón, Dalton se interesó por
ubicación del otra cuestión: la composición del aire.
complemento  A medida que se aceptaba la importancia y la
Hiperbática circunstancial se altera utilidad de su teoría atómica,
atómica,  Dalton empezó a
en la oración
aparece sujeto recibir
antes del(cuando Desde honores.
los inicios de su carrera,
carrera, Bohr fue muy hábil
o del verbo). en conseguir financiación para sus proyectos.
El rector felicitó
felicitó   a los ingresantes; la directora,
directora,  a
Se usa cuando el verbo los padres.
está omitido por haber El Perú es un banco mundial de recursos genéticos;
Elíptica sido mencionado con en papa somos somos   los primeros; en maíz,maíz,  igual; en
anterioridad o estar camélidos andinos, andinos, los mejores del mundo.
sobrentendido. Los que no tengan invitación,
invitación, por esa puerta.
Ocho por siete,
siete, cincuenta y seis.
Acompaña a las Creo que he desarrollado bien la prueba final, sin
conjunciones embargo,, tengo dudas.
embargo
adversativass (pero
adversativa pero,, mas
 mas,,  El aislamiento los ha hecho vulnerables a los virus,
sino,,  sino que,
sino que,  sin por eso, huyen
eso, huyen del contacto con otros.
embargo),
embargo ), explicativas Sabía que era peligroso, mas mas no
 no tenía miedo.
Conjuntiva (es decir , o sea),
sea), ilativas Volvió muy decepcionado, es decir, nodecir, no le fue bien
(por ello,
ello ,  luego,
luego ,  en en la prueba.

CEPRE-UNI HUMANIDADES 13
 

CICLO INTENSIVO ESCOLAR NACIONAL Material de Estudio Nº 1 

consecuencia )
consecuencia) y
(pues,, 
algunas causales (pues
ya que),
que), en enunciados
breves.
Se usa delante de la Pagó el traje, el bolso y los zapatos, y  salió de la
conjunciones y, o 
o  cuando tienda.
introduce una idea No sé si postular a Mecatrónica o Arquitect
Arquitectura,
ura, o  
distinta de la prepararme otro ciclo.
enumeración o cuando
indica una conclusión o Pintaron las de
disposición paredes de la habitación,
los muebles, pusieroncambiaron la
alfombras
una consecuencia. nuevas, y  quedaron encantados con el resultado.

Asimismo, si la conjunción consta de una sola palabra, se le antepone una sola coma; si
consta de más de una palabra, dicha conjunción va entre comas. Por otro lado, las
conjunciones que terminan en qu
quee llevan una sola coma antes de la conjunción. Ejemplos:
   Abordó temprano el tren en Grau,
Grau, pero
pero llegó
 llegó tarde a sus clases.
   Ayer estaba todo bien,
bien, sin embargo, hoy
embargo, hoy no desea verme.
  Ha decidido retomar su dieta,
dieta , debido a que aún
que aún no adelgaza.
No se debe escribir la coma entre el sujeto y el verbo, aunque el sujeto sea muy extenso o se
presente una frase explicativa en el sujeto. Ejemplos:
T-302,,  postularán en febrero. (X)
La mitad de mis alumnos del Cepre-UNI del aula T-302
sujeto verbo

T-302  postularán en febrero.(X)


La mitad de mis alumnos del Cepre-UNI del aula T-302 
sujeto verbo

El entrenador argentino Ricardo Gareca, Tigre, agradeció a la prensa nacional. (√) 


Gareca , el Tigre,
sujeto verbo

2. EL
EL PUNTO Y C COMA
OMA ( ; )
Indica una pausa mayor que la de la coma y menor que la del punto.

2.
2.1
1UUsos
sos del pu
punto
nto y ccoma
oma
  La
primera parte de los Comentarios reales de los
incas, la cual consta de nueve libros, se publica en Lisboa;
Lisboa;  
Separa elementos de una la segunda parte, en 1617.
A. enumeración compleja   Algunos representan
representantes
tes de la escuela mercantilista
que incluye comas. son Thomas Mun, político inglés; Colbert, quien defiende
el proteccionismo económico;
económico;  Midelsen, el que dio origen
a la expresión balanza comercial y política estatal.
  El nuevo alcalde inauguró el túnel Santa Rosa; Rosa ; la
teniente alcaldesa supervisó el puente Mirones.
  Esteban vivió un tiempo en Inglaterra;
Inglaterra ;   actualmente,
Enlaza proposiciones 
proposiciones 
B. yuxtapuestas.. 
yuxtapuestas
reside en Colombia.
  El hedonismo sostiene la percepción de los valores
guiada por el placer ;  el utilitarismo defiende la valoración
orientada al beneficio.

CEPRE-UNI HUMANIDADES 14
 

CICLO INTENSIVO ESCOLAR NACIONAL Material de Estudio Nº 1 

C. Enlaza proposiciones   La atención no puede fragmentarse y concentrarse en


coordinadas adversativas dos o más estímulos distintos; peropero   sí puede pasar
(pero, sin embargo),
embargo ), progresivamente
progresivamen te de un estímulo a otro.
(es decir, o
explicativas (es   La hernia inguinal puede resultar peligrosa;
peligrosa ;  ya que el
que el
sea)) e ilativas (por
sea (por ello,
ello ,  intestino delgado puede atascarse y cortar su suministro
luego,,  en consecuencia)
luego consecuencia) de sangre.
cuando las ideas son   El método cartesiano,
cartesiano,  expuesto en el Discurso del
extensas o tienen método y en las Reglas para la dirección del espíritu, tiene
interna. 
puntuación interna.  como propósito
absoluto
absoluto;;  es decir,advertir
facilitar en
a la todas lasnítida
mente la cosas lo más
percepción
de los asuntos.

Asimismo, en enumeraciones complejas de más de dos elementos, se usa el punto y coma o


la coma delante de la conjunción y. Ejemplos:
 Adoro leer a Neruda,
Neruda, poeta chileno;
chi leno;   a Vallejo,
Vallejo, el Chol
Cholo;
o; y
 y a Bécquer, autor de Los ojos
verdes.. 
verdes

 Adoro leer a Neruda


Neruda,, poeta chileno; 
chileno;   a Vallejo, el Cholo, y
Cholo, y a Bécquer, autor de Los ojos
verdes..
verdes

3. LOS DOS PUNTOS ( : ) 

Usos Ejemplos
  Hoy se sabe que el átomo está formado por tres
 An ti ci pad
 Anti padaa (con
unidades menores: 
menores:   protones, electrones y
una palabra que
neutrones.
presenta o
  La geometría parte de ciertos conceptos
introduce a los
básicos: plano, punta, recta.
elementos de una

enumeración)   Llevó todo
selección
selección: lo borrador,
:  lápiz, n ece
ecesario
sario para
 para rendir
tajador, regla,laetc.
prueba de
A.   El actual sílabo y las preguntas para el banco: banco :  
estos son algunos puntos que se abordaron en la
primera reunión.
Enumeración   Hospitales, prisiones, hacienda pública,
Cierre  
Cierre
agricultura::   nada
agricultura nada   escapó al ímpetu reformador de
Lavoisier.
  Natural, sana y equilibrada:
equilibrada: así 
así  debe ser una
buena alimentación.
   Antoine Lavoisier afirmó:
afirmó:  “Solo debemos confiar
en los hechos. Estos nos son presentados por la
Cita textual antecedida por el verbo 
verbo  naturaleza y no pueden engañar”. 
B.   Ya lo dijo Ortega y Gasset:Gasset:   “La claridad es la
cortesía del filósofo”. 
Fórmulas de saludo   Querida hija:/ Nos vemos… 
(documentos o cartas)   Estimados alumnos:/ Les comunicamos…  

CEPRE-UNI HUMANIDADES 15
 

CICLO INTENSIVO ESCOLAR NACIONAL Material de Estudio Nº 1 

  Sr. padre de familia:


familia: / / Reciba mi saludo…
saludo… 

causa-efecto   Subió la tarifa del Metropolitano: la población


marchará hacia el Congreso.
  Fue a preguntarle a la profesora:
profesora:   no había
C. Relación de…  efecto-causa  
efecto-causa comprendido el ejercicio.
(en oraciones
yuxtapuestas)   El eje terrestre es denominado línea de los polos:
polos:  
ejemplificación  
ejemplificación  une los polos norte y sur.
Oposición   Nina no es una persona:
persona :  es la gatita de Pierina.

Usos incorrectos Ejemplos


Ejemplos  
  Erasmo de Rotterdam fue el principal
a. las preposiciones
Después de las preposiciones
representante del:
del: Humanismo.
 Humanismo.
  Heráclito propuso que:
que: el
 el universo es un eterno
b. Después de la conjunción «que»
fuego.

L OS PUNTOS SUSPENSIVOS ( … )


4. LOS

Usos  
Ejemplos
Niels Bohr afirmó: “ Si
Si alguien no queda confundido
Indican la omisión de un fragmento  por la física cuántica, entendido bien”. 
cuántica, es […] entendido
A. en una cita textual.   El nuevo alcalde nos dijo: “Espero que todos
estemos contentos con esta nueva obra y…”. 
Indican la omisión de información,
  Me gustas cuando callas  
sea deducible (refrán, poema) o

B. no.
no.     Un tonto siempre encuentra   …

  Te tengo que contar que  


C. Expresa duda, te
temor,
mor, ironía, etc.
 

El maestro ya sabe  el ejercicio  


… …

  Han entregado a las aulas las calculadoras, las


Se emplea en enumeraciones
D. incompletas con el mismo valor de fichas
 
ópticas  para la prueba final.

Las lenguas que nacieron del latín son el sardo, el


la palabra etcétera.
etcétera.  
italiano, el francés, el castellano   …

EVALUACIÓN  

1. Indique qué alternativa carece de uso E) Al culminar la secundaria, los jóvenes migran
adecuado de la coma hiperbática. de sus comunidades a las ciudades para
A) Por la pérdida del sedentarismo, la vivienda y continuar sus estudios.
la agricultura dejaron de ser necesarias
necesarias..
B) Con una mejor educación, los procesos de 2. Marque la opción que presenta adecuado uso
entendimiento y adaptación de las comunidades de los dos puntos.
permitirán un desarrollo más equilibrado. A) El coordinador del taller nos comunicó que:
C) A Giordano, lo felicitaron sus padres y los libros los entregarán mañana.
maestros por su maestría virtual en educación B) Anunciaron la subida del pasaje escolar:
en España. marcharán los jóvenes por las calle
D) Como en la música y en la moda, las ciencias C) La joven médica no pudo: salvar la vida del
también están ofreciendo temas de moda. menor llegó sin vida al nosocomio.
D) El profesor nos indicará: cuándo, dónde y
cómo rendir la prueba final del curso.

CEPRE-UNI HUMANIDADES 16
 

CICLO INTENSIVO ESCOLAR NACIONAL Material de Estudio Nº 1 

E) Los isómeros se clasifican como: isómeros de B) A fines de febrero; se mudará a: Pacaraos


función, posición y de cadena. (Huaral), pero regresará en otoño.
C) Antón cenó con su familia de todo: pollo y
3. Identifique la oración con uso adecuado de los chifa, etc…, y aún siente hambre. 
signos de puntuación. D) Raúl, mi mejor amigo, ingresó a la UNI, su
A) En el 2017; la Organización de las Naciones novia, a la PUCP y yo, a la UNMSM.
Unidas (ONU) decidió proclamar a 2019 como: E) El dolor abdominal se relaciona con muchos
el Año Internacional de la Tabla Periódica. procesos; como estreñimiento, gases, etc.
B) En 1869, se conocían 63 elementos
químicos; pero Mendeleev predijo que: se 7. Identifique el enunciado con uso adecuado de
descubrirían más para completar los vacíos de los signos de puntuación.
su tabla. A) El profesor nos habló sobre Vargas Llosa; el
C) En la actualidad, se conocen 118 elementos nuevo tutor, la orientación vocacional y el
químicos: los primeros 94 existen naturalmente, director, conversó sobre la beca.
el resto; por laboratorio. B) Salvador del Solar el nuevo jefe de la PCM,
D) Los países que más elementos químicos han calificó como “natural” que… duden de su
descubierto son: Inglaterra, EE.UU., Suecia y gestión.
Francia; y con menos índice: México y Costa C) El 21 de diciembre del 2003, en el Estadio
Rica. Nacional, César Vallejo derrotó 9-4 a DEA.
E) Las últimas adiciones a la tabla periódica en D) El rector manifestó que: “LA OCAD es
2016 son cuatros elementos superpesados: responsable de … de los postulantes”. 
nihonio, moscovio, teneso y oganesón. E) Ingresaron todos mis amigos Antonio a
Mecánica; Luis, a Física; Ana, a Ambiental; … 
4. ¿Qué enunciado requiere mayor cantidad de
signos de puntuación? 8. Marque la alternativa que evidencia uso
A) Mi padre Darío quien era policía era un adecuado de los dos puntos.
aficionado al ajedrez. A) Ernesto fue operado de emergencia: presenta
B) Este jueves ha sido el mejor día de todos un cuadro de aneurisma.
querida hija. B) La políglota se presentó en: Cuarto poder y
C) Llevan casados mucho tiempo 20 años pero ATV noticias el lunes.
nunca logran entenderse. C) Ha esperado por su nombramiento durante:
D) Parece que últimamente le va muy bien en ocho años, Leonardo.
cepre regresa triste a casa. D) José estudiará en el aula A-303 en el turno:
E) Sus palabras fueron No lo haré pero al final tarde durante este ciclo.
nos ayudó. E) La química estudia la composición interna de:
la materia.
5. ¿Cuál es la alternativa que presenta uso
inadecuado de los signos de puntuación? 9. Señale la alternativa que presenta uso
A) Guillermo
premios Óscar:del Toro
mejor (mexicano)
vestuario ganó
y mejor dos
música. correcto de los
A) Si y solo si tesignos de puntuación.
esfuerzas mucho; alcanzarás
B) En la Guerra Fría, se usó un programa de el éxito, estimado alumno.
ayuda económica y alianzas militares: el plan B) Las menestras te brindan hierro: las
Marshall. verduras te proporciona
proporcionann antioxidantes.
C) La Revolución Industrial produjo un cambio C) Iremos al Castillo de Chancay, a los baños
social, político y en el estilo de la vida de la termales, a las lomas, etc....
humanidad. D) Es tan
t an «inteligente» que obtuvo el primer
D) Mi familia visitó museos, huacas e iglesias; no puesto en la Facultad de Química.
obstante, no quedaron muy complacida. E) El pescado, las frutas, las menestras, el
E) El productor y conductor viste muy elegante: agua son necesarios en la dieta.
lleva un “short” y sandalias”. 
10. ¿Qué oración presenta correcto uso de la
6. ¿Qué opción presenta mayor cantidad de coma?
errores de signos de puntuación? A) La playa Agua Dulce situada en el distrito de
A) Federico desea organizar: una actividad pro- Chorrillos, no está bien cuidada.
fondos para la promoción 2022. B) El estudiante del octavo ciclo de la Facultad
de Contabilidad, obtuvo una beca.

CEPRE-UNI HUMANIDADES 17
 

CICLO INTENSIVO ESCOLAR NACIONAL Material de Estudio Nº 1 

C) El respeto de las personas se gana; la E) Mónica compró colores, acuarelas,


subordinación de ellas, se exige. plumones, crayolas y pintó un bodegón.
D) Profesor Fernando Casanova, ¿veinticinco
por cinco, ciento veinticinco
veinticinco??

TEMA V. EL SUSTANTIVO: RECONOCIMIENTO Y CLASES (SIMPLES, COMPUESTOS,


COLECTIVOS E INDIVIDUALES).
INDIVIDUAL ES). GÉNERO Y NÚMERO DE LOS SUSTANTIVOS SIMPLES
Y COMPUESTOS, SIGLAS Y ACRÓNIMOS

1. EL SUSTANTIVO
Es la palabra que nombra a la persona, animal o cosa. Puede estar constituid
constituidoo por uno o más
significado.  Ejemplos:
morfemas, que son unidades mínimas con significado.  Ejemplos:  
1.1 Reconocimiento del sustantivo
¿Cómo se reconoce un sustantivo?
  Suele estar
estar acomp
acompañado
añado de artículos, posesivos, numerales u otros det
determinantes.
erminantes.
La multitud,
La  multitud, mis ahijadas, estos
estos directores,
 directores, varias
varias prácticas
 prácticas
El corcho nuevo,
nuevo, un
 un gran
gran saludo,
 saludo, la antigua
antigua y
 y amplia
amplia aula
 aula

2. FORMACIÓN DEL GÉNERO Y DEL NÚMERO 

2.1 Género
El género de los sustantiv
sustantivosos es una pr
propiedad
opiedad gramatical que los clasifica como masculinos
o femeninos. Suelen ser masculinos los sustantivos terminados en -o   (plan o , ángul
plano o )  y,
ángulo
femeninos los terminados en -a (arm armaa, áre
áreaa).
No obstante, existen sustantivos masculinos terminados en -a. Ejemplos: (el) aroma, clima,
dogma, trauma, tema, cisma, teorema, problema .  Además, hay sustantivos femeninos
terminados en -o .  Ejemplos:  (la)  libido, nao, seo, mano; así como los que provienen de
acortamientos, como  (la) disco (de discoteca), foto (de fotografía), moto (de motocicleta),
polio (de poliomielitis), quimio (de quimioterapia). 

2.1.1 Reglas para formar el género

Criterios  
Criterios Reglas MASCULINO  Ejemplos 
MASCULINO  Ejemplos   FEMENINO
FEMENINO  
gallo
gall o abad in a 
gallin
gall abadesa
abad esa
Se añade un sufijo a
colegial diácono
diácon o colegiala
colegial a  diacon
diaconisa
isa
la raíz.
Morfológico  
Morfológico anfitrión  
anfitrión  jabalí  
 jabalí anfitriona
anfitrion a   jabalin
 jabalin a 
el
el postulante
 postulante   la postulante
la  postulante  
el testigo la testigo
la  testigo
Se antepone un
el
el piloto
 piloto   la piloto
 piloto  
determinante al
el
el cónyuge
 cónyuge la cónyuge
la  cónyuge
sustantivo.
el
el portavoz
 portavoz   la portavoz
la  portavoz  
el
el pianista
 pianista la pianista
la  pianista  
Sintáctico  
Sintáctico
la hormiga machomacho   la hormiga hembra hembra  
la culebra macho
culebra macho la culebra hembra
culebra hembra
Se pospone un
el mosquito macho el mosquito hembrahembra  
adjetivo al sustantivo
el espárrago macho el espárrago hembra
(epiceno). la ruda macho la ruda hembra
ruda hembra
el ajo macho el ajo hembra

CEPRE-UNI HUMANIDADES 18
 

CICLO INTENSIVO ESCOLAR NACIONAL Material de Estudio Nº 1 

el vástago masculino
vástago masculino el vástago femenino
el personaje varón
varón   el personaje mujer  
carnero  
carnero oveja  
oveja
Se usa una palabra caballo yegua
Lexicológico  
Lexicológico diferente chivo cabra
(heterónimo). fray sor
 jinete amazona

  Algunos sustantivos aceptan dos formas para señalar el género.


A) El jefe
El  jefe / l a jefe, jef a  B) El fiscal
El  fiscal / la
la fiscal, fiscala 
 fiscal, fiscala
C) El
 El gerente
 gerente / l a gerente, gerenta
gerenta D) El presidente
El  presidente / la la presidente, presidenta 
 presidente, presidenta
E) El médico / la la médico, médica 
 médico, médica F) El cliente / la cliente, clienta
clienta
G) El modista, modisto  / l a modista
El  modista, modisto H) El juez
El  juez / la
la juez,
 juez, jueza
jueza 
I) El políglota
El  políglota, políglot
polígloto o  / la  políglota 
la políglota J) El aprendiz / la aprendiz, aprendiza
aprendiza

2.
2.1.
1.2
2 La variación del género y el significado
signi ficado de los su
sustanti vos  
stantivos
CLASES DE SUSTANTIVO 
IGUO 
 AMBIGUO
 AMB HOMÓNIMO/POLISÉMICO 
Sea masculino o femenino,
el significado del El significado del sustantivo varía de acuerdo al género.
sustantivo no varía.
El/la maratón
El/la  maratón   El editorial firmado)  ≠  la editorial
editorial  (artículo no firmado)  editorial  (casa editora) 
editora) 
El/la interrogante
El/la  interrogante   El
El pendiente
 pendiente (arete) ≠ la pendiente
la  pendiente (inclinación)
El/la cochambre El  corte (de “cortar”) 
El corte ≠ la corte
la  corte (tribunal)
El/la mar 
El/la  mar   El cura (sacerdote)
(sacerdote)   ≠ la cura
la  cura (curación)
(curación)  
armazón  
El/la armazón El parte (escrito breve) ≠ la parte (porción de un todo)
El/la azúcar   El cólera (enfermedad)
(enfermedad)   ≠ la cólera (ira)
(ira)  
vodka  
El/la vodka El final (término de algo) final  (última competición)
≠ la final 

2.
2.1.
1.3
3 El género
género de
d e las siglas
sigl as y acróni mos  
acrónimos
El artículo concuerda con el género del primer sustantivo que forma parte de la sigla o el
acrónimo. Ejemplos:

El MTPE
El  MTPE (Ministerio de Trabajo y Promoción del Empleo ) 
El DNI (documento  nacional de identidad ) 
documento nacional
La ONG
Es (organización
una propiedad  node
gramatical gubernamental ) 2.2.
los sustantivos y Número  
otras categorías gramaticales que se
presenta en dos formas: singular y plural. El número plural se puede formar con un morfema
(-s o -es) o un determinante.
2.
2.2.
2.1
1 Reglas
Reglas ppara
ara la formación plu ral  
formaci ón d el plural
  Los plurales de régimen, espécimen  y carácter   son regímenes, especímenes  y
caracteres, respectivamente.
respectivamente.

Ejemplos  
Ejemplos
N.°  Contexto  
Contexto Regla  
Regla
Singular   Plural 
Plural 
avemaría avemaríass  
avemaría
Unidad formada por dos Siguen las
1. palabras escritas en una reglas generales parabién    
parabién
paramilitar  parabienes
parabien es 
es   
paramilitar 
es 
sola. de pluralización.
bocacalle bocacalless  
bocacalle

CEPRE-UNI HUMANIDADES 19
 

CICLO INTENSIVO ESCOLAR NACIONAL Material de Estudio Nº 1 

niño prodigio 
prodigio  niños prodigio
niños prodigio  
Unidad formada por dos
Solo se pluraliza hombre rana 
rana  hombress  rana
hombre  rana  
2. sustantivos escritos por
el primero. coche bomba 
bomba  cochess  bomba
coche  bomba  
separado
dormitorio 
ciudad dormitorio  ciudades
ciudad dormitorio  
es dormitorio
cortaúñas 
el cortaúñas   cortaúñas  
lo s  cortaúñas
La segunda palabra del
Se pluraliza con el abrelatas 
abrelatas  los abrelatas
abrelatas  
3. compuesto tiene forma
el artículo. el portaligas 
portaligas  los portaligas
portaligas  
plural o termina en -todo
todo..
el sabelotodo 
sabelotodo  lo s  sabelotodo
 sabelotodo  
2.
2.2.
2.2
2 Sustanti
Sustanti vos con
co n más de un plur al
Hay sustantivos que presentan más de una forma plural. Ejemplos: club clubss /club
/clubes
es;;
bistecss /bisté
bistec /bistéss ; ítems
ítems /lo
loss   ítem/ítemes
ítem/ítemes;; accésits
accésits /los
/los   accésit; fans
fans /fan
/fanes
es;; tests
tests /l os
os   test;
debutss /debú
debut /debúss ; coñacs
coñacs /coñás.
2.
2.2.4 Pluralización de palabras co mpuestas  
2.4
  Asimismo, si el segundo sustantiv
sustantivo
o de una unidad formada por dos sustantivos se
puede emplear también con valor de adjetivo, hay dos formas de pluralizarlo. Ejemplos:
— Avion
 Aviones
es espía
 espía / aviones
aviones espía
 espíass   —Disco
Discoss  pirata / discos
discos  pirata
 piratass  
—País
Países
es  satélite / país
países
es satélite
 satélitess   —Momento
Momentoss  clave / momentos
momento s  clave
 clavess  
—Empresa
Empresass  líder / empresas
empresas líder e es 
s  —Programa
Programass  piloto / programas
programas piloto
pilotoss
2.
2.2.
2.5
5 Otros casos para marcar el núm
número
ero
Ejemplos  
Ejemplos
N.°  
N.° Casos  
Casos Reglas  
Reglas
Singular   Plural  
Plural
Se pluraliza con el el DNI
el  DNI   lo s  DNI
 DNI  
1. Siglas  
Siglas artículo u otro esa AFP
esa  AFP   esas AFP
esas  AFP  
determinante. la ONG
la  ONG   las ONG
ONG  
 Ac ró ni
 Acró nimo
mo s Se someten a las reglas pyme ovni  
ovni pymess   ovni
pyme ovniss  
2. lexicalizados y generales de láser   foto  
foto láser es 
es   foto
fotoss  
palabrass acortadas
palabra acortadas   pluralización. uci  
uci bici  
bici uciss  
uci biciss  
bici
Usualmente, si la
Cnel. (coronel) Cneles.  
Cneles.
abreviatura se forma con
pról. (prólogo)
(prólogo)   próls.
próls.
las primeras letras, se
3.  Abrev
 Ab reviat
iatur
uras
as   Cía. (compañía) Cíass . 
Cía
pluraliza con -s
-s;; en otros
Ing. (ingeniero)
Ing. (ingeniero) Ingss .
Ing
casos, se usan las reglas
generales.

CEPRE-UNI HUMANIDADES 20
 

CICLO INTENSIVO ESCOLAR NACIONAL Material  de Estudio Nº 1 


Ejemplos
N.°  Regla  Contexto  
Singular   Plural  
Palabras terminadas en sofá bidé sofás
sofás bidés  
bidés
vocal átona y en -á,- é, cámara plató cámarass
cámara platóss
plató

-ó   comité capó  
capó comitéss
comité capóss  
capó
Palabras del registro
coloquial o las cañí cañís
cañís
popurrí popurrís
popurrís
castellanizadas gilí gilíss
gilí
vermú vermúss  
vermú
procedentes de otras menú menúss
menú
Se tutú tutúss  
tutú
lenguas terminadas en -i -i   champú champúss  
champú
añade  –

1. -s
-s     ú 
oPalabras castellanizadas espray 
espray  espráis  
espráis
procedentes de lenguas  jersey  jerséiss
 jerséi
extranjeras que terminan (yérsey
yérsey)) (yersei
yerseiss )
en -y
-y   cambian a - a -ii cuando gay gaiss
gai
se pluralizan. yóquey  
yóquey yoqueiss  
yoquei
Palabras terminadas en
icebergs  
icebergs
doble consonante o en iceberg tic
ticss  
récordss  
récord
consonantes diferentes de récord tic
tic   ziguratss
zigurat
mamutss  
mamut
-d, -j, -z, -l, -r, -n, -s, mamut   zigurat
mamut
cómicss  
cómic
-x, -y, -ch 
-ch  cómic  
cómic
ceutí  
ceutí ceutíes  
ceutíes
Gentilicios terminados en -
hindú hindúes
hindú es
í o -ú
-ú  
bagdadí  
bagdadí bagdadíes
bagdadí es  

Originariamente
castellanas terminadas en carey    
carey
convoy
convoy  careyes
carey
convoyes 
convoyes   
es 
 –y  mamey  
mamey mameyes
mamey es  
2. Se Monosílabos o palabras
añade tos 
tos  toses  
toses
agudas terminadas en
-es  
-es fax  
fax faxes
fax es  
-s   o  -x
-x,, excepto relax
relax   (los
anís  
anís anises
anis es  
relax)) 
relax
Palabras terminadas en -j, deidad cónsul deidades
deidades cónsules
cónsules
-z, -
-z, -l,l, -d, -r, -n que no sean reloj máster relojes
relojes máster es
es
esdrújulas cáliz canon cálices
cálic es cánones
cánon es  
maniquí maniquís /es
maniquís
Se gurú gurús /es
gurús
Palabras terminadas en -í 
-í  rubí rubíss /es
rubí
3. añade tabú tabúss /es
tabú
o  ú   ají ajís
ajís /es
-s
-s o
 o -es
-es   iglú  
iglú iglúss /es 
iglú es  
 –

colibrí  
colibrí colibríss /es 
colibrí es 
el
Palabras polisilábicas ómnibus 
ómnibus  lo s  ómnibus
 ómnibus  
el tórax los tórax
terminadas en -s o  -x
-x que
 que la dosis
el clímax las dosis
las  dosis
el lo s  clímax
no son agudas lo s  fórceps
fórceps
Se Palabras terminadas en -j, el dóberman
dóberman   los dóberman
dóberman  
antepone -z, 
-z,  -l, -d, -r, -n que son el asíndeton los asíndeton
4.
artículo  
artículo
esdrújulas el polisíndeton lo s  polisíndeton
 polisíndeton  
el álter ego los álter ego
el lapsus línguae los lapsus línguae
Las locuciones latinas
el pluralia tántum los pluralia tántum
el currículum vítae los currículum vítae

EVALUACIÓN  

CEPRE-UNI HUMANIDADES 21
 

CICLO INTENSIVO ESCOLAR NACIONAL Material de Estudio Nº 1 

1. Marque el enunciado que evidencia adecuada A) Aquel estilista obsequió varios champúes
formación del género de los sustantivos. importados.
I. La polígloto entregó las traducciones a la B) Crearán ciudades satélite para el sector A en
Reniec. Lima.
II. La médica realizó una operación en la uci. C) La cantante Thalía presentó popurrís de
III. El choque le ocasionó una coma a la testigo. baladas.
IV. La Concytec otorgará una beca a la D) Se actualizará el modelo de los currículum
autodidacta. vítae.
A) IIII y IV B) Solo II C) I, II y III E) Remataron seis ómnibus de la empresa Sol
D) II y IV E) II y III de Oro.
2. ¿Qué oración(es) presenta(n) uso incorrecto 7. Identifique el enunciado que evidencia uso
número? 
del sustantivo en cuanto a su género o número?  adecuado del género de la sigla o acrónimo.
I. Las pyme contrataron al modista para A) La INEI no publicó el número de habitantes en
confeccionar algunos yerseis. Lima.
II. La Sunat le va a imponer este año veinte UIT´s B) Me prepararé en la Cepre-UNI el próximo
por vender discos pirata. verano, Jesús.
III. La sustentación de la tesis de pregrado C) Antonio solicitó ante la Reniec su partida de
presentó varios momentos claves. matrimonio.
IV. Los mexicanos prefieren los mameyes; los D) El domingo, el OCAD entregó el resultado de
iraquíes, la sopa de picadillo. la prueba.
A) Solo I B) Solo II C) III y IV E) La ONG programó visitas guidas por el Día
D) Solo IV E) I y II del Niño.

3. Indique la alternativa que presenta 8. Indique la alternativa en la cual hay uso


incorrectamente el género femenino del incorrecto del género o número del sustantivo.
sustantivo. A) La juez Díaz le dio varios ultimátums al
A) La jirafa hembra B) La fiscala corrupta acusado.
C) La elefante preñada B) Los magísters en Biología brindaron con
D) Una palmera hembra E) La azúcar rubia vermúes.
C) Varios ítemes de la Biblia los leyó la cónyuge.
4. Elija la alternativa que evidencie sustantivos D) Los gurús recurrieron al estudio de la ruda
compuestos adecuadamente pluraliza
pluralizados.
dos. macho.
A) Les colocaron baipases a veinte niños genios. E) El interrogante de los sabelotodo fue
B) Desactivaron coches bomba en dos inesperado.
bocacalles.
C) En las horas puntas, se tomaban sus 9. ¿Qué latinismo está mal pluralizado?
tentempiés. A) Los lapsus línguae B) Réquiemes
D) Las piloto.
programa empresas líder presentaron dos C) Los álter ego D) Cuórums E) Hábitats
E) Dañaron sus faldas pantalones con los 10. Marque la alternativa que evidencia uso
abrelatas. correcto de grafías.
A) La voleybolista absorbió mucha harina
5. Identif
Identifique
ique la serie en la cual no hay error de accidentalmente.
pluralización. B) La exhumación de aquel hermitaño fue
A)  Bisturíes, manís, vermúes
A) sorpresiva.
B) Los autobús, lo loss TLC´s, los láser C) Almorzé con Lucecita en ese garaje de la
C) Pines, fanes, los vals esquina.
D)
D)  Los relax, los currículum, pírsines D) Permaneció meditabundo, pues quizo hilar
E) Tres ómnibus, córneres, ítemes su idea.
E) Le practicaron una cirugía de emergencia al
6. Indique la alternativa que presenta el vicerrector.
sustantivo subrayado mal pluralizado. Historia

CEPRE-UNI HUMANIDADES 22
 

CICLO INTENSIVO ESCOLAR NACIONAL Material de Estudio Nº 1 

Literatura 
TEMA I: GÉNER
GÉNEROS
OS Y SUBGÉNEROS LITERARIO
LITERARIOS. S. FIGURAS LITERARIAS:
DEFINICIÓN
DEFINICIÓN Y CLASES. LITERATURA GRIEGA. CARACTER
CARACTERÍSTICAS,
ÍSTICAS,
REPRESENTANTES. ÉPICA: HOMERO (ILÍADA
(ILÍADA Y
 Y ODISEA
ODISEA).
). TRAGEDIA: SÓFOCLES
SÓFOCLES
(EDIPO REY)
REY) 

1. GÉNEROS LITERARIOS
Se denominan géneros literarios a las distintas clases en las que se pueden agrupar las obras
literarias por sus características comunes: épico, lírico, dramático, narrativo y expositivo.

1.1. Género épico


El género épico nació en la Edad Antigua como una composición en verso. Narra objetivamente
hechos gloriosos y hazañas de héroes con cualidades sobrehumanas.

Los principales subgéneros o especies del género épico son los siguientes:
Epopeya. Es una extensa composición en verso que Las epopeyas fueron difundidas por
relata los actos heroicos y gloriosos de personajes los rapsodas en la Edad Antigua; y
mitológicos de un periodo histórico remoto. Por los cantares de gesta, por los juglares
ejemplo: la Ilíada y la Odisea, de Homero; la Eneida, de en la Edad Media. 
Media.  
Virgilio.
Cantar de gesta. Es una composición en verso de origen popular, nacional y anónimo,
referida a las hazañas de un héroe medieval. Por ejemplo: el Poema de Mío Cid (España), el
Cantar de los Nibelungos (Alemania), el Cantar de Roldán (Francia).

Poema épico. Es una composición escrita en verso que surgió a fines de la Edad Media. Se
caracteriza por el empleo de un lenguaje culto y por sus constantes referencias cristianas. Por
ejemplo: la Divina Comedia, de Dante Alighieri; Jerusalén Libertada, de Torcuato Tasso.
1.2. Género lírico
El género lírico se caracteriza por su subjetividad, ya que expresa (tradicionalmente en verso
o eventualmente en prosa) el mundo interior del poeta ( sentimientos, emociones,
pensamientos).
Los principales subgéneros o especies del género lírico son los siguientes:
Oda. Poema que elogia o alaba, con un lenguaje entusiasta, las cualidades de algún ser u
objeto. Por ejemplo: Odas elementales, de Pablo Neruda; «A la vida retirada», de Fray Luis
de León. 
Elegía. Composición que expresa el lamento del poeta ante la muerte de un ser querido. Por
ejemplo: Coplas por la muerte de su padre, de Jorge Manrique; Llanto por Ignacio Sánchez
Mejías, de Federico García Lorca.
Égloga. También llamada bucólica, es un poema referido al campo, los pastores y sus
sentimientos amorosos. Por ejemplo: Églogas, de Garcilaso de la Vega; Bucólicas, de Virgilio.
Epigrama. Composición poética de extensión breve que se caracteriza por su tono irónico y
sus finales ingeniosos. Por ejemplo: Epigramas, de Ernesto Cardenal.

CEPRE-UNI HUMANIDADES 23
 

CICLO INTENSIVO ESCOLAR NACIONAL Material de Estudio Nº 1 

Sátira. Poema que critica de forma ácida y mordaz los vicios, defectos y costumbres de un
personaje o la sociedad, con un fin moralizante. Por ejemplo: «A una nariz», de Francisco
Quevedo; «El carnaval de Lima», de Felipe Pardo y Aliaga.
1.3. Género dramático
El género dramático abarca las obras destinadas a la representación escénica en espacios
diseñados para ello (teatros). Estas se componen de diálogos y acciones de los personajes, y
acotaciones del autor (aclaraciones
(aclaraciones sobre los detalles o el ambiente de la escena).
Los principales subgéneros o especies del género dramático son los siguientes:
Tragedia.  Obra dramática centrada en hechos funestos y En el género dramático, el
protagonizada por personajes elevados o ilustres (reyes, autor crea un texto, ya sea en
príncipes, aristócratas). Su final se relaciona con la desdicha verso o en prosa, para ser
o la muerte. Por ejemplo: Edipo rey, de Sófocles; Bodas de interpretado en escena. 
escena.  
sangre, de García Lorca.
Comedia.  Obra dramática centrada en hechos burlescos y protagonizada por personajes
sencillos de la vida cotidiana (campesinos, labradores, comerciant
comerciantes).
es). Su desenlace es feliz.
Por ejemplo: Ña Catita, de Manuel Ascencio Segura.
Drama.  Obra que mezcla los elementos de la tragedia y la comedia: hechos funestos y
burlescos;
sueño personajes
, de Calderón deelevados
la Barca;yFuenteovejuna,
comunes; final entre trágico
de Lope y feliz.Ollantay
de Vega; Por ejemplo: La vida es
 (Anónimo).
1.4. Género narrativo
El género narrativo, en sus orígenes, estuvo vinculado al género épico. Se diferencia de este por el
empleo de la prosa y por su mayor variedad temática. Abarca las obras que narran acontecimientos
reales o ficticios (novelas históricas, policiales, humorísticas, de terror, etc.).

Los dos principales subgéneros o especies del género narrativo son los siguientes:
Cuento.  Narración breve protagonizada por uno o pocos personajes. Se caracteriza por
desarrollar un solo tema. Por ejemplo: «Warmakuyay», de José María Arguedas.
Novela.  Narración extensa en la que intervienen varios personajes. Se caracteriza por
desarrollar dos o más temas. Por ejemplo: Los perros hambrientos, de Ciro Alegría.
1.
1.5.
5. G
Género
énero exposit ivo
El género expositivo tiene por finalidad
f inalidad incentiv
incentivar
ar la capacidad crítica y reflexiva del lector a través del
desarrollo de un tema específico. Se manifiesta en prosa.
pr osa.
El principal subgénero o especie del género expositivo es el siguiente:
Ensayo. Obra que desarrolla un tema alrededor de una hipótesis sustentada a través de
argumentos. No posee la rigurosidad metódica de un texto científico; es, más bien, de carácter
subjetivo, puesto que evidencia la interpretación
interpretación personal de la realidad que hace un autor.
2. FIGURAS LITERARIAS
Las figuras literarias o retóricas son recursos de los que se vale un autor para dar mayor expresividad
y originalidad a sus composiciones orales y escritas.

2.1.. Metáfora
2.1 Metáfor a

CEPRE-UNI HUMANIDADES 24
 

CICLO INTENSIVO ESCOLAR NACIONAL Material de Estudio Nº 1 

Figura que opera con relaciones de semejanza, mediante la cual una palabra o frase es comparada o
sustituida por otra con la que mantiene un rasgo en común.
Tipos de metáfora:
A) «a» es «b». Consiste en la comparación directa entre dos palabras o términos.
Ejemplos:
   «Nuestras vidas son los ríos / que van a dar a la mar / que es el morir» (Jorge Manrique).
   «Era la sed y el hambre, y tú fuiste
fuiste la fruta» (Pablo Neruda).

B) «a» sustituye a «b». En este caso, una idea reemplaza


r eemplaza a otra.
Ejemplos:
  «coged de vuestra alegre primavera

el dulce fruto, antes que el tiempo airado


cubra de nieve la hermosa cumbre» (Garcilaso de la Vega).

2.2. Símil
2.2.
Figura que consiste en la comparación explícita entre dos términos. A diferencia de la
metáfora, emplea, necesariamente,
necesariamente, un elemento de comparación (como, cual, así como).

  «Para que
 que tú me oigas mis palabras / se adelgazan a veces / como  las huellas de las
gaviotas en las playas» (Pablo Neruda).

2.3. Hipérbole
Figura que radica en la exageración de los atributos, defectos o acciones de un ser u objeto;
ya sea para el elogio o la burla.
Ejemplos:
  «Oh más dura
 dura que mármol a mis ququejas
ejas / y al encendido fuego en que me qquemo
uemo /
más helada que nieve, Galatea» (Garcilaso de la Vega).

2.4. Anáfor a
2.4.
Figura que consiste en la repetición de una o más palabras al inicio de los versos.
Ejemplo:
  «Para mi corazón basta tu pec
 pecho,
ho, / para tu libertad bas
bastan
tan mis alas» (P
(Pablo
ablo Neruda).

2.5. Hipérbaton
Figura que consiste en la alteración del orden convencional de las palabras en una oración.
Ejemplo:
  «Del salón en el áángulo
 ngulo oscuro» (Gustavo Adolfo Bécquer).

3. LITERATURA GRIEGA
La literatura griega es el conjunto de composiciones orales y escritas en griego antiguo que se
desarrolló desde el s. IX a. C. hasta el s. II d. C. Es considerada clásica por servir de paradigma o
modelo a movimientos literarios posteriores.

CEPRE-UNI HUMANIDADES 25
 

CICLO INTENSIVO ESCOLAR NACIONAL Material de Estudio Nº 1 

3.1. Características
   Antt ro po
 An pocén trii ca.  El hombre es el centro del universo.
céntr
Las obras muestran un profundo conocimiento de la naturaleza
humana (deseos, temores, pasiones, conflictos, etc.).
  Racional.  Las emociones son reguladas por la razón. La
literatura se adecua a determinadas normas.

  Fatalista.  Los hombres y los dioses están subordinados
inevitablemente al Hado (Destino).
   Mitológica.  Recrea mitos pertenecientes a un pasado
remoto, en la Edad Heroica. Las obras relatan el mundo de
los dioses del Olimpo, los semidioses y los héroes
legendarios.
   Equilibrada.  Se busca la simetría, la armonía y la proporción
entre la forma y el contenido de las obras. Se emplea,
además, un lenguaje mesurado y preciso.

3.2.. Repr
3.2 Representant
esentantes
es
ÉPOCA GÉNERO   AUTOR   OBRAS 
Homero Ilíada,, Odisea
Ilíada Odisea  
 ARCAIC A O
 ARCAICA Épico
JÓNICA Hesíodo
Píndaro Teogonía, Los trabajos
Epinicios 
Epinicios   y los días 
días  
(IX - VI a. C.) Lírico
Safo Odas 
Odas 
Esquilo Orestíada,, Prometeo
Orestíada
CLÁSICA encadenado  
encadenado
O Edipo rey,
rey, Edipo en Colono,
Dramático Sófocles
 ÁTICA  Antígona, Electra 
Electra 
(V - IV a. C.) 
C.)  Eurípides bacantes, Medea
Las bacantes, Medea  
Aristófanes Las nubes,
nubes, Las avispas 
avispas 

4. LA ÉPICA EN GRECIA
La épica floreció en la época arcaica (s. IX - VI a. C.). Se caracterizó por difundirse de forma oral y
por incorporar mitos griegos de tiempos remotos. Su máximo representante fue Homero.

4.1.. Homero (s. VIII a. C.) 


4.1
Fue un notable poeta griego de la Antigüedad. Hasta hoy se
desconoce con exactitud el tiempo y lugar de su nacimiento;
asimismo se pone en cuestión su autoría de las dos principales
epopeyas griegas: la Ilíada y la Odisea. Este debate es conocido
como la “cuestión homérica”.
 A. Ilíada
Ilíada  
   Pertenece al género épico y la especie epopeya.
   Consta de 24 cant
cantos
os o rapsodias (15 693 versos).
   Ilíada proviene del vocablo griego ilión’ (Troya).

  El tema central es la cólera ddee Aquiles


Aquiles..

CEPRE-UNI HUMANIDADES 26
 

CICLO INTENSIVO ESCOLAR NACIONAL Material de Estudio Nº 1 

   Presenta un amplio uso de epítetos. Por ejemplo: Aquiles, el de los pies ligeros; Agamenón,
el de los ojos de perro; Héctor, el de tremolante casco; Briseida, la de hermosas mejillas;
Zeus, el que amontona las nubes; Helena, la de los níveos brazos. 

 Argu
 Ar gumen
mento
to de lla
a Ilíada
Ilíada::

La historia comienza cuando una terrible


peste enviada por Apolo asola el
campamento  aqueo. Tras una debida
consulta al adivino Calcas, se determina
que la causa es la afrenta cometida contra
Crises, sacerdote de Apolo y padre de
Criseida, quien había sido tomada como
botín de guerra por el rey Agamenón. Tras
un largo debate, el rey acepta devolver a
Criseida, pero decide subsanar su pérdida
tomando a la concubina  preferida de
Aquiles, la bella Briseida. El héroe griego,
preso de la ira, decide no volver a la lucha.
Su ausencia permite el avance triunfal de los troyanos, apoyados por Hera, Afrodita, Ares y
Apolo. Ante
enfrentar tal situación,
al enemigo. Patroclo,
Así, gana gran
terreno amigo
hasta llegardea las
Aquiles, pidedeaTroya;
murallas este sus armas para
sin embargo, allí
el príncipe troyano Héctor le da muerte.

Furioso, Aquiles regresa a la lucha para vengar la muerte de su amigo. Es así que los aqueos,
apoyados por Hera, Atenea y Poseidón, avanzan causando grandes estragos en las filas
troyanas. Aquiles logra enfrentar a Héctor y lo mata, luego lo ata a su carruaje y lo arrastra
alrededor de los muros de Troya. El viejo rey Príamo, padre de Héctor, se interna en el
campamento aqueo y le ruega a Aquiles que le devuelva el cuerpo de su hijo. Él, conmovido,
accede. La obra culmina con los funerales de Héctor, que interrumpen la guerra por once
días.

B. Odisea
Odisea   El tema principal de la Odisea
Odisea  
  Pertenece al género épico y a la espec
especie
ie epo
epopeya.
peya. es el retorno de Ulises. Otro

  Consta de 24 cant
cantos
os o rapsodias (12 110 versos). tema: el amor a la familia y a
la patria.
  Con posteridad, fue dividida en tres partes: “Telemaquia”,
“Aventuras” y “Venganza”. 
  Presenta uso de epítetos. Por ejemplo: Odiseo, fecundo en ardides, padre de Telémaco;
Atenea, la de los ojos de lechuza. 
 Argu
 Ar gumen
mento
to de lla
a Odisea
Odisea::

CEPRE-UNI HUMANIDADES 27
 

CICLO INTENSIVO ESCOLAR NACIONAL Material de Estudio Nº 1 

El hogar de Ulises (Odiseo) se halla


invadido por hombres que pretenden la
mano de su mujer, Penélope, para así
apoderarse de Ítaca. Telémaco, hijo de
Penélope y Ulises, desea expulsarlos y
para ello cuenta con la ayuda de Atenea,
quien le aconseja viajar en busca de
noticias de suno
su progenitor padre. Así, seenentera
ha muerto de que
la guerra de
Troya. Ulises, mientras tanto, se encuentra
preso en la isla de Ogigia, a manos de la
ninfa Calipso. Pese a estar ahí ya siete años, nunca ha dejado de pensar en su familia, a la
que no ve desde hace dos décadas. Gracias a los dioses, por fin emprende el viaje de retorno
a su patria, pero su embarcación es hundida por Poseidón, enfadado porque el griego cegó
a su hijo Polifemo. Ulises naufraga en la isla de los feacios, cuyo rey es Alcínoo. La hija de
este, Nausícaa, atiende al héroe y lo lleva al palacio. Allí, Ulises cuenta a sus anfitriones su
paso por la tierra de los lotófagos, su llegada a la isla de los cíclopes (donde se encontraba
Polifemo), y la ayuda que recibe del dios Eolo. Narra, además, cómo había llegado a la isla
de la hechicera Circe, su visita al Hades, su huida de las sirenas, de Escila y Caribdis, y su
llegada a la isla de Calipso.

Al oír su historia, Alcínoo decide ayudarlo. Es así que, ya en Ítaca, tras contactarse con su fiel
sirviente Eumeo y su hijo Telémaco, Ulises entra al palacio aparentando ser un mendigo.
Entre tanto, Penélope anuncia que se casará con quien logre pasar una flecha a través de
doce aros en fila. La prueba debe realizarse con el arco que le perteneció a su esposo.
Ninguno de los presentes logra la hazaña, excepto Ulises que pide una oportunidad.
Realizada la proeza, el héroe se descubre y da muerte a los pretendientes. Luego, se dirige
hacia Penélope, quien duda de la identidad de su esposo. El héroe la convence refiriéndole
cómo había construido su lecho nupcial, algo que solo ambos conocía
conocían.n. Reunida la familia se
restablece la paz en Ítaca.
5. LA TRAGEDIA EN GRECIA
El género dramático alcanzó gran desarrollo en Atenas durante el Siglo de Pericles (s. V a. C.).
Destaca principalmente la tragedia, cuyos principales exponentes fueron Esquilo y Sófocles.

5.1. Sófocles
Máximo exponente de la tragedia en el Siglo de Pericles (s. V a. C.).
Nació en Colona. Incorporó al tercer actor en escena (lo cual brindó
mayor agilidad a los diálogos) y desarrolló la psicología de sus
personajes. Obras: Edipo en Colono, Antígona, Electra.

 A. Edipo rey


   Pertenece al género dramá
dramático
tico y a la espec
especie
ie tragedia
tragedia..
   Está escrita en verso y no tiene actos. Sus personajes son
humanos.
   Tiene como ant
antecedente
ecedente el mito de Edipo, que cuenta su ascensión al poder. 
   El tema central es el destino del hombre det
determinado
erminado por los dioses. 

 Argu
 Ar gumen
mento
to de Edipo rey:
rey :

CEPRE-UNI HUMANIDADES 28
 

CICLO INTENSIVO ESCOLAR NACIONAL Material de Estudio Nº 1 

Una terrible peste asola al pueblo de Tebas. Edipo, el rey,


envió a su cuñado, Creonte, a consultar el oráculo de Delfos
para saber cómo actuar. Cuando este regresó, hizo pública
la respuesta: la peste solo cesaría cuando se castigara al
asesino de Layo, el rey anterior. Creonte pide ayuda al viejo
adivino Tiresias. Al principio, el anciano se negó, pero ante
las desesperadas ofensas de Edipo, terminó por hacerle
terribles presagios.
habían coludido paraEdipo pensó que
destronarlo, peroCreonte
Yocasta, y Tiresias
la reina, se
lo
tranquilizó diciéndole que los oráculos no siempre se
cumplían, pues Layo estaba destinado a morir a manos de
su hijo y, sin embargo, terminó siendo asesinado en un cruce
de tres caminos. A Edipo lo asaltó un recuerdo: antes de
llegar a Tebas, él tuvo un altercado similar. Para esclarecer
las cosas, mandó a llamar al único testigo de la muerte de
Layo: un viejo pastor.
Entretanto, llegó al pueblo un mensajero de Corinto
anunciando la muerte del rey Pólibo, supuesto padre de
Edipo. En esas tierras, esperan al protagonist
protagonistaa para que ocupe el trono. Sin embargo, algo le
impide aceptar: recuerda que, en su juventud, el oráculo predijo que él mataría a su padre y
se casaría con su madre, y teme que, yendo a Corinto, se cumpla su destino. El mensajero,
para calmarlo, le revela que es adoptado: él mismo lo recibió de manos de un pastor de Layo
cuando solo era un bebé. En ese preciso momento, llega el viejo pastor, que, además de
testigo clave, es también quien salvó la vida de Edipo, pues, en vez de matarlo, como quería
Layo, decidió entregarlo al mensajero corintio.
Yocasta se da cuenta de lo que ha ocurrido y, desesperada, huye hacia el palacio. Bajo
amenazas, el pastor revela la terrible verdad: Edipo es hijo de Yocasta y Layo; ha asesinado
a su padre y se ha casado con su madre. El rey, desconsolado, también se refugia en el
palacio. Un criado sale a escena y relata que Yocasta se ha ahorcado y que Edipo se ha
arrancado los ojos. Finalmente, el protagonista lamenta su trágico destino y pide a Creonte
que lo destierre.

  Evaluación
Eva luación 1
1. ¿A qué especie lírica pertenece el siguiente D) Edipo rey, Ña Catita, Orestíada
poema?: “Por no sé qué capricho / Filis juró E) “A una nariz”, Ollantay, Ña Catita 
Catita 
olvidarme. / Pasados pocos días, / Hizo otra vez
las paces. / Pero fue tan gustoso / Aquel feliz 3. Relacione correctamente: figura literaria-
instante, / Que le digo mil veces: 'Filis, vuelve a verso.
olvidarme, / Con tal que a pocos días / Vuelvas I. Anáfora II. Metáfora
paces'” (Mariano Melgar). 
a hacer las paces' III. Hipérbaton IV. Símil
A) Sátira B) Epigrama C) Oda a) Sus muslos se me escapaban / como peces
D) Elegía E) Bucólica sorprendidos.
b) Mi cuarto es una / manzana, / con sus / libros,
2. Designe la alternativa que incluya solo obras / con su cáscara.
del género dramático. c) Ves aquí un prado lleno de verdura, / ves aquí
A) Bucólicas, Églogas, Epigramas una espesura.
B) Ilíada, Ensayos, “Warma kuyay”  d) Volverán del amor en tus oídos / las palabras
C) Divina comedia, Odisea, Eneida ardientes a sonar.

CEPRE-UNI HUMANIDADES 29
 

CICLO INTENSIVO ESCOLAR NACIONAL Material de Estudio Nº 1 

A) I-c, II-b, III-a, IV-d B) I-a, II-b, III-c, IV-d


C) I-c, II-b, III-d, IV-a D) I-d, II-a, III-b, IV-c 5. ____________ es el máximo exponente de la
E) I-c, II-a, III-b, IV-d tragedia del denominado Siglo de Pericles e
incorporó al tercer actor en escena.
4. En la epopeya Ilíada,
Ilíada, de
 de Homero, los aqueos A) Homero B) Eurípides C) Esquilo
sufrieron por una peste que fue enviada por el D) Sófocles E) Aristófanes
dios olímpico ___________, quien los castigó
por el mal comportamiento de Agamenón. 1B 2D 3C 4C 5D
A) Zeus B) Hera C) Apolo
D) Poseidón Atenea 
E) Atenea 

TEMA II: LITERATURA MEDIEVAL.CARACTERÍSTICAS, REPRESENTANTES. DANTE


 ALIGHIERI
 AL IGHIERI (DIVINA COMEDIA).
COMEDIA ). RENACIMIENTO Y BARROCO. WILLIAM
SHAKESPEARE (HAMLET
(HAMLET))

1. LITERATURA MEDIEVAL
MEDIEVAL
Se denomina Literatura medieval al conjunto de expresiones literarias que se desarrolló en
Europa durante la Edad Media o Medioevo (siglo V d. C a XV d. C.).

1.1. Características

  Carácter teocéntrico.
del universo.  Dios, el
De este modo, enarte
el sentido cristiano
medieval plasmadellas
término,
l as se presenta
doctrinas como
de la Iglesia el centro
católica.
  Empleo de lenguas romances. Si bien el latín es el idioma oficial de la Iglesia católica,
las principales obras literarias del Medioevo se componen mediante el empleo de lenguas
romances (como el español, el italiano o el francés) que derivan del latín vulgar. 
  Espíritu nacionalista.  Surge el cantar de gesta, especie que manifiesta un profundo
carácter nacional e histórico y representa la fundación de las naciones europeas. 

1.2.. Repr
1.2 Representant
esentantes
es

PAÍS AUTOR OBRA ESPECIE


ESPECIE
Italia Dante Alighieri Divina comedia Poema épico
España Anónimo Poema de Mio Cid Cantar de gesta
Francia Anónimo Cantar de Roldán Cantar de gesta
Alemania Anónimo Cantar de los Nibelungos Cantar de gesta
1.3. Dante Alighieri (1265-1321) 
Nació en Florencia, Italia. Perteneció a una familia de nobles. Se
sabe muy poco acerca de su educación, aunque sus libros reflejan
una vasta erudición que comprendía casi todo el conocimiento de su
época. Ocupó diversos cargos políticos. Fue desterrado en 1302.
Murió en Rávena.
Obras.  Divina Comedia,  Vida nueva, De vulgar elocuencia, La
monarquía.
1.4. Divina comedia 
comedia 
Obra cumbre de Dante Alighieri. Fue escrita entre 1307 y 1319. Un detalle que caracteriza la
obra es que
propósito está escrita
principal enespiritualmente
reformar toscano y no en latín
a los (lengua Propone
hombres. culta de la
la posible
época).
pos Tiene como
ible reconciliación

CEPRE-UNI HUMANIDADES 30
 

CICLO INTENSIVO ESCOLAR NACIONAL Material de Estudio Nº 1 

con Dios, siempre y cuando se respeten los preceptos cristianos. Su título original fue
Comedia. Giovanni Boccaccio fue quien la denominó “divina”, aludiendo a su perfección
formal y a la naturaleza de su contenido.
Características:
  Pertenece al ggénero
énero épico: es un poema ép
épico
ico de eviden
evidente
te carácte
carácterr religioso.
  Está narrada en primera persona (Da
(Dante
nte es el protag
protagonista).
onista).

  Se divide
cantos, enuno
más tresintroductorio
partes (Infierno
(Infierno,, Purgatorio
al Infierno y Paraí
Paraíso),
(100 cantos enso), cada una compuesta por 33
total).
  El número tres tie
tiene
ne una co
connotación
nnotación religio
religiosa:
sa: representa a la Santí
Santísima
sima Trinidad.
  Se emplea el terceto (estrofa de ttresres versos) y el verso endecasíla
endecasílabobo (de once sílabas
sílabas).
).
Está compuesto por 14, 233 versos.

 Argu
 Ar gumen
mento
to de lla
a Divina comedia
Infierno. El protagonista, Dante, se halla extraviado en una selva
oscura (símbolo de la vida en el pecado). Al avanzar, le impiden el
paso tres fieras: una loba (avaricia), un león (soberbia) y una
pantera (lujuria). De pronto aparece el poeta Virgilio (razón), quien
ha sido enviado por Beatriz (fe y gracia). Este lo lleva por un
camino distinto y, posteriormente, lo guía a través de nueve
círculos, donde se encuentran padeciendo terribles penas las
almas de los condenados. Dante observa a una serie de
personajes bíblicos, históricos y mitológicos. En el noveno círculo
se encuentran aquellos que, según el autor, cometieron el peor de
los pecados: la traición. Aquí se halla Lucifer.
Purgatorio. Por medio de una barca, Dante y Virgilio llegan a una playa (Ante purgatorio).
Luego inician su recorrido por siete cornisas ascendentes
ascendentes.. El purgatorio es el espacio donde
las almas aún viven bajo el peso de los sufrimientos, sin embargo, abrazan la esperanza de
que pronto alcanzarán la felicidad plena. Se encuentran en un escenario de contrición y
purificación. Al final del Purgatorio, Virgilio se despide de Dante, puesto que él no puede
ingresar al Paraíso.
Paraíso. Dante tiene como guía a Beatriz. Este lugar está compuesto por nueve cielos
móviles,
encuentranconcebidos según
los espíritus la división
elegidos tolemaica,
por sus accionesy un cielo inmóvil
terrenales. En eloEmpíreo,
Empíreo;los
en espíritus
ellos se
bienaventurados rodean la gloria de Dios. Finalmente, Dante percibe la inexpresable
presencia de Dios.
  Tema principal. En un plano literal, es el viaje de Dante por el Infierno, el Purgatorio y el
Paraíso. En un plano alegórico, es la visión sobre los excesos del hombre pecador (Dante)
y su salvación vía la razón (Virgilio) y la fe (Beatriz). 
  Temas secundarios. Las pugnas políticas, la divinización del amor, la maldad de ciertos
religiosos.

2. RENACIMIENTO
El Renacimiento fue un movimiento cultural y artístico que se originó en Italia, en el siglo XV, y se
desarrolló por toda Europa durante el siglo XVI.

2.1. Características

CEPRE-UNI HUMANIDADES 31
 

CICLO INTENSIVO ESCOLAR NACIONAL Material de Estudio Nº 1 

  Humanista. Surge la figura del humanista, quien respetó, rev revaloró


aloró y emprendió el estudio
minucioso de la Antigüedad clásica.
   An
 Antt ro po
pocén
céntr co.. El Renacimiento hizo del hombre, en su ser y destino mundanos, el gran
trii co
tema de su arte y literatura, de sus reflexiones filosóficas y de su curiosidad científica.
  Racionalista. Frente al misticismo de la Edad Media, el Renacimiento desarrolló el culto a
la razón, manifiesto en su espíritu científico y eminentemente crítico. 
  Clasicista.   El Renacimiento significó un regreso a los ideales clásicos grecolatinos
(equilibrio, simetría, armonía).
2.2.. Repr
2.2 Representant
esentantes
es

PAÍS AUTOR OBRA ESPEC


ESPECIE
IE
Francia Michel de Montaigne Ensayos Ensayo (inició la especie)
Nicolás Maquiavelo El príncipe Ensayo
Italia Ludovico Ariosto Orlando furioso Poema épico
Torcuato Tasso Jerusalén libertada Poema épico
España Garcilas
Garcilasoo de la Vega Églogas Égloga
Holanda Erasmo de Rotterdam Elogio de la locura Ensayo

3. BARROCO
El Barroco fue un movimiento cultural y artístico que surgió como una reacción frente al
Renacimiento. Se desarrolló, principalmente,
principalmente, durante el siglo XVII.

3.1. Características
  Pesimista.  El artista del Barroco afronta una grave crisis espiritual, producto del
desmoronamiento de la doctrina cristiana, que había ordenado el mundo medieval. El
espíritu pesimista se evidencia en algunos temas literarios como la fugacidad de las cosas
o la vida vista como lucha, sueño o mentira.
  Originalidad. El arte ofrece la posibilidad de inmortalizarse y de afirmar la individualidad.
(Shakespeare emplea la original técnica del teatro dentro del teatro).
  Estilo intrincado.  Estilo donde abunda el empleo de figuras literarias y recursos
innovadores (complejidad formal).
  Ruptura del equilibrio. Consistió en el rechazo de la simetría y la armonía renacentistas,
elementos propiamente clásicos.

3.2.. Repr
3.2 Representant
esentantes
es
PAÍS AUTOR OBRA GÉNERO
GÉNERO
Inglaterraa
Inglaterr William Shakespeare Hamlet Dramático
Luis de Góngora y Argote Soledades Lírico
España
Miguel de Cervantes El Quijote 
Quijote  Narrativo

3.3. William Shakespeare (1564-1616)


Es uno de los dramaturgos más importantes de la literatura universal
y el más prestigioso de la literatura en lengua inglesa. Nació en
Stratford-upon-Avon. En su juventud viajó a Londres donde se
desarrolló como actor, director y empresario de una compañía
teatral. Falleció en 1616.
Obras:  

CEPRE-UNI HUMANIDADES 32
 

CICLO INTENSIVO ESCOLAR NACIONAL Material de Estudio Nº 1 

GÉNERO OBRAS
o
Tragedias Romeo y Julieta,
Julieta, Hamlet
Hamlet,, Otelo
Otelo,, Macbeth
Macbeth,, El rey Lear , Antonio y
ci
t
Cleopatra..
Cleopatra
á Comedias Sueño de una noche de verano,
verano, El mercader de Venecia.
Venecia.
m
a
r
D
Dramas Enrique VIII,
VIII, Ricardo III.
III.
(históricos)
Lírico “Venus y Adonis”, Sonetos
Sonetos..

 Ar queti
 Arqu etipo
poss en la ob
obra
ra d
dee Wil
Williliam
am Sh
Shakes
akespear e:  
peare:
   Hamlet representa
representa la duda; O Otelo,
telo, los celos; Macbe
Macbeth,
th, la ambici
ambición;
ón; Romeo y Julieta, el
amor puro; el rey Lear, la ingratitud filial.
3.4. Hamlet
Hamlet  
Hamlet parte de la recreación de un texto previo: una crónica sobre los reyes de Dinamarca
compuesta por Saxo Grammaticus (s. XII). Sin embargo, es Shakespeare quien dota de vida
a esta historia, especialmente, al elaborar el conflicto interior que atormenta al protagonista.
Características:
William Shakespeare es
  Pertenece al género dramá
dramático
tico y a la espec
especie
ie tragedia
tragedia.. considerado una figura
  Está compuesta por cinco actos. de transición entre el
 Renacimiento y el
   Desarrolla
Hace uso de la la técnica
psic ologíadel
psicología de monólog
monólogo o en es
los personajes.escena.
cena. Barroco.
  Utiliza el recurso del teatro ddentro
entro del teatro.
 Ar gumen
 Argu mentoto de Hamlet
Hamlet, príncipe de Dinamarca, se lamenta por la muerte de su
padre y cuestiona las razones por las que su madre se casó al
poco tiempo con su tío Claudio. En medio de este conflicto
interior, el príncipe recibe la noticia de que el fantasma de su
padre pasea por la explanada del castillo. El espectro le revela
que Claudio lo ha asesinado mientras dormía y le pide que
vengue su muerte, pero sin dañar a su madre. El príncipe quiere
vengar a su padre y, para ocultar sus intenciones, se finge loco.
Aprovechando la presencia de una compañía de teatro
ambulante,
muerte de su Hamlet decide
padre. representar
A mitad en escena
de la función, la verdadera
el rey Claudio,
perturbado por el desarrollo de la
l a trama, la suspende y el príncipe confirma las revelaciones
de la aparición. Después de la función, Hamlet confronta a su madre
m adre y mata accidentalmente
al indiscreto Polonio (viejo chambelán) al confundirlo con Claudio. El rey envía a Hamlet a
Inglaterra y da la orden de que lo maten en dicho país. Hamlet logra escapar falsificando las
órdenes del rey y regresa a Dinamarca, entonces se entera de que Ofelia (hija de Polonio),
perdida en la locura, se ha ahogado. Laertes (hijo de Polonio) desea vengar la muerte de su
padre y Claudio lo convence de enfrentar al príncipe en un duelo utilizando un arma
envenenada. Para el combate, Claudio prepara también una copa de vino envenenada para
que Hamlet la beba, si acaso gana la pelea. Durante el combate, Gertrudis bebe de la copa.
Hamlet es herido por Laertes, pero en medio de la batalla se confunden las espadas y el
príncipe lo hiere con su propia arma. La reina muere. Laertes sabe que va a morir y decide
confesar. Hamlet, moribundo, hiere al rey con la espada de Laertes y le hace beber de la copa

CEPRE-UNI HUMANIDADES 33
 

CICLO INTENSIVO ESCOLAR NACIONAL Material de Estudio Nº 1 

envenenada. El rey muere al igual que Laertes y Hamlet. Tras la tragedia, Fortimbrás accede
al trono de Dinamarca.

Temas principales: la duda y la venganza.


Temas secundarios: el amor, la locura y la muerte.

Evaluación 2
1. Dentro de la Literatura medieval se A) I-d, II-a, III-c, IV-b B) I-d, II-a, III-b, IV-c
desarrollaron dos especies literarias C) I-c, II-a, III-b, IV-d D) I-a, II-b, III-c, IV-d
importantes: el cantar de gesta y el poema épico. E) I-d, II-c, III-a, IV-b
Mientras que en el primer subgénero destaca la
obra________; en el otro sobresale _______. 4. Señale la(s) afirmación(es) correcta(s)
A) Cantar de Roldán -
Roldán - De vulgar elocuencia 
elocuencia  respecto al siguiente fragmento de la Divina
B) Poema de Mío Cid Cid  –
– Divina comedia 
comedia  comedia:  “A mitad del camino de la vida, / en
comedia: e n una
monarquía  –
C) La monarquía  –Poema de Mío Cid  Cid  selva oscura me encontraba / porque mi ruta
Nibelungos  –
D) Cantar de los Nibelungos  – Vida nueva 
nueva  había extraviado // ¡Cuán dura cosa es decir cuál
libertada  –
E) Jerusalén libertada  –Elogio de la locura era / esta salvaje selva, áspera y fuerte / que me
vuelve el temor al pensamiento!”.
pensamiento!”. 
2. Respecto a la Divina Comedia, Comedia, señale I. Emplea el terceto
t erceto y el verso endecasílabo.
verdadero o falso según corresponda. II. La selva oscura simboliza la vida en pecado.
I. Busca reformar la espiritualidad del hombre III. Evidencia narración en tercera persona.
renacentista. A) Solo I B) I y III C) II y III D) III E) I y II
II. Emplea el italiano toscano para su
composición literaria. 5. Sobre Hamlet,
Hamlet,   de William Shakespeare,
III. Es un cantar de gesta de evidente carácter señale la afirmación incompatible.
nacionalista. A) Pertenece a la especie literaria drama,
IV. Se divide en tres partes: Infierno, Purgatorio compuesta por cinco actos.
y Paraíso. B) Desarrolla el arquetipo de la duda y la
A) FVFV B) FVVF C) VFFV venganza en el protagonista.
D) VFVF E) FVVV C) Emplea la técnica dramatúrgica denominada
“teatro dentro del teatro”. 
3. Establezca la relación correcta: personaje- D) Presenta soliloquios, que la enriquecen, de
alegoría. una profundidad reflexiva.
I. Virgilio a. La fe y la gracia E) Recrea una crónica medieval basada en los
II. Beatriz b. El hombre pecador monarcas de Dinamarca.
III. Dante c. La lujuria 1B 2A 3B 4E 5A
IV. La pantera razón  
d. La razón

TEMA III: REALISMO. CARACTERÍSTICAS. GUSTAVE FLAUBERT (MADAME


( MADAME
BOVARY);
BOVARY); FEDOR DOSTOIEVSKI (CRIMEN
(CRIMEN Y CASTIGO).
CASTIGO). VANGUARDISMO.
NARRATIVA CONTEMPORÁNEA. FRANZ KAFKA (LA ( LA METAMORFOSIS).
METAMORFOSIS). ERNEST
HEMINGWAY (EL MAR) 
(EL VIEJO Y EL MAR)

1. REALISMO
Es un movimiento literario que surgió a mediados del siglo XIX en Francia, en contraposición al
Romanticismo. Privilegió
Privilegió la temática social.

1.1.
1.1. C
Contexto
ontexto histó rico y socioc ultural
  Surge el pensamiento
 pensamiento positivista, el cual as
asume
ume como verdadero co
conocimiento
nocimiento aq
aquel
uel que
se adquiere a través del método científico.

CEPRE-UNI HUMANIDADES 34
 

CICLO INTENSIVO ESCOLAR NACIONAL Material de Estudio Nº 1 

  Se produce
 produce el desarrollo de la Revolución Industrial multiplicando las didiferencias
ferencias sociales.
  Surgen las ideologías capitalista y ssocialista.
 ocialista. En 184
1848,
8, se publica el Manifiesto del Partido
Comunista de Marx y Engels.

1.
1.2.
2. Ca
Característi
racterísticas
cas del Re
Realismo
alismo
  Objetividad.  La razón predomina sobre la pasión o los sentimientos
 sentimientos.. Evidencian, además,
una visión científica.
  Descripción detallada. Las obras muestran una minuciosa descripción de la sociedad de

su tiempo (ambientes, personajes).


  Lengua
 Lenguajeje sencillo . Los autores emplean un lenguaje sobrio y directo.
  Narrador omnisciente.  Se recurre al emple
 empleoo del narrador oomnisciente
mnisciente (total) y el
impersonal.

1.3.. Repr
1.3 Representant
esentantes
es
País
País Auto
Autorr Obras
Francia Stendhal (Henry Beyle) Rojo y negro, La cartuja de Parma
Honoré de Balzac La comedia humana (abarca 97 novelas). 
novelas). 
Gustave Flaubert Madame Bovary, La educación sentimental
Rusia Fedor Dostoievski Humillados y ofendidos, Crimen y castigo
León Tolstoi La guerra y la paz, Ana Karenina

2. Gustave Flaubert (1821-1880) 


Nace en Ruan, Francia. Concibió la escritura como un proceso de
mucho trabajo y autocrítica. Al publicar Madame Bovary, enfrentó un
proceso legal debido a que se le calificó de inmoral.
Obras:  Madame Bovary; Salambó; La educación sentimental 

Características principales:
Afán por el perfeccionamient
perfeccionamientoo formal. 
  Establece un juicio crítico sobre los valores bburgueses.
urgueses.
  Utiliza el narrad
narrador
or oomnisciente
mnisciente e im
impersonal.
personal.  Flaubert es considerado uno de
los precursores de la novela
 Argu
 Ar gumen
mento
to de Ma
Madame
dame Bovary
Bov ary moderna por tener una clara
conciencia de la perfección
técnica.  
técnica.
Charles Bovary es un muchacho agobiado por la presencia
pr esencia
de su madre, por ello busca librarse de ella y acepta casarse con una mujer mayor, Eloísa.
Ya convertido en el médico de Tostes, Charles enviuda. Tiempo después, contrae matrimonio
con Emma Rouault. Su esposa es una mujer llena de sueños e ilusiones. Sus lecturas de
novelas románticas la han convencido de que el matrimonio es el inicio de una vida plena y
feliz. No obstante, se enfrenta a una decepcionante realidad, pues su esposo es un hombre
desapasionado y monótono. Esta situación lleva a Emma a un estado bastante crítico, por lo
que Charles decide mudarse a Yonville- l’ Abbaye. En este lugar nace su pequeña hija, Berta,
y Emma conoce al joven León Dupuis. Rápidamente se produce una intensa atracción entre
ambos. No obstante, ninguno tiene el coraje de expresar sus afectos. Al poco tiempo, el joven
León decide marcharse. Tiempo después, conoce a Rodolfo Boulanger, un hacendado
intrépido y aventurero, que la seduce. Así, Emma inicia una vida plena de goce y sobresaltos
al lado de su amante, pero la relación termina cuando Emma le propone huir juntos. Ella,

abrumadaresulta
esfuerzo por elvano.
rechazo,
Viaja decide retomar
con Charles la relación
a la ciudad afectiva
de Rouen, consesu
y allí esposo, pero
reencuentra todo
con León

CEPRE-UNI HUMANIDADES 35
 

CICLO INTENSIVO ESCOLAR NACIONAL Material de Estudio Nº 1 

Dupuis e inicia con él un tórrido romance. Asimismo, Emma, quien adquirió todo tipo de telas
y objetos, se vio envuelta en una cuantiosa deuda que no puede pagar. Abatida por la
desesperación y la deshonra, se suicida ingiriendo arsénico. Casi al final de la novela, Charles
Bovary descubre las cartas escritas por los amantes de Emma, y al poco tiempo muere. 

3. Fe
Fedor Dost oievski (1821-1881)
dor Dostoievski
Nace en Moscú. En 1849, fue condenado a muerte por colaborar con
grupos liberales
su ejecución. y revolucionarios.
Estuvo Se le indultóen
cuatro años encarcelado momentos antes en
Siberia. Murió de
San Petersburgo.
Obras:   Humillados y ofendidos (su primera gran novela), Crimen y
castigo (su novela más reconocida), Los hermanos Karamazov (su
última novela). Otras obras importantes: El idiota, El jugador ,
Memorias del subsuelo.

Características principales:
  Presenta uuna
na gran preocupación por la moral religios
religiosa.
a.
  Describe la sociedad rusa, ssumida
umida en crisis y miseria.
  Sus obras tiende
tiendenn a lo teat
teatral,
ral, debido a la gran importa
importancia
ncia de los diá
diálogos.
logos.
  Sus obras más importantes ssee presentan mediante el fformato
ormato del fo
folletín.
lletín.

 Argu
 Ar gumen
mento
to de Crimen y castigo
El estudiante Rodion Raskolnikov, azorado por la miseria e Por la exploración de la psique
impulsado por ideas de superioridad, asesina con un hacha de sus personajes, Dostoievski
a la vieja usurera Aliona Ivanovna y a su hermana Lizaveta. es considerado el iniciador de la
Este hecho desencadena en Rodion un largo periodo de novela psicológica. 
psicológica. 
crisis marcado por el sentimiento de culpa. Posteriormente,
Rodion entra en contacto con el juez Porfirio Petrovich, encargado de las investigaciones
sobre el asesinato de la anciana, pues pretende recuperar unos objetos que había dejado a
Aliona Ivanovna. El juez ha investigado a Rodion y tiene sospechas de él, pero no cuenta con
pruebas. Poco tiempo después, Rodion confiesa su crimen a Sonia, hija mayor de
Marmeladov, un hombre alcoholizado a quien conoció en una taberna. Entre el joven
estudiante y Sonia surge una relación afectiva. Ella era una joven noble y abnegada que se
había
Rodionsumido en la prostitución
a los textos bíblicos quepara apoyar asobre
le hablaban su empobrecida familia.
la redención. PasadoEllaelacerca
tiempo,aleljoven
juez
Petrovich le informa a Raskolnikov que tiene la certeza de que él es el asesino. Después de
reflexionar, Rodion se despide de su familia y se entrega. Sonia lo acompaña a confesar su
crimen. Finalmente, es condenado a ocho años de trabajo forzado en Siberia. Sonia viajará a
esa ciudad para visitarlo con frecuencia.
Tema central : el conflicto ético entre una moral cristiana humanitaria y una intelectual o
antihumanitaria.
4. VANGUARDISMO
El Vanguardismo es el nombre genérico de todo un conjunto de escuelas artísticas o
“ismos de vanguardia” que surgieron en Europa en las primeras décadas del siglo XX. Se
caracterizaron por oponerse al arte tradicional y al pensamiento racionalista del siglo XIX.
Los ismos de vanguardia que más destacaron fueron los siguientes:

CEPRE-UNI HUMANIDADES 36
 

CICLO INTENSIVO ESCOLAR NACIONAL Material de Estudio Nº 1 

Rindió culto a la máquina, a su velocidad y su potencia, como nuevo símbolo


Futurismo
de belleza.
Cubismo Jugó con las formas y la perspectiva de la palabra como imagen.
Expresionismo Se caracterizó por privilegiar una visión subjetiva de la realidad y su énfasis en
lo fantástico, lo absurdo y lo irracional.
Dadaísmo Se distinguió por su profunda rebeldía y tendencia a lo caótico y lo lúdico.
Experimentó con una poesía que desarticulara el lenguaje.
Fue influenciado por el psicoanálisis de Sigmund Freud. Presentó temas
Surrealismo oníricos y practicó la escritura automática.
5. NARRATIVA CONTEMPORÁNEA
CONTEMPORÁNEA
La Narrativa contemporánea es el conjunto de vertientes innovadoras que aparecieron durante la
primera mitad del siglo XX. Estuvieron relacionad
relacionadas
as con el espíritu vanguardista de esos años por
su carácter experimen tal. 
experimental.

5.
5.1.
1. Ca
Característic
racterísticas
as de la
l a Narrati
Narrativa
va con
contempor
temporánea
ánea
  Visión in tegra
tegrall del ser humano. Consideraron aspectos como el psicológico, el ético, el
religioso, el político, el emocional, todo lo que concierne al hombre.
  Expe rimentación lingüística. Ensayaron con los diferentes niveles de la lengua (desde el
Experimentación
habla popular hasta el habla culta).
  Temátic
Temática a variad
variada.a. Abordaron distintos temas (psicológicos, filosóficos, políticos, policiales,
eróticos, etc.) sin limitarse, estrictamente, a la preocupación social del Realismo del siglo

  XIX.
Innovaciones técnicas. Emplearon diversas técnicas narrativas. Estas innovaciones se
relacionan con los cambios radicales del contexto histórico.

5.2. Principales innovaciones narrativas

TÉCNICA
TÉCNI NA RRATIVA  
CA NARRATIVA DESCRIPCIÓN
La conciencia del personaje es representada a través de un discurso
Monólogo interior
complejo y caótico.
Multiplicidad de voces Aparecen diferentes voces en la narración.
Ruptura cronológica Se quiebra la lineali
linealidad
dad del relato.
Historias paralelas Se relatan diferentes historias que parecen independi
independientes.
entes.
Narrador parcial Este narrador solo relata desde su propia perspectiva.

5.3.. Repr
5.3 Representant
esentantes
es
PAÍS AUTOR OBRAS
Irlanda James Joyce Ulises, Dublineses
Albert Camus (Nobel 1957) El extranjero
Francia Jean Paul Sartre (Nobel, 1964) La náusea
Marcel Proust En busca del tiempo perdido
Franz Kafka La metamorfosis
 Al
 Aleman
emania
ia El proceso

CEPRE-UNI HUMANIDADES 37
 

CICLO INTENSIVO ESCOLAR NACIONAL Material de Estudio Nº 1 

6. FRANZ
FRANZ KAFK
KAFKAA (1883-1924)

Nació en Praga, Checoslovaquia, pero desarrolló toda su obra en


lengua alemana. Sostuvo una relación conflictiva con su padre, lo cual
se refleja en el texto Carta al padre. Antes de morir, le solicitó a su
íntimo amigo Max Brod que quemara sus escritos, pero este se
encargó de difundir su obra.

padre.   La metamorfosis,  América, El proceso, El castillo, Carta al


Obras:

6.
6.1.
1. C
Características
aracterísticas d
dee su obra
  Muestra una actitud cuestionadora de los sistemas Kafka evidencia en La
opresores. metamorfosis la influencia del
  Explora el mundo ddee los sueños y las pesadillas
pesadillas.. expresionismo,, pues transgrede la
expresionismo
  Desarrolla temas como la soledad, la frustra
frustración
ción y lógica cotidiana por medio de
escenas absurdas.
la culpa.

6.
6.2.
2. Argumento de La metamorfos
metamorfos is

Una mañana, Gregorio Samsa despierta convertido en insecto, por ello esa madrugada no ha
podido
casa el cumplir con su labor
jefe de personal paradeindagar
viajante de comercio.
sobre La situación
su ausencia. empeora cuando
Ante la insistencia llega a su
de sus familiares
y luego de mucho esfuerzo, Gregorio logra abrir la puerta de su habitación. Su nuevo estado
alarma a todos: su madre se desmaya, mientras que el jefe de personal sale huyendo
presuroso. Su padre lo obliga a
ingresar a su habitación con un
bastón en la mano. Durante las
primeras semanas, Grete, su
 joven hermana, se encarga de
llevarle los alimentos. Él
permanece encerrado en su
dormitorio oyendo de rato en rato
el nuevo proyecto de vida de su
familia. Evoca también pequeñas
escenas de su vida. Un día, en
que su madre y Grete sacaban
los muebles de su habitación,
Gregorio decide impedirlo. Esto provoca un nuevo desvanecimiento en su madre. Cuando
regresa su padre, lo conmina a volver a su habitación arrojándole numerosas manzanas. Una
de estas lo hiere al incrustarse en su caparazón.
Debido a su precaria situación económica, la familia aloja a tres huéspedes en su casa. Una
tarde, Grete toca el violín para ellos y Gregorio sale nuevamente de su encierro para oír de
cerca a su hermana. Los huéspedes descubren la existencia de Gregorio y, sobresaltados,
deciden marcharse sin pagar un centavo. Ante los lamentos de su hermana, esa madrugada,
Gregorio Samsa ansía su propia muerte. Al amanecer, la empleada anuncia la tragedia:
Gregorio ha muerto. La familia decide realizar un pequeño viaje para superar sus penas y
pensar en el futuro.

CEPRE-UNI HUMANIDADES 38
 

CICLO INTENSIVO ESCOLAR NACIONAL Material de Estudio Nº 1 

Tema central:  la alienación o la deshumanizac


deshumanización
ión del hombre moderno.
Temas
Te secundario s:  el autoritarismo, la incomunicación, la marginación.
mas secundarios:

7. ERNEST HEMINGWAY (1899-1961)

Notable escritor estadounidense nacido en Oak Park, Illinois. Desde


pequeño se dedicó
tauromaquia, lo quease
la refleja
pesca yenlasus
caza. Fue un
relatos y sugran
libroaficionado
Muerte ena la
la
tarde. Ganó el Premio Nobel de Literatura en 1954. En 1961, se
suicidó con una escopeta de caza.

7.
7.1.
1. C
Características
aracterísticas d de
e su obra
  Estilo periodíst
periodístico:
ico: presenta párrafos breves con frases ccortas
ortas y direct
directas.
as.
  Presenta ppersonajes
ersonajes vvencidos
encidos y marcados ppor
or la fatalidad.
  Alude a la lucha sim
simbólica
bólica entre el hombre y la naturalez
naturaleza.
a.
  Define a sus personajes a ttravés
ravés de acc
acciones
iones y diálogos Hemingway tiende en su
sencillos. narrativa a lo autobiográfico,
pues la nutre con sus
pasiones y experiencias: la
Obras:   Adiós
Adiós a las armas , Por quién doblan las campanas, bohemia, la pesca, la caza, la
El
7.2.viejo
7.2. y el mar , de
Argumento Fiesta.
El viejo y el mar tauromaquia, la guerra.

Santiago, un viejo pescador, no ha conseguido


pescar nada desde hace ochenta y cuatro días.
A Manolín, un muchacho que solía acompañarlo
en sus salidas al mar, sus padres lo han instado
a tomar otro bote. A pesar de ello, su amistad
continúa. Juntos comen y conversan sobre su
labor de pescadores, el béisbol y vivencias del
pasado. Una mañana, Manolín ayuda al viejo
Santiago a zarpar. Ya en medio del mar,
Santiago coge un pequeño pez y habla consigo
mismo sin perder las esperanzas. Al mediodía,
se da cuenta de que un enorme pez ha picado el anzuelo. Este tiene una descomunal fuerza,
pues arrastra el bote mar adentro. Así se inicia una lucha prolongada. Esto le recuerda a
Santiago algunos hechos de su vida como aquella vez que tuvo que pulsear su fuerza con el
negro de Cienfuegos, a quien derrotó después de todo un día de mucho esfuerzo. Al
amanecer, el viejo Santiago hunde su arpón en el corazón del pez espada. Posteriormente,
lo sujeta a un costado del bote e inicia el retorno a la playa. Sin embargo, la sangre del pez
atrae a los tiburones. Se inicia otro duro enfrentamiento. Los tiburon
tiburones
es logran su propósito. El
viejo Santiago llega al puerto, de madrugada, solo con el espinazo y la cabeza del pez. Al
final, Manolín va a verlo llevándole comida y trata de animarlo diciéndole que volverán a
pescar juntos. Un rato después, el viejo duerme y sueña con los leones marinos.

Tema central: la perseverancia.


Tema
Temas secundarios: la esperanza, la amistad, la naturaleza

CEPRE-UNI HUMANIDADES 39
 

CICLO INTENSIVO ESCOLAR NACIONAL Material de Estudio Nº 1 

Evaluación 3

1. Respecto al argumento de Madame Bovary,


Bovary, A) I-c, II-a, III-d, IV-b B) I-b, II-d, III-a, IV-c
de Gustave Flaubert, señale la alternativa C) I-c II-a, III-b, IV-d D) I-c, II-d, III-b, IV-a
incorrecta. E) I-a, II-b, III-c, IV-d
A) Emma le propone a Rodolfo huir juntos, pero
es rechazada. 4. El Vanguardismo, conjunto de tendencias
B) Charles, luego de la muerte de Emma, se artísticas de carácter innovador, se desarrolló a
casa con Eloísa. inicios del siglo XX en Europa. Este influenció en
C) El nacimiento de la pequeña Berta se produce
produ ce la Narrativa contemporánea a través de
en Yonville. A) la descripción detallada y objetiva de la
D) La protagonista, finalmente, se suicida realidad.
ingiriendo arsénico. B) el empleo exclusivo de un narrador
E) El esposo de Emma se muestra como un omnisciente.
hombre monótono. C) la narración de los hechos gloriosos del
pasado.
2. Señale la alternativa que incluye afirmaciones D) el evidente carácter experimental en sus
correctas respecto a Gustave Flaubert. obras.
I. Aspira a la perfección formal de la novela. E) la temática orientada solo a la muerte y el
II. Es un destacado autor del Romanticismo. amor.
III. Escribe la obra La educación sentimental.
sentimental.
IV. Tiende a lo teatral en su obra narrativa. 5. Señale la afirmación correcta con respecto
A) III y IV B) I y III C) II, III y IV D) II y IV a La metamorfosis,
metamorfosis, de Franz Kafka.
E) I, II, III A) El jefe prohíbe que la familia abandone la
casa.
3. Respecto a Crimen y castigo, castigo, de Fedor B) Grete, en todo momento, protege a su
Dostoievsky, relacione correctamente: hermano.
personaje-característica. C) La madre hiere al protagonista con unos
I. Porfirio Petrovich II. Sonia Marmeladov
Marmeladovaa frutos.
III. Rodion Raskolnikov IV. Aliona Ivanovna D) Los inquilinos aceptan quedarse en la
a) Representa la luz de la esperanza cristiana vivienda.
b) Prestamista que es considerada usurera E) Gregorio muere a causa del abandono
c) Juez que sigue los pasos del protagonista familiar.
d) Estudiante condenado a trabajo forzado 1B 2B 3A 4D 5E

CEPRE-UNI HUMANIDADES 40
 

CICLO INTENSIVO ESCOLAR NACIONAL Material de Estudio Nº 1 

TEMA IV: LITERATURA MEDIEVAL ESPAÑOLA. POEMA DE MÍO CID.


CID. SIGLO DE ORO:
GARCILASO DE LA VEGA ( ÉGLOGA I ). LOPE DE VEGA (FUENTEOVEJUNA). MIGUEL
“ ”

DE CERVANTES SAA VEDRA (EL QUIJOTE). 


CERVANTES SAAVEDRA
1. LITERATURA MEDIEVAL ESPAÑOLA
La Literatura medieval española comprende las obras compuestas entre los siglos XII y XV. Destaca
en este periodo la aparición de los mesteres de juglaría y clerecía.

1.
1.1.
1. principales
Las Ve
Verti
rtientes li terarias   entre las vertientes más representativas de la Literatura medieval
entes diferencias
española son las siguientes:
Verti
Ve rtiente
ente Mester
Mester de jugl
juglaría
aría Mester
Mester de clerecía
Cronología s. XII-XIII s. XIII-XV
Traducción Oficio del juglar Oficio del clérigo
 Auto
 Au to r Anónimo (juglares) Conocido (clérigos)
Tendencia Popular (cantares de gesta) Culta (lenguaje refinado)
Temática Histórica (estilo realista) Religiosa (carácter didáctico)
Transmisión Oral Escrita
Versificación Irregular Regular
Rima Asonante Consonante
- Milagros de Nuestra Señora,
Señora, de Gonzalo
Obras de Berceo. Fragmento: “Quería, aunque era
- Poema de Mio Cid 
(anónimo) Cid  oía susmalo, muchosiempre
sermones a Santalos
María, / / La
acogía.
- Mocedades de Rodrigo saludaba siempre diciendo cada día: / "Ave,
(anónimo) llena de gracia que pariste al Mesías".

- Libro de buen amor , de Juan Ruiz,


Arcipreste de Hita.

1.2. Poema de Mio Cid (anónimo)


Cid  (anónimo)
Esta obra posee gran valor histórico, lingüístico y literario, ya que nos narra las hazañas de
Rodrigo Díaz de Vivar, quien participó en la Reconquista de España; además, está compuest
compuestaa
en castellano antiguo, lo que permite conocer esta lengua en su etapa de formación. Se
desconoce al autor. Per Abbat fue el copista quien transcribió el manuscrito en 1207.

Características
Autor: anónimo Género: épico Especie: cantar de gesta
Estructura: tres cantares Estilo: realista Versos: 3735
Rima: asonante Métrica: irregular Carácter: nacionali
nacionalista
sta

 Argu
 Ar gumen
mento
to del Poema de Mio Cid 
Cid  

CEPRE-UNI HUMANIDADES 41
 

CICLO INTENSIVO ESCOLAR NACIONAL Material de Estudio Nº 1 

Primer cantar: DeDesti


stierro
erro de Mio Cid
El rey Alfonso VI de Castilla ordena el destierro de Rodrigo
Díaz de Vivar,
Vivar, el Cid. Este se dirig
dirigee a Burgos donde uuna
na
niña le anuncia que nadie puede ayudarlo. Luego, con la
ayuda de Martín Antolínez, el Cid saca provecho de dos
prestamistas judíos: Raquel y Vidas. Posteriormente, deja a
su familia en el Monasterio de San Pedro de Cardeña y
conquista Alcócer, Calatayud y Barcelona. Es en esta
ciudad donde vence al conde Ramón de Berenguer y le
gana la espada Colada.

Segundo cantar: Bodas de las hijas del Cid


El Cid logra recuperar la ciudad de Valencia y envía con Alvar Fáñez Minaya muchos
presentes al rey, con el objetivo de mostrar su fidelidad a la corona. El rey, conmovido por las
atenciones del Cid, lo perdona a orillas del río Tajo. A raíz de la creciente fama de Rodrigo
Díaz de Vivar, los infantes de Carrión, don Diego y don Fernando, solicitan como esposas a
sus hijas, doña Elvira y doña Sol. El Cid acepta con hondo pesar. Las bodas se celebran
durante quince días.
Tercer
Te rcer cantar: La afrenta de Corpes
La cobardía de los infantes de Carrión se evidencia cuando un león escapa de su jaula y
cuando
espada el rey Búcar
Tizona. Los de Marruecos
infantes intenta del
se marchan recuperar Valencia.
lugar con En estapero
sus esposas, batalla, el robledal
en el Cid ganade
la
Corpes las azotan salvajemente y las abandonan. El Cid solicita justicia al rey, quien convoca
a las Cortes de Toledo, donde los vasallos del campeador vencen a los infantes. Se realizan
las nuevas bodas de las hijas del Cid con los infantes de Navarra y Aragón.
Tema central. La recuperación de la honra.
Temas secundarios. La fidelidad, la guerra santa, el amor familiar.
f amiliar.
2. SIGLO DE ORO
El Siglo de Oro fue la etapa de mayor esplendor cultural en España. Abarcó dos periodos:
Renacimiento (s. XVI) y Barroco (s. XVII). Destacaron los géneros lírico, dramático y narrativo.

2.
2.1.
1. El Rena
Renacim
cimiento
iento español
El Renacimiento es la primera etapa del Siglo de Oro. Lo más destacado es la aparición y el desarrollo
de dos escuelas poéticas: italiana y salmantina.
2.
2.2.
2. E
Escuelas
scuelas poéticas
p oéticas rrepresentativas
epresentativas
Escuela itali
italiana
ana Escuela salmantin a
Fue influenciada por la lírica italiana (Francesco Escuela poética cultivada en Salamanca y
Petrarca). Empleó el verso endecasílabo, la lira, el derivada de la Escuela italiana (influencia de
soneto y la octava real. Además, trabajó especies Garcilaso). Destacó por la concisión de su
poéticas como la elegía, la oda, la canción y la lenguaje. Su temática fue reflexiva y de
égloga. carácter filosófico.
Representantes
Garcilaso de la Vega Fray Luis de León
2.3.. Repr
2.3 Representant
esentantes
es
a) Garci laso de la Vega (1503-1536) 
Garcilaso Garcilaso de la Vega desarrolla
Nació en Toledo. Hizo triunfar el verso endecasílabo el tema del amor no
introducido por el Marqués de Santillana y Juan Boscán. correspondido.
Participó en muchas batallas siempre al servicio del rey

CEPRE-UNI HUMANIDADES 42
 

CICLO INTENSIVO ESCOLAR NACIONAL Material de Estudio Nº 1 

Carlos I de España. Durante el asalto a la fortaleza de Provenza, Garcilaso fue herido


gravemente y falleció.
Obras: tres églogas, dos elegías, una epístola, cinco
odas y cuarenta sonetos.
Características
Característi cas de su obra:
  Estuvo influenciada por Francesco Petrarca. En
España,
las representa
normas del versoel“al
esplendor de la lírica
itálico modo”.   dentro de
  Trabaja tem
temas
as ligados al amor y la mitología.
  Sus poemas evidencian ritmo, equilibrio y
musicalidad.
Tópicos latinos emple
empleados
ados por Garcil
Garcilaso:
aso:
  Carpe diem 
diem   (aprovecha el día). Este tópico es tomado de la obra del poeta latino Horacio.
Aconseja gozar la belleza y la juventud, pues todo lo terrenal es efímero.
  Beatus ille 
ille  (dichoso aquel). Hace elogio a la vida sencilla y apacible del campo, en contraste
con la ciudad llena de vicios.
  Locus amoenus (lugar
amoenus  (lugar ameno). Alude a la descripción de una naturaleza idealizada, un espacio
armónico de atmósfera bucólica (relativo al campo y a los pastores). 

Égloga I: “Salicio y Nemoroso” 


La égloga es una especie lírica de carácter pastoril con personajes idealizados. La también
llamada “Égloga primera” de Garcilaso de la Vega está escrita en 30 estancias y consta de
421 versos. Su temática se centra en la vida y los amores de dos pastores: Salicio se lamenta
por el rechazo de la pastora Galatea; y Nemoroso, por la muerte de su amada Elisa.

Salicio:
El dulce lamentar de dos pastores, ¡Oh más dura que mármol a mis quejas,
Salicio juntamente y Nemoroso, y al encendido fuego en que me quemo
he de contar, sus quejas imitando; más helada que nieve, Galatea!
cuyas ovejas al cantar sabroso Estoy muriendo, y aún la vida temo;
estaban muy atentas, los amores, témola con razón, pues tú me dejas;
(de pacer olvidadas) escuchando. que no hay, sin ti, el vivir para qué sea.

3. TEATRO BARROCO DEL SIGLO DE ORO


El teatro del Siglo de Oro presentó dos tendencias: la popular, con piezas de temática nacionalista y
lenguaje sencillo; y la cortesana, con obras de densidad filosófica y lenguaje refinado. Durante el
Barroco, el teatro alcanzó su mayor esplendor con autores como Félix Lope de Vega, Pedro Calderón
de la Barca y Tirso de Molina.

3.
3.1.
1. C
Características
aracterísticas del teatro b
barroco
arroco del Siglo de
de O
Oro
ro
  Público vasto. El público del teatro barroco fue socialmente heterogéneo. Los espacios
idóneos para la representación fueron los corrales de comedias.
  Ruptur
Ruptura unid adess clásic as. Quebraron las unidades de tiempo, lugar y acción.
a de las unidade
  Consolidación d el drama. Para brindar un mayor disfrute al espectador, se difundió esta
especie, mezcla de lo trágico y lo cómico.

  estróficas:
Uso de distintas estrofas.
redondillas, El reales,
octavas teatro barroco se escribió
silvas, liras, enentre
sonetos, verso, con distintas formas
otras.
  Pluralidad temática. Exploró temas diversos, atendiendo siempre al gusto del público.

CEPRE-UNI HUMANIDADES 43
 

CICLO INTENSIVO ESCOLAR NACIONAL Material de Estudio Nº 1 

3.2. Féli
3.2. Félixx Lope
L ope d
dee Vega (1562-1635) 
Vega
Escritor madrileño. Fue conocido como el Fénix de los Ingenios, por ser
un escritor de gran talento. Cervantes lo llamó el Monstruo de la
Naturaleza, por su prolífica producción literaria.
Obras:
GÉNERO OBRAS
Épico La Dragontea,
Dragontea, El Isidro 
Isidro 
Lírico Rimas humanas, Rimas sacras, Romancero espiritual 
espiritual  
Dramático   Fuenteovejuna
Dramático Fuenteovejuna;; El mejor alcalde, el Rey; El caballero de
Olmedo  
Olmedo

Características
Característi cas de su obra:
  Representante de la tendencia popular, pues priorizó el entrete
entretenimiento
nimiento del público. 
  Fue el organizador del nuevo teatro español del siglo XVII. 
  Privilegió la acción sobre los personajes.
  Incorporó al personaje gracioso dentro del teatro español.
  Hizo uso de la polimetría (diversidad de met
metros
ros y estrofas). 
 Ar gumen
 Argu mento
to de Fuenteovejuna
El comendador de Fuenteovejuna, Fernán Gómez, excede los privilegios de su cargo:

atropella losFrondoso
campesino derechosestá
de declarando
los pobladores y abusa
su amor de las aparece
a Laurencia, campesinas. UnGómez
Fernán día, cuando el
e intenta
abusar de la joven. Frondoso lo impide amenazándolo con una ballesta. Posteriormente, en
medio de la boda de Laurencia y Frondoso, el comendador irrumpe con su séquito, secuestra
a Laurencia (para abusar de ella) y apresa a Frondoso. Tras esto, los pobladores dialogan y,
en medio de su reunión, aparece Laurencia y recrimina duramente a los varones por haber
sido incapaces de protegerla.
El pueblo entero se rebela; se dirige a la Casa de la Encomienda, toma el edificio y mata a
Fernán Gómez. Los reyes católicos envían a un juez para que investigue el suceso. A la
interrogante de quién mató al comendador, el juez obtiene siempre la misma respuesta:
“Fuenteovejuna lo hizo”. Los labradores explican al rey los motivos de su acción y le reiteran
su fidelidad. Al comprender que se ha hecho justicia, el rey perdona al pueblo de
Fuenteovejuna.

Tema
masprincipal:
Temas
Te el honor
secundarios:
secundario villanodeo poder,
s:  el abuso campesino.
la justicia popular, el amor.
4. NARRATIVA BARROCA
La narrativa barroca alcanzó su mayor esplendor con la publicación de ElEl   ingenioso
ingenioso   hidalgo don
Quijote  de La Mancha (publicada
Quijote Mancha (publicada en dos partes, en 1605 y 1615) de Miguel de Cervantes Saavedra.
No obstante, la narrativa barroca se caracterizó porque existieron diversos tipos de novela:
sentimental, pastoril, de caballerías y picaresca.

4.1. Miguel d e Ce
4.1. Cervan tes Saavedra (1547-1616)
rvantes
Nació en Alcalá de Henares. Participó en la batalla de Lepanto en la
cual fue herido, quedando con el brazo izquierdo inutilizado; por ello,
se le conoció como el Manco de Lepanto. Murió en Madrid, sumido en
la pobreza y sin reconocimiento por su obra.
Obras:

CEPRE-UNI HUMANIDADES 44
 

CICLO INTENSIVO ESCOLAR NACIONAL Material de Estudio Nº 1 

GÉNERO OBRAS
Narrativo El ingenioso hidalgo don Quijote de La Mancha;
Mancha; las doce Novelas Ejemplares,
Ejemplares,
donde destacan: Rinconete y Cortadillo y
Cortadillo y La gitanilla.
gitanilla.
Lírico Viaje del Parnaso. 
Parnaso. 
Dramático El cerco de Numancia (tragedia),
Numancia (tragedia), Pedro de Urdemalas (comedia)
Urdemalas (comedia).. 
 Ar gumen
 Argu mento
to de El ingenios o hidalgo d
don
on Quijot e de La M
Mancha
ancha
Primera parte (1605). El hidalgo Alonso Quijano pierde el juicio a causa de la lectura
incesante
un caballerodeandante.
libros dePara
caballería;
ello, seentonces,
nombra adecide convertirse
sí mismo en
don Quijote
Q uijote El tema central de
de La Mancha. Se propone ayudar a los más necesitados y El ingenioso hidalgo don
restablecer la justicia donde haga falta. Promete dedicar todas sus Quijote Mancha es
de La Mancha es
la lucha entre lo ideal y
hazañas a su dama, Dulcinea del Toboso, quien en realidad es una lo material. 
material. 
labradora llamada Aldonza Lorenzo.
En su primera salida, se arma caballero en una posada, que confunde en su locura con un
castillo. Posteriormente decide regresar a casa para proveerse de recursos, pero es apaleado
en el camino. Una vez en casa, el Quijote se da cuenta de que necesita un escudero;
entonces, se dirige a la casa del campesino Sancho Panza y lo convence de que lo acompañe.
En su segunda salida, el Quijote emprende nuevas aventuras: la batalla contra los molinos de
viento, la lucha contra el escudero vizcaíno, su enfrentamient
enfrentamientoo contra un rebaño de ovejas y
la preparación del bálsamo de fierabrás. Además, hace penitencia en Sierra Morena y
finalmente regresa a su pueblo en una jaula mediante el engaño de sus amigos, el cura y el
barbero.
Segunda parte (1615). En
(1615). En la tercera salida, se dirigen al encuentro con Dulcinea. Sancho convence
a don Quijote de que su amada ha sido encantada y por esa razón luce como una campesina. En su
búsqueda por desencantarla llegan a Aragón, donde unos duques, muy divertidos con la locura de
ambos, deciden burlarse designando a Sancho como gobernador de la Ínsula Barataria. El último
enfrentamiento del Quijote se da contra el Caballero de la Blanca Luna, que resulta ser su vecino,
Sansón Carrasco. Este derrota al hidalgo y lo obliga a regresar a casa. Al poco tiempo de su rretorno,
etorno,
don Quijote cae enfermo y recobra la cordura antes de morir.
Característi
Características
cas de El ingenioso
ingenios o hidalgo d on Quijote
Quijo te de La M
Mancha
ancha
Estilo. Emplea tres registros primordiales: el lenguaje del narrador (culto), el lenguaje señorial (de don
Quijote) y el lenguaje coloquial (de Sancho).
no mi as. Son nociones opuestas y complemen
 Antiti nomi
 An complementarias,
tarias, como: locura/cordura y ser/parecer.
Intercambio de psico logías. Hacia el final de la obra, se produce un intercambi
intercambio o de psicologías: Don
Quijote, el soñador e idealista, se contagia del pragmatismo de su escudero; y Sancho, a su vez, se
impregna de la ensoñación de su amo. A este fenómeno se le conoce como la quijotización
quijotización de
 de Sancho
y la sanchificación
sanchificación de
 de don Quijote.

Evaluación 4

1. Respecto a la Literatura española medieval


medieval,, I. El Cid pierde su honor en el primer y tercer cantar.
indique verdadero o falso según correspond
corresponda.a. II. El contexto histórico es la Reconquista de España.
I. Desarrolló dos vertientes: juglaría y clerecía. III. Los infantes de Carrión vencen al Cid en Valencia.
II. Rechazó la temática
t emática histórica y nacional. IV. El rey perdona al protagonista a orillas
or illas del río Tajo.
III. Se transmitió de manera oral y escrita. A) I, II y III B) II y III C) I, II y IV D) III y IV E) I
IV. Empleó la rima asonante y consonante.
A) VFVV B) FFVV C) VFVF 3. Respecto a la “Égloga I”, relacio ne
D) FVFV E) FVFF correctamente  –característica.
correctamente:: pastor 
I. Salicio II. Nemoroso
2. Identifique los enunciados correctos sobre el a. Dirige sus pesares a Galatea.
argumento del Poema de Mio Cid. 
Cid.  b. Llora la muerte de su amor.
c. Lamenta el rechazo de su amada.

CEPRE-UNI HUMANIDADES 45
 

CICLO INTENSIVO ESCOLAR NACIONAL Material de Estudio Nº 1 

d. Su tristeza se debe a Elisa. 5. ¿Qué gigantes? -dijo Sancho Panza. Aquellos


A) I- a, c; II- b, d B) I- b, c; II- a, d que allí ves -respondió su amo- de los brazos
C) I- a, d; II- b, c D) I- c, d; II- a, b largos, que los suelen tener algunos de casi dos
E) I- b, d; II- a, c leguas. Mire, vuestra merced, -respondió
Sancho- que aquellos que allí se parecen no son
4. ¿No se ven aquí los golpes / de la sangre y las gigantes, sino molinos de viento, y lo que en
señales? / ¿Vosotros sois hombres nobles? / ellos parecen brazos son las aspas, que,
¿Vosotros padres y deudos? / ¿Vosotros, que no volteadas del viento, hacen andar la piedra del
se os rompen / las entrañas de dolor, / de verme molino.  
molino.
en tantos dolores? / Ovejas sois, bien lo dice / de Según el fragmento anterior, de la novela El
Fuenteovejuna el hombre. Según los versos ingenioso hidalgo don Quijote de La Mancha,Mancha,
anteriores, del drama Fuenteovejuna,
Fuenteovejuna,   ¿Cuál es identifique la característica que en ella resalta.
el objetivo ante lo mencionado por Laurencia? A) La sanchific
sanchificación
ación B) Antinomias
A) Dar a conocer el nombre Fuenteovejuna. C) La quijotizaci
quijotización
ón D) Pesimismo
B) Calificar como cobardes a los hombres. E) Uso de la polimetría
C) Incitar la rebelión frente al comendador.
D) Describir los golpes a su padre Esteban. 1A 2C 3A 4C 5B
E) Culpar a los hombres nobles y deudos.

TEMA V: NUEVA NARRATIVA LATINOAMERICANA: JORGE LUIS BORGES


(FICCIONES
FICCIONES);
); JUAN RULFO (PEDRO
(PEDRO   PÁRAMO
PÁRAMO).
). EL BOOM LATINOAMERICANO.
CARACTERÍSTICAS, REPRESENTANTES. GABRIEL GARCÍA MÁRQUEZ (CIEN
( CIEN AÑOS
SOLEDAD). 
DE SOLEDAD).

1. NUEVA NARRATIVA LATINOAMERICANA


Se desarrolló desde 1920 hasta 1970, aproximadamente. Se caracterizó por el uso de las
innovaciones técnicas propias de la narrativa contemporánea.

Presenta tres etapas:


Etapa Características
Emergente Aparecen de modo incipiente algunas obras cuyo lenguaje muestra
(1920-1930) un afán renovador.
De consolidaci
consolidación
ón Las obras desplazan al Regionalismo, presentando nuevas
(1940-1950) modalidadess narrativas.
modalidade
De apogeo Conocido como Boom, fue un fenómeno narrativo con obras de alta
(1960-1970) calidad y de un gran éxito editorial.

1.
1.1.
1. C
Características
aracterísticas de lla
a etapa
etapa de cons olidación
olid ación

  Transculturación narrativa.  Reactualización de tradiciones, mitos y leyendas, vistos


desde una óptica moderna, empleando para ello las innovaciones técnicas.
  Innovaciones técnicas. Asimilan las técnicas de la narrativa europea y norteamericana:
multiplicidad de voces, ruptura del orden cronológico, etc.
  Experimentación lingüística.  Se busca explotar la capacidad expresiva del idioma
utilizando la oralidad y lo coloquial.
  Cosmopolitismo narrativo. La urbe empieza a desplazar el espacio regional (pampas,
llanos, etc.) como lugar de representación. Se observa un alejamiento de las ciudades
natales y un acercamiento a Europa.

País
País Jorge Auto
Autorr
Luis Borges Ficciones, El Aleph Obras
Argentina
Ernesto Sábato El túnel, Sobre héroes y tumbas

CEPRE-UNI HUMANIDADES 46
 

CICLO INTENSIVO ESCOLAR NACIONAL Material de Estudio Nº 1 

México Juan Rulfo El llano en llamas,


llamas, Pedro Páramo
Uruguay Juan Carlos Onetti La vida breve, Juntacadáveres
Paraguay Augusto Roa Bastos Yo, el supremo; Hijo de hombre
Cuba Alejo Carpentier El siglo de las luces, El reino de este mundo
Guatemala Miguel Ángel Asturias El señor presidente, Hombres de maíz

1.
1.2.
2. Jor
Jorge
ge Luis Bo rges (1899-1986)
Borges
Nació en Buenos Aires. Inicialmente escribió poesía ultraísta y
posteriormente se inclinó por el ensayo y el cuento, donde incorporó
elementos filosóficos y fantásticos. Ganó numerosas distinciones
literarias, entre ellas el Premio Cervantes (1979). En varias ocasiones,
fue candidato a ganar el Premio Nobel; sin embargo, nunca lo obtuvo.

Características
Característi cas de su obra
  Emplea un lenguaje culto e incorpora refe referencias
rencias eruditas (enciclopedias an
antiguas)
tiguas) y
pseudoeruditas (libros inexistentes, citas o referencias inventadas).
  Uso recurrente de símbolos
símbolos:: el tigre (el temor), el espejo (el doble), la bibliotec
bibliotecaa (el
conocimiento), la espada (la muerte) y el laberinto (el caos).
  Inserta reflex
reflexiones
iones filosóficas y metafís
metafísicas,
icas, también elementos fa
fantásticos.
ntásticos.
  Presenta influencia de la narrativa policial que se evidencia en el empleo del dato
escondido y en sus finales inesperados.
Género Obras
Narrativo Historia universal de la infamia (1935),  Ficciones (1944),
(1935), Ficciones  El aleph (1949),
(1944), El  El
(1949), El
informe de Brodie (1970),  El libro de arena (1975).
(1970), El (1975).  
Lírico Fervor de Buenos Aires (1923),  Elogio de la sombra (1969).
(1923), Elogio (1969).  
Ensayo Historia de la eternidad (1936),  Nueva refutación del tiempo (1947),
(1936), Nueva  Otras
(1947), Otras
inquisiciones (1952).
(1952).  
Ficciones (1944)
Ficciones 
  Son 17 cuentos en 2 secciones: El jardín de los senderos que se bifurcan y Artificios. 
  Emplea el na
narrador
rrador personaje
personaje;; incorpora citas y notas a pie de pá
página.
gina.
  Se observa un
unaa mezcla de géneros literarios, el principal eje
ejemplo
mplo es «Pierre Menard, aut
autor
or
del Quijote», cuento que está escrito como un ensayo.
1.3. Juan Rulfo (1917-1986)
1.3.
Nació en México. Fue un escritor, guionista y fotógrafo. En 1953, publicó
su libro de cuentos El llano en llamas y, transcurridos dos años, la novela
Pedro Páramo, con la cual se consagró como uno de los narradores más
importantes. Asimismo, se le considera el precursor del realismo mágico.
Características
Característi cas de su obra
  Utiliza magistralmente much
muchas
as de las innovaciones técnic
técnicas
as de la novelística europe
europea.
a.
Desaparece por completo el narrador omnisciente.
  Incorpora ele
elementos
mentos míticos mediante un simbolismo sutil.
  Brinda un juicio crítico y analiza las cons
consecuencias
ecuencias de la Revoluc
Revolución
ión mexicana (1910),
además del problema de la posesión de tierras.

CEPRE-UNI HUMANIDADES 47
 

CICLO INTENSIVO ESCOLAR NACIONAL Material de Estudio Nº 1 

 Ar gumen
 Argu mento
to de Pe
Pedro Páramo (1955)
dro Páramo
Juan Preciado, por encargo de su madre, viaja a Comala para buscar
a su padre, Pedro Páramo, a quien no conoce: cree descender al
infierno debido al intenso calor y desolación que rodea al pueblo. Se
encuentra con Abundio, arriero y uno de los tantos hijos bastardos de
Pedro Páramo, quien le indica que busque a doña Eduviges Dyada.
Tras un largo recorrido por Comala, Juan Preciado descubre que se
encuentra en un pueblo lleno de ánimas en pena que aparentan estar vivas: él también fallece
y se convierte en una de las voces que colabora con la narración.
Se nos informa así de la vida de Pedro Páramo, de su niñez así como de su matrimonio trunco
con Susana San Juan y de la muerte de su hijo Miguel Páramo. Se resalta también la forma
en que aquel se apodera de Comala y cómo deja morir de hambre a sus pobladores.
Finalmente, Pedro Páramo muere apuñalado por su hijo Abundio, desmoronándose
desmoronándose “como si
fuera un montón de piedras”.

2. EL BOOM LATINOAMERI
L ATINOAMERICANO
CANO
Se denomina Boom latinoamericano a la etapa de apogeo de la Nueva narrativa latinoamericana.
Se distingue por la consagración internacional de la narrativa en las décadas del 60 y el 70.  

2.1. Características
  Los escritores latinoamericanos son reconocidos a nivel La industria editorial cumplió
internacional. Esto seidiomas
obras a diferentes hace evidente
y en laenentrega
la traducción de sus
de diversos un rollatinoamericano.
decisivo en el Boom
premios.
  La labor del es
escritor
critor adquiere perfiles profesio
profesionales.
nales. La lit
literatura
eratura se cconvierte
onvierte en uunn medio
de subsistencia para estos narradores.
  Una constante preocupación
preocupaci ón política se hace evidente en los relatos, como consecuencia
del establecimiento de las diferentes dictaduras latinoamericanas.
  Se lleva a su máx
máxima
ima expresión las in
innovaciones
novaciones ttécnicas
écnicas de la Narrat
Narrativaiva contemporáne
contemporánea, a,
y se crean técnicas narrativas propias.

2.
2.2.
2. Princi
Principales
pales técnicas narrativas
narrati vas
TÉCNICA
CLASIFICACIÓN DEFINICIÓN
NARRATIVA
Representación del fluir de la conciencia
TÉCNICAS DE L
TÉCNICAS LAA Monólogo interior del personaje.
Acción narrativa relatada por distintos
NARRATIVA Multiplicidad de voces
personajes.
CONTEMPORÁNEA
Alteración, mezcla o simultaneidad de los
Ruptura cronológica
planos temporales.
Incorporación de elementos mágicos o
TÉCNICAS Realismo mágico
sorprendentes relatados con naturalida
naturalidad.
d.
LATINOAMERICANAS
ORIGINALES  
ORIGINALES Transculturación Reactualización de mitos o leyendas
narrativa ancestrales a través de técnicas modernas.

2.3.. Represent
2.3 Representantes
antes
PAÍS AUTOR OBRAS
Colombia Gabriel García Márquez
Cien años de soledad; El otoño del patriarca 
patriarca  
(Nobel, 1982)
México Carlos Fuentes La muerte de Artemio Cruz; La región más
transparente  
transparente
 Argen
 Ar gentiti na Julio Cortázar Rayuela; Todos los fuegos el fuego (cuentos)

CEPRE-UNI HUMANIDADES 48
 

CICLO INTENSIVO ESCOLAR NACIONAL Material de Estudio Nº 1 

Mario Vargas Llosa


Perú mundo 
La casa verde; La guerra del fin del mundo 
(Nobel, 2010)
2.4.. Gabriel García Má
2.4 Márq uez (1927-2014) 
rquez

Nació en Aracataca, un pequeño pueblo ubicado en el Caribe


colombiano. En 1967, alcanzó la consagración internaciona
internacionall
con la publicación de su novela Cien años de soledad.
Además de cuentista, novelista y guionista, se dedicó
también al periodismo. Ganó el Premio Nobel de Literatura
en 1982. Falleció en México, D. F. el 17 de abril de 2014.

Características
Característi cas de su obra
  Rescata la cultura popular
popular de los pueblos del Caribe Se considera a García
colombiano. Márquez como el
máximo representante
  Emplea el tiempo mítico, también denominado tiempo circular. del realismo mágico.
De esta manera, abandona la narración lineal; y en su lugar,
presenta una constante reactualización o repetición de los acontecimientos.
  Muestra una visión par
particular
ticular de la historia de su país y de Latinoam
Latinoamérica,
érica, espec
especialmente
ialmente
en su novela Cien años de soledad.
  Emplea un lenguaje variado. Destac
Destacaa el tono lílírico
rico y humorís
humorístico
tico en sus libros más
relevantes.

Obras:
  Ciclo Macondo. Novelas: La hojarasca, La mala hora, El coronel no tiene quién le escriba,
Cien años de soledad. Cuentos: Los funerales de la Mamá Grande , Doce cuentos
peregrinos, La increíble y triste historia de la
l a cándida Eréndira y de su abuela desalmada. 
  Otras novelas: El otoño del patriarca, Crónica de una muerte anunciada, El amor en los
tiempos del cólera; El general en su laberinto;
l aberinto; Memoria de mis putas tristes.  

 Ar gumen
 Argu mento
to de Cien años
años de soledad
Esta extraordinaria novela inicia con una evocación del El tema central de Cien años
coronel Aureliano Buendía, quien recuerda los primeros años de soledad es la soledad y el
de Macondo. Los esposos José Arcadio Buendía y Úrsula incesto. Otros temas
Iguarán son quienes, junto con otras jóvenes familias, fundan importantes son la familia, la
locura y el amor.
el pueblo.
temor deLos cónyuges
procrear erancon
un hijo primos, y Úrsula,
cola de cerdo, inspirada por
evita durante
año y medio que el matrimonio se consume. La pareja abandona su pueblo natal para alejarse
del fantasma del compadre Prudencio Aguilar, asesinado por José Arcadio al haberse burlado
de su hombría. En el camino de viaje, la pareja engendra a José Arcadio; y ya en Macondo,
nacen Aureliano (luego conocido como el coronel Aureliano Buendía) y Amaranta.

CEPRE-UNI HUMANIDADES 49
 

CICLO INTENSIVO ESCOLAR NACIONAL Material de Estudio Nº 1 

Macondo permanece aislado de la modernidad durante


mucho tiempo y sus pobladores solo tienen contacto con un
grupo de gitanos que aparecen de vez en cuando. Su líder,
Melquíades, entabla una profunda amistad con el patriarca del
pueblo, José Arcadio Buendía. Los gitanos introducen en
Macondo novedades como el hielo y el imán. Además,
Melquíades le regala al fundador del pueblo un laboratorio de
alquimia y unos manuscritos redactados en una lengua
enigmática. Ya viejo, José Arcadio Buendía muere amarrado
a un árbol, preso de la locura. Tras su muerte, se produce una
lluvia de flores amarillas.

José Arcadio hijo se casa con Rebeca, una prima lejana que
vive en casa de los Buendía, debido a lo cual los jóvenes son
arrojados de casa por Úrsula, que no aprueba el matrimonio.
Poco tiempo después, José Arcadio aparece muerto.
Aureliano, por otra parte, se convierte en el coronel Aureliano Buendía y, tras la trágica muerte
de su joven esposa Remedios, se entrega a la revolución. El coronel promueve treinta y dos
levantamientos armados y los pierde todos. La novela transcurre a lo largo de cien años y
narra las vicisitudes de siete generaciones de la familia Buendía.

Al
tíafinal de la novela,
Amaranta Úrsula.Aureliano
Fruto de Babilonia, descendiente
esa relación, de los
nace un niño conBuendía, tiene amores
cola de cerdo, que escon su
luego
devorado por las hormigas. Tras mucho esfuerzo, Aureliano Babilonia consigue descifrar el
sentido de los viejos pergaminos entregados por Melquíades a su antepasado José Arcadio
Buendía. Mientras lee, saltándose varias páginas, comprende que él jamás saldrá de ese
cuarto porque “estaba previsto que la ciudad de los espejos (o los espejismos) sería arrasada
por el viento y desterrada de la memoria de los hombres en el instante en que Aureliano
Babilonia acabara de descifrar los pergaminos, y que todo lo escrito en ellos era irrepetible
desde siempre y para siempre, porque las estirpes condenadas a cien años de soledad no
tenían una segunda oportunidad sobre la tierra”.  

Evaluación
Evaluación 5

1. Señale la con
incongruencia afirmación
la Nuevaque narrativa
presenta A) Usa un lenguaje culto e incorpora el
enciclopedismo.
latinoamericana. B) Emplea influencia del relato policial y el dato
I. Emplea, inicialmente, en algunas narraciones oculto.
un lenguaje renovador. C) Utiliza un lenguaje sobrio y sencillo en su
II. Desplaza al Regionalismo y presenta nuevas narrativa.
modalidadess narrativas.
modalidade D) Toma elementos fantásticos, filosóficos y
III. Muestra un uso mesurado de figuras literarias metafísicos.
o retóricas en sus textos. E) Inserta símbolos como el tigre, el espejo y la
IV. Resulta de la asociación de la narrativa de biblioteca
alta calidad y el éxito editorial.
V. Alcanza su apogeo con el fenómeno literario 3. Señale la alternativa correcta respecto al
conocido como el Boom. argumento de Pedro Páramo.
A) I B) II C) III D) IV E) V A) Pedro Páramo hereda sus tierras a Abundio.
B) Juan Preciado conoce a su padre en el viaje.
2. Marque incorrectas
afirmaciones la alternativa
sobre laque
obra presenta
de Jorge C)
D) Miguel Páramo muere
El protagonista es despreciado
apuñaladoporpor
susu
padre.
hijo.
Luis Borges. E) Comala desaparece tras una tempestad.

CEPRE-UNI HUMANIDADES 50
 

CICLO INTENSIVO ESCOLAR NACIONAL Material de Estudio Nº 1 

4. Respecto a Gabriel García Márquez, es


incorrecto afirmar que
A) destacó en dos géneros: lírico y narrativo.
B) recibió el Premio Nobel de Literatura (1982).
C) deja de lado la denominada narración lineal.
D) empleó el realismo mágico en su narrativa.
E) fue representan
representante
te del Boom latinoamericano
latinoamericano..

5. Al final de Cien años de soledad,


soledad, _______ es
quien logra descifrar los viejos pergaminos que
le entregaron a José Arcadio Buendía.
A) el coronel Aureliano Buendía
B) el gitano Melquiades
C) Prudencio Aguilar
D) Aureliano Babilonia
E) Remedios La Bella

1C 2C 3C 4A 5D

CEPRE-UNI HUMANIDADES 51
 

CICLO INTENSIVO ESCOLAR NACIONAL Material de Estudio Nº 1 

Historia
TEMA I. PREHISTORIA Y HOMINIZACIÓN HASTA MESOPOTAMIA, EGIPTO

1. Homi
Hominización
nización

Es el proceso evolutivo que desarrollaron antiguos primates hasta


adquirir características que los definieron como humanos. Este
comprendió un conjunto de mutaciones biológico-anatómicas, así
como, logros culturales y tecnológicos.

1.1. Principales planteamientos sobre la evolución

La Selección natural (Charles Señala que la naturaleza impone condiciones donde las
Darwin) especies más aptas logran la supervivencia.
El papel del trabajo Consideró el trabajo como el factor más importante, que hizo
(F
(Friedrich
riedrich Enge ls)  
Engels) posible el proceso de hominización.

1.
1.2.
2. Princi
Principales
pales factores de la hominiza
homini zación
ción

  El bipedismo. Fue el desplazamient


desplazamientoo con
las extremidades
liberar las manosinferiores
y brazos.que hizo
Ello posible
facilitó la
posición erguida.
   La es
especialización
pecialización del pulg
pulgar
ar op
oponible.
onible. Esto
favoreció la producción de herramientas.
   El crecimiento craneal y cerebral. Su
desarrollo acrecentó la creatividad y el
desarrollo de capacidades complejas, entre
ellas, el pensamiento abstracto.
   El lenguaje art
articulado.
iculado. Est
Estoo incrementó la
acumulación de conocimientos y la
socialización.

1.3. ncias
evide Principales
evidencias fósil es especies
fósiles homínidas y
Halladas principalmente
en el valle del Rift (entre Etiopia, Kenia y
Tanzania) en África. Le corresponden los
siguientes géneros:
 A. Género
Gén ero  Au
 Aust
stral
ralop
opitithec us . Son homínidos
hecus
que se destacan por dominar el bípedismo.
Entre sus especies destaca el:
 Au strao
 Aust raopipitec
tecus
us Se considera una de las especies más antiguas de este género.
anamensis  
anamensis
 Aust
 Au stral
ral opit
op itecu
ecuss Destacan el esqueleto de Lucy
Lucy (Etiopía), pisadas de Laetoli (Tanzania)
 (Etiopía), las pisadas de Laetoli (Tanzania) y la
afarensis  
afarensis Niña de Selam. 
Selam. 

B. Género Homo. Presenta un mayor crecimiento de la masa encefálica y capacidad para


elaborar herramientas. Entre sus especies se encuentra el

CEPRE-UNI HUMANIDADES 52
 

CICLO INTENSIVO ESCOLAR NACIONAL Material de Estudio Nº 1 

  Homo hábilis. Es la primera especie humana. Asimismo, iniciaron la producción de


herramientas 
  Homo erectus . Fue el primer humano que migró del continente africano y ocupar Asia y
Europa. Además, uso el fuego que ayudó a la cocción de sus alimentos.
  Homo neanderthalensis. Sus restos se encuentran desde Europa hasta medio oriente.
Muestra un gran tamaño cerebral y evidencia la capacidad de lenguaje articulado incipiente.
  Homo sapiens. El resto más antiguo de esta especie corresponde al hombre de Herto
(Etiopía) de
(Francia). donde
Los procede
sapiens el hombre
poblaron moderno.
Oceanía Otro ejemplar fue el hombre de Cromagnon
y América.
2. La Prehistor
Prehistoria
ia
Época que abarca desde el surgimiento de la humanidad hasta la aparición de la escritura.
Piedr a 
2.1. La Edad de Piedra
2.1.
Es la etapa en el que predominó el uso de la piedra. En esta época surgen las Comunidades
primitivas, cuyas características esenciales son: la propiedad colectiva, es decir carencia de
toda idea de propiedad privada, y ausencia de jerarquías sociales. Por ser un periodo de larga
duración, presenta tres periodos:
 A. Paleo
Paleoll íti co .  Su desarrollo se produjo a fines del Pleistoceno y presenta las siguientes
ítico
características:
   La economía depredadora o ddee su subsistencia,
bsistencia, caracterizada por la caz
cazaa y la recolec
recolección
ción
de plantas.
   La organización social fueron las bandas, grupos nómades sin liderazgo definido.
El paleolítico es periodizado en tres fases:
 A.1. in fer iorr . Es la fase del Homo hábilis y del Homo erectus. Aparece la industria lítica y la
ferio
manipulación del fuego.
 A.2. med
medioio . Predominó el Homo neanderthalensis, quien manifestó las
tempranas ideas mágico-religiosas mediante los primeros entierros
humanos.
 A.3. su per io r. Predominó el Homo sapiens creador del arco y la flecha,
perio
así como el arte rupestre o parietal, cuyas primeras evidencias se
encuentran en la cueva de Altamira (España) y Lascaux (Francia).
También el arte mobiliar, a través de las venus paleolíticas (culto a la
fertilidad femenina). Las bandas de esta especie desarrollaron el
poblamiento americano.

B. Mesolítico. Se produjo en el contexto de variación


climática mundial: el paso del Pleistoceno al Holoceno,
lo cual generó el fin de las glaciaciones y la extinción
de la megafauna.
m egafauna. Entre sus características tenemos:
   Presencia de recolección y caza selectiva, la
producción incipiente con la horticultura y el primer
paso a la domesticación de animales.
   Organización en clanes
clanes,, que practicaron la
trashumancia o seminomadismo.
   El impulso de la industria microlítica ligada a la caza y pesca.

C. Neolítico.  Se produjo un mayor avance técnico con la pulimentación de la piedra, sin


embargo, el proceso más resaltante fue la Revolución Neolítica que consistió en el paso de
una economía de subsistencia a una economía productiva. Tuvo como centros de desarrollo

CEPRE-UNI HUMANIDADES 53
 

CICLO INTENSIVO ESCOLAR NACIONAL Material de Estudio Nº 1 

originario la Media luna fértil  (Egipto y Mesopotamia), el Lejano oriente  (China e India),
Mesoamérica (México) y los Andes centrales (Perú).
Característi
Ca racterísticas
cas de la R
Revoluci
evoluciónón Neolítica
  El desarrollo de una economía productiva (agricultura y ganadería), que permitió el
surgimiento del excedente productivo que facilitó
el trueque y la aparición de artesanos.
  La forma de vvida ida sedentaria, la cual se
manifestó en las nuevas actividades domésticas
como la producción de tejidos, cerámica,
construcción con barro, etc.
  La organiz
organización
ación en ttribus.
ribus. Las tierras
continuaban siendo colectivas y su usufructo se
repartía entre los miembros de las comunidades.
El neolítico en Europa tuvo como expresión
2.2. Edad de los Metales
2.2. arquitectónica las construcciones megalíticas,
Surgió en el cercano oriente y se extendió de destacando el Stonehenge en Inglaterra.
modo discontinuo a otras regiones.
 A. Edad de Cob re . Se utilizó simultáneamente el uso de la piedra y los metales. En este
Cobre
periodo se consolidaron los cambios desarrollados durante la Revolución neolítica, por
ejemplo:
- La sociedad con jerarquías sociales: conformada por sacerdotes, artesanos, campesinos.
- El surgimiento del Estado organizado por los sacerdotes (teocracia).
- Se inició la Revolución urbana (ciudades) en Jericó (Jordania), Catal Hüyük (Turquía), etc.
- Aparición del comercio a raíz de la producción generada por los artesanos en torno a las
ciudades.
B. Edad de Bronce. Esta aleación mejoró herramientas y armas. El uso y la necesidad de
este material motivaron una expansión comercial a diversas regiones, como en Egipto,
Mesopotamia, India, Creta, Grecia, etc. Se expandió la Revolución urbana y fue inventada la
escritura. Así, surgieron las civilizaciones esclavistas.
C. Edad de Hierro. Este metal desplazó al bronce y su uso fue expandido por los hitit
hititas.
as. En
Europa destacaron, las fases de Hallstat (Austria)  y La Tene (Suiza). Su uso consolidó los
imperios esclavistas

3.
SeEdad
Einició
dad Antigua
Ant
conigua
la aparición de la escritura (3300 a.C.) y culminó con la caída del Imperio romano
de Occidente (476 d.C.). En este periodo se manifestó una nueva formación economía social:
el esclavismo.
3.1. Mesopotamia. En esta región (actual Iraq) surgieron civilizaciones que se desarrollaron
entre los ríos Éufrates y Tigris. 
3.
3.1.
1.1.
1. De
Desarrol
sarrollo
lo histór
hi stórico
ico
sum erios. Constituyeron la primera civilización de
Los sumerios.
la región. Se ubicaron en Sumer (zona sur)
organizándose en ciudades-estados, tales como Uruk,
Ur, Lagash, etc. En su interior construyeron zigurats
(templos) y los enci-patesi (rey sacerdote) ordenaban la
vida local. También
la contabilidad, inventaron
el arado la escritura cuneiforme,
y la rueda.
r ueda.

CEPRE-UNI HUMANIDADES 54
 

CICLO INTENSIVO ESCOLAR NACIONAL Material de Estudio Nº 1 

  Los acadios.  Destacó Sargón I, quien fundó el Imperio acadio, el primero de la


antigüedad. 

   Primer Imperio babilónico. La ciudad


de Babilonia fue el centro político,
comercial y cultural de la época.
Hammurabi aplicó normas estatales
(Código de Hammurabi), como la Ley del
talión y para consolidar el imperio impuso
el culto a Marduk (Dios de la guerra). Este
imperio cayó por una serie de invasiones
lideradas por los hititas y los
l os kasitas (tribus
de origen indoeuropea). 
  Imperio asirio.  Sobresalió
Asurbanipal, quien logró la máxima
expansión territorial (observar mapa), a la
vez organizó la biblioteca de Nínive, la más antigua de la historia. Fueron vencidos por la
alianza medo-babilónica de Ciaxares y Nabopalasar.
  Se
Segundo
gundo Impe rio babilónico. Nabucodono
Imperio Nabucodonosor
sor II fue el emperador más resaltante. Este
mandó a construir el templo de Marduck. Fue conquistado por los persas bajo el mando de
Ciro “el Grande”. 
3.
3.1.
1.2.
2. Aspectos
Aspectos cul cultur
turales
ales
   Ar
 Arqu
quititect
ectur a. Utilizaron adobe y ladrillo e introdujeron arcos y bóvedas. Construyeron
ura.
palacios y templos (zigurats).
  Escultura. Destacó la del rey Gudea (sumeria) y los toros alados de Khorsabad (asiria).
  Religión. Fue politeísta y antropomorfa. Sus principales dioses fueron Ishtar, Assur (dios
asirio), Marduk (shamash).
  Escritura. Fue de tipo cuneiforme, descifrada por Rawlinson 
  Ciencias. En ingeniería, promovieron canales, diques y reservorios. Así mismo,
desarrollaron el calendario lunar, establecieron la división de la hora y los días de la semana,
observaron los eclipses y en matemática dieron a conocer la numeración sexagesimal.
  Literatura. Destacó la epopeya del Gilgamesh, en donde se narra el diluvio.

3.2. Egipto. Se desarrolló al noreste del África, a lo largo del valle del río Nilo. La inundación
aportaba limo (fertilizante) que permitió una mayor productividad agrícola. La región estuvo
conformada por el Bajo Egipto (norte) y Alto Egipto (sur).
3.2.1 Periodos históricos
ást ic o. La población estuvo organizada en nomos o Estados locales autónomos
 A. Pred in ástic
que vivían en situación tribal, a lo largo del Bajo y Alto Egipto.  
B. Imperio arcaico o thinita.  Surge a partir de la unificación la ciudad de Thinis, las dos
regiones de Egipto fueron unificadas por primera vez con Narmer considerado el primer faraón
de la dinastia I.
C. Imperio antiguo o menfita . Tuvo importancia la ciudad de Menfis, en el bajo Egipto.
Destacó la pirámide escalonada del faraón Zosser en Saqara, además, se construyeron las
pirámides de Gizeh y la Esfinge. Se inició la crisis por la pérdida central de poder sobre los
nomos.

CEPRE-UNI HUMANIDADES 55
 

CICLO INTENSIVO ESCOLAR NACIONAL Material de Estudio Nº 1 

D. Imperio medio ebano. Desde la ciudad de Tebas (Alto Egipto), Mentuhotep II logró la
m edio o ttebano
segunda unificación. Se estableció el culto a Amón- Ra, divinidad imperial. El periodo llegó a
su fin con la invasión de los hicsos.

E. Imperio
Imperio nuevo o n eotebano. Desde Tebas, Amosis
neotebano
I expulsó a los hicsos y, posteriormente, se inició el
esplendor y máximo desarrollo del esclavismo en
Egipto. Destacaron los siguientes faraones:
   Tutmosis III. Logró la máxima
m áxima expansión territorial
de Egipto.
    Am
 Amenoenofifiss IV. Realizó una reforma religiosa
imponiendo el monoteísmo (dios Atón), contra el poder
sacerdotal. Esta se realizó desde la ciudad de Tell el
 Amarna.
   Tutankamon . Restableció el politeísmo. En la batalla de Qadesh se hizo uso
del caballo y carro de guerra como
   Ramsés II. Firmó el Tratado de Paz de Qadesh con elemento tecnológico del ejército
los hititas, considerado el más antiguo del mundo. faraónico.
Posteriormente esta civilización decayó por las
invasiones de varios pueblos como los asirios, persas, griegos y romanos.
3.
3.2.
2.2
2 Aportes cult
c ulturales
urales
  Escritura. Jeroglífica, usada por la nobleza y
hallada en los templos y tumbas, Hierática, de carácter El speos fue una edificación funeraria
subterránea, combinación de templo e
resumido usado por los escribas y demótica, escritura hipogeo, perforada en la roca. Abu
tardía usada en actividades comerciales.  Simbel (Siglo XIII a. C.) es un ejemplo
  Ciencias. En matemática desarrollaron la geometría y la de ello
decimal. 
numeración decimal. 
   As
 Astr
tr on
onomomía.ía. Desarrollaron solar.  
 Desarrollaron el calendario solar.
   Ar
 Arqu
quitit ect
ectur
ura.
a. De carácter religioso, monumental y con sentido de eternidad. Destacaron las
tumbas como las pirámides (Saqqara, Keops, Kefren, Micerino), mastabas y los hipogeos
(subterráneos)) y los templos de Karnak, Luxor y Abu Simbel.
(subterráneos
  Religión. Fue politeísta, creían en la vida después de la muerte y el juicio de las almas.
Destacaron dioses como Amon-Ra, Osiris, Seth e Isis. Isis.  
  Medicina. Realizaron
Realizaron   la momificación de sus muertos, conocieron sobre traumatología y
ginecología.
ginecología. 

  El libro de los muertos.
  Literatura.
  Escultura. Los colosos de Memnon, el Escriba sentado, la Esfinge. 
Esfinge.  
  Orfebrería. Destaca el tesoro de la tumba
t umba de Tutankamon excavad
excavadaa por Howard Carter en 1922.

EVALUACIÓN

1. Los primeros homínidos se dedicaron a


actividades de subsistencia en zonas 2. La continuidad de la economía de
focalizadas como el ________, mientras que los subsistencia en el mesolítico se relaciona con la
homínidos más avanzados se desarrollaron en forma de vida seminómade, pero con el
un contexto de ________. surgimiento de la domesticación de plantas, la
A) istmo de Beringia – glaciación vida social se organizó en torno a
B) valle del Rift – glaciación A) cuevas o grutas. B) aldeas agrícolas.
C) aldeas horticultoras.
C) África
D) desierto del Sahara
oriental  – enfriamiento
 – variación climática global D) aldeas sedentarias. E) comunidades agrícolas.
E) Cercano oriente – era de los metales

CEPRE-UNI HUMANIDADES 56
 

CICLO INTENSIVO ESCOLAR NACIONAL Material de Estudio Nº 1 

3. En el neolítico, la Revolución agrícola modificó 7. En los inicios de la civilización egipcia se


la organización económica y social del hombre. construyeron tumbas como ___________ y en
Como resultado de lo anterior surgió la parte final de esta importante cultura las
inmediatamente tumbas construidas fueron ______________.
A) el Estado teocrático
t eocrático militar. A) las pirámides – los hipogeos
B) el excedente productiv
productivo.
o. B) las mastabas – los speos
C) la Edad de bronce. C) los hipogeos – las mastabas
D) el comercio y la navegación. D) los hipogeos – las pirámides
E) De
4. las las
civilizac
civilizaciones
iones proposiciones
siguientes esclavistas. sobre la Edad E) las mastabas – los speos
de los metales, marque la alternativa correcta 8. Respecto a sus aportes culturales
I. La consolidación de los imperios esclavistas se de Mesopotamia identifique el valor de verdad
produjo en la Edad de hierro. (V o F) de los siguientes enunci
enunciados
ados
II. La Revolución urbana y la invención de la I. Entre sus principales dioses
escritura corresponden a la Edad de bronce destacaron Marduck e Ishtar.
III. La civilización caldeo  –  asiria y egipcia II. Los zigurats fueron templos donde vivió
surgieron en la Edad de cobre el enci - patesi.
A) I B) Solo II C) I y II D) IIII y III E) I, II y III III. Las esculturas princip
principales
ales fueron los colosos
de Memnon.
5. ¿Cuál fue el evento histórico que hizo posible IV. La escritura fue de tipo cuneiforme y
el tránsito de la forma de vida de subsistencia, descifrada por Rawlinson.
dependiente absoluto a la naturaleza, al estilo de A) VVFF B) VFVV C) FFVV D) FFVV E) VVFV
vida civilizada?
9. Uno de los principales
A) El fin
B) El surgimiento de del
de la época la agricultura
Pleistoceno periodos de Mesopotamia fue el primer imperio
C) La invención de la escritura cuneiforme babilónico. Señale, lo que corresponda, al
D) La difusión del hierro a Cercano oriente periodo mencionado.
E) La consolidación del sedentarismo I. Nabucodonosor II ordenó construir el templot emplo
de Marduck.
6. Durante el Imperio nuevo o Neotebano se II. La invasión de los kasitas culminó con este
alcanzó el apogeo territorial y cultural de periodo.
Egipto. Identifique, lo que corresponda, al III. Hammurabi estableció un código de leyes.
periodo mencionad
m encionado.o. IV. Alcanzó la máxima expansión con
I. Tutankamon reestableció el politeísmo en Asurbanipal.
Egipto. A) I, III y IV B) II Y III C) I, II y IV
II. La máxima expansión fue alcanzada D) Solo I y III E) II, III y IV
por Ramses II. 10. La Edad antigua inició con la aparición de la
III. El monoteísmo fue establecido desde la escritura aproximadamente 3300 ac y
ciudad decon
IV. Inició Amarna.
la unificación realizada por Menes culminó con __________________.
A) la caída de Roma de oriente
o Narmer. B) la invención de la metalurgia
A) I, II y III B) I y III C) I, II y IV C) el surgimiento de la Edad de hierro.
D) Solo II y IV E) II, III y IV D) la caída de Roma de occidente
E) el fin de la cultura greco romana.
 
TEMA II. DEL POBLAMIENT
POBLA MIENTO
O AMERICANO HASTA EL DESARROLL
DESARROLLOO REGIO
REGIONAL
NAL

1. Poblami
Poblamiento
ento americano

CEPRE-UNI HUMANIDADES 57
 

CICLO INTENSIVO ESCOLAR NACIONAL Material de Estudio Nº 1 

Fueron migraciones de cazadores-recolectores, agrupados en bandas, hacia el continente


americano en procura de elementos de subsistencia. Estas bandas, llegaron desde diversos
puntos de Asia durante el paleolítico superior y, paulatinamente, se adaptaron a las diversas
realidades ecológicas de América.
Proyecto genoma humano.
humano . El hallazgo del niño Anzick en Montana (EE.UU.), poblador de más de 11
mil años, comprueba que el poblamiento inicial se dio en Beringia, ya que su ADN ha confirmado que
los indígenas americanos actuales son descendientes directos de los primeros pobladores de América.
 Así Beringia es considerada
considerada la principal ru
ruta
ta de poblamiento. 
poblamiento.

1.
1.1
1 Princi pales teorías de la procedencia
proc edencia del homb
hombrere americano
A inicios del siglo XX comenzaron a plantearse, desde la antropología, diversas teorías que
explicaban la existencia de población nativa en América precolombina a partir de procesos
migratorios. En la actualidad, dos son las teorías que presentas fundamentos consistentes.

2. Precerámico
Precerámico andino
andi no
Es el periodo que abarca desde el momento inicial de adaptación de las bandas paleolíticas
hasta el surgimiento de la vida productiva y la civilización en los Andes centrales.
2.1. Peri
2.1. Periodo
odo lít
lític
ico
o (12 000 –8000 a.C.)
Los cazadores y recolectores practicaron una economía de subsistencia en los diversos
espacios

ecológicos
  la costa, ocuparonandinos.
valles, Por ejemplo,
lomas
lomas, en para cacería y el litoral para aprovech
, pampas aprovechar
ar los
recursos marinos.
  la sierra, ocuparon valles intera
interandinos
ndinos y mesetas log
logrando
rando acceder a la recolecció
recolecciónn de
vegetales, la caza de cérvidos y, especialmente, de camélidos.
Durante este periodo, que se ubica en el contexto de fines del Pleistoceno e inicios del
Holoceno, podemos resaltar los principales hallazgos encontrados.

HALLAZGO UBICACIÓN CARACTERÍSTICAS


OGICO 
 ARQUEOLOGICO
 ARQUEOL
Piquimachay 
Piquimachay  Instrumental lítico más antiguo y, por tanto, evidencia la
Ayacucho
(Fase Ayacucho) 
Ayacucho)  primera presencia humana pleistocénica en nuestro territorio.
Utensilios de piedra usado para raspar escamas del pescado,
Huaca Prieta La Libertad trozos de tejido
calibrados en másde junco
de 13restos de plantas
000 años y animales
antes del presente.marinos,
Puntas líticas (puntas Paiján), asociadas al taller lítico de
Tradición Lambayeque
Chivateros. En Paiján (La Libertad) también se descubrieron
paijanense hasta Lima
entierros humanos.
Pinturas rupestres con escenas
Toquepala Tacna
de cacería colectiva o Chaku
Chaku..
Restos óseos enterrados con
evidencias de ritos funerarios
Lauricocha Huánuco (ideas mágico-religiosas) y
pinturas rupestres donde se
aprecia la cacería selectiva.

2.2
2.2.
Se .inició
Peri
Periodo
odo
conarcaic
arcaicoo (8000
(8000 –1500dea.C.)
la domesticación  
plantas y animales, así como el mayor desarrollo del
marisqueo, que trajo consigo el paulatino proceso de sedentarismo. Presenta dos fases:

CEPRE-UNI HUMANIDADES 58
 

CICLO INTENSIVO ESCOLAR NACIONAL Material de Estudio Nº 1 

2.2.1. Arcaico inferior (8000 –3000 a.C.)


En esta fase dieron las primeras formas de producción de alimentos mediante la horticultura
y el pastoreo; esto permitió el abandono paulatino del nomadismo.
Por otra parte, en la costa, los abundantes recursos marinos condicionaron la aparición de las
primeras formas de ocupación aldeana en la costa. Por otra parte, los cazadores de la región
andina se convirtieron gradualmente en pastores y abandonaron la trashumancia.

HALLAZGO UBICACIÓN  CARACTERÍSTICAS  


CARACTERÍSTICAS
Nanchoc Se encontraron restos de Muestran las evidencias
Cajamarca calabaza y maní. más antiguas de la
Guitarrero II Restos de frejoles y pallares y horticultura en el
Ancash Precerámico.
tejidos con fibras vegetales.
Encontró restos de chozas, con actividad marisquera y
Chilca  
Chilca Lima
horticultora.
Santo Domingo Se hallaron flautas de hueso, redes para la pesca y viviendas
Ica
semisubterráneas.
Se hallaron restos óseos de llamas en corrales y un alto nivel de
Telarmachay Junín su consumo. Se considera el domesticador de camélidos más
antiguo.
Piquimachay II Se encontraron huesos y corrales de cuyes, considerado el
Ayacucho
primer cunicultor de los Andes.
2.2.2. Arcaico superior (3000  – 1500 a.C.)
Se caracterizó fundamentalmente por el inicio de la economía productiva (Revolución
agrícola). Se masificó y extendió cultivos como la papa y algodón; también se domesticó el
maíz. Este avance trajo cambios sociales como el sedentarismo y la aparición de
comunidades agrícolas (los primeros ayllus). Paralelamente, el desarrollo de la domesticación
de animales permitió la presencia de la ganadería, especialment
especialmentee de camélidos.

Los excedentes productivos posibilitaron la aparición de formas complejas de organización


de vida económica que implicaron la jerarquización social (élite y trabajadores), así como la
especialización artesanal que produjo la Revolución urbana, es decir, el surgimiento de la
ciudad.

HALLAZGO UBICACIÓN   CARACTERÍSTICAS


Presenta arquitectura monumental con plaza hundida de tipo circular
Sechín Bajo Ancash
y cuadrangular. Fue utilizado como cementerio de la élite.
Fue un centro ceremonial que consumía recursos marinos y realizó
Bandurria Lima
intercambios con Caral (recurso marinos-redes de pesca).
Se ubicó en el valle de Supe. Contiene arquitectura ceremonial con
Caral Lima connotaciones urbanas debido a su producción permanente de redes
de pesca. Asimismo, se hallaron flautas, estatuillas, quipus y shikras.
Huaca Prieta La Realizó tejidos de algodón sin telar y mates pirograbados,
Libertad relacionados con la agricultura.
Templos con fogones donde se halló escultura religiosa: las manos
Kotosh mito Huánuco
cruzadas.
En el Arcaico superior, las élites, especialistas en el manejo del tiempo, mediante calendarios
solares, organizaron las jefaturas sacerdotales,
sacerdotales, asumieron el control de la población y de sus
excedentes, lo cual permitió la edificación de construcciones arquitectónica
arquitectónicas.
s.

CEPRE-UNI HUMANIDADES 59
 

CICLO INTENSIVO ESCOLAR NACIONAL Material de Estudio Nº 1 

Así aparecieron los Estados prístinos (Estado liderado por una jefatura sacerdotal) y la
Revolución urbana.
urbana. Estos fueron los fact
factores
ores determinan
determinante
te para el surg
surgimiento
imiento de la
civilización andina.
3. Altas Cultur as de
dell Anti
Antiguo
guo Perú
Es el periodo donde se fortalecieron los elementos culturales como la tecnología que permitió
un mejor dominio del medio geográfico y expresiones artísticas de alta calidad. El origen de
las Altas culturas fue debatido a través de las siguientes teorías:
Inmigracionista ( Autor: Max Uhle)    Auct
 Au ctoc
octo
toni
nism
smo
o ((Au
Auto
to r: J.C. Tel lo ) 
Tello
La Alta cultura andina tiene una fuerte La civilización andina se formó por la
influencia de Centroamérica. influencia amazónica en la región de los Andes centrales.
 Aloc
 Al octo
to nist
ni sta
a (Au
(Auto
to r: F. Kau
Kauffff man
mann)
n)   Hologenista (Autor: Lui s G. Lumbreras)
Lumbreras)  
La civilización andina proviene de la cultura La civilizaci
civilización
ón andina fue resultado de la síntesis de
Valdivia (Ecuador). factores externos y principal
principalmente
mente internos.

En cuanto a la periodización de las Altas culturas andinas se utiliza el siguiente esquema.

Horizonte Intermedio Horizonte medio 


medio  Intermedio Horizonte
Según Jhon Rowe 
Rowe 
temprano  
temprano temprano  
temprano tardío  
tardío tardío  
tardío

Chavín Moche, Chimú


Civilizaciones Wari
Paracas Tiahuanaco Chincha Incas
andinas
Nazca Aimaras

andino 
Según Luis G. Formativo andino  Desarrollo Estados
Imperio wari 
wari  Tahuantinsuyo  
Tahuantinsuyo
Lumbreras  
Lumbreras Regional  
Regional regionales  
regionales

4. HORIZONTE TEMPRANO O FORMATIVO ANDINO

4.
4.1.
1. Horizonte temprano / Formativo An dino
din o
Es el periodo de consolidación de los Estados teocráticos quienes organizaron el trabajo
colectivo de los ayllus. En este periodo además se masificó el cultivo del maíz, se
construyeron templos
ubicamos diversos ende
tipos U ydesarrollos.
se rindió culto
  al dios felino. Este periodo presenta fases donde

Formativo inferior
Formativo Formativo
Formativo medio Forma
Formativo
tivo final
Kunturwasi (Cajamarca), Sechín Paracas (Ica), Vicus
(Ancash), Garagay (Lima), Cantería Chavín (Piura), Pucará (sur
(Lima). andino)

4.1.1 Chavín

CEPRE-UNI HUMANIDADES 60
 

CICLO INTENSIVO ESCOLAR NACIONAL Material de Estudio Nº 1 

  El centro de peregrinación fue el


templo de Chavín de Huantar ubicado
entre los ríos Mosna y Wachecsa, en el
Callejón de Conchucos (Ancash). Su
economía estuvo basada en la actividad
agrícola con irrigación artificial (obras
hidráulicas)
  Su expresión política fue a través de
un Estado de tipo teocrático donde los
sacerdotes usaron sus conocimientos
astronómicos para un mayor control del
excedente productivo y de la población.
La cerámica chavín: forma globular, asa gollete y un solo
  Entre sus manifestaciones culturales pico (imagen izquierda) y de cuencos con decoración
destacaron la litoescultura, como el (imagen derecha).
Lanzón monolítico, la estela de
Raimondi, el obelisco Tello y las cabezas clavas, además presentó cerámica de tipo ceremonial
usada en sus cultos.
4.1.2. Paracas
Ubicada en la provincia de Pisco (Ica). El hallazgo de tumbas presenta dos etapas:
Paracas
Paracas Cavernas
Cavernas Pa
Paracas
racas Ne
Necró
crópoli
poliss
Su centro fue Tajahuana. Destacaron sus tumbas en Su centro fue Topará. Presenta tumbas
forma
pinturade copa invertida
postcocción y su cerámica
cerámic
o fugitiva. a polícroma
Mantiene con rectangulares
la influencia y semi-subterráneas.
cerámica fue pre cocción. Se aprecia La
el
del estilo felínico abandono del patrón felínico por estilos
naturalistas.
4.
4.1.
1.2.
2.1.
1. Manif
Manifestaciones
estaciones cul
cultur
turales
ales paracas
  Textilería. En la fase Cavernas, presenta diseños relacionados al culto
felínico, pero la etapa Necrópolis muestra el mejor desarrollo del arte textil.
  Arquitect
Arquitectura.
ura. Destacó la construcci
construcción
ón de las huacas Soto y Santa
Rosa en su fase Topará.
  Medicina. Momificaban cadáveres, realizaban las trepanaciones y las
deformaciones de cráneos, estas últimas para diferenciar las clases
sociales.

4.
4.2.
2. IIntermedi
ntermedio o ttemprano
emprano
Conocido como el periodo de los Desarrollos regionales, destacó la presencia de teocracias-
militares y el alto desarrollo artesanal, en especial en la alfarería.

4.2.1. Moche
  Ubicación. Su centro principal estuvo en el valle de Moche (La Libertad) y de Sipán
(Lambayeque)
  Economía. De base agrícola, destacaron canales de riego como Ascope y La Cumbre,
además se practicó la pesca en caballitos de totora.
  Cerámica.  Los huacos expresan escenas de su vida, destacaron los huacos retratos,
sexuales y clínicos.
   Arqu
 Ar quii tect ur a. En sus construcciones usaron el adobe. Destacaron las siguientes:
tec t ura
- Huaca del Sol, centro administrativo.
- Huaca de la Luna, centro religioso.
-- Huaca
Huaca Rajada, se se
Cao Viejo, halló la tumba
encontró del señor
la tumba deSeñora
de la Sipán. de Cao.
   Murales. Destacan
Murales.  Destacan La rebelión de las cosas contra el hombre,
hombre , y el Dios degollador o Aia Paec 
Paec 

CEPRE-UNI HUMANIDADES 61
 

CICLO INTENSIVO ESCOLAR NACIONAL Material de Estudio Nº 1 

  Metalurgia.. Trabajaron el oro, la plata y el cobre. Utilizaron técnicas de repujado y laminado.


Metalurgia

4.2.2. Nazca
   Ubicación. En el valle del rio Grande donde se encuentra su centro principal Cahuachi.
Su estado fue teocrático-militar. 
   Economía.  Para la agricultura utilizaron sistemas de riego denominados galerías
filtrantes que captaban el agua de la napa freática.

al  vacío.
Cerámica. Destaca la policromía, su carácter pictórico y el uso de la técnica del horror
  Geoglifos. Los dibujos y las líneas gigantescas en las pampas de Nazca y Palpa. En
su interpretación destaca la idea de un calendario astronómico con fines agrícolas (María
Reiche) y la de caminos ceremoniales.

Evaluación

1. Determine el valor de verdad (V) o (F) de los escenas de caza de animales y herramientas
siguientes enunciados respecto a las líticas corresponden a
características del proceso de poblamiento A) Toquepala. B) Paiján C) Lauricocha.
americano. D) Pikimachay. E) Nanchoc.
I. Las primeras oleadas migratorias humanas se
dieron a fines del Pleistocen
Pleistoceno.
o. 5. Pikimachay I corresponde a las evidencias
II. Es explicada científicamente por la teoría arqueológicas de instrumental lítico de
asiática y la teoría de noratlántica. cazadores andinos que data del periodo final del
III. Tuvo un único foco de poblamiento, el cual Pleistoceno. La importancia de este resto radica
está ubicado en Europa. en que
IV. Una consecuencia
consecu encia del proceso fue el inicio de A) constata la temprana presencia humana en el
la Revolución neolítica. Perú.
A) VFVF B) FFVV C) FVFF D) VVFV E) VVFF B) comprueba la validez de la ruta noratlántica.
C) demuestra el
2. La especie Homo sapiens ingresó a América origen de la
desde ______________, debido a la necesidad civilización en
de recursos económicos. Este proceso de los Andes.
produjo a finales del _______________. D) es un
A) un solo foco migratoria de Asia  – Paleolítico antecedente
inferior inmediato de la
B) todos los continentes – Paleolítico superior Revolución
C) varios focos migratorios de Asia  – Paleolítico urbana.
superior E) el
D) el continente africano – Paleolítico superior poblamiento
E) el continente euroasiático – Neolítico
euroasiático inicial del Perú
se inició en la
3. El periodo lítico fue el de mayor duración de la costa norte.
etapa del Pre cerámico. Respecto a lo anterior,
indique las características de dicho periodo. 6. En periodo Intermedio temprano es
I. El Homo sapiens se adaptó a varios denominado el de grandes maestros artesanos,
ecosistemas andinos. debido a que destacó_____________________,
II. Ubicamos los restos arqueológicos más siendo la cultura ___________ aquella que
antiguos. destacó en este estilo.
III. La economía estuvo sustentada en la A) la producción alfarera, por ejemplo, la
agricultura incipiente. cerámica escultórica - Moche
A) III B) I y II C) I y III D) II y III E) I, II y III B) el uso del laminado y martillado en la
metalurgia – Paracas necrópolis
4. Los hallazgos arqueológicos de restos C) la construcción de centros urbanos
humanos de gran antigüedad, arte rupestre con ceremoniales – Chavín

CEPRE-UNI HUMANIDADES 62
 

CICLO INTENSIVO ESCOLAR NACIONAL Material de Estudio Nº 1 

D) la elaboración de esculturas líticas religiosas A) VFV B)


 – Paracas cavernas VVV C) FFF
E) el desarrollo de la cerámica polícroma  –  D) VVF E)
Mochica FVF
7. El centro ceremonial más importante de la
cultura nazca fue 10. La
A) Huaca de la luna. B) Topará. C) Cahuachi.
D) Viñaque. E) Tambo de Mora.
8. Es una característica propia del desarrollo de
la cultura nazca.
A) La cerámica antropomor
antropomorfafa monócroma.
B) El impulso de hidráulica subterránea.
C) El mayor auge de la metalurgia.
D) La elaboraci
elaboración
ón de fardos ffunerarios
unerarios..
E) La elaboración de estelas y obeliscos.
consolidación del ayllu, la difusión inicial del
9. De las siguientes proposiciones determine el maíz y formación de la primera cultura
valor de verdad (V o F) según corresponda. panandina corresponde al periodo
I. Paracas caverna expresó su dominio político a A) Arcaico superior. B) Intermedio tardío.
través de un Estado teocrático.
t eocrático. C) Horizonte medio. D) Intermedio temprano.
II. La técnica del horror al vacío fue utilizada en E) Horizonte temprano.
la cerámica Moche.
III. Los acueductos en Nazca sirvieron para
aprovechar el agua subterránea.
 

TEMA III. DE GRECIA HASTA EDAD MEDIA 

1. Grecia. Se desarrolló en la Península balcánica, en el extremo sur de Europa. 

1.1. Evolución histórica


 A. Etapa
Et apa preh ic a. El de las primeras civilizaciones en Grecia
p reh elén ica.

Civilización cretense o minoica. Se asentó en la Civilización mi cénica o aquea


aquea.. Destacaron
isla de Creta. El palacio de Cnosos fue su centro las ciudades de Micenas y Tirinto. Las
administrativo y sus reyes fueron denominados rivalidades comerciales en el Egeo,
Minos. Controlaron
estableciendo las rutaso Estado
la talasocracia del Mar Egeo desencadenaron
marítimo- la Guerra
a.C.). En arquitectura, destacade la
Troya (1250
Puerta de
comercial. Fueron invadidos por los aqueos. leones. Esta civilización llegó a su fin con
los leones. 
la invasión de los dorios.
B. Etapa helénica

  Periodo oscuro. Se produjo las invasiones de pueblos indoeuropeos (jonios, dorios y


eolios), estos pueblos dieron origen a la Grecia clásica.  

  Periodo arcaico. Los griegos se organizaron en Polis o ciudades estados. El aumento


demográfico generó la colonización griega en el Mediterráneo. 

  Periodo clásico. Se llevó a cabo las guerras médicas, el Siglo de Pericles, y la guerra
del Peloponeso. La civilización griega alcanzó su máximo apogeo destacando las principales
polis. 
Esparta Atenas

CEPRE-UNI HUMANIDADES 63
 

CICLO INTENSIVO ESCOLAR NACIONAL Material de Estudio Nº 1 

Fue un Estado dirigido por una aristocracia Fue cuna de la democracia. La actividad
terrateniente. Se consideraron descendientes de los más importante fue el comercio. Fueron
poderoso.. 
dorios. Constituyeron un ejército terrestre poderoso descendientes de los jonios. Destacaron
por su poderosa flota naval. 
naval.  

Las guerras médicas El Siglo de Pe


Peric
ricles
les Las guerras del Pe
Pelopo
loponeso
neso
Fue el enfrentamiento entre Atenas logró auge político y Conflicto entre Esparta y Atenas por
persas y griegos por el control cultural. Se consolidó la la hegemonía en Grecia. Atenas
comercial en el Egeo. Los persas democracia y se lideró la Liga de Delos y Esparta la
fueron vencidos en las batallas construyeron distintas obras Liga del Peloponeso. Esta guerra
de Maratón, Salamina y Platea. públicas como el Partenón y provocó la crisis de las polis griegas
Erection.

C. Macedonia
Macedonia y su Imperio
  Filipo IIII.. Organizó un poderoso ejército: la
l a falange macedónica. Su objetivo fue conquistar
y unificar Grecia y creó la liga de Corinto para la conquista del Imperio Persa.
   Al
 Alejan
ejandr
dro o Magn o. Lideró la Liga de Corinto conquistando al Imperio persa extendió su
Magno.
dominio hasta la India, lo cual permitió la formación de un gran Imperio. Estas conquistas
unieron Occidente y Oriente fusionándose ambas culturas, lo que dio origen al Helenismo. En
este periodo Alejandría remplazó a la ciudad de Atenas como foco cultural.

1.
1.2.
2. A
AbarcóAspecto
spectos s cul
cultur
las ciencias, turales
ales   filosofía, letras, etc. Su máximo aporte fue la democracia. Es
artes,
considerada la base de la civilización occidental.
  Escultura.  Representaron figuras humanas, llenas de vida, movimiento y acción.
Destacaron Fidias y Mirón.
   Ar
 Arqu
quii tect ur a. Utilizaron columnas destacando los estilos dórico y jónico. Construyeron
tec t ura.
palacios, templos, teatros, gimnasios, plazas públicas, estadios (sede de las olimpiadas),
hipódromos, bibliotecas, museos, etc.
  Matemática. Destacó Pitágoras y Euclides, Padre de la geometría.
Matemática. Destacó
   Destacó Heródoto con su obra Los nueve libros de historia.
Historia. Destacó
Historia. historia.
  Medicina. Destacó
Medicina.  Destacó Hipócrates, Padre de la medicina.
  Filosofía. Destacaron
Filosofía.  Destacaron los filósofos Sócrates, Platón y Aristóteles.
  Física. Destacó
Física.  Destacó los aportes de Arquímedes.
  Química. Destacó
Química.  Destacó los aportes de Demócrito, quien planteó sobre el átomo.
2. Roma. Se desarrolló en la Península itálica, en torno al río Tiber.
Sociedad . Se organizó de la siguiente manera:
2.1.. Sociedad.
2.1
Patricios.  Fueron la nobleza aristocrática quienes controlaban los latifundios y esclavos. La
Patricios. 
pertenencia a una gens (familia) le brindaba privilegios.
Plebeyos. En
Plebeyos.  En un principio fueron la mayoría social carente de derechos. Posteriormente ganaron
sus derechos a ser reconocidos ciudadanos romanos.
Clientes. Plebeyos pobres prote
protegidos
gidos por los patricio
patricios.
s.
Esclavos.. Representados por los prisioneros de guerra o comprados en los mercados. Desde el
Esclavos
periodo republicano romana constituyeron la mayoría social.

2.2. Desarrollo histórico


2.2.1.. Mon
2.2.1 Monarq
arquía
uía (753 -509 a.C.).
a.C.).
Estuvo gobernada
legendaria, por reyes
destacando Rómuloquequien
pertenecieron a dos
fundo Roma; y ladinastías.
dinastía La dinastía
etrusca latina o
o histórica,
destacando Tarquino Prisco en cuyo reinado se construyó la Cloaca máxima. 
CEPRE UNI HUMANIDADES 64

CICLO INTENSIVO ESCOLAR NACIONAL Material de Estudio Nº 1 

2.2.2. Repú
2.2.2. Repúbl
blic
icaa (509-
(509-27
27 a.C.)
a.C.)
Régimen político que permitió mayor intervención
i ntervención de los ciudadanos: participando y eligiendo
funcionarios públicos.
públicos. Las institu
instituciones
ciones del poder (Sena
(Senadodo y magistratura
magistraturas)
s) estuvieron en
manos de patricios.
 A. Org anizaci
ani zación
ón po
polít ic a. Se organizó a través del Senado, cuya función fue legislar y
lític
asesorar a los cónsules; las magistraturas, eran funcionarios como el Cónsul, Dictador,
Tribuno de la plebe, Censor, Pretor, Cuestor, Edil y finalmente las asambleas o comicios.
Conflict os y reformas. 
B. Conflict
  Huelga del monte ssacro.acro. los plebeyos lucharon por la obtención de sus derechos, los
cuales fueron reconocidos inicialmente con la Ley de las XII Tablas. Así al aprobarse distintas
leyes, estas se convirtieron en la base del Derecho romano.
  Guerras pú púnicas
nicas (2
(246
46 y 216 a.C.
a.C.).
). Con
Conflicto
flicto entre ro
romanos
manos y cartagineses por el control
del mediterráneo occidental. Destacó la batalla de Zama.
  Reformas de los hermanos Graco. Tiberio y Cayo Graco promovieron dos importantes
leyes: la Agraria y la Frumentaria.
  Guerra civil. Enfrentamiento entre el Partido popular y el Partido senatorial.
  Primer triunv
triunvirato.
irato. IIntegrado
ntegrado por Ju
Julio
lio César, Pompeyo y Craso. Se enfrentaron Julio
César y Pompeyo en la batalla de Farsalia. 

  Segundo
Octavio triunvirato.
y Marco Integrado
Antonio en la batallapor
de Oct
Octavio,
avio,.  Marco Antonio y Lé
Actium Lépido.
pido. Se enfrentaron

2.2.3. El Im
2.2.3. Imper
perio
io (27 a.C.-
a.C.- 476 d.C.)
El gobernante principal fue el emperador, quien concentró los poderes políticos,
administrativos, religiosos y militares.

 Alto
 Al to imper
im perio
io o Pr
Prin
in ci
cipad
pado
o Baj
Bajoo iimp
mp eri
erioo o Dom
Domin
inado
ado
Fue la etapa de máximo apogeo del Imperio. Etapa de crisis producto del fin de la expansión y la
  Au
 Augu
guststo.  Siglo de Augusto (Pax romana).
o. Siglo crisis del esclavismo.
Florecimiento cultural de Roma.   Constantino “el Grande”. Decretó la tolerancia
  Nerón. Incendió Roma y persiguió a los religiosa (Edicto de Milán).
cristianos.   Teodosio
Teodosio.. Dividió el Imperio entre sus hijos
  Tito.
Tito. Destruyó
 Destruyó Jerusalén y llevó a cabo la Honorio (parte occidental) y Arcadio (parte oriental
oriental).
).
diáspora
Palestina). (expulsión de los judíos de Además, decretó
el cristianismo fueelconvertido
Edicto de en
Tesalónica con el cual
religión oficial.
  Trajano
Trajano.. Logró la máxima expansión Rómulo Augústulo.
Augústulo . Fue derrocado por Odoacro,
territorial del Imperio. rey de los hérulos (476 d.C.). Esta etapa terminó con
las invasiones bárbaras.
2.
2.3.
3. A
Apor
portes
tes cul
cultur ales. Fueron herederos de la cultura griega 
turales.
CEPRE UNI HUMANIDADES 65

CICLO INTENSIVO ESCOLAR NACIONAL Material de Estudio Nº 1 

  Derecho : Principal aporte. Se aprobaron


leyes como la ley canuleya que permitió el
matrimonio entre patricios y plebeyos, ley
licinia que permitió a los plebeyos acceder al
cargo de cónsul y la ley ogulnia que permitió a
los plebeyos asumir cargos religiosos o
sacerdotales.

  qu ititect
 Arqu
 Ar
Vesta, electur
uraa: construyeron
Anfiteatro el Templo
Flavio (coliseo romano)dey
el circo máximo. Además, construyeron
acueductos, arcos de triunfo, etc.
  Poesía: destacaron Virgilio, autor de La
Eneida, Horacio y Ovidio.
  Historia: destacó Tito Livio autor de la
historia de Roma.
  Filosofía: destacaron Seneca y Marco La arquitectura y la ingeniería evidenciaron el poderío
Aurelio. im erio
erio roma
romano
no

3. La Edad
Edad m
media
edia
Comprende desde la caída del Imperio romano de Occidente, en 476 d.C. con el
derrocamiento del emperador Rómulo Augústulo, por parte de los hérulos hasta la caída del
Imperio romano de Oriente en 1453, cuando los turcos otomanos tomaron Constantinopl
Constantinopla.
a.
3.
3.1.
1. La
Lass iinvasion
nvasiones
es bárbaras
Se denominaban bárbaros, a los pueblos que vivían en los alrededores del Imperio romano.
Así tenemos a los germanos, hunos, etc. Se produjeron dos formas de invasión, que fueron
las siguientes:
Pacífica (Siglo III  IV)
 – Viol
Violenta
enta (Siglo IV  V)  –

Debido a la crisis del esclavismo, los romanos El incremento de la población bárbara, la


permitieron el ingreso de los germanos, a quienes necesidad de tierras y el avance de los hunos
les asignaron labores agropecuarias (sistema del (Atila), agudizó el proceso de invasiones a
colonato) y militares Roma.
.
3.
3.2.
2. Los reinos Germánicos
Surgieron como consecuencia de las invasiones bárbaras y
sentaron las bases de la denominada civilización occidental. ¿
En losSabías que? se
monasterios
Tuvieron como base económica la agricultura, establecieron la conservó la cultura greco – 
monarquía electiva y el catolicismo se convirtió en la religión romana a inicios de la
Edad Media
oficial de estos reinos a través de la conversión de sus
gobernantes.
einos germánicos  
Principaless rreinos
Principale
  Visigodo (España). Vencidos por los árabes.
  Ostrogodo. (Italia), derro
derrocaron
caron a los hérulos y luego vencidos
vencidos por Bizanc
Bizancio.
io.
  Franco (Francia). Vencier
Vencieron
on a los musulmanes en la batalla de Poitiers.
  Anglo y Sajón (Gran Bretaña). Unificado
Unificadoss por los vikingos o normandos (Siglo XI).
  Vándalo (Túnez). Fueron incorporados al Imperio bizantino. 

3.
3.3.
3. E
Ell Imperio bizantino
Tuvo como
controló base del
el paso el Imperio romano
comercio de la de Oriente
seda siendode
y especies suEuropa
capital Constantinopla.
a Asia Entre susEsta ciudad,
principales
emperadores tenemos a:
CEPRE UNI HUMANIDADES 66

CICLO INTENSIVO ESCOLAR NACIONAL Material de Estudio Nº 1 

- Justiniano (527-565). LLogró


ogró el apogeo im
imperial,
perial, rec
recopiló
opiló la legislación ro
romana
mana en el Corpus
Iuris Civilis y ordenó la construcción de la basílica de Santa Sofía.
- Constantino XI XI (1449-1453). Último emperad
emperador or quien murió en la toma ddee Constantino
Constantinopla
pla
por los turcos otomanos liderados por el sultán Mohamed II.

Entre sus principales manifestaciones culturales


tenemos:
- Religión.  Fundaron la iglesia ortodoxa desde
1054 cuando se produjo el Cisma de Oriente
que consistió en la ruptura con el Papado de
Roma. 
- Arquitectura: Sus principales referentes
arquitectónicos fueron la basílica de Santa Sofía
y el hipódromo. Además, realizaron mosaicos
como expresiones artísticas.

3.
3.4.
4. Los árabes y el iislam
slam
La expansión del islam se originó en la Península arábiga entre los valles del Yemen y Hedjaz,
regiones aptas para la agricultura y el desarrollo del comercio.

3.4.1. Periodificación
Pre islámico
Los árabes vivían organizados
Islámico
Se inicia con la Hégira, que fue la huida de Mahoma de la Meca
en clanes politeístas (beduinos) a Medina (622 d.C). Este evento marcó el inicio del calendario
en permanentes rivalidades. Su musulmán. El logro del profeta fue la unificación de la Península
mayor actividad económica fue arábiga a través del islam. Tras la muerte de Mahoma los que
el comercio. asumieron el gobierno se les denomino califas

3.
3.4.
4.2.
2. Evoluci
Evoluciónón hi
histór
stór ica del mundo
mund o árabe
El califa dirigió una monarquía absolutista, teocrática y militar, organizando el Imperio
administrativamente en provincias o emiratos. Fueron vencidos en el siglo X, por los turcos
selyúcidas. Los Califatos fueron los sig
siguientes:
uientes:

Califato Ortodoxo Califato Omeya Califato Abasida


Su capital fue Medina e Su capital fue Damasco, Su capital fue Bagdad. Alcanzaron el
iniciaron su expansión fuera de llegaron hasta la península esplendor de la civilización islámica.
la península arábiga. ibérica alcanzando la Con su crisis surgieron nuevos
Desatacaron Abu Beker y Alí. máxima expansión. califatos como el Fatimí (Egipto) y
Córdoba (España).

3.
3.4.
4.3
3 Aspectos cul turalestur ales
Los árabes sintetizaron el legado bizantino, hindú y persa, que fue introducido a Europa
Occidental mediante España, destacaron en los campos siguientes:
  Literatura. La obra más sobresaliente es Las mil y una noches.
  Química. Desarrollaron la alquimia 
  Matemática. Desarrollo del álgebra y difundieron la numeración arábiga 
  Medicina. Estudiaron la anatomía humana, destacando Avicena con su obra Canon
de la Medicina. 
   Arqu
 Ar quii tect ur a. Destacan las mezquitas (templos) y los alcázares (palacios militares).
tec t ura.
  Religión. Fundaron el islam que significa sumisión a  Alá. Es de carácter monoteísta,
su libro sagrado es el Corán que contiene la doctrina islámica que regula diversos aspectos
CEPRE UNI HUMANIDADES 67

CICLO INTENSIVO ESCOLAR NACIONAL Material de Estudio Nº 1 

de la vida social, jurídica y religiosa de los musulmanes. Tienen cinco pilares que son: fe,
oración, ayuno, limosna y peregrinación a la Meca.
4. El Imperio carolingio
Tuvo como base el reino Franco. El rey Carlomagno inició la gran expansión, formando el
Imperio carolingio (768-814), con su capital en Aquisgrán. Fue coronado como emperador por
el papa León III, aquello significó la restauración temporal del Imperio romano de Occidente.

4.1.
4.1. Administración
Administ ración imperial
Carlomagno estableció el vasallaje para el nombramiento de autoridades ( vassi dominicci)
quienes se encargaban de administrar de la siguiente manera:
Era la división de mayor jerarquía, conformada por varios condados y marcas al
Ducados
duque. 
mando de un duque. 
Condados Eran las provincias internas administrados por un conde.
Eran las provincias limítrofes o fronterizas, de carácter militar, gobernadas por los
Marcas
marqueses.
El Imperio contaba con los Missi dominici (inspectores o supervisores), quienes vigilaban el
cumplimiento de las leyes denominadas Capitulares, dictadas por Carlomagno y aprobados
por los nobles reunidos en los Campos de Mayo.
4.
4.2.
2. Aspecto cul
cultur
tural
al
Se produjo un lento resurgimiento de la cultura clásica
representado en las letras, ciencias, artes mediante la
educación en escuelas destacando las siguientes:
  Palatina. Funcionó en Aquisgrán, donde se formaba
a los nobles.
  Conventual. Funcionaban en monasterios y
religiosos. 
catedrales formándose religiosos. 
  Parroquial. Funcionaban en las parroquias dirigida a
los campesinos.
4.3. Fin del Imperi
4.3. Imperio o
Luego de la muerte de Carlomagno, los hijos de Luis
el Piadoso, se repartieron el imperio en el tratado de Verdún
(843): Lotario I, el centro (Italia), Carlos el Calvo  la actual
Francia y Luis el Germánico  el este. En este último se
estableció el Sacro Imperio Romano Germánico (Siglo X  – 
1808).
5. El
El feudalismo
feudalis mo
Fue el régimen social basado en la tenencia de tierras
(feudos), relaciones de dependencia personal (vasallaje) y
la confiscación de rentas a los campesinos (servidumbre).
Esta estructura alcanzo su apogeo entre el siglo IX y XIII.
5.1. Causas
  Desintegración del Imperio carolingio.
  Segunda inv
invasión
asión de bbárbaros
árbaros (normandos
(normandos,, sarracenos
sarracenos,, eslavos a Occident
Occidente).
e).
  Debilitamiento del pode
poderr real, frente al poder local.
5.2. Sociedad feudal
Fue de carácter estamental privilegiando a la nobleza y al clero y se organizó en los siguientes
estamentos:
Bellatores
Bellator es Oratores Laborator
Laboratores
es
CEPRE-UNI HUMANIDADES 68

CICLO INTENSIVO ESCOLAR NACIONAL Material de Estudio Nº 1 

Eran propietarios de la tierra Eran las autoridades Eran campesinos sometidos a la


y estuvieron conformados eclesiásticas. Se dividía en entrega de rentas y de fuerza de
por la nobleza guerrera: alto y bajo clero. Se trabajo en provecho de los
señores feudales, reyes y encargaron de la difusión de bellatores y oratores. Estuvo
vasallos. la cultura feudal a través de conformado por los villanos (libres)
la educación. y los siervos de la gleba (adscritos
a la tierra).

5.3. Características


   Economía
Predominioautárquica, que entre
de la vida rural sobreellassiglo
iglo X
XII a XIII alcanzó su auge económico.
urbana.
   Poliarquía que fortaleció el poder local (feudal) frente al real (monárquico).

5.
5.4.
4. R
Relaciones
elaciones de feudo vasallaje y de servidum
servidumbre
bre
Relacion
Re laciones
es feudo vasalláticas
Homenaje Investidura
Ceremonia que consistió en la juramentación Ceremonia donde el señor feudal entregaba el
de lealtad del vasallo a un señor feudal quien feudo o beneficio al vasallo, a cambio que este se
recibía ayuda militar y monetaria del primero. comprometía a auxiliar
auxiliarlo
lo militarmente.
militarmente.

Relaciones de servidumbre
Los campesinos eran obligados a entregar rentas al señor feudal y a los vasallos. Las principales
rentas fueron: la corvea (trabajo gratuito en los feudos), la gabela (pago en cantidades de sal), las
banalidades (entrega de especies por usar los instrumentos del castillo feudal), el pecho o talla
(entrega de tributo monetario). El derecho de la pernada o pernocta que permitió a los feudales
afectar la intimidad de las mujeres campesinas. 

(1096 – 1291)
6. Las cruzadas (1096
Fueron campañas militares emprendidas por los cristianos liderados por los feudales contra
los musulmanes liderados por los turcos. Aquellos pretendieron expandir el régimen feudal a
Oriente. Estas campañas se iniciaron en el concilio de Clermont convocado por el Papa
Urbano II. Entre las principales cruzadas tenemos:

I cruzada 
cruzada  Fue la principal victoria para los cristianos porque recuperaron temporalmente
Jerusalén.
III cruzada 
cruzada  Participaron reyes europeos destacando Ricardo Corazón de León, Felipe Augusto
y Federico Barbaroja.
IV cruzada 
cruzada  Tuvo como objetivo la invasión de Constantinopla.
CEPRE-UNI HUMANIDADES 69

CICLO INTENSIVO ESCOLAR NACIONAL Material de Estudio Nº 1 

En el desenlace de las cruzadas se produjo la gran derrota de los señores feudales y sus
vasallos, marcando así la crisis del feudalismo, y la reactivación del comercio entre oriente y
occidente por el Mediterráneo.

7. La cri
crisis
sis del siglo
sigl o XIV y el fin d e la EEdad
dad Me
Media
dia
A fines de la Edad Media se produjo una crisis agrícola que
condujo a hambrunas, que se agravaron con la epidemia
de la peste negra (1348-1350). En política, destacaron la
Guerra de los Cien Años (1377-1453) entre Francia e
Inglaterra, la Reconquista española (que expulsarían a los
musulmanes en 1492) y la corrupción de la Iglesia católica
que produjo el Cisma de Occidente (1377-1415).
Finalmente, la Edad Media llegó a su fin f in con la caída de Constantinopla en 1453.

EVALUACION
1. Durante el periodo clásico, el mundo griego se II. Palacio de
consolidó con el Cnosos b. Cretense
de Atenas y Esparta.
predominio de Atenas Esparta.  Entre estas III. Partenón c. Mic
dos polis existían similitudes entre las que énica

I.podemos mencionar
dioses antropomorfos A) Ic, IIa y IIIb D) Ic, IIb y IIIa
II. arquitectura B) Ib, IIc y IIIa E) Ia, IIc y IIIb C) Ib, IIa y IIIc
III. origen étnico
IV. actividade
actividadess económicas 5. La filosof
filosofía
ía fue uno de
A) I, III y IV C) I, II y IV B) I y IV D) I y II los resaltantes aspectos en el que destacaron
E) I, II y III los griegos. Entre sus
principales representantes tenemos a
2. La etapa helénica griega ocurrió durante el A) Arquímedes, Polibio y Aristóteles.
periodo ______, mientras que la crisis de las B) Virgilio, Hipócrates y Demócrito.
polis griegas ocurrió a consecuencia de C) Sócrates, Platón y Aristóteles.
las Guerras ______________. D) Tucídides, Euclides y Sócrates.
A) Arcaico – púnicas E) Pitágoras, Heródoto y Plutarco.
B) Oscuro – del Peloponeso
C) Cretense – del Peloponeso 6. De las siguientes proposiciones, identifique
D) Clásico – médicas
E) Helenístico  – médicas aquellos
surgimientoquedelson considerados causas para el
feudalismo.
I. Primeras invasiones bárbaras.
3. Durante la etapa helénica se originó la II. Desintegración del Imperio carolingio
carolingio..
civilización griega producto de la invasión de III. Reactivación del comercio mercantil.
tribus indoeuropeas entre las que destacaron IV. Debilidad del poder real, frente al local.
A) samnitas, ecuos y eolios. A) III y IV B) II y IV C) Solo IV
B) cretenses, dorios y jonios. D) I y IV E) I, II y III
C) aqueos, micénicos y latinos.
D) volscos, celtas y fenicios. 7.Las cruzadas fueron campañas militares
E) dorios, jonios y eolios. emprendidas por los cristianos contra los
musulmanes. Identifique las consecuencias de
4. Con respecto a la arquitectura desarrollada aquel proceso
por los griegos, relacione correctamente la obra I. El debilitamiento
debilitamiento político de los señores
arquitectónica
arquitect ónica y el periodo en el qu quee ffue
ue feudales.
construido. II. La segunda invasión bárbara a Europa
I. Puerta de los Leones a. Clásica occidental.
CEPRE-UNI HUMANIDADES 70

CICLO INTENSIVO ESCOLAR NACIONAL Material de Estudio Nº 1 

III. La reactivación del comercio entre Oriente y 9. Respecto a la crisis del siglo XIV, señale la
Occidente. verdad (V) o falsedad (F) según correspond
corresponda. a.
IV. La decadencia del poder económico de la I. La crisis agrícola condujo a hambrunas.
burguesía. II. La peste negra
negra agravó la crisi
crisiss de este siglo
siglo..
A) II, III y IV B) I y II C) Solo III D) II y IV III. La a sociedad urbana decayó frente al rural.
E) I y III IV. El Cisma de Oriente debilitó el poder papal.
A) VVFF B) VFVF C) VVFV D) FFFV
8. Dentro de las relaciones de dependencia E) VVVF
personal,
pagar rentaslos campesinos
a los nobles.estuvieron
Identifiqueobligados
la relacióna 10. Durante el Renacimiento carolingio se
correcta de las rentas con su respectiva produjo
característica. A) el impulso de la cultura clásica mediante la
I. Corvea a. Tributo monetario educación.
II. Gabela b. Pago en cantidades de sal B) la desintegración
desintegración del Imperio carolingio
carolingio..
III. Talla c. Trabajo gratuito en los feudos C) la fundación de la ciudad de Aquisgrán.
A) Ic, IIb, IIIa B) Ib, IIa, IIIc C) Ib, IIc, IIIa D) la crisis de la educación parroquial.
D) Ic, IIa, IIIb E) Ia, IIb, IIIc E) la aparición de la burguesía comercial.

TEMA IV. DEL HORIZONTE MEDIO AL TAHUANTINSUYO.

I.
SeHori
Horizonte
zonte medio
caracterizó por la expansión de tradiciones culturales hacia diversas regiones desde la
sierra sur que permitió la segunda síntesis cultural. Destacan los desarrollos estatales de
Tiahuanaco y Wari.

1.1. Tiahuanaco
1.1.
Los inicios del poblamiento en la región se remontan al Formativo, 1200 a.C. y posteriormente
destacó la presencia de Pucará

  ntro político . Se desarrolló en la meseta altiplánica del Collao cuyo centro de poder
Centro
Ce
fue la ciudad de Tiahuanaco o Taipicala.
  Economía. Se basó en la ganadería y en un agro limitado. Realizaron intercambios
con poblados de diversas regiones a través del sistema del control vertical de pisos ecológicos
o dominio discontinuo de territorios.
  Tecnología agrícola. Construyeron camellones (waru waru), campos agrícolas
elevados para proteger su frontera agrícola evitando las
Los waru waru permitieron la
heladas.  ex an
ansió
sión
n de
de la
la ffro
ront
nter
era
a a ríc
rícol
ola
a
   Arqu
 Ar quii tect ur a. Emplearon grandes bloques de
tec t ura
piedra simétrica y pulimentada; utilizando grapas de cobre les dieron consistencia y solidez.
Los recintos más importantes son Kalassasaya, Akapana, El Templete y Puma Punku.
  Cerámica.  Destacaron los pebeteros o zahumerios y vasos ceremoniales. En estos
objetos aplicaron decoraciones con figuras zoomorfas, fitomorfas y emblemas geométricos.
  Ideología. Rinden culto al Dios de los Báculos, representado como el dios llorón o de
las varas. Forma de dominación de la elite que lograron expandir a través del control vertical
de pisos ecológicos
El control vertical de un máximo de pisos ecológicos o dominio discontinu
discontinuoo territorial, fue propuesto por John
Murra. Este es un sistema donde los ayllus altoandinos, carentes de variedad de recursos, establecían colonias
o enclaves en otras regiones a diversas altitudes para obtenerlos, complementar su dieta y otras necesidades.
CEPRE-UNI HUMANIDADES 71

CICLO INTENSIVO ESCOLAR NACIONAL Material de Estudio Nº 1 

1.2. Wari
  Origen del Estado . Se originó tomando como base demográfica el
desarrollo del pueblo Huarpa quienes asimilaron e introdujeron en su
desarrollo tradiciones propias de los Nazca (elementos urbanos,
patrones alfareros y textiles) y Tiahuanaco (aspecto religioso, debido
al prestigio del dios altiplánico).
  Ce ntro político. La capital fue la ciudad de Viñaque (Huanta). Fue
Centro
un Estado centralista y expansivo alcanzando la costa, sierra y ceja de
selva (Cuzco).
  Carácter del Estado.  Luis G. Lumbreras, sostiene que Wari se
constituyó en un Imperio panandino
  Cerámica.  La alfarería fue policroma. Robles Moqo fue el estilo principal. Destacaron los
tazones ceremoniales.
   Ar
 Arqu
quititect
ectur a.  Construyeron ciudades (las cabezas de región) para fortalecer su dominio en
ura.
diversas regiones. Entre estos centros urbano-administrativos tenemos a Viñaque,
Wiracochapampa, Pachacámac y Pikillacta. De esta
forma, lograron la expansión de un patrón urbano
serrano. Además, construyeron depósitos de alimentos
estatales.
  Ideología.   Su divinidad fue el Dios de los báculos
bizco, relacionado directamente a la agricultura de papa
y maíz.
  Re
Red ial. Fue un sistema de caminos utilizados para
d vvial.
la movilización del ejército, de los y demás funcionarios
del Estado y además unió las cabeceras de región.
2. Intermedio
Intermedio tardío
Este periodo, denominado como Estados regionales, está caracterizado por el gobierno a
través de reinos y señoríos. Es posterior al final del poder del Estado Wari. En este contexto,
la actividad agrícola continuó como la principal actividad económica, aunque ahora dirigida
desde centros urbanos regionales. La actividad alfarera decae en calidad, pero mejora su
producción en la cantidad.

2.1 Chincha
  Centro político . Constituyó una confederación de Estados. Destacaron Tambo de Mora y
Centro
Lurinchincha. Fueron conquistados pacíficamente
pacíficamente por los incas

  Economía . Destacó la actividad comercial con rutas marítimas, hasta Ecuador para traer
el mullu, y la terrestre hacia el Collao para obtener fibra de camélidos.
  Cultura. Practicaron la xilografía confeccionando remos ornamentales, utensilios de
labranza y timones.

2.2 Chimú
  Centro político . Su capital fue Chan Chan. Fueron conquista
Centro conquistados
dos mediante la fuerza por el
Inca Pachacútec.
  Economía . Fue agrícola, complementada con la pesca y el comercio. Se construyeron los
wachaques (chacras hundidas).
  Cerámica.  Predominó la cantidad y no la calidad, para ello utilizaron moldes.
  Orfebrería. Fueron grandes orfebres, forjaron brazaletes, orejeras, cuchillos rituales o
tumis. Recibieron influencia de la cultura Sicán o Lambayeque.

   Ar
 Arqu
quii tect ur a. Se usaron tapiales decorados con frisos. Destaca la ciudad de Chan Chan.
tec t ura
2.3.. Chachapoyas
2.3
CEPRE-UNI HUMANIDADES 72

CICLO INTENSIVO ESCOLAR NACIONAL Material de Estudio Nº 1 

  Centro político . Destacaron Kuelap y Gran Pajatén centros urbano-administrativos, de


carácter militar. Sometidos por Túpac Inca Yupanqui.
   Ar
 Arqu
quii tect
tec t ura
ur a funer
fu nerari
ariaa. Destacan los purunmachus, sarcófagos de arcilla para la nobleza
guerrera, y torres funerarias (en forma de chullpas) decoradas como los pinchudos,  ídolos
tallados en madera (símbolo de la fertilidad).

2.4. Reinos aimaras


  Ce
losCentro
ntroypolítico
collas . Crearon
los lupacas. unaconquistados
Fueron confederación dePachacútec.
por reinos alrededor del Titicaca, destacando
  Te
Tecnol
cnología
ogía a agrícol
grícolaa. Continuaron las técnicas de los pueblos de la meseta del Collao, la
ganadería de altura, los waru waru, la deshidratación de alimentos y aplicaron el sistema de
archipiélagos ecológicos o enclaves.
  Cultura.  Construyeron una monumental arquitectura funeraria para la nobleza,
denominada chullpas.

3. El
El Tahuantinsu yo
Durante el Tercer horizonte, el Tahuantinsuyo se constituyó en la mayor civilización de los
Andes, que se desarrolló en el sector centro-occidental de América del Sur. Se trató de la
mayor síntesis de todas las culturas andinas y su originalidad no está en las aportaciones
culturales, sino en la organización administrativ
administrativaa y po
política.
lítica.
3.1. Límites y división
El Tahuantinsuyo políticaen cuatro suyos demarcadas por
fue dividido
ceques y antiguas delimitaciones de los ayllus (ríos, lagos,
lagunas, montañas, etc) que partían desde el centro de la ciudad
del Cuzco (Ombligo del mundo). Los límites fueron los
siguientes: al norte el río Ancasmayo, al sur el río Maule al este
la Ceja de la selva, al oeste el océano Pacífico y al sureste Cuyo,
Tucumán. Los suyos fueron los siguientes:
Chinchaysuyo. Fue el suyo de mayor población y una zona
estratégica por la presencia de comerciantes, navegantes, orfebres,
etc.
Collasuyo. Situada hacia la meseta del Titicaca y Tucumán. Fue el
suyo de mayor extensión.
Contisuyo. Fue el suyo de menor extensión y de gran importancia
en lo religioso, ocupaba desde el sudeste de Cuzco hasta el
Pacífico.
 Antiti su yo
 An yo.. Fue el suyo más septentrional y estaba situado en la ceja
de selva. Allí el Estado obtenían las hojas de coca, plumas, así como
la yuca y frutas.
3.2. Origen
3.2. Orig en
Para explicar el origen de los incas, se debe tomar en cuenta su origen
mítico, que se explica a través de leyendas, mitos; y su origen histórico
donde intervienen los estudios etnohistóricos, arqueológicos etc.
CEPRE-UNI HUMANIDADES 73

CICLO INTENSIVO ESCOLAR NACIONAL Material de Estudio Nº 1 

Origen legendario. Encontramos la leyenda de Manco Cápac y Mama Ocllo, recogida por Garcilaso
de la Vega en la crónica Comentarios Reales y
Reales y el mito de los hermanos Ayar, recogida por Juan de
Betanzos en la crónica Suma y narración de los incas.
incas.
Origen histórico. 
histórico.  Para el historiador Waldemar Espinoza, la nobleza inca procede de la cultura
Tiahuanaco (lengua Puquina
Puquina).
).

Din astías. El Tahuantinsuyo fue gobernado por dos dinastías diferentes. 


3.3.. Dinastías.
3.3

Hurin Cuzco
Cuzco (Bajo Cuzco) Hanan
Hanan Cuz
Cuzco
co (Alto Cuzco)
Se dedicó inicialmente a asuntos políticos, pero, Esta dinastía a diferencia de los hurin se
posteriormente quedó relegada a asuntos religiosos. De dedicó más a asuntos políticos,
la misma forma, esta dinastía se inició con Manco Cápac administrativos y militares del Estado.
y culminó con Cápac Yupanqui. Fue iniciada por Inca Roca.

3.4. Proceso histórico


Inicialmente conquistaron
conquistaron las etnias ubicadas en el valle de Acamama y durante el resto del
siglo XV lograron imponerse también sobre distintos señoríos como los chancas, chinchas,
chimúes, lupacas, etc. Sin embargo, la conquista no solo fue a través de la guerra, también
aplicaron la diplomacia y las recompensas. Los periodos que se desarrollaron fueron:

Curacazgo. Asentamiento
Confederación. y fundación
Fue el periodo donde sedelestablecieron
Cuzco. Destacaron
alianzas Manco Cápac con
(matrimonios) y Sinchi Roca.
los grupos
locales del Cuzco formando una confederació
confederación.
n. Destacaron:
  Lloque Yupanqui: Inició este periodo
  Huiracoch
Huiracocha:a: Avance de los chancas hacia el Cuzco, pero fueron derrotados en la batalla de
Yahuarpampa por Cusi Yupanqui (Pachacútec)
Imperial. Destacaron las siguientes sapa incas 
incas (equivalente a emperadores) 
emperadores) 
  Pachacúte
Pachacútec,c, fundador y principal organizador del imperio.
  Túpac Inca Yupanqui: conquistó a los Chachapoyas, estableció el límite en el río Maule y realizó
un viaje por vía marítima
m arítima a Oceanía y las islas Galápagos.
  Huayna Cápac: loglogró
ró la máxima expansión territorial.

3.
3.5.
5. O
Organización
rganización económi
económica
ca

3.
3.5.1. Princip ios.  La organización económica se basó en dos principios que fueron:
5.1.
-Reciprocidad. Consistió en la mutua prestación de energía humana y servicios dentro del ayllu.
Según Waldemar Espinoza, podemos identificar dos tipos:
  Simétrica.
Simétrica. Se ayllu . 
 Se realizaba entre los mismos miembros del ayllu.
   As
 Asim
imétr
étric
ica.
a. Se hatunrunas   y el
 Se realizaba entre los hatunrunas del ayllu con su curaca o entre los hatunrunas 
Estado imperial.
-Redistribución. Consistió en el reparto de bienes que el Estado imperial hacía entre los ayllus, de
lo producido por ellos mismos gracias al trabajo de la mita. En este principio como se observa
interviene el Estado.

3.
3.5.
5.2.
2. Distr
Distribuc
ibución
ión d e las tierras
Inca Comprendían las tierras del Estado los incas y su panaca.
Sol Fueron tierras destinadas al sustento del culto y a los sacerdotes.
Fueron repartidos entre los hatunrunas, cada miembro recibía un topo, la extensión de este
 Ayllll u
 Ay era variable y dependía mucho de la calidad de la tierra y del clima.
CEPRE-UNI HUMANIDADES 74

CICLO INTENSIVO ESCOLAR NACIONAL Material de Estudio Nº 1 

3.5.3. Formas de trabajo. En el Tahuantinsuyo, la producción y la construcción de obras


públicas dependían de las formas de trabajo. Estas fueron las siguientes:
Mita Minka Ayni
Forma de prestación de Forma de trabajo colectivo entre Forma de prestación de
energía humana por turnos, ayllus en beneficio de la energía humana recíproca, en
destinada a la ejecución de comunidad, como la forma colectiva y obligatoria en
servicios y la producción de construcción de depósitos, beneficio particular dentro de
bienes en favor del Estado. etc. 
caminos, puentes, acequias, etc.  ayllus. 
los ayllus. 

3.5.4 Tecnologías agrícolas. Supieron aprovechar y mejorar los avances de las culturas
anteriores a ellos, ampliando su frontera agrícola 
Fueron terrazas artificiales o plataformas superpuestas sostenidas por muros
 Anden
 An denes
es construidos en las faldas de los cerros. Los andenes más famosos son Pisac,
Ollantaytambo y el centro de experimentación agrícola de Moray.
Camellones Fueron chacras elevadas. Tecnología agrícola para evitar la helada creando
(Waru
(Waru w aru
aru)) microclimas para proteger a las plantas.
Tecnología agrícola de la cultura Chimú y Sicán, fueron chacras hundidas cuyo
Wachaques
objetivo fue aprovechar la napa freática.
Control vertical Fue el establecimiento de enclaves en los diferentes pisos ecológicos, su
de pisos objetivo fue obtener productos diversos.
Instrumentos Los instrumentos de trabajo más importantes fueron la chaquitaclla, la raucana
agrícolas entre otros.

3.
3.4.
4. O
Organización
rganización políti
política
ca
El Estado fue de carácter imperial, centralista y multiétnico, administrado por diferentes
autoridades que estuvieron organizadas jerárquicamente. Entre ellas tenemos.
Fue considerado hijo del Sol y en el residía el poder absoluto. Concentraba en sí la
Sapainca
totalidad del poder político y religioso.
Conformaban el consejo imperial y fueron gobernadores máximos en cada uno de sus
 Apoc
 Ap ocun
unaa
respectivos suyos.
Gobernadores de las provincias o huamanis del Imperio. También fueron
Tocricut
denominadoss Apunchic.
denominado
Visitadores que vigilaban el cumplimiento de las leyes al interior del imperio, fue
Tucuiricui
considerado como “los ojos y oídos del inca”.  
Era el jefe de un ayllu, en caso de guerra se denominaba Sinchi. Era el vínculo entre
Curaca
los ayllus
ayllus y el aparato burocrático.

3.
3.5.
5. O
Organización
rganización soci
social
al
Se caracterizó por ser jerárquica, con diferencias notables entre los diversos grupos sociales.

Nobleza 
Conformado por el Sapainca y su panaca. Esta última estaba constituida por todos
De sangre los miembros de la familia del inca, el auqui heredero al trono imperial con el que
se establecía el correinado no pertenecía a la panaca.
Provincial  
Provincial Fueron las élites incorporadas al imperio.

Pueblo  
Estuvieron organizados en ayllus (célula básica de la sociedad) además,
constituyeron
constitu yeron la mayoría de la población del Tahuantins uyo.  El Estado imperial
Tahuantinsuyo.
Hatunrunas
etc.  
disponía de los comunes hatunrunas para ser mitayos, acllas, chasquis, etc. 
Eran sirvientes exclusivos, abandonaban sus ayllus para siempre y se dedicaban
Yanaconas  
Yanaconas
nobleza.  
por el resto de su vida al servicio de la nobleza.
CEPRE-UNI HUMANIDADES 75

CICLO INTENSIVO ESCOLAR NACIONAL Material de Estudio Nº 1 

Eran mujeres que iban a los acllahuasis dedicándose a la preparación de textiles y


 Acllll as
 Ac
mamaconas 
chicha dirigidos por las mamaconas 

Mitmas Personas trasladas


traslada s de un lugar a otro para colonizar los pueblos recién
conquistados
Chasquis Mensajeros que transportaban
transportaban noticias u objetos por todo el territorio

3.
3.6.
6.quit
M
Manifestacion
 Arqu
 Ar anifestaciones
itect
ectur
ura. es cult
culturales
urales
a. Se caracterizó por su sencillez, solidez, simetría,
monumental, planificada y el uso de la decoración trapezoidal, así
como diseñaron patrones de asentamientos propios. El principal
material utilizado fue la piedra destacando Machupicchu,
Saysayhuaman,
Saysayhuama n, O
Ollantaytambo
llantaytambo,, Coricancha
Coricancha,, Tumipampa, Cuzco etc.
Textilería. La producción textil fue masificada. Existieron dos tipos de
tejidos el cumbi, que era un tejido fino a diferencia del tejido abasca
de uso popular.
Educación. Se instruía en el Yachayhuasi a cargo de los amautas.
elitista 
La educación fue elitista  Anden de moray en Cuzco
Cosmovisión. Existieron tres mundos relacionados entre sí. Arybalo o urpo
- Hanan Pacha (mundo de arriba) lugar donde habitaban las
divinidades,
- Kay Pacha (mundo terrenal) era el mundo de aquí tiempo presente,
- Uku Pacha (mundo de abajo) lo fértil y lo agrícola dependen de
este mundo.
Cerámica. La cerámica más notable fue el Urpo o Arybalo, de
carácter utilitario. Tuvo principalmente una decoración geométrica y
pictórica.
Religión. Se caracterizó por ser politeísta y panteísta siendo el Dios
principal el sol (Inti). Destacaron además Killa(Luna), Illapa (rayo) y
Pariacacca (lluvia).

3.
3.7.
7. Guerra civil entre Huá
Huáscar
scar y Atahualpa
At ahualpa ((Crisi
Crisiss fifinal)
nal)

La sucesión al trono de Huayna Cápac se vio marcada por un conflicto entre las panacas
gobernantes. Se desató un enfrentamiento entre la facción hurin (Huáscar) y hanan
(Atahualpa), que debilitó el Imperio. Todo ello se agudizó por la
l a alianza de los españoles y las
diferentes etnias sometidas por los incas como tallanes, chancas, huancas, chachapoyas,
entre otros.
Evaluación
1. El Horizonte medio se caracterizó por la altiplánico y desarrolló su economía
expansión de tradiciones culturales hacia basado en
diversas regiones de la sierra sur. ¿Qué A) la ganadería y el agro limitado.
sistemas estatales destacaron en este periodo? B) la alfarería y el comercio.
I. Chavín. II. Tiahuanaco. III. Moche. C) la agricultura intensiva y la pesca.
IV. Wari V. Chincha. D) el avance tecnológico y la medicina.
A) I, II y III B) II, III y IV C) II y IV E) los sistemas de regadío y el comercio.
D) III, IV y V E) Solo II
3. La construcción
construc ción de camellones
2. La cultura Tiahuanaco, al desarrollarse en la o waru waru , en Tiahuanaco , tuvieron como
meseta del Collao, aprovechó su medio objetivo
CEPRE-UNI HUMANIDADES 76

CICLO INTENSIVO ESCOLAR NACIONAL Material de Estudio Nº 1 

A) proteger su frontera agrícola para reducir el I. Curacal


impacto de las heladas. II. Confederación cusqueña.
cusqueña.
B) controlar los sistemas de regadío agrícola en III. Imperial
los suelos. a. Expansión por el altiplano
C) intercambiar técnicas agrícolas con las b. Gobierno de Inca Roca
culturas vecinas. c. Cusi Yupanqui asume el poder
D) aprovechar la napa freática para realizar IIIb   C) Ic. IIb y IIIa
A) Ib, IIc y III B) Ia, IIc y IIIb
cultivos en zonas frías. D) Ib, IIa y IIIc E) Ia
Ia,, IIb y IIIc
E) desarrollar
productos formas de
altoandinos
altoandinos.. deshidratación de los 8. Respecto a la a la arquitectura incai
incaica
ca
marque verdadero (V) o falso (F) según
4. El origen de la cultura
cultura Wari, una sí
síntesis
ntesis corresponda.
de los aportes culturales de I. Los materiales de construcción eran
I. Huarpa, el desarrollo de su base demográfica.
impuestos por el imperio.
II. Moche, el aporte de su alfarería II. El Cuzco fue edificado en forma de Puma.
representativa. III. El uso de la decoración trapezoidal señalaba
III. Nazca, sus elementos urbanos, alfareros y el carácter imperial.
textiles. IV. Seguían el diseño de construcción wari.
IV. Tiahuanaco, el aspecto religioso culto a Dios A) VVFF B) FFVV C) FVVF D) FVVV E) VVVV 
VVVV  
llorón.
A) I, II y III B) I y II C) II D) II y IV E) I, III,IV 9. la Asamblea imperial estaba conformada por
5. Determine que culturas forman parte del los __________ y los gobernantes de las
período Intermedio tardío provincias eran los _________.
I. Chincha II. Chimú III. Moche A) Apu Quispay
B) Purej  – Willac Umu.
- Apucunas.
IV. Tiahuanaco V. Chachapoyas
A) I, II y IV B) II, III y IV C) I, II, y V C) Apocunas  – Apunchic.
D) III, IV, V E) I, III y V D) Curacas - Tucuyricuy
6. Establezca la relación entre gobiernos incas E) Sinchis- Auqui
y sus respectivas obras. 10. Los incas aprovecharon todas las
I. Pachacutec a. Koricancha tecnologías agrícolas disponibles en los
II. Tupac Yupanqui b. Máxima expansión tertitorios conquistados.
conquistados. Dentro de las que
III. Huayna Capac c. Mayores conquistas podemos destacar
A) Ic, IIb y IIIa IIIb   C) Ib. IIc y IIIa
B) Ia, IIc y IIIb A) los andenes y camellones.
D) Ib, IIa y IIIc E) Ia, IIb y IIIc B) los wachaques y jircas.
C) los ceques y huachos.
7. Relacione las etapas de la historia inca con D) los tinkuy y las piruas.
los siguientes eventos sucedidos en estas. E) los ushnus y bancales. 
bancales. 

TEMA V. EDAD MODERNA Y REVOLUCIONES BURGUESAS DEL S. XVIII.


La Edad Moderna
Es el periodo que inició con la Toma de Constantinopla (1453) y finalizó con la Toma de la
Bastilla, en el proceso de la Revolución francesa (1789). Presenta resumir los grandes
cambios ocurridos esta Edad, tales como:
  El surgimiento del capita
capitalismo
lismo en su fas
fasee mercant
mercantil.
il. Aunque aquello coe
coexistió
xistió ccon
on
elementos de la economía feudal europea.
  La cecentralización
ntralización política representada por la
lass mo
monarquías
narquías europeas que devinieron en
modelos absolutistas.
  El cues
cuestionamiento
tionamiento de la escolástica por par
partete de intelectuales en ddiversos
iversos campos del
saber.

1. El
El desarrollo
desarro llo de la burgu esía
CEPRE-UNI HUMANIDADES 77

CICLO INTENSIVO ESCOLAR NACIONAL Material de Estudio Nº 1 

La burguesía, clase social promotora del capitalismo, surgió en el contexto del renacimiento
urbano y estuvo íntimamente ligado al comercio, la banca y la producción manufacturera. Los
elementos que hicieron posible el desarrollo temprano de esta clase fueron:
1.
1.1.
1. Los burgos
bu rgos
Eran las ciudades medievales que alcanzaron su autonomía frente al control feudal mediante
las cartas o fueros. El eje económico y social de los burgos fueron los gremios.

1.
1.2
2 Los
Eran gremios de artesanos especializados en un oficio. Eran liderados
asociaciones l iderados por los maestros
(propietarios del taller), que dominaban el trabajo de los oficiales (asalariados) y aprendices.
1.
1.2.
2. La
Lass ligas
li gas comerciales
com erciales La crisis del siglo XIV
Fueron las alianzas de los burgos orientadas al control de las rutas detuvo el avance de los
de comercio y la defensa común. Entre las más importantes burgos. En el siglo XV la
tenemos las siguientes: lenta recuperación
  La Liga Lomb
Lombarda
arda (Venecia, Génova y Florencia), controló el comercial por el
Mediterráneo fue alterada
Mediterráneo. por la caída de
  La Hansa teutteutónica
ónica (Bremen, Ha
Hamburgo,
mburgo, Lubeck), controló el Constantinopla.
Báltico.
2. Las primeras expresion
expresiones
es intelectuales burguesas
El desarrolloy culturales
intelectuales de los burgos marcó el escenario
que cuestionaron para
la escolástica el surgimiento
(filosofía manifestaciones
que justificó el feudalismo).
f eudalismo).
Aquellas manifestaciones fueron el humanismo y el renacimiento, las cuales presentan las
siguientes causas:
  El crecimiento de los burgos gracias al avance del comercio y la banca.
  El accionar de los mecenas (burgueses) que patrocinaron
patrocinar on a los intelectuales.
intelectu ales.
  El avance técnico expresado en el uso de la imprenta.
2.1. El Humanis mo (Siglo XIV-
2.1. XIV-XV)
XV)
Fue el movimiento intelectual que buscó situar al ser humano como centro de su reflexión y
su lugar en el mundo. Surgió en el norte de Italia y luego, mediante las Academias y las
Universidades, se difundió por Europa.
Características 


   El uso
antropocentris
antropocentrismo
mogrecolatina
de la cultura que se sustentó
como en el estudio
fuente integral del
de producción hombre..
académica

(Florencia):: Sonetos para Laura 


Francesco Petrarca (Florencia) Laura 
Principales
humanistas GiovanniMaquiavelo
Nicolás (Florencia): El
Bocaccio (Florencia): El Decamerón 
Decamerón
Príncipe   
Príncipe 
Erasmo de Rotterdam (Países Bajos): Elogio de la locura 
locura 
CEPRE-UNI HUMANIDADES 78

CICLO INTENSIVO ESCOLAR NACIONAL Material de Estudio Nº 1 

Tomás Moro (Inglaterra): Utopía


Utopía  

2.2 El Renacimiento (Siglo XV  – XVI)


Fue el movimiento cultural que se expresó en las artes, ciencias, literatura, etc.

Características
  Tomó como modelo el mundo clásico. * Exaltó la anatomía humana.

  Innovó en pintura con la técnica de la perspectiva. * Estudio de la naturaleza.


naturaleza.
Leonardo Da Vincci. Pintura: La última cena, la Gioconda; arquitectura,
escultura e ingenierí
ingeniería)
a)
Principales
Miguel Ángel Buonarotti. Pintura: Los frescos de la Capilla Sixtina en Roma;
renacentistas
escultura: El David.
Rafael Sanzio. Pintura: La Escuela de Atenas.
3. La expansió
expansiónn europea
euro pea ((fin
fines
es del s. XV - ini
inicio
cioss del s. XV
XVII)
II)
Fue un proceso de exploración y asentamiento de colonias y factorías que permitió la
expansión del capitalismo mercantil hacia otros continentes 
3.1. Causas y factores
  La búsqueda de nuevas rutas comerciales que agilice Oriente. 
agilicenn el comercial a Oriente. 
 Mediterráneo.  
  La caída de Constantinopla (1453) que encareció el comercio en el Mediterráneo.


   El
El avance
respaldotécnico:
políticoperfección
de las monarquías europeas
de la brújula, armasade
brújula, losfuego,
proyectos de navegación.
la cartografía, los portulanos o cartas
náuticas, la carabela, etc. 

3.
3.2.
2. Princi
Principales
pales expl
exploracion
oracion es geográficas
 A. Port
Po rtug
ugal
al
El rey Enrique (El navegante) fundó la Escuela náutica de Sagres.  Sus exploraciones se
iniciaron en las costas occidentales de África. Las exploraciones más destacadas estuvieron
a cargo de
  Bartolomé Días, llegó al cabo de las Tormentas o Buena
Buena Esperanza (1488).
  Vasco da Gama, llegó a la India (1498).
  Pedro Alvares Cabral, llegó
llegó a las costas de Brasil (1500).  España y
B. España  Portugal

Tras la victoria sobre las musulmanas en Granada (1492), los Reyes acordaron la
repartición del
Católicos respaldaron el proyecto de Cristóbal Colón hacia las Indias
mediante la Capitulación de Santa Fe. Los cuatro viajes de Colón mundo con el
permitieron la colonización sobre América, la cual significó
signifi có la dominación Tratado de
de territorios y de la población nativa. Posteriormente, la corona española Tordesillas
respaldó el primer viaje de circunnavegación por la parte de Hernando de (1494).
Magallanes y continuada por Sebastián Elcano, que le permitió colonizar
Filipinas.
3.
3.3.
3. C
Consecuenci
onsecuencias
as de la expansió n europea
eur opea
  Surge el capital
capitalismo
ismo mercan
mercantil
til y la cuenca del Atlántico ala
alanzó
nzó su auge co
comercial.
mercial.
  Emergen los Impe
Imperios
rios ultramari
ultramarinos
nos y los conflic
conflictos
tos entre las potenc
potencias
ias coloni
colonizadoras.
zadoras.
  Disminución de la población nativa por las enfermedades
enfermedades,, las guerras y los trabajos forzados.
Lo anterior influyó en la introducción de esclavos afros.

  Mestizaje biológico y cultural. 
CEPRE-UNI HUMANIDADES 79

CICLO INTENSIVO ESCOLAR NACIONAL Material de Estudio Nº 1 

La ciencia moderna (siglo XVII) permitió afirmar la concepción del mundo a partir de leyes naturales.
Sobresalieron los trabajos de Copérnico, Descartes, Newton, Kepler, etc. Su labor permitió superar la
concepción religiosa y fatalista del mundo, especialmente el geocentrismo.

4. El Antigu o régimen (siglo


(sig lo XVI – XVIII)

Fue el conjunto de instituciones políticas


políticas,, sociales y económicas que predominaron en Europa
hasta la Revolución francesa. Estas instituciones fueron
  La monarquía ab absolutista,
solutista, enc
encarnada
arnada por la fig
figura
ura del rey, quien eera
ra el encargado de cceder
eder
privilegios a los súbditos. El territorio o reino era patrimonio del monarca.
  La sociedad esta
estamental,
mental, organizada eenn grupos privilegiados (noble (nobleza za y clero, quienes
estuvieron exentas del pago de impuestos) y no privilegiados (Tercer Estado o pueblo), que
evidenció una marcada desigualdad social.
  La economía pre indus
industrial,
trial, en la cual predominab
predominabaa la vida rural. En las ciudad
ciudades
es se
estableció el mercantilismo organizado por la monarquía.

Las monarquías europeas desde el siglo XV organizaron el Estado moderno, es decir, Estados
centralizados,
centralizados, cuya unidad política estuvo representada por el monarca.
monarca. Estos fueron importantes
para el surgimiento del Estado-nación 
surgimiento

Principios
Providencialismo . El poder
Providencialismo. de las monarquía
del rey derivabamonarquías
del mandatos de
absolutis
Dios. tas
Centralismo. Concentración
Centralismo. Estado. 
 Concentración de los poderes del Estado. 
Regalismo.. Prevaleció el poder del rey sobre la Iglesia.
Regalismo
Bases del Estado
Estado mo derno .
moderno
Una burocracia profesional, representa al Estado en todo el territorio, además se encargó del
cobro de impuestos.
La diplomacia, al mando de embajadores, para mantener la paz.
Un ejército permanente, para mantener la autoridad real en el país.
Los impuestos, dinero cobrado a los súbditos a favor del rey.

Principales monarquías

Monarquía francesa. Representada por la Dinastía Borbón


Luís Sentó las bases del absolutismo con medidas tales como la creación de las
XIII intendencias para el cobro de impuestos. Intervino en la Guerra de los treinta años  
(1618-1648) para alcanzar la hegemonía en Europa.
Con la Paz de Westfalia, su gobierno hegemonizó la política europea. Además,
Luís estableció por medio de su ministro Colbert un proteccionismo económico que
XIV fortaleció el mercantilismo francés (colbertismo). También ordenó la construcción del
Palacio de Versalles. Se le identifica con la frase: “ El Estado soy yo”. Con él, el
absolutismo alcanzó su auge.

Monarquía ing lesa. Representa


Representada
da por la Dinastía Estuardo, cuyo iintento
ntento de establecer el
absolutismo fracasó en la Revolución de 1688.
El Parlamento, conformado por políticos liberales (burgueses y nobles)
Revolución
derrotó a la corona inglesa, estableciendo la monarquía constitucional,
1688 de basada en el parlamentarismo. Así surgió el primer Estado liberal de la
gloriosa
historia.
CEPRE-UNI HUMANIDADES 80

CICLO INTENSIVO ESCOLAR NACIONAL Material de Estudio Nº 1 

5. La ililust
ust ración
raci ón (S. XVIII)
XVIII)

Fue un movimiento ideológico y cultural que cuestionó, mediante el


racionalismo, el Antiguo régimen. Se sustentó en principios de
libertad individual, igualdad jurídica, derecho de la propiedad privada
y la fraternidad.
5.
5.1.
1. P
Prin
rincipales
cipales planteamientos
p lanteamientos
   Criticó el abs
absolutismo
olutismo propo
proponiendo
niendo la división de poderes (Esta
(Estado
do
liberal). Se proponían una monarquía constitucional o sistema
republicano.
   Cuestionó lo esta
estamental,
mental, proponien
proponiendodo una sociedad ccon on mayor
movilidad social.
   Buscó la separación ddel
el Estado con la IgIglesia
lesia (Estado laico). Campesino sosteniendo a los
   Defendió la tolerancia religiosa y la libertad de prensa
prensa..

Representan tes  
5.2.. Representantes
5.2
  Voltaire. En Cartas filosóficas planteó una crítica al
Voltaire.   Francias Quesnay.
Quesnay. Señaló a la
absolutismo. agricultura como base de la riqueza
  Montesquieu.
Montesquieu. En
 En El espíritu de las leyes planteó la (Fisiocracia).
Estado. 
división de poderes del Estado.    Ad
 Adam
am Sm
Smitit h . En su obra La Riqueza
  Rousseau
Rousseau.. En El contrato social sustento
sustento los de la Naciones (1776)
Naciones (1776) sentó las bases del
principios de la soberanía popular. liberalismo económico. Además, se le
  Diderot y D’Alembert. Autores de La enciclopedia, considera como el Padre de la economía
obra de difusión de la ilustración. moderna.

5.
5.3.
3. El De
Despot
spotism
ismo
o ilu
ilustr
strado.
ado.

Fueron las monarquías absolutistas de Europa


que, inspiradas en ideas de la ilustración,
realizaron reformas al interior de sus dominios
para evitar el estallido de revoluciones sociales.
Estas reformas implicaron hacer más eficiente a
la burocracia, flexibilizar la justicia y mejorar la
educación en beneficio de los súbditos. El lema
de este régimen fue: Todo para el pueblo, pero
sin el pueblo. Destacaron
- Prusia. Federico II, Máximo exponente
- Austria. María Teresa y José II.
- España. Carlos III, implantó reformas en sus
colonias americanas.
- Rusia. Catalina II.

6. Las revoluc
revoluc ion
iones
es burg uesas del sig
siglo
lo XV
XVIII
III
Fueron procesos históricos, liderados por la burguesía, que progresivamente eliminaron el
Antiguo Régimen y establecieron una sociedad basada en los ideales del pensamiento
ilustrado, así como un régimen económico capitalista liberal.
6.
6.11 La independencia de las tr
trece
ece coloni
colonias
as de Norteaméric
Norteamérica a
CEPRE-UNI HUMANIDADES 81

CICLO INTENSIVO ESCOLAR NACIONAL Material de Estudio Nº 1 

Fue el conflicto que se desarrolló de 1775 a 1783. Alcanzó el rango de revolución política
porque permitió la toma del poder por parte de la burguesía norteamericana al derrotar al
Imperio británico.

Antecedent es 
6.1.1. Antecedentes
6.1.1.
A inicios del s. XVII, migrantes ingleses (protestantes, cató
católicos
licos y otros) fundaron colonias en
Norteamérica.
A partir de la guerra de los
l os Siete Años (1756-1763), el gobierno inglés incrementó el control y
dominación en sus colonias que afectó a los colonos.

6.1.2. Causas
  El incre
incremento
mento de im
impuestos
puestos y limita
limitaciones
ciones comerciales a loloss co
colonos.
lonos. Esta presión fiscal
afectó a la importación de té, textiles, café, índigo, vino y otros productos.
  La ausencia de representantes de las colonias en el Parla Parlamento
mento inglés.
  La ininfluencia
fluencia de la ilustración. Intelectual
Intelectuales
es como Benjamín Franklin, Thomas Jefferson
Jefferson,,
consideraron como absolutista la política inglesa.

6.
6.1.
1.3.
3. El
El proc eso de independencia

 A. Inic
In ic i o: el Mo
Motín
tín del té en Bost on (1773). Fue la protesta de los colonos por la pretensión
Bo ston
del gobierno británico de monopolizar la venta del té. En respuesta, el Parlamento británico
impuso las Leyes intolerables: nombramiento de un gobernador y la obligación del pago por
los daños ocasionados. La respuesta de los colonos fue convocar a los congresos
continentales en Filadelfia.

B. Los cong resos continentales 


I Congreso
Congr eso Contin
Cont inental
ental (1774)
(1774) II Congr
Congreso
eso Cont
Contin
inental
ental (17
(1775-
75-178
1781)
1)
Las colonias rechazaron la política Se declaró la guerra y fue constituido el Ejército
intervencionista del Parlamento británico Continental al mando de G. Washington y, finalmente, se
y rompieron su relación comercial. adoptó la Declaración de independencia (4 (4 de julio de
1776), inspirada en los ideales del pensamiento liberal.

C. Desarrollo
Desarrollo de la guerra de ind
independencia
ependencia

  Batalla de Saratoga (1777). Triunfo importante del Ejército Continental sobre el inglés. 
  Batalla de Yorkt
Yorktown
own (1
(1781).
781). Triu
Triunfo
nfo fifinal
nal de los colon
colonos
os co
conn apoy
apoyoo de Francia, Países
Bajos y España, interesados en acabar con el poder británico.
  Tratado de Versalles (1783). Inglaterra reconoció la independencia de las ccolonias.
olonias. De
este modo, estas fueron las primeras que se independizaron de su metrópoli.

6.1.4. Cons
6.1.4. Consecuenci as  
ecuencias
  Se aprobó la Constitució
Constitución
n de EE. UU.
(1787) y se conformó la República Federal,
siendo George Washington (1789  – 1797) su
primer presidente.
  Se reconoció el derecho a la ciudadanía,
aunque restringida a los afroamericanos en los
Estados del Sur, así como de las mujeres.
  La independencia de EE. UU. impactó en
el estallido de la Revolución francesa y en la
CEPRE-UNI HUMANIDADES 82

CICLO INTENSIVO ESCOLAR NACIONAL Material de Estudio Nº 1 

independencia de las colonias españolas en Hispanoamérica, puesto que demostró la


viabilidad del pensamiento liberal. 

La Edad Contempo ránea


Tiene como punto de partida el 14 de julio de 178
17899 con la toma de La Bast
Bastilla.
illa. En esta Edad
se produjeron eventos como la Revolución Industrial, el imperialismo colonial, las guerras
mundiales, la Revolución rusa, la Guerra fría y el triunfo de la economía de mercado a nivel
global.
6.
6.2
2 La Re
Revoluc
voluc ión francesa
Fue una revolución política que permitió a la burguesía la toma del poder y eliminar las bases
del Antiguo Régimen.

6.1.1. Causas

 A. Econ ómii cas  


Ec onóm
   Crisis fiscal por los ggastos
astos en guerras, como en la guerra de los siete años y el apoyo a
la independencia de las trece colonias, y el despilfarro en la
l a Corte de Versalles.
   La crisis agrícola de los aaños
ños 1787 y 1788
1788,, que redujo la producción de cereale
cerealess e
incrementó el hambre.

B. Políticas  
La crisis del absolutismo representada en el rey Luis XVI y la marginación política contra la
burguesía.

C. Sociales
La desigualdad entre estamentos que privilegiaba a las minorías (el clero y la nobleza:
exoneradas de impuestos) y obligaba a las mayorías (el tercer estado o estado llano:
burgueses, artesanos y campesinos) a pagar impuestos, tributos y rentas.

D. Ideológi
Ideológi cas
Influencia de la ilustración que cuestionó
las instituciones del Antiguo Régimen.

6.
6.1.
1.2.
2. Desa
Desarro
rro llo de la revoluc ión
 A. Etapa
Et apa mon
m onár
árqu ic a (1789 – 1792)
quic

Los Estados Generales. Asamblea 


conformada por representantes de los
estamentos. Se buscó dar solución a la
crisis fiscal mediante una reforma tributaria
que extendía el pago de impuestos a los
privilegiados. La medida fue rechazada
r echazada por
estos y el Estado llano se retiró.
r etiró.

La Asamblea
Asambl ea N acional. Fue convocada por los representes del tercer estado en el Salón del
Nacional.
 juego de la pelota , donde juraron darle una Constitución política a Francia. Al integrarse los
privilegiados a esta asamblea, se convirtió en Constituyente.
La Asamblea Nacional
Nacional Constitu
Const itu yente
CEPRE-UNI HUMANIDADES 83

CICLO INTENSIVO ESCOLAR NACIONAL Material de Estudio Nº 1 

  El pueblo parisino asaltó la Bastilla (14 de julio), símbolo del absolutismo, lo que
incrementó el estado de violencia.
  La revolución se expandió en las áreas rurales, en
donde los campesinos asaltaron castillos y propiedades
feudales (el Gran miedo). 
  Para concontener
tener la ola de vioviolencia,
lencia, ssee aprob
aprobóó la
Declaración de los Derechos del Hombre y del Ciudadano
(la
de ciudadanía reemplazóelalAntiguo
eliminar legalmente conceptoRégimen.
de súbdito) con el fin
Incluso, fse
in
estableció la Constitución civil del clero.
   Finalmente fue promulgada la Constitución de 1791,
que estableció una Monarquía constitucional. Luis XVI dejó de ser rey de Francia  para
convertirse en rey de los franceses; es decir, su poder provenía del pueblo.  

La Asamblea Legislativa
   Conformada por clubes políticos: los fuldenses
(monarquistas), los girondinos (republicanos moderados) y los
 jacobinos (republicanos
(republicanos radicales)
   Se produjo la invasión de Austria y Prusia al territorio francés
en apoyo a la familia real.
   Ante la ttraición
raición de Lui
Luiss X
XVI,
VI, ssee arres
arrestó
tó a la familia rea
reall y fue
encerrada en las Tullerías.

B. Etapa republi
repub licana
cana (17
(1792
92 – 1804)

La Convención Nacional
A partir de 1792, la revolución entró en una fase radical y acabó
con los rezagos del Antiguo Régimen.

Fase gir ondina


Fase
  Liderada por la gran burguesía, estableció la Primera República francesa y fue ejecutado, en la
guillotina, Luis XVI, acusado de traición a la Patria.
  En la región de la Vendee
Vendee surgió
 surgió un movimiento contrarrevolucionario de campesinos católicos
y realistas. Aquello y la crisis económica permitió el ascenso de los jacobinos (pequeña burguesía)
al poder
Fase   régimen del terror  
político.El
político. 
jacobina:
  Con los jacobinos la re revolución
volución entró a su fase más
radical, quienes contaron con el apoyo de los sans culottes.
culottes.
Para contener la contrarrevolución interna establecieron el
Comité de Salvación Pública,
Pública, liderado por Maximiliano
Robespierre, y el Tribunal Revolucionario 
Revolucionario  encargado de
ordenar el aguillotinamiento
aguillotinamiento de
 de los nobles, aunque la mayoría
de las víctimas fueron miembros del tercer estado.
  Luego de la destitución de M. Robespierr
Robespierre,e, retornaron los
girondinos al poder (Reacción termidoriana).

El Directorio
  Fue limitada la participación ppolítica,
olítica, porque se ex
excluyó
cluyó del derecho al su
sufragio
fragio a los
sectores populares. Así se generó un clima de mayor descontento
descontento social.
CEPRE-UNI HUMANIDADES 84

CICLO INTENSIVO ESCOLAR NACIONAL Material de Estudio Nº 1 

  En el plano exterior, el ejército rev


revolucionario,
olucionario, liderado por Napole
Napoleón
ón Bonaparte, obt
obtuvo
uvo
victorias importantes en Egipto y afectó el poder naval de Inglaterra (lideró a las coaliciones:
alianza de monarquías europeas contrarrevolucionarias).
  En 1799, los girondinos respaldaron el golpe de Estado de Napoleón Bonaparte
(denominado 18 Brumario) para estabilizar a Francia en todos sus frentes.

El Consulado
Se instauró tras el
revolucionarias y 18 Brumario
evitar . El principal
el retorno objetivo de Sus
del absolutismo. Napoleón fue consolidar
reformas las conquistas
se consagraron con el
Código Civil: derecho a la propiedad, igualdad ante la ley, libertad de trabajo, legalización del
matrimonio civil y el divorcio.
C. El
El Imperi
Imperioo
En 1804, Napoleón instauró el Imperio. Las guerras napoleónicas permitieron la difusión de
la Revolución francesa enfrentándose a las coaliciones de los Estados europeos. Napoleón
fue derrotado definitivamente
definitivamente en la batalla de Waterloo (1815).

6.1.3 Consecuencias
Políticas Sociales
Fue abolida la monarquía absolutista y los privilegios Fueron abolidos la servidumbre y los
de la nobleza y del clero. Se separó el poder del derechos feudales, estableciendo la
Estado respecto a la Iglesia (Estado laico). igualdad ante la ley.
Económicas Culturales
Se confiscó el patrimonio del clero y fue suprimido el Se difundió la libertad de culto y expresión.
diezmo. Además, se abolieron los poderes de los Asimismo, se expandieron las bases del
gremios en las ciudades. liberalismo.

Evaluación
.
1. La Edad Moderna se inició con la toma de 4. En cuál de los siguientes enunciados
Constantinopla y concluyó con aparecen representantes del Renacimiento
A) el inicio del mercantilismo. (XV-XVI).
B) el nacimiento comercial
comercial.. A) Maquiavelo y Rotterdam
C) la caída de Roma Occidental. B) Pettrarca y Bocaccio
D) la toma de la Bastilla. C) Buonarotti y Sanzio
E) la independencia de los EEUU. D) Da Vinci y Petrarca
2. Identifica los enunciados que correspo
correspondennden E) Moro y Maquiavelo
al desarrollo de la burguesía. 5. Señale el enunciado que corresponde al
I. Aumento del poder clerical. pensamiento Humanista.
II. Surgimiento de ligas y gremios. A) Exaltó la anatomía humana.
III. Fortalecimiento de la escolástica. B) Fue patrocinado por los mecenas.
IV. Expansión del feudalismo. C) Se expresó en arquitectura y pintura.
A) I y II B) II C) III D) I y IV E) II, III y IV D) Desarrolló la técnica de la perspectiva.
E) Da Vinci fue su máximo representante.
3. Relacione adecuadamente a los
representantes del Humanismo y sus 6. La instauración del régimen denominado el
respectivas obras. Terror  en
  en Francia estuvo simbolizado por el uso
I. Petrarca a. El Decamerón de la guillotina. Este periodo llegó a su fin con
II. Bocaccio b. Sonetos a Laura A) el golpe de Estado de Napoleón.
III. Maquiave
Maquiavelo
lo c. El Príncipe B) la toma del poder por parte de los
A) Ic-IIa-IIIb B) Ia-IIb-IIIc C) Ic-IIb-IIIa girondinos.
D) Ia-IIc-IIIb E) Ib-IIa-IIIc C) las acciones contrarrevolucionarias de las
coaliciones.
CEPRE-UNI HUMANIDADES 85

CICLO INTENSIVO ESCOLAR NACIONAL Material de Estudio Nº 1 

D) la alianza entre absolutistas y gran


burguesía.
E) el surgimiento de la primera república
francesa.
7. La lucha por la independencia
norteamericanaa se inició tras la declaración del
norteamerican
4 de Julio en ___y finaliza en la batalla de ____.
A) Filadelfia –-  Saratoga
C) Yorktown B) Saratoga - Yorktown
D) Yorktown - Washingto
W ashingtonn
E) Filadelfia - Gettysburg
8. Dentro de la Revolución francesa de 1789 se
elaboró un calendario que buscaba enaltecer las
ideas renovadoras republicanas. En 1792, se
decretó el nacimiento de la era republicana con
el Año I Revolucionario donde se
A) ejecutó al rey Luis XVI.
B) inició el gobierno del terror.
C) formó el Consulado.
D) promulgó el Código Civil francés.
E) gobernó la asamblea nacional.
9. Respecto de los aportes de la Revolución
francesa tenemos
I. Constitución política
II. Declaración de Derechos
III. Estado laico
IV. Economía mercantilista
A) I B) II C) III D) I, II y III E) I, III y IV.
CEPRE-UNI HUMANIDADES 86

CICLO INTENSIVO ESCOLAR NACIONAL Material de Estudio Nº 1 

Geografía 
Geografía 
Tema 01: Teoría geográfica y elementos para la localización. Representación del espacio
geográfico

1. Teoría
Teoría geogr
geográfica
áfica

La Geografía
donde es la ciencia
se desarrolla. social que
Investiga estudiade
relaciones la interrelación entre laespacial
interdependencia sociedadentre
(humanidad) y elfísicos,
elementos medio
biológicos y humanos en un geosistema zonal, regional, continental y global. Los localiza, describe,
explica, compara e integra en un geosistema mayor. 

Geosistema: unidad funcional donde se interrelacionan componentes abióticos (litosfera, hidrosfera y la


atmósfera), bióticos (biosfera) y antrópicos (humanidad) con una dinámica particular. El enfoque sistémico es
la base para entender las relaciones hombre-medio y las transformaciones que produce.

1.1. Objeto d e e
1.1. est
studio
udio de la Geografía. Es el espacio geográfico: entorno del planeta modificado por
acción humana donde realiza sus actividades. Involucra partes superficiales del planeta conformadas
por elementos naturales y culturales. En él hallamos hechos y fenómenos
f enómenos geográ
geográficos.
ficos.

Hecho geogr áfico . Es toda acción que acontece en la Tierra y se caracteriza por su larga duración. Uno de
Hecho
sus principales agentes es la acción antrópica, que modifica la superficie terrestre en función a sus
necesidades. Ejemplo: asentamientos humanos, construcción de represas, montañas, etc.
Fenómeno geográfico. Son alteraciones violentas producidas por la naturaleza. Se caracterizan por ser
impredecibles, incontrolables y de corta duración; sin embargo, sus efectos son duraderos. Ejemplos:
inundaciones, terremotos, huaycos, huracanes, tsunamis, etc.  

1.2.
1.2. P
Princi
rinci pios geográficos. Son los fundamentos que permiten el desarrollo de investigaciones en
este campo. Los seis principios que fundamentan a esta ciencia son los siguientes:

Localización. Se encarga de ubicar, de manera exacta, el hecho o fenómeno sucedido en el espacio


geográfico. Emplea el sistema de coordenadas geográficas, que usa aspectos espaciales: latitud,
longitud, altitud, límites y superficie. Es el principio básico de esta ciencia, porque de no localizar el
objeto de estudio, no se podría estudiar. Ejemplo: localizar la ciudad de Huancayo.
Descripción. Permite establecer las características que poseen un fenómeno o hecho geográfico
por medio de la observación.  Ej. que características naturales-sociales presentan los cerros de Lima.  
Comparación
dos objetos deoestudio
analogía. Consiste
del mismo tipoenodeterminar
forma. Estounpermite
conjunto de semejanzas
comprender y diferencias
los posibles efectosentre
que
presentará uno de ellos. Ejemplo: si se activa un volcán en el sur del Perú, podemos estudiar los
volcanes activos de la región para entender cómo será su desarrollo.  
Explicación o causalidad. Busca establecer la causa que inició un fenómeno u hecho geográfico y
proponer una explicación coherente de su alcance y consecuencias. Ejemplo: ¿Cómo incide el clima
población?  
del norte peruano en las actividades de su población? 
 Actiti vidad
 Ac vi dad o evo
evolu
lu ción
ci ón.. Permite explicar la dinámica de los fenómenos y hechos geográficos por
acción de los diversos agentes (hombre y naturaleza). Ejemplo: la alteración del medio geográfico
en las playas del sur de Lima por la expansión urbana en los últimos 20 años.
Relación
Re lación o con exión. Explica las diversas relaciones e interdependencias que se generan entre los
fenómenos o hechos geográficos. Permite entender que existe una relación lógica entre las
permanencias y variaciones que sufre el espacio geográfico y no por cambios al azar. Ejemplo: ¿Qué
relación muestra la zona volcánica del sur peruano con la cordillera de los Andes y las fosas
marinas?  
marinas?

2. Localización
Localización geográfica: línea
líneass y círc
círculos
ulos i maginarios
CEPRE UNI HUMANIDADES 87

CICLO INTENSIVO ESCOLAR NACIONAL Material de Estudio Nº 1 

2.1. Polos geográficos. Son los puntos extremos del eje de rotación del planeta. Se localizan en las
áreas climáticas de baja temperatura e indican el punto de mayor latitud en el planeta. Debido a la
posición inclinada del planeta, presentan días y noches prolongados con duración hasta de 6 meses.
2.2.. Eje tterrestr
2.2 errestre.
e. O línea de los polos: une los polos geográficos norte y sur. Es
la línea geodésica más importante sobre la cual gira la Tierra. Su movimiento lo
realiza de oeste a este. En dirección norte, apunta hacia la Estrella Polar y, al sur,
apunta

a lainclinación
 Presenta constelación
deCruz delcon
23°27’ Sur.respecto a la perpendicular de la eclíptica
(ver línea en el gráfico), 66° 33’ con respecto al plano de la eclíptica y cruza
imaginariamente
imaginaria mente el centro de la Tierra.
 La inclinación del eje terrestre junto con el movimiento de traslación causan, en
el planeta, la desigual distribución de la luz y el calor en un año; asimismo,
provocan las estaciones (solsticios y equinoccio
equinoccios).
s).
2.3. Ecuador terrestre. Es el círculo máximo de la Tierra: llamado línea equinoccial. Es una línea
geodésica circular de 360º de longitud y divide al planeta en 2 hemisferios: norte (boreal, septentrional)
y sur (austral, meridional). Su equidistancia de los polos es la base para determinar los valores de
latitud, cuyo valor parte de 00°00’00’’ hasta los 90° como máximo valor. 
 Único lugar del planeta con igual duración entre día y noche.
 Cuando los rayos solares inciden en el ecuador, de modo perpendicular, el 21 de marzo y 23 de
setiembre se producen los equinoccios. Estas fechas dan inicio a las estaciones de verano y otoño
según el hemisferio donde se ubique una persona.
2. 4. Paralelos. Son circunferencias paralelas y menores al
ecuador que recorren la Tierra. Disminuyen su tamaño al
acercarse a los polos. Su posición permite expresar los valores de
latitud entre 0º (en el ecuador) y 90º (polos).
  Los más importantes son el trópico de Cáncer (23°27’N), trópico
de Capricornio (23°27’S), el círculo polar ártico (66°33’N) y el
círculo polar antártico (66°33’ S). Estos valores de latitud
delimitan las zonas térmicas del planeta.
Existen tres zonas térmicas en el planeta distribuidas en función a su posición latitudinal. La zona tropical o
tórrida, delimitada por los trópicos. Las zonas templadas norte y sur delimitadas por los trópicos (de la zona
tropical) y círculos polares correspondientes (de la zona fría). Las zonas frías o polares norte y sur delimitadas
por los círculos polares correspondientes

2.5. Meridianos
2.5.
  Son semicircunferen
semicircunferencias
cias (180°) que van de polo a polo. Su máxima curvatura al cruzar el ecuador
y se unen en los polos. Asimismo, forman ángulos rectos en el punto de cruce con los paralelos.
  Señalan los valores de longitud entre 0º, en el meridiano de Greenwich, y 180º, en el antemeridiano
base. Estos dos son los meridianos más importantes.
Meridiano de Greenwich. 
Greenwich.   O meridiano base, divide a la Tierra en dos
hemisferios. Determina los 0° de longitud. Usado para calcular la hora
internacional. Establece los hemisferios Oeste (Occidente, Poniente) y Este
(Oriente, Levante). Recorre Europa, África y Antártida.
180º. O antimeridiano base y Línea Internacional del Tiempo.
Meridiano de 180º.
Es usado para determinar el cambio de fecha de un hemisferio a otro.
Presenta diversas curvaturas para no recorrer ninguna superficie terrestre y
así evitar problemas horarios. Recorre el océano Pacífico.  

Círculo
Círculo s y semicírculo Continentes que recorren 
recorren  
Ecuador   América del Sur, África, Asia (insular) y Oceanía
Trópico de Cáncer   América del Norte, Asia y África
CEPRE UNI HUMANIDADES 88

CICLO INTENSIVO ESCOLAR NACIONAL Material de Estudio Nº 1 

Trópico de Capricornio   América del Sur, Oceanía y África


Círculo polar ártico   América del Norte, Europa y Asia
Círculo polar antártico   Antártida
Greenwich   Europa, África y Antártida
2.6. Coordenadas geográficas. Es un sistema de localización que indica la posición de un punto en
la superficie terrestre. Para ello, utiliza dos pautas de referencia: latitud y longitud. Aunque puede
usarse, en este aspecto, también la altitud. Las coordenadas geográficas permitenpermiten reconocer la
localización
localizaci
establecerónsuabsoluta.
latitud y Para localiza
localizar
longitud. Porrejemplo,
algún punto correctamente,
la ciudad de París seenubica
la superficie terrestre,
a 48°51'24"N con debemos
02°20'27"E; mientras que la ciudad de Lima se localiz a a 12°02’36’’S y a 77°01’42’’W. 

Latitud. Distancia angular de un punto respecto al ecuador. Se mide de 0º en ecuador hasta 90º
90 º en los polos
(S).  
norte o sur. Se establece con direcciones norte (N) o sur (S). 
Longitud. Distancia angular de un punto al meridiano de 0º. Se mide de 0º hasta 180º (antimeridiano) hacia
el hemisferio oeste y este. Se establece con direcciones oeste (W)y este (E). 
(E).  
Glosario
 Alti
 Al titu
tu d . Elevación o altura sobre el nivel del mar: distancia vertical desde el nivel del
d el mar (m s. n. m.). Puede
Pue de
ser positiva, sobre el nivel del mar, o negativa, bajo el nivel del mar.
 An típ od
 Antíp oda a. Punto de la superficie terrestre o lugar del globo terrestre diametralmente opuesto con respecto
a determinado punto en la superficie. Los puntos antípodas presentan latitudes equivalentes, días y noches
de igual duración. Asimismo, presentan horas opuestas (6 am. en uno y 6 pm en otro).
 Periecos
Periecos.. Puntos ubicados en un mismo paralelo, pero separados por 180°. Presentan igual estación y
latitud.
 An tec os.
 Antec os . Puntos en un mismo meridiano. Presentan igual latitud con diferente dirección (norte-sur), tienen
igual longitud y son equidistantes del ecuador. Ambos registran la misma hora y fecha.

3. Represe
Representación
ntación del espacio geográfico

Cartografía. Disciplina que se ocupa de la representación de los espacios de la superficie terrestre a través
de instrumentos: mapas, cartas y planos. En nuestro país, el Instituto Geográfico Nacional o IGN es la
institución oficial que se encarga del proceso cartográfico del país.

3.1.
3.1. Ele
Elementos
mentos cartográfico s
3.1.
3.1. Proye
Proyeccion
ccion es cartog ráficas
Por la forma de la Tierra, cercana a la esférica, se requiere de cálculos
matemáticos para hacer coincidir la forma real del terreno con su
representación gráfica. La proyección cartográfica es un sistema de
representación gráfico que establece una relación ordenada entre los
puntos de la superficie terrestre y los de una superficie plana. Estos
puntos se localizan con uso de una red de meridianos y paralelos en
forma de malla. Las proyecciones siempre muestran aproximaciones a
la realidad; pero no pueden mostrarlas sin caer en inexactitudes por la
forma del planeta. Las proyecciones cartográficas, según su origen, son
las siguientes:

 A. Cil
Cilínd
índ ri
rica
ca y UTM. La proyección cilíndrica normal es aquella que
resulta cuando la superficie de proyección es convexa. Esta, como un
cilindro, envuelve
envuelve y ttoca
oca tangencialmen
tangencialmente
te la esfera terrestre. Las zonas que
se representan con mayor fidelidad son las próximas al ecuador (baja
latitud 0º-30º); sin embargo, al avanzar a los polos, la superficie se
distorsiona.

 tangente
La proyecció
proyección
a la nlínea
normal de Mer
Mercator
ecuatorial (uncator se
s e realiza cua
paralelo). cuando
ndo el cilindr
cilindroo es
CEPRE UNI HUMANIDADES 89

CICLO INTENSIVO ESCOLAR NACIONAL Material de Estudio Nº 1 

 La proyección UTM es cilíndrica transversal. Se da cuando el cilindro es tangente a un meridiano.


Trabaja con 60 husos UTM (áreas en torno a un meridiano) de 6° de longitud cada uno. Esta
proyección da origen al Sistema de Coordenadas UTM que, a diferencia del sistema de coordenadas
geográficas, localiza en función a zonas UTM. Las distancias en este sistema se expresan en metros
únicamente al nivel del mar, que es la base de la proyección del elipsoide de referencia.

B. Proyección cónica. Esta proyección, de tipo normal, se realiza al


establecer
del eje de un cono cuyo
la Tierra. Las vértice
zonas se
queubica en un puntomejor,
se representan de prolongación
para esta
proyección, son las de latitud media, entre 30º y 60º, en la Tierra. En este
caso, los paralelos se trazan de forma equidistante y los meridianos son
rectas que se trazan hacia los polos.
C. Proyección
Proyección acimutal norm al o polar. Esta proyección se realiza al colocar
un plano tangente a uno de los polos terrestres y representar el área en el
plano. Representan con mayor fidelidad las zonas polares, zonas de alta
latitud (60º-90º); pero la superficie se distorsiona conforme nos alejamos de
ellos. Las proyecciones acimutales pueden aparecer en tres posiciones u
orientaciones: polar, ecuatorial y oblicua.
3.2. Símbolos cartográficos. Se usan para representar de manera simbólica objetos, lugares o
cualquier información en una representación cartográfica. Entre ellos tenemos pueblos, fronteras, etc.
Se presentan en un recuadro conocido como leyenda. Tienen por característica ser convencionales.
3.3. Escala. Es la relación matemática que existe entre las dimensiones reales de un área y las del
dibujo. Es la razón o equivalencia entre la distancia en el mapa y la distancia sobre el terreno. Esto se
concreta de acuerdo al propósito del mapa. Las formas de escala son las siguientes:

Escala numérica. Es expresada en cifras que Escala gráfica o lineal. Es una barra dividida en
indican la proporción entre la distancia entre dos segmentos iguales. Representa la longitud sobre el
puntos en el mapa y la correspondencia del mapa y las unidades terrestres de distancia (metros o
terreno. Por ejemplo, 1:100000 indica que una kilómetros).
unidad de medida en el mapa (1 cm) representa
100 000 de las mismas unidades (100 000 cm)
en la superficie terrestre. Este tipo de escala
también puede expresarse a través de una
fracción, por ejemplo 1/100 000. 
000.  

La gran escala es de 1/1 000 a 1/20 000, la mediana escala es de 1/ 25 000 a 1/200 000 y la pequeña escala
es de 1/250 000 a 1/1000 000.

4. Documentos cartográfico s
4.1. Mapa. Es la representación gráfica, a escala pequeña, de una porción de la superficie terrestre
que muestra algunos atributos de la realidad. Es un instrumento analógico diseñado para el registro,
exposición, análisis y, en general, comprensión de hechos geográficos y sus relaciones espaciales.
Su función es representar, sobre una superficie plana, una parte de la superficie terrestre.
Algunas de sus principales características de los mapas son las siguientes:
  Representan áreas de mayor extensión que otros documentos (Tierra, continente, país, etc.).
  Tienen en cuenta la forma esférica de la Tierra y se basan en la aplicación de proyecciones.
  Presentan información general por la extensión de territorio que abarcan.
  Son bidimensionales: en el mapa se indica valores de latitud y longitud de un lugar.
  Presentan deformaciones, porque
porque deben pasar de un sistema curvo a uno plano.
  Se elaboran en base a escalas pequeñas (1:200 000 000 a más).
Clasifi
Clasifi cación de los mapas. Según su contenido pueden ser los siguientes:
CEPRE UNI HUMANIDADES 90

CICLO INTENSIVO ESCOLAR NACIONAL Material de Estudio Nº 1 

 A. Mapas gen general


erales.
es. Es el tipo básico usado para representar áreas de la superficie terrestre. Se
caracterizan porque muestran relaciones espaciales entre diferentes elementos geográficos (naturales
o artificiales). Entre estos tenemos montañas, cursos de agua, redes de transporte, poblados, etc.
Asimismo, fronteras políticas como límites de ciudades, provincias
provincias o países. Debido a la gran cantidad
de información que poseen, se utilizan a menudo como mapas generales de consulta.
Los mapas generales de escala grande son llamados mapas topográficos. Tanto los mapas generales y/o
topográficos se consideran, habitualmente, complementarios y opuestos a los mapas temáticos. En el caso
de mapas topográficos, se elaboran de modo tridimensionales, con uso de curvas de nivel.
B. Mapas temáticos. Ofrecen información sobre temas específicos del área a estudiar (clima,
bosques, aspectos económicos, etc.). Llamados mapas de aplicación o de información particular.
Normalmente se elaboran a escalas pequeñas. Destacan los siguientes:
  Climáticos: di
 distribución
stribución de clima
climass
  Demográf
 Demográficos:
icos: distribución y densidad de una población
  Ecológicos: distribución de los ecosi
 ecosistemas
stemas
  Físicos: distribuci
 distribuciónón de aspectos físicos relev
relevantes
antes del territorio (relieve,
(relieve, hidrografía, etc.)
  Geomorfológic
 Geomorfológicos: os: distribució
distribuciónn de los diferentes tipos de relie
relieve
ve
  Hidrográficos: di
 distribución
stribución de masas de agua ccomo omo ríos y lagos
lagos
  Meteorológi
 Meteorológicos:
cos: distribución y carac
características
terísticas del tiempo atmos
atmosférico
férico
  Orográficos: dis
 distribución
tribución de llas
as cordillera
cordillerass y montañas
  Políticos: di
 división
visión político-adminis
político-administrativa
trativa
  Turísticos: distri

distribución
bución ddee atractivos tu
turísticos
rísticos
  Viales: vías de comunicación

4.2. Cartas topográficas. Son representaciones detalladas de la superficie con descripción de


posición, forma, dimensiones e identifica accidentes del terreno. Son tridimensionales y usan altitud,
latitud y longitud. Elaboradas para espacios marítimos, aéreos o terrestres. Presentan como
características:
 Se realizan con uso de escalas medianas.
 Representan
 Represe ntan áreas de mediana extensión (provincia
(provincias,
s, distritos, etc.).
 Son tridimensionales,
tridimensionales, porque determinan latitud, longitud y altitud.
 Usan curvas de nivel o líneas hipsométricas para representar altitud.
 Contienen gran información que permite tener una idea real sobre el
terreno.
 Presentan regular deformació
deformación.
n. Si en una carta las líne líneas
as se
encuentran más
pendiente. Por cerca; quiere
el contrario, decir separación
su mayor que el terreno presenta
señala mayor es más suave.
que la pendiente

Elementos altimétri cos. Determinan la variación del nivel y formas del terreno.
Elementos
- Cota. Es
Cota. Es el número, en las cartas, que indica la altura de un punto sobre el nivel del mar.
- Curvas
Curvas de nivel o línea
líneass hi psométric as.
as. Es una línea imaginaria que se traza para representar el
relieve de un lugar. Unen dos puntos de igual altitud. Algunas de sus características son las
siguientes:
 Las curvas de nivel no se cruzan entre si.

 Deben de ser lineas cerradas.


  Cuando se acercan indican un relieve más pronunciado en la superficie (mayor pendiente). La


máxima pendiente del terreno se produce cuando las curvas se unen en una misma.
 En el caso de produndidades de mares y océanos, se las denomina líneas isóbatas.

El IGN señala el uso de escala 1:100 000 para la elaboración de la Carta Nacional del Perú y, en el caso del
Mapa Oficial del Perú, el uso de la escala 1:1 000 000. También puede elaborarlos con otras escalas según
los requerimientos y las labores a desarrollar.
CEPRE-UNI HUMANIDADES 91

CICLO INTENSIVO ESCOLAR NACIONAL Material de Estudio Nº 1 

4.3. Planos. Representas áreas pequeñas de la superficie (ciudades, distritos, urbanizaci


urbanizaciones,
ones, etc.).
  Presentan información detallada del terreno y se usan escalas grandes (menor reducción).
  Son bidimensionale
bidimensionales:s: latitud y longitud. Según la necesidad pueden presentan curvas de nivel:
plano topográfico. Carecen de deformación.

Evaluación

1. Para estudiar todo fenómeno


f enómeno o hecho 4. En el análisis que desarrolla la Geografía, el
geográfico se debe establecer su posición en la estudio de una cuenca es considerado como un
superficie terrestre; es decir, se aplica el  _____; mientras
mientras que una inu
inundación
ndación
principio de corresponde a un _____.
A) conexión. B) descripció
descripción.
n. C) causalida
causalidad.
d. A) geosistema - hecho geográfico
D) activida
actividad.
d. E) localización.
localización. B) fenómeno geográfico - biosistema
C) hecho geográfico - fenómeno geográfico
2. Con respecto a las características de los D) geosistema - fenómeno geográfico
paralelos, marque la proposición correcta. E) biosistema – hecho geográfico
A) Son equidistantes al ecuador.
B) Fijan valores de longitud. 5. En una carta geográfica, las curvas de nivel
C) Todos son de igual distancia. se caracterizan por
D) Son semicírculos imaginarios. A) representar zonas de fuerte pendiente con
E) Se pueden trazar hasta cinco. su cercanía.
B) su representación gráfica donde las líneas
3. El meridiano de Greenwich se caracteriza por se cruzan.
ser C) ser represen
r epresentadas
tadas en forma de líneas rectas.
A) la línea de cambio de fecha. D) ser líneas que representan medidas de
B) el lugar donde producen equinoccios. longitud.
C) la referencia para la altitud. E) su elaboración porque unen dos puntos de
D) la base para determinar la longitud. diversa altitud.
E) el semicírculo usado para fijar latitudes.
1E 2A 3D 4C 5A

Tema
Tema II
II:: Perú e
en
n el contexto geopolític o. El P
Perú
erú y los acuerdos de integración

1. Perú
Perú en el cont
contexto
exto geopolítico. La ubicación del Perú otorga una importante ventaja con relación
ael otros países
contexto sudamericanos
continental porsituación
posee una su proyección haciaen
estratégica distintas cuencas
el continente internacionales.
internacionales.
sudamericano. El Perúa en
Respecto su
ubicación, el Perú se encuentra bajo las siguientes pautas:
   En el hemisferio sur con respect
respectoo a la línea ecuatorial.
   En el hemisferio occidental con respecto al meridiano de Greenwich
Greenwich..
   En la zona tropical del sur.
   En la parte centro occidental de América del Sur.

El Perú
Perú en el contexto geopolítico m
mundial.
undial. Nuestro país se caracteriza por lo siguiente:

Marítimo. Puesto que limita, por el oeste, con el océano Pacífico y le corresponde el territorio
terr itorio llamado Mar de
Grau, tiene acceso directo al entorno a través de un litoral de más de 3 mil km con recursos muy diversos.
Esta situación convierte al Perú en un país con importante potencial pesquero.

 An dino
 Andi no.. Es un país andino central. La cordillera de los Andes tiene una importante extensión en nuestro país:
lo recorre longitudinalmente. Esta geoforma es columna vertebral del territorio, principal divisoria de aguas
que separa cuencas y provoca la gran biodiversidad (variedad climática, florística y faunística) del país. El
CEPRE-UNI HUMANIDADES 92

CICLO INTENSIVO ESCOLAR NACIONAL Material de Estudio Nº 1 

Estado debe asumir la responsabilidad de explotar en forma racional y sostenible los recursos y sus atr
atractivos.
activos.
Asimismo, permite impulsar la integración subregional, promover el desarrollo económico y social de los
países miembros del CAN.  
 Am azón ic o. Abarca más de la mitad del territorio
 Amazón terr itorio nacional. Somos el 2° país con mayor
m ayor superficie de bosque
amazónico. Sus ríos interconectan cuencas internacionales del Amazonas, Orinoco, La Plata, nos permite
vincular y expandir nuestras posibilidades de desarrollo comercial. 
Eje medio
medio del Pacífico
Pacífico sur. Su ubicación central en la costa del continente, por nuestra ubicación en torno
al océano Pacífico, permite gozar del flujo comercial que se vive en la actualidad hacia otros sectores de
América, Asia, Oceanía. Por ello, mejorar e incrementar la infraestructura portuaria es importante para
fortalecer el desarrollo nacional.
Bioceánico. Su acceso a dos grandes océanos hace que conecte con mercados mundiales.
 Océano Pacífico: más extenso del mundo, con él se accede a países de América, Asia y Oceanía.
 Océano Atlántico: al este, a través del río Amazonas, permite el acceso a países del Atlántico y
del Medio Oriente. Este acceso se encuentra limitado porque falta incrementar las carreteras de
penetración del norte, centro y sur hasta los puertos fluviales.
 An tár tico
 Antár ti co.. El Perú posee un territorio asignado en este continente. Tiene acceso desde el 1 de abril de 1981
como parte del Tratado Antártico. El Perú muestra presencia a través de la estación científica Machu Picchu.
Desde 2018, la política nacional en este sector es liderada por la Dirección General de Soberanía, Límites y
Asuntos Antárticos del Ministerio de Relaciones Exteriores.

2. Dimensiones espaciales. Con base a la Constitución Política, el territorio peruano comprende el


suelo, el subsuelo, el dominio marítimo y el espacio aéreo que lo rodea.
Superficie continental: 1 285 215.60 Km 2  Superficie terrestre: 1 280 085.92 Km2 
Superficie lacustre: 4 996. 28 Km2  Superficie insular: 133.40 Km2 
Superficie marítima: 626 240.00 Km 2  Perímetro: 10 153 Km

3. Puntos extremos. Permite conocer aquellos lugares del territorio peruano que tienen algún récord
nacional debido a su localización. Los más importantes son los siguientes:
Más boreal Talweg del río Putumayo en la localidad de Güeppí, Loreto, a 0º01’48’
0º01’48’’S
’S 
Más austral Punto de la Concordia, Tacna, a 18º21’08’’S 
Más oriental Confluencia de los ríos Heath y Madre de Dios a 68º39’27’’Oeste. 
Máss occidental
Má Punta Balcones o Pariñas, Piura, a 81º19’34.5’’W  
Más alto: Nevado Huascarán, Áncash, 6 768 m s. n. m.
Más bajo Depresión Bayóvar, Piura, a 37 m s. n. m.

4. Organiza ción política y administr ativa. La Ley de Bases de la


Organización
Descentralización, promulgada en 2002, regula la estructura y
organización del Estado en forma democrática, descentralizada y
desconcentrada, correspondiente al Gobierno Nacional, gobiernos
regionales y gobiernos locales. El territorio de la República del Perú
está integrado por regiones políticas (25), departamentos (24),
provincias (195), distritos (1840) y centros poblados (2300), en cuyas
circunscripciones se constituye y organiza el Estado y gobierno a nivel
nacional, regional y local, conforme a sus competencias y autonomía
propias, preservando la unidad e integridad del Estado y la nación. El
gobierno nacional tiene jurisdicción en todo el territorio de la
República; los gobiernos regionales y los gobiernos municipales la
tienen en su respectiva circunscripc
circunscripción
ión territorial.
CEPRE-UNI HUMANIDADES 93

CICLO INTENSIVO ESCOLAR NACIONAL Material de Estudio Nº 1 

4.1. Proceso de regionalización. La Ley 27783, de Bases de la Descentralización, define a las


regiones como unidades territoriales geoeconómicas, con diversidad de recursos naturales, sociales
e institucionales, integradas de modo histórico, económico, administrativo, ambiental y cultural, que
comportan distintos niveles de desarrollo, especialización y competitividad productiva, sobre cuyas
circunscripciones
circunscripcio nes se constituyen y organizan gobiernos regionales.

Este proceso se inicia eligiendo gobiernos regionales en los actuales departamentos y la Provincia
Constitucional del Callao,
República no integra conforme
ninguna región, ala Ley. Se han formado
competencia la tiene la25Municipalidad
regiones políticas. La capital
Metropolitana de la
de Lima.
No hay que confundirla con la región Lima, cuya capital es Huacho.

La regionalización es un proceso que se ha tratado en diversos momentos; porque es un paso importante


para su modernización y descentralización. En 2005, se dio el primer paso democrático para la integración en
macrorregiones políticas; sin embargo, el referéndum fracasó por la negativa de la mayoría de regiones (15
de 16). A la fecha, hay diversas tentativas para reorganizar el proceso, pero sin resultados satisfactorios.

4.2. Municipalidades. El Ley 27783 define las municipalidades como órganos de gobierno local que
se ejercen en circunscripciones provinciales y distritales de cada una de las regiones del país, con las
atribuciones, competencias y funciones que les asigna la Constitución Política, la Ley Orgánica de
Municipalidades y la presente Ley. En la capital de la República, el gobierno local lo ejerce la
Municipalidad Metropolitana de Lima. En centros poblados funcionan municipalidades conforme a ley.

Regiones
políticas Capitales Regiones
políticas Capitales
1 Ancash Huaraz 14 La Libertad Trujillo
2 Amazonas Chachapoyas 15 Lima Huacho
3 Apurímac Abancay 16 Loreto Iquitos
4 Arequipa Arequipa 17 Moquegua Moquegua
5 Ayacucho Ayacucho 18 Pasco Cerro de Pasco
6 Cajamarca Cajamarca 19 Madre de Dios Puerto Maldonado
7 Callao Callao 20 Piura Piura
8 Cusco Cuzco 21 Puno Puno
9 Huancavelic
Huancavelicaa Huancavelica 22 San Martín Moyobamba
10 Huánuco Huánuco 23 Tacna Tacna
11 Ica Ica 24 Tumbes Tumbes
12 Junín Huancayo 25 Ucayali Pucallpa
13 Lima Metropolitana: capital Lima. Caso de régimen
Lambayeque Chiclayo especial, pues no conforma región política.
Fron teras del Perú. La frontera es la franja exterior y vulnerable, adyacente al límite internacional,
5. Fron
que envuelve el territorio del Estado y donde llega su soberanía. La frontera se puede delimitar a
etc. 
través de hitos, montañas, ríos, etc. 
Fijación de las fron teras. Las fronteras son franjas territoriales que están fijadas en ambos lados del territorio
y se establecen a través de un tratado bilateral. Se pueden dividir en,
- Natur
Natur ales. En
ales. En este caso, el punto de referencia sobre un terreno está establecido por accidentes naturales.
Entre ellos tenemos el divortium aquarum (cadenas montañosas que separan las cuencas), el talweg (línea
río).  
que une los puntos más profundos del río) y el áleo (línea que corre a igual distancia de las orillas de un río). 
Artificiales. Construcciones elaboradas por el ser humano. Pueden ser hitos, muros, canales, etc. 
- Artificiales. Construcciones etc.  

Límite: Pa
País
ís Re
Regiones
giones Accidentes Acuerdos Año
Ecuador Loreto, Amazonas, Boca Capones - Protocolo de Río 1942
Rio Zarumilla
Norte 1529 Km Piura, Tumbes
Cajamarca Cordillera del Cóndor de Janeiro
- Acta de Brasilia 1999
CEPRE-UNI HUMANIDADES 94

CICLO INTENSIVO ESCOLAR NACIONAL Material de Estudio Nº 1 

Noreste Colombia
Loreto Río Putumayo Salomón-Lozano 1922
1506 Km
Este Brasil Loreto, Ucayali, Ríos Amazonas, Yavarí, río Velarde-Río 1909
2822 Km Madre de Dios Breu, río Acre, río Yaverija Branco
Sureste Bolivia Puno, Tacna, Río Heath, río Suches, río Polo-Bustamante 1909
1047 Km Madre de Dios Desaguadero,
Desaguade ro, Lago Titicaca
Chile Meseta de Ancomarca,
A ncomarca, Rada y Gamio- 1929
Sur 169 Km Tacna Cordillera El Frayle,
Blanca, Pascana delLaguna
Hueso Figueroa Larraín
Tratado de Lima  o
- En 2011, el Acuerdo de Límites Marítimos entre Perú y Ecuador delimitó formalmente estos
espacios entre ambos países.
- En 2014, se emitió el fallo de la Corte de La Haya sobre la delimitación marítima entre Perú y Chile.
Sus puntos se vienen llevando a cabo a través de las reuniones 2+2 entre ministros de Relaciones
Exteriores y Defensa de ambos países.

5.1.
5.1. Espacios front erizos 

- Región fronteriza. Espacio fronterizo de mayor extensión. Es de ámbito subnacional para


programaciónn y gestión del desarrollo. Coincide con las regiones políticas. Se caracteriza porque hay
programació
vinculación cultural entre pobladores, organización de actividades en torno a un centro urbano o capital
regional, integración y proyección a cada uno de los países fronterizos, etc. En Perú, de sus 25
regiones, 9 son fronterizas. Estas son Tumbes, Piura, Cajamarca, Amazonas, Loreto, Ucayali, Madre
de Dios, Puno y Tacna.
- Zona de frontera. Espacio menor a la región fronteriza. En ella, existen centros urbanos y ejes de
articulación dotados de equipamientos básicos y roles funcionales diferenciados que facilitan la
ejecución de acciones de desarrollo. Asimismo, proporcionan soporte y apoyo al área de frontera. Es
el espacio de articulación entre el área de frontera y la región política fronteriza.

- Área de frontera. Es la manifestación tangible del fenómeno fronterizo a un nivel local. Puede ser
un distrito o un área designada. En ella tenemos aduanas, puestos de vigilancia, servicios como
cambio de moneda, etc. Es el espacio de mayor vulnerabilidad ante otro Estado.
En el Perú hay 25 provincias y 81 distritos fronterizos. La mayor vulnerabilidad la
presentan los distritos de orientales por falta de vías y medios
m edios de comunicación, y la
escasa intervención estatal.

5.2. Integración
Estados en sus fronteriza. Es un proceso
espacios fronterizos orgánicoBusca
colindantes. acordado por dos
promover el
desarrollo sobre la base del aprovechamiento complementario de
potencialidades,
potencialidades, recursos, costos y beneficios compartidos.

5.3. Desarrollo fronterizo. Es el proceso de satisfacción de necesidades


básicas de la población en los espacios de frontera y su incorporación a la
dinámica del desarrollo nacional, mediante el despliegue de iniciativas
públicas y privadas orientadas hacia los campos económico, ambiental,
social, cultural e institucion
institucional.
al. Así como el ffortalecimi
ortalecimiento
ento de las capacidad
capacidades
es
de gestión local y regional, según criterios de sostenibilidad, desarrollo
humano y seguridad nacional.

6. El
El Perú y los acuerdos de integración
Los organismos internacionales son asociaciones voluntarias de Estados por acuerdo internacional,
dotadas
colectivosdey órganos
capacespermanentes, propios
de expresar una e independientes,
voluntad jurídicamenteencargados de gestionar
distinta de las unos intereses
de sus miembros. En la
actualidad, no es posible que los Estados mantengan una vida aislada de los demás. Por eso, tienen
CEPRE-UNI HUMANIDADES 95

CICLO INTENSIVO ESCOLAR NACIONAL Material de Estudio Nº 1 

que concertar, pactar en los aspectos económico, político, social, cultural, etc., y crear condiciones
mundo. 
necesarias para recibir un mejor trato de parte de las grandes potencias del mundo. 

Relaciones internacionales. El Estado, en ejercicio del poder nacional emanado del pueblo, realiza
Relaciones
su actividad política dentro del límite de sus fronteras dirigidas a alcanzar los objetivos nacionales que
permitan, a su vez, el logro del bienestar social y la seguridad integral de la nación. Sin embargo, el
logro de los objetivos nacionales requiere de una actividad política estatal más allá de sus fronteras,
que le permita establecer relaciones con
Relaciones Sonotros Estados. Así
las relaciones surgen
entre doslas relaciones
Estados. .
internacionales.
internacionales
Ejemplo Perú-Rusia;
bilaterales Perú-China.
Tipos de Relaciones Los Estados se vinculan en un ámbito más amplio y motivados
relaciones multilaterales por intereses comunes. Ejemplo OEA, ONU. 
ONU. 
internacionales Resultan de la convergencia de objetivos e intereses
Relaciones
fundamentalmente de carácter económico (Cooperación y
comunitarias
ayuda mutua). Ejemplo SELA, Aladi, CAN, Unasur .
6.1. Acuerdos de integración. 
integración.  Permiten unir esfuerzos para conseguir la solución conjunta a
problemas o buscar beneficios comunes entre las naciones que lo suscriben. De esta manera, es
posible alcanzar metas en diversos campos que los Estados demanden.
Tratado de Libre Comercio (TLC).
(TLC) . Los tratados de libre comercio son acuerdos binacionales o
regionales donde se busca eliminar o reducir los aranceles para poder ampliar el mercado de bienes
yunservicios
tratado entre los comercio
de libre países participantes.
se encuentranEntreEE.
losUU.,
principales
Chile, países
Canadá,conSingapur,
los que elAustralia,
Perú ha firmado
China,
Unión Europea, EFTA, Chile. Aún se encuentra en negociaciones un TLC con la Unión Económica
Euroasiática (que incluye a Rusia), India e Indonesia. El diario Gestión señaló (10 octubre 2017)
que a la fecha tenemos 19 Tratados de Libre Comercio.
Foro de Cooperación Asia-Pacífico (APEC). (APEC). Es un foro multinacional tiene como objetivo
promover el comercio, la cooperación, el desarrollo regional y asesoría técnica de los países y
territorios de la cuenca del Pacifico. En el Perú, la APEC se celebró el año 2008. Agrupa a las 21
economías más dinámicas de la cuenca del Pacífico y Perú es una de ellas. Tiene su sede en
Singapur. En 2018, se desarrolló en Papúa Nueva Guinea.
Comunidad de Estados Latinoamericanos y Caribeños (Celac). Nació en 2010, en la XXI
Cumbre del Grupo de Río, que fue su antecesor. Es un mecanismo intergubernamental de ámbito
regional para el diálogo y concertación política. Incluye 33 países de América Latina y el Caribe.
Dentro de sus objetivos, busca mejorar las relaciones en Latinoamérica con base en el respeto al
Derecho Internacional, la igualdad soberana de los Estados y afrontar el reto de la integración.
Asimismo,
conforman el no uso
todos de la fuerza,
los países la preservación
de América, de de
a excepción la democracia y el yrespeto
Estados Unidos a los DD. HH. Lo
Canadá.
 Asoc
 As ociac
iació
iónn Est
Estrat
ratégi
égica
ca Amér
Américic a Lat
Latin
ina,
a, el Cari
Caribe
be y la Uni
Unión
ón Eur
Europopea
ea (AL
(ALC-UE)
C-UE).. Es una
asociación birregional de Estados que busca el diálogo, cooperación y concertación política para
promover el desarrollo en temas económicos, sociales, culturales, educativos, etc., a través del
desarrollo sostenible. Surge de la unión de los países que conforman el UE y la Celac. Promueve la
Cumbre de América Latina, el Caribe y la Unión Europea, reunión de mandatarios celebrada cada
dos años. La primera de ellas se llevó a cabo en Río de Janeiro en 1999. En esta cumbre se busca
discutir y analizar problemas comunes entre los grupos reunidos. Asimismo, ver los avances
alcanzados en integración económica, reducción de pobreza, justicia social y equidad. El Perú fue
país anfitrión el año 2008. Sus acciones son coordinadas, desde 2011, por la Fundación EU-LAC
Hamburgo. 
con sede en Hamburgo. 

7. Organizaciones internacionales. Su función es encontrar soluciones para las tareas políticas,


económicas, militares o sociales que los miembros acordaron manejar. Organizaciones como la ONU,
la UE y la OEA son asociaciones multilaterales de carácter público intergubernamental; pero existen
entes independientes de los gobiernos como Amnistía Internacional
I nternacional y Greenpeace.
CEPRE-UNI HUMANIDADES 96

CICLO INTENSIVO ESCOLAR NACIONAL Material de Estudio Nº 1 

7.1. Organizació
7.1. Organización n de llas
as Naciones Unidas (ONU
(ONU).
). Fundada oficialmente, el 24 de octubre de 1945,
cuando representantes de 51 Estados firmaron el acta fundacional: la Carta de la ONU. Es la mayor
organización
organización internacional multilateral del mundo. Su sede principal se encuentra en New York. En la
actualidad, 193 Estados son miembros de esta organización y están representados en el órgano
deliberante: la Asamblea General. Cada uno de ellos es un miembro de la Asamblea General. Los
Estados son admitidos como miembros de las Naciones Unidas por una decisión de la Asamblea
General a recomendación
Los principales del
órganos de la Consejo dedeSeguridad. 
Organización Seguridad.   Unidas son los siguientes:
las Naciones
- Asamblea General - Consejo de Seguridad
- Consejo Económico y Social - Consejo de Administración Fiduciaria
- Corte Internacional de Justicia - Secretaría de la ONU
Todos ellos se crearon en 1945 al fundarse la ONU.

Entre sus propósitos primordial


primordiales
es tenemos los siguientes (www.un.org):
 Mantener la paz y la seguridad internacional. Con este objetivo, la Organización intenta prevenir
conflictos y poner de acuerdo a las partes implicadas. El Consejo de Seguridad es el máximo
responsable de la paz y la seguridad internacional.
 Lograr la cooperación internacional en la solución de los problemas de carácter económico, social,
cultural o humanitario
humanitario..
 El desarrollo y estímulo para el respeto a los derechos humanos: libertades fundamentales de todos,
sin distinción por motivos de raza, sexo, idioma o religión.

  Crear condiciones bajo las cuales puedan mantenerse la justicia y el respeto a las obligaciones
emanadas de los tratados y de otras fuentes del derecho internacional.

La Declaración Universal de los Derechos Humanos (1948) fue el primer documento legal de
protección
protecci ón de estos derechos. Junt
Juntoo con el Pacto Intern
Internacional
acional de Derechos Civi
Civiles
les y Políticos
(1966) y el Pacto Internacional de Derechos Económicos, Sociales y Culturales (1966). Los tres
instrumentos forman la llamada Carta Internacional de los Derechos Humanos.

 As amblea
 Asamb lea Gener al
al.. Es el órgano representante, normativo y deliberativo de la ONU. Tiene representación
universal por estar representados sus 193 Estados Miembros. Estos se reúnen cada año, en setiembre. La
toma de decisiones en la Asamblea General requiere una mayoría de dos tercios cuando se trata de asuntos
de vital importancia: temas referidos a la paz y la seguridad, admisión de nuevos miembros y asuntos
presupuestarios. Las decisiones en otras cuestiones se toman por mayoría simple. Cada año, se elige a un
presidente, que ejerce el cargo durante un año.  
Corte Internacional
Internacional de Ju sticia. Llamada Tribunal Internacional de Justicia es el órgano judicial
ju dicial principal de
la ONU. Está
Estados encargada
y emitir de decidir,
opiniones conforme
consultivas sobrealcuestiones
Derecho Internacional, controversias
jurídicas planteadas de ordeno jurídico
por órganos entre
instituciones
especializadas de la ONU. Su sede se encuentra en La Haya, Países Bajos.
Secretaría.. Está encabezada por el Secretario General. Realiza el trabajo estipulado por la Asamblea y los
Secretaría
otros órganos. El personal de la ONU es reclutado a nivel internacional y local, y trabaja en lugares de destino
y en las misiones de paz en todo el mundo. Desde la fundación de las Naciones Unidas, cientos de hombres
y mujeres abnegados han perdido su vida al servicio
ser vicio de la causa de la paz. El peruano
p eruano Javier Pérez de Cuéllar
ocupó el cargo entre 1982 a 1991.  

7.2. Unión de Naciones Sudamericanas (Unasur). Creada en 2007. Con sede en Quito. Entre sus
objetivos está construir, de modo participativo y consensuado, un espacio de integración cultural,
social, económica y político, con respeto por la realidad de cada nación. Asimismo, eliminar la
desigualdad, impulsar el desarrollo con equidad e inclusión, fortalecer la democracia y reducir
asimetrías existentes, considerando la soberanía e independencia de los Estados. También alude a
la cooperación comercial, financiera y energética para el uso sostenible de recursos. Los integrantes
de este organismo son los países de Brasil, Chile, Colombia, Ecuador, Guyana, Paraguay, Uruguay,
Surinam, Perú y Venezuela.
CEPRE-UNI HUMANIDADES 97

CICLO INTENSIVO ESCOLAR NACIONAL Material de Estudio Nº 1 

La Comunidad Sudamericana de Naciones (CSN), creada en Cuzco, en 2004. El 17 de abril del 2007, los
presidentes de Sudamérica, reunidos en la isla Margarita, decidieron renombrar a la comunidad como Unión
de Naciones Sudamericanas (Unasur).
En Unasur intervienen países de la CAN, del Mercosur, del Caricom y Chile, que no forma parte de
ningún bloque comercial. Entre sus objetivos destaca la concertación y coordinación política,
ciudadana y diplomática en la región y la convergencia entre Mercosur y CAN en una zona de libre
comercio.
En 2018, Colombia, Argentina, Brasil, Chile, Paraguay y Perú anunciaron su retiro temporal.
7.3. Comuni dad And
7.3. Andina
ina de Naciones (CAN). Es un organismo regional. Constituido en 1969, con la
firma del Acuerdo de Cartagena, como Pacto Andino. Su sede se ubica en la ciudad de Lima (Perú).
En 1996, se estableció como Comunidad Andina de Naciones. Entre sus objetivos busca organizar
una mejor integración económica de los Estados andinos para favorecer el desarrollo equilibrado y
armónico de la región, eliminar diferencias en el nivel de vida y establecer un mercado común
latinoamericano. Impulsa el equilibrio de aspectos sociales, culturales, económicos, políticos y
comerciales. Sus países miembros son Bolivia, Ecuador, Colombia y Perú. Como países asociados
están Argentina, Brasil, Chile, Paraguay y Uruguay. España es un país observador. Venezuela se
retiró en 2006.
7.4. Mercado Común del Sur (Mercosur) . Es un proceso de
integración regional.
regional. Se ffundó
undó con el Tratado de Asunción firmado en Mercosur tiene como
Estados asociados a Chile,
1991. Su sede está en Montevideo, Uruguay. Está integrado por
Argentina, Brasil, Paraguay, Uruguay. Venezuela fue suspendida y Colombia,
GuyanaPerú, Ecuador,
y Surinam.
Bolivia está en proceso de adhesión. Tiene los siguientes objetivos:
 Propiciar un espacio común que genere oportunidades comerciales y de inversiones a través de la
integración competitiva
competitiva de las economías nacionales al mercado internacional.
 La libre circulación de bienes, servicios y factores productivos entre los países, a través, entre otros,
de la eliminación de derechos aduaneros y restricciones no arancelarias a la circulación de
mercaderías y de cualquier otra medida equivalente.
 El establecimiento de un arancel externo común y la adopción de una política comercial común con
relación a terceros Estados o agrupaciones de Estados y la coordinación de posiciones en foros
económicos comerciales regionales e internacionales.
7.5. Sistema Económico Latinoamericano (SELA). Formado en 1975 mediante el Convenio
Venezuela. Es un organismo intergubernamental
Constitutivo de Panamá. Con sede en Caracas, Venezuela.
regional. Entre objetivos está promover un sistema de consulta y coordinación para concretar
posiciones y estrategias comunes de América Latina y el Caribe en lo económico. Asimismo, impulsar
la integración y la cooperación. Representa a sus miembros en conjunto, en foros mundiales como
ante terceros. Lo conforman 26 países: de América del Sur, América Central, México y países
caribeños.
7.6.. Organización del Tratado d
7.6 de
eCCoop
ooperación
eración Am azónica (OTC(OTCA).A). En 1978, se elaboró el Tratado
de Cooperación Amazónica. En 1995, sus 8 miembros decidieron poner en práctica la OTCA para
fortalecer e implementar los objetivos del tratado. Sus países miembros son Perú, Brasil, Bolivia,
Ecuador, Colombia, Venezuela, Guyana y Surinam. Su sede en Brasilia, Brasil. Busca promover el
desarrollo armónico de los territorios amazónicos. Para ello, incentiva el desarrollo sostenible, la
inclusión social de la región, la incorporación de sus territorios a las respectivas economías nacionales.
Todo esto con miras para lograr un equilibrio entre el crecimiento económico y el medio ambiente
mediante los principios del desarrollo sostenible.

7.7. Organización d e E
7.7. Estado
stadoss Ameri canos (OE
(OEA).
A). Es antecedido por la 1ª Conferencia Internacional
Americana de Washington en 1890; por lo que es el más antiguo sistema de integración política
regional
Tiene sudel mundo.
sede La OEA fueEstá
en Washington. creada en Bogotá,por
conformada en35
1948, cuando
Estados se suscribió ladeCarta
independientes de la Entre
América. OEA.
1962 y 2009, Cuba fue expulsada; sin embargo, en ese año, se dejó sin valor la exclusión. Entre sus
CEPRE-UNI HUMANIDADES 98

CICLO INTENSIVO ESCOLAR NACIONAL Material de Estudio Nº 1 

objetivos destaca promover la paz y seguridad, la democracia, los derechos humanos, fomentar la
solidaridad,
solidaridad, ffortalecer
ortalecer la colaboración y defender su soberanía, integridad territorial e independencia
de sus Estados miembros. Al mismo tiempo, el desarrollo social y económico del continente. Aunque
su papel actual está siendo cuestionado por no cumplir con sus objetivos.

La Corte Interamericana de Derechos Humanos (Corte IDH) es el órgano judicial de la OEA que es autónomo
frente a sus demás órganos, con sede en San José de Costa Rica. Aplica e interpreta la Convención
Americana sobre Derechos
Sistema Interamericano de Humanos
DerechosyHumanos.
otros tratados
Losde derechos
idiomas humanos
oficiales de laa los cuales
Corte son se
el somete
español,el francés,
llamado
inglés y portugués.

Evaluación

1. Sobre la localización del Perú, señale las A) Protocolo de Río B) Velarde  – Río Branco
proposiciones
proposicion es verdaderas
verdaderas.. C) Tratado de Lima D) Tratado de Ancón
I. Sus actividades son propias de un país E) Polo – Bustamante
templado.
II. Se encuentra en el hemisferio boreal. 4. País que fue expulsado de la Organización de
III. Está en la parte centro occidental de Estados Americanos (OEA) en la década de los
Sudamérica. 60.
IV. Se encuentra al oeste del meridiano base. A) Venezuela B) Guatemala C) Haití
A) I, II B) II, III C) I, III D) II, IV E) III, IV D) Cuba E) Bolivia

2. En cuanto a los espacios fronterizos, dónde se 5. Los países de la cuenca del Río de la Plata
puede aplicar las medidas más específicas y forman parte de un organismo internacional
urgentes para mostrar una presencia ante los conocido como ________, el cual tiene un
países vecinos. componente fundamentalmente __________.
A) Región fronteriza B) Zona de frontera A) Mercosur - económic
económicoo B) CAN - científico
C) Frontera viva D) Área de frontera C) APEC – militar D) Unasur - político
E) Puntos de delimitación E) Aladi - medioambiental

3. Es el tratado de límites por el cual nuestro


nuest ro país 1E 2D 3C 4D 5A
cede a perpetuidad el territorio de Arica.

Tema
Tema III: Ge
Geogr
ografía
afía H
Humana.
umana. Dinámica pob lacio
lacional
nal

1. Demografía. Es la ciencia que estudia estadísticamente la estructura, dinámica y característica


característicass de
la población humana. Entre sus fines está analizar las dimensiones de las poblaciones, conocer su
estructura y determinar los fenómenos demográficos que condicionan la vida de cada población.

Demografía cuantitativa. Estudia factores como tasa de natalidad, mortalidad y migraciones. 


migraciones.  
Demografía cualitativa. Estudia características como sexo, edad, educación y salud.
1.1.
1.1. Fue
Fuentes
ntes de i nformació n d emográfica
 Censo.
Censo.   Principal fuente de datos demográficos. Describe estadísticamente la distribución de la
población por edad, sexo, nivel de instrucción, ingreso per cápita y sus características cuantitativas.
 Registro civil.
civil . Organismo administrativo que coordina y ejecuta actividades en materia registral de
la población. Desarrolla la inscripción de hechos vitales: nacimientos, matrimonios y defunciones. En
Perú, esta función la desarrolla el Registro Nacional de Identificación y Estado Civil (Reniec).
 Mue
Muestreos-encuestas
streos-encuestas (proyecciones).
(proyecciones). Son
 Son herramientas de investigación estadística específica de
la población. Su función básica es determinar que parte de una población debe examinarse con la
finalidad de hacer inferencias sobre dicha población.
CEPRE-UNI HUMANIDADES 99

CICLO INTENSIVO ESCOLAR NACIONAL Material de Estudio Nº 1 

El Instituto Nacional de Estadística e Informática o INEI es el organismo estatal encargado de producir y


difundir información estadística oficial que el país necesita con calidad, oportunidad y cobertura requerida.
Elabora los censos de población, de vivienda, de empresas, agrarios, etc. Proporciona información
inform ación estadística
para el diseño, monitoreo y evaluación de políticas públicas y el proceso de toma
tom a de decisiones de los agentes
socioeconómicos, el sector público y la comunidad en general en el Perú.
1.2. Dinámica demográfica o poblacional. Es el desarrollo y
los cambios que registra la población en un determinado tiempo
y espacio. Para determinar el ritmo de evolución de la población
se emplean los censos. Asimismo, estos se pueden analizar
mediante las pirámides poblacionales.
1.3. Tipos de pirámides poblacionales
- Expansiva
Expansiva.. Con base ancha y cima muy angosta. En este tipo
de población predomina la de tipo joven y menor la de tipo
longeva. Esto indica que hay una alta natalidad y elevada
mortalidad.
- Regresiva
Regresiva.. Con base más estrecha que el cuerpo central. Muestra que la población joven tiene
una tendencia a disminuir.
- Estacionaria
Estacionaria.. Posee igualdad entre población joven y adulta: muestra una tendencia al equilibrio.
2. Distri
Distri bución de la población en el mundo. La distribución desigual de la población en el mundo
considera las siguientes causas naturales ( Clima, altitud y latitud, topografía, tipo de suelo,
abundancia
la población de agua) ypodemos
mundial, humanasmencionar
 (  Culturales, industrializ
industrialización,
lo siguiente: ación, eevolución
volución demográfica)
demográfica).. En cuanto a
 Se calcula, hoy en día, una población de mayor a 7 700 millones de habitantes (ONU, 2019). En
2011, se estimaba era de 7 000 millones.
 Asia es el continente más poblado. Posee el 60% de la población mundial (4 500 millones aprox.).
Los países más poblados son China (más de 1 398 millones) e India (más de 1 373 millones). Ambos
suman el 37% de la población mundial.
 África es el segundo continente con mayor población (más de 1 300 millones). Donde Nigeria es el
país más poblado, seguido por Etiopía y Egipto.
 América
 Amé rica es el tercer continente más poblado con más de 1 028 millones. Los países más poblados,
al 2017, son (1) Estados Unidos, (2) Brasil, (3) México, (4) Colombia, (5) Argentina, (6) Canadá, (7)
Perú. América Latina posee alrededor de 9% de la población mundial con un aproximado de 660
millones.
 El continente europeo en cuarto lugar con un aproximado de
habitantes. 
743 millones de habitantes. 
  Oceanía es el continente menos poblado del mundo con 41
millones de habitantes.

3. Población en el Perú. El 25 de junio de 2018, el Instituto


Nacional de Estadística e Informática (INEI), presentó los
resultados del último censo: XII Censo de Población, VII de
Vivienda y III de Comunidades Indígenas o Censo peruano de
2017. 
- Ritmo de crecimiento de la población peruana. El INEI
señala que la tasa de crecimiento promedio anual referente a la
población en el periodo 2007-2017 es de 1.0%. El ritmo decreciente poblacional se explica por la
reducción de niveles de fecundidad como consecuencia de las políticas de control de natalidad. Se
muestra una tendencia decreciente del ritmo de crecimiento poblacional en los últimos 56 años.

- Características de la población peruana. Según el censo 2017, la población peruana es adulta


(15 a 60 años), abarca la mayor proporción con 61,7% del total. Se observa un ligero incremento en
CEPRE-UNI HUMANIDADES 100

CICLO INTENSIVO ESCOLAR NACIONAL Material de Estudio Nº 1 

la densidad poblacional (24,3 Hab./Km2) y la disminución del índice de fecundidad (2,2) en


comparación con las cifras del censo realizado en el año 1993.
Gruposs de edad
Grupo 1993
1993 2017
2017
TOTAL 22 1555,4 (miles)
(mi les) 31 237,3 (miles)
0-14 37,0 % 26,4 %
15-59 56% 61,7%
60 y más 7,0% 11,9%

- Estructura de la población peruana por


edad y sexo. Según el Censo del 2017, la Los distritos más poblados están en Lima y Callao. San
población masculina del Perú fue de 14 Juan de Lurigancho (1 038 495 de pobladores), seguido
millones 450 mil 757 (49,2%) y la población por San Martín de Porres, Ate, Comas, Callao, Villa María
femenina fue de 14 millones 931 mil 127 del Triunfo, Villa El Salvador, San Juan de Miraflores,
(50,8%).

- Indicadores demográficos. Vienen a ser toda la información que se obtiene y dispone para conocer
estadísticamente la población en una determinada región. Los principales indicadores demográficos
son los siguientes:

Población absolut a. Número total de habitantes en un territorio (distrito, provincia,


provincia, país, etc.).
Población relativa. Llamada también densidad poblacional. Se refiere al grado de concentración
2
de la población
tenemos o el número
que considerar de habitantes
a la población existentes
absoluta y el áreapor
totalKm
de .laPara obtener este indicador,
superficie.
Tasa
Ta sa de natali dad. Es el número de nacimientos ocurridos en un año por cada 1000 habitantes
en un territorio y durante un año específico.
Esperanza de vida. Es el indicador que muestra el número promedio de años que le espera vivir
a un habitante. Está determinado por el nivel de desarrollo del país. En el Perú, la esperanza de
vida es de 75 años (al 2018).
Tasa de crecimiento. Se refiere al ritmo en el cual varía la población de un país. Se obtiene de
la suma (1) de la diferencia entre la tasa de natalidad y la tasa de mortalidad con (2) la diferencia
entre la población que entra al país y la que sale de él. TC= (TBN-TBM) + (Inmigración -
Emigración)
Tasa de mortalidad. Conformada por el número anual medio de muertes por cada 1 000
habitantes.
Tasa
Ta sa de fecundi
fecundidad.
dad. Es el promedio de hijos nacidos vivos por mujer. Se refiere a la relación que
existe entrefemenina
población el número
ende nacimientos
edad fértil en elocurridos en un cierto período (en un año) y la cantidad de
mismo periodo.

Los principales indicadores geográficos del Perú y del mundo son los siguientes:

PERÚ MUNDO
MAYOR MENOR MAYOR MENOR
Población absoluta Lima Madre de Dios China Vaticano
Población rela
r elativa
tiva Callao Madre de Dios Mónaco Mongolia
Tasa
Tasa de natali
natalidad
dad Ucayali Huancavelica Níger y Angola Letonia
Esperanza de vida Callao Huancavelica Japón Swazilandia
Tasa
Tasa de ccrecimiento
recimiento Lima Madre de Dios Sudán del Sur Vaticano
Tasa de mortalidad Huancavelica Lima, Callao Ucrania, Bulgaria Qatar
Tasa de fecundidad Huancavelica Callao Níger Singapur
Además, la población peruana presenta los siguientes datos demográficos:
Regiones
Regiones con m ayor población ur bana Callao, Lima
CEPRE-UNI HUMANIDADES 101

CICLO INTENSIVO ESCOLAR NACIONAL Material de Estudio Nº 1 

Regiones con mayor pob lación rur


Regiones rural
al Huancavelica, Cajamarca
Huancavelica,
Regiones
Regiones con mayor pob lación masculin a Madre de Dios, San Martín
Regiones
Regiones co n mayor pob lación femenina Lambayeque, Huancavelica
Ciudadess capitales más pobl adas
Ciudade adas Lima, Arequipa, Trujillo,
Ciudadess capitales menos po bladas
Ciudade Chachapoyas,, Huancaveli
Chachapoyas Huancavelicaca
4. Migración. Es el fenómeno de desplazamiento que realiza una determinada población humana
desde un lugar
migración es unodede
origen a otro destino
los factores y lleva
que afecta la consigo
dinámicaundecambio de la yresidencia
crecimiento habitual.
la composición de La
la
población.. Entre sus principales causas tenemos las siguientes:
población
Geográficas o naturales. Sismos, Humanas. Los movimientos migratorios se dan por
sequías, inundaciones, variaciones guerras, persecuciones políticas, religiosas, terrorismo,
climáticas, huaycos, erupciones mejoras económicas, busca de trabajo, mejores
volcánicas.  
volcánicas. vida. 
servicios, mejores condiciones de vida. 
4.1. Clasificación de las migraciones. La direccionalidad establece que la población se desplaza, a
nivel interno como externo; es decir, dentro del mismo territorio
t erritorio o lo hace cruzando fronteras político-
administrativas. Así puede realizar la inmigración, ingreso o llegada de población hacia un nuevo lugar
de residencia, o la emigración, salida de la población del propio país o la propia región para
establecerse en otro lugar diferente de residencia.
De acuerdo a la información que el INEI señala al 2018, durante el periodo 1990-2017, el número de
peruanos que salieron y no han retornado al país es 3 089 123. Asimismo, indicó que 51% son
mujeres, 49% hombres y aproximadamente un 10% de la población peruana ha migrado. Chile lidera
la lista de países de primer destino que declaran los emigrantes peruanos, representando el 28,7%,
seguido por EE. UU. (16,3%). Respecto a la residencia de peruanos, 64% vive en el continente
americano (Norteamérica
(Norteamérica 33,3%, Sudamérica 32,1% y Centroamérica 1,0%), 28,8% en Europa, 4,2%
en Asia, 0,6% en Oceanía.
De los peruanos emigrantes por país de residencia, según el INEI, tenemos a EE. UU. con el 30,9%,
Argentina 14,5%, España 14,3%, Chile 11,2%, Italia10,0%, Japón 3,9%. En función a ciudades,
10,7% residen en Buenos Aires, 9,2% en Santiago de Chile; 6,8% en Madrid; 4,6% en Nueva Jersey,
3,8 % en Milán. Más del 40 % de peruanos en el exterior envían remesas. En el año 2018, las
remesas totalizaron 3 225 millones de dólares, cifra superior en 5% en comparación a las del 2017.
Las remesas representaron el 1,4 % del Producto Bruto Interno. Entre 2000-2017, los peruanos que
retornaron al Perú fueron 313 708.

Sobre los inmigrantes, al 2017, el INEI señaló que en el país residen más de 150 000: 24,6% de
Venezuela, 15,3% de Colombia, 7,3% de España, 6,8% de Estados Unidos. Asimismo, para fines de
abril de 2019, la población venezolana ha llegado a 750 000 personas en el territorio peruano, según
la Superintendencia Nacional de Migraciones.
Según El Comercio (1/mayo/19), de la población venezolana en el país 331 mil son ciudadanos venezolanos
que cuentan con el Permiso Temporal de Permanencia (PTP) y 160 mil lo vienen tramitando. El resto no tiene
ni lo tramita. Más de 152 mil no se encuentran sujetos al PTP. A fines
f ines de marzo, 224
2 24 666 cuentan con calidad
migratoria de turista, la cual es válida por un periodo de 183 días
El Permiso Temporal de Permanencia (PTP) es un documento emitido por la Superintendencia Nacional de
Migraciones que permite acreditar la situación migratoria regular en el país de ciudadanos venezolanos por
el plazo de un año y desarrollar actividades en el marco de legislación peruana.

5. Calidad
Calidad d
dee vida en el Perú. Está en función de la mejora del nivel de renta, condiciones de vida,
de trabajo y la mejora del ambiente. Puede estar determinada por los siguientes factores:
Factores materiales. Son los recursos que una persona tiene como los ingresos económicos
disponibles, posición en el mercado, salud.
CEPRE-UNI HUMANIDADES 102

CICLO INTENSIVO ESCOLAR NACIONAL Material de Estudio Nº 1 

Factores ambientales. Presencia y acceso a servicios, grado de seguridad y criminalidad,


transporte y movilización, habilidad
habilidad para servirse de nuevas ttecnologí
ecnologías
as que mejoran la vida.

En 2017, el Programa de la Naciones Unidas para el Desarrollo o PNUD publicó el Informe sobre Desarrollo
Humano y señaló el índice de desarrollo humano o IDH
ID H del Perú en 0,750, con lo cual ocupa el puesto 89 de
189 países. Ubica al Perú como un país con IDH alto, pero por debajo del promedio. De ello, los mayores
niveles de IDH se encuentran en Lima y Callao; pero los menores índices están en Apurímac, Ayacucho y
Huancavelica. Chile y Argentina son los países que lideran el ranking regional, mientras el Perú se ubica en
el noveno puesto.

6. Pobl
Población
ación eco
económi
nómi camente activa (PEA ). Se refiere a aquellas personas comprendidas entre los
(PEA).
14 años a más que se encuentran en condicione
condicioness de ttrabajar.
rabajar. Para fines censales, se considera que
la población económicamente activa está conformada por las siguientes categorías:
- Personas ocupadas. Es la población que estuvo participando en la generación de un bien económico
o en la prestación de un servicio.
- Personas desocupadas. La población que no encontró un empleo.

- El censo 2017 señaló que, del 100% de población, el 77% corresponde de la Población en Edad de Trabajar
(PET), el 56% a la PEA y 53% a la PEA ocupada.
- Del 100% de la PEA, los ocupados son el 96.0% y los desocupados buscando un empleo el 4.0%.
- El 39,5% de la PEA cumple labores en el sector servicios, el 24,9% en agricultura, el 18,3% en comercio,
el 9,5% en manufactura y el 6,2% en construcción.

Evaluación

1. Elementos como característica


característicass de I. La esperanza de vida en el Perú supera los
individuos, sexo, edad, educación y salud son 70 años.
estudiadas por la demografía de tipo II. Considera la mejora del nivel de renta,
A) cuantitativa.
cuantitativa. B) sanitaria
sanitaria.. C) guberna
gubernamental.
mental. condiciones de vida, de trabajo y ambientales.
D) cualitativa. E) económica.
económica. III. Ica y Callao muestra fuer
fuerte
te retroceso en su
nivel de calidad de vida
2. Es el país con mayor población absoluta en A) VVV B) FFF C) VVF D) FFV E) VFV
América Latina.
A) México B) Estados Unidos C) Brasil 5. Respecto a las fuentes de información
D) Canadá E) Colombia demográfica, la principal fuente de obtención de
información se obtiene de __________; sin
3. Los indicadores
densidad queson
poblacional permiten hallar la embargo,
establecerante su ausencia, sedepuede
el comportamiento la dinámica
A) natalidad y mortalidad. poblacional a través de __________.
B) natalidad y población femenina. A) censos – muestreos-e
muestreos-encuestas
ncuestas
C) mortalidad y población relativa. B) registros civiles - censos
D) población absoluta y superficie. C) muestreos-encuestas - censos
E) migración y población total. D) censos – registros públicos
E) actas electorales - censos
4. Marque la alternativa que contiene
enunciados verdaderos respecto a la calidad de
vida en el Perú. 1D 2B 3D 4C 5A

Tema IV: Estado peruano. Estado. G


Tema Gobierno.
obierno. Poderes públi cos y ór
órganos
ganos autón omos. Reforma
del Estado. Centralización
Centralización y descentraliza
descentralización.
ción. Regiones y gobiernos locales. Ética públic
pública.
a.
CEPRE-UNI HUMANIDADES 103

CICLO INTENSIVO ESCOLAR NACIONAL Material de Estudio Nº 1 

1. Estado peruano. Es Es la organización jurídico-política de la sociedad, la cual es concebida como


Nación. Incluye su gobierno, sus institucion
instituciones leyes.  La actual Constitución Política del
es públicas y sus leyes.
Perú, promulgada en 1993, entre los artículos 43° al 49°, señala las siguientes caracterí
características:
sticas:
   El Estado peruano se organiza en una república democrática,
democrática , social, independiente y soberana.
   El Estado es uno e indivisible. Su gobierno es unitario, representativo y descentralizado. Se
organiza con base a la separación de poderes.
   Son deberes primordiales del Estado: defender la soberanía nacional, garantizar
garantiza r la plena vigencia
de los derechos humanos, proteger a la población de amenazas contra su seguridad, promover el
bienestar general que se fundamenta en la justicia y el desarrollo integral y equilibrado de la Nación.
   El poder del Estado emana del pueblo. Quienes lo ejercen lo hacen con las limitaciones y
responsabilidades
responsabili dades que la Constitución y las leyes establecen.
   La defensa de los interese
interesess del Estado está a cargo de los Procura
Procuradores
dores Público
Públicoss conforme a ley.
   La capital de la República del Perú es la ciudad de Lima. Su capital histórica es la ciudad del Cusco.
Elementos pr
Elementos princip
incip ale
aless del Estado
- Población
Población:: grupo de personas que habitan un territorio determinado.
- Territorio: lugar geográfico en el que habita una población. Está constituido por suelo, subsuelo,
espacio aéreo y dominio marítimo donde se realiza la actividad estatal y ejerce sus potestades.
- Soberanía
Soberanía:: estructura organizada asumida por cada país para el ejercicio del poder del Estado. Es
la potestad de un Estado de hacer que, en su territorio, imperen la ley y las decisiones de su gobierno.
2. Estructura del Estado peruano. Se constituye por un conjunto de instituciones y organismos
debidamente interrelacionados. El Estado se basa en la separación de poderes.
2.1. Poder legislativo. Este poder reside en el Congreso de la República. El cual consta de cámara
única. El número de congresistas es ciento treinta (desde 2011). Sus integrantes son elegidos por
cinco años mediante proceso electoral organizado conforme a ley. Su función
f unción es a tiempo completo.
Los congresistas representan a la Nación. No están sujetos a mandato imperativo ni a interpelación.
Tiene funciones legislativas, especiales y de control político.
Los candidatos a la Presidencia de la República no pueden integrar las listas de candidatos a congresistas.
Los candidatos a vicepresidentes pueden ser simultáneamente candidatos a una representación en el
Congreso (Art. 90°).
Principales atribucion es del PPoder
oder Legislativo
  Dar ley
leyes
es y resoluciones legislativa
legislativas,
s, e interpretar, modificar o dero
derogar
gar las existentes.
  Velar por el respeto de la Constitución y de las leyes.
  Aprobar tratados de conformidad con la Constitución
Constitución..
  Aprobar el Presupuesto y la Cuenta General.
  Autorizar empréstitos, conforme a la Constitución.
  Ejercer el derecho de amnistía.
  Aprobar la demarcación territorial que proponga el P Poder
oder Ejecutivo.
  Prestar consentimiento para el ingreso de tropas extranjeras en el territori
territorioo de la República, sin
que afecte la soberanía nacional.
  Autorizar al presidente de la Republica para salir del país.
2.2. Poder ejecutivo. Está integrado por el presidente de la República, quien es el jefe del Estado y
personifica a la Nación, el Consejo de Ministros y demás autoridades elegidas. Para ser elegido
presidente de la República se requiere ser peruano por nacimiento, tener más de 35 años de edad al
momento de la postulación y gozar del derecho de sufragio. Es elegido por sufragio directo. Desde el
5 de noviembre del 2000 no hay reelección presidencial.
La vacancia presidencial (art. 113°) se produce por lo siguiente: El ejercicio de la Presidencia de la
1. Muerte del presidente de la República. República se suspende (Art. 114°) por
los siguientes motivos:
CEPRE-UNI HUMANIDADES 104

CICLO INTENSIVO ESCOLAR NACIONAL Material de Estudio Nº 1 

2. Permanente incapacidad moral o física, declarada por el 1. Incapacidad temporal del


Congreso. presidente, declarado por el
3. Aceptación de su renuncia por el Congreso. Congreso.
4. Salir del territorio nacional sin permiso del Congreso o no regresar 2. Hallarse este sometido a proceso
a él dentro del plazo fijado.  judicial, conforme al artículo 117º de
5. Destitución, si es sancionado por una infracción mencionada en la Constitución.
el artículo 117º de la Constitución.
Artículo 117°. El presidente de la República solo puede ser acusado, durante su período, por traición a la
patria; por impedir elecciones presidenciales, parlamentarias, regionales o municipales; por disolver el
Congreso, salvo en los casos previstos en el art. 134° de la Constitución, y por impedir su reunión o
funcionamiento, o los del Jurado Nacional de Elecciones y otros organismos del sistema electoral.
Principales funciones
func iones del presidente
pr esidente de la RRepública:
epública:
 Cumplir
 Cumpl ir y hacer cumplir la Constitución, leyes, tratados y demás disposiciones legales
legales..
 Representar
 Rep resentar al Estado, dentro y ffuera
uera de la República.
 Dirigirr la política general del gobierno.
 Dirigi
 Velar
 Ve lar por el orden interno y la seguridad exterior.
 Convocar
 Convoc ar al Congreso a legislatura extraordinaria.
 Administrar la hacienda pública.
 Dictar medidas extraordinarias, con decretos de urgencia con fuerza de ley, en materia económica y
financiera, con cargo de dar cuenta al Congreso.
 Conceder indultos y conmutar penas.

 Conferir condecoraciones en nombre de la Nación, con acuerdo del Consejo de Ministros.


2.3.. Poder judi
2.3 cial. La potestad de administrar justicia emana del pueblo y la ejerce el Poder Judicial
judicial.
a través de sus órganos jerárquicos con arreglo a la Constitución y a las leyes.
En todo proceso, de existir incompatibilidad entre una norma constitucional y una norma legal, los jueces
prefieren la primera. Igualmente, prefieren la norma legal sobre toda otra norma de rango inferior.
Órganos juris diccio nale
naless de acuerdo a su jerarquía
jerarquía::
 Corte Suprema de Justicia: tribunal de mayor jerarquía, su presidente lo es del Poder Judicial.
 Superiores 
 Las Cortes Superiores 
 Mixtos  
 Los Juzgados Especializados o Mixtos 
 Letrados 
 Los Juzgados de Paz Letrados 
 Los juzgados de paz: dirigidos por un juez que no es abogado. Se dan en zonas alejadas.
No son revisables, en sede judicial, las resoluciones del Jurado Nacional de Elecciones, en materia electoral,
ni las del Consejo Nacional de la Magistratura, para evaluación y ratificación de jueces.
3. Organismos const itucio nales aautónom
utónomos.
os. La Constitución política, para salvaguardia del Estado
de Derecho y mayor eficiencia en ejecución de labores, ha constituido algunos organismos autónomos
que no dependen de ninguno de los poderes del Estado y se rigen por sus propias leyes orgánicas.
3.1.
3.1. O
Organismos
rganismos autónomos de carácte
carácterr económi co
 A. Ban
Banco
co Cent
Central Reser va del Perú (BCRP). Institución encargada de preservar la estabilidad
ral de Reserva
monetaria y su autonomía. Es dirigido por un directorio de siete miembros, cuyo presidente es
designado por el Poder Ejecutivo. Algunas de sus funciones son las siguientes:
- Regular la moneda y el crédito del sistema financiero. - Administrar las reservas internaciona
internacionales.
les.
- Informar
Informar perió
periódicamente
dicamente sobre llas
as finanz
finanzas
as naci
nacionales.
onales. - Emitir billetes y moned
monedas.
as.

B. Superintendenci
Superin tendenci a de Banca, Seguros y AFP ((SBS).
SBS). Institución de derecho público con autonomía
funcional reconocida por la Constitución Política. El Poder Ejecutivo designa al Superintendente de
Banca y Seguros por el plazo correspondiente
correspondiente a su período constitucion
constitucional.
al. Tiene como funciones:
CEPRE-UNI HUMANIDADES 105

CICLO INTENSIVO ESCOLAR NACIONAL Material de Estudio Nº 1 

   La regulación y supervisión del sistema financiero, de seguros y del Sistema Privado de


Pensiones. Ejerce el control de las empresas bancarias y de seguros, cajas de ahorro, mutuales,
cajas rurales autorizadas a operar en el sistema financiero.
   Preservar los intereses de los depositantes, de los asegurados y de los afiliados al Sistema
Privado de Pensiones.
   Prevenir y de
detectar
tectar el lavado de ac
activos
tivos y fina
financiamiento
nciamiento del terrorismo.

C. Cont
Contralor
raloría
ía G
General
eneral de la R Repúbli ca (CGRP).. Máxima autoridad del Sistema Nacional de Control.
epúbli ca (CGRP)
Para realizar con eficiencia sus funciones, tiene autonomía administrativa, funcional, económica y
financiera. Está representada por el Contralor General de la República que es propuesto por el
presidente de la República y designado por la Comisión Permanente del Congreso por un periodo de
7 años. Entre sus principal
principales
es funciones tenemos las siguientes:
  Supervisa, vigila, verifica la correcta aplicación de políticas públicas, uso de recursos y bienes del
Estado.
  Supervisar la legalidad de la ejecución del presupuesto del Estado, de las operaciones de la deuda
pública y de los actos de las instituciones sujetas a control.
  Supervisa las operaciones de la deuda pública y de los actos de las instituciones sujetas a control.
3.2.
3.2. O
Organismos
rganismos autónomos de carácte
carácterr ju dicial
D. Defensoría del Pueblo. Órgano autonomo que se organiza a nivel nacional. Es conducido por el
Defensor del Pueblo, que es elegido y removido por el Congreso con el voto de los dos tercios de su
número legal. Es elegido por cinco años y goza de inmunidad, igual que los congresistas. Entre sus
principales funciones tenemos las siguientes:
  Defiende los derechos constitucionales y fundament
fundamentales
ales de la persona y de la comunidad.
  Supervisa el cumplimient
cumplimientoo de los deberes de la administración estatal y la prestación de los
servicios públicos a la ciudadanía
Los órganos públicos están obligados a colaborar con la Defensoría del Pueblo cuando lo requiera.
E. Ministerio Público (MPFN). Es el organismo que representa a la sociedad ante los tribunales de
 justicia, la defensa de la legalidad y de los intereses públicos tutelados por el Derecho. Lo preside el
Fiscal de la Nación que es elegido por la junta de fiscales supremos. Su función dura tres años con
opción a prorroga por dos más. Algunas de sus funciones son las siguientes:
   Represent
Representaa a la sociedad ante los tribunales de justicia.
   Promueve la acción judicial en defensa de la legalidad y los interese
interesess públicos.
   Conducir desde un inicio la investigac
investigación
ión del delito.

  Emitir dictamen previ
previo
o a las resoluciones judiciales
judiciales en los casos que la ley contemple.
contemple.
   Velar por la independenci
independenciaa de los órganos jurisdiccional
jurisdiccionales
es y la recta administración de justicia.
   Ejercer iniciativa en la formación de las leyes y dar cuenta, al Congreso o al presidente de la
República, de los vacíos o defectos de la legislación.
F. Consejo Nacional de la Magistratura (CNM). Era el organismo encargado de la selección y el
nombramiento
nombramien to de los jueces y ffiscales.
iscales. El CNM estaba conformado por siete integrantes denominados
consejeros, elegidos por 5 años y representan a organismos e instituciones diversas para lograr su
autonomía. Sus principales funciones son las siguientes:
  Nombrar por co
 comisión
misión perma
permanente,
nente, prev
previo io concurso público, a jueces y fisca
fiscales
les de todos los
niveles. Asimismo, su ratificación cada siete años.
  Destituir a los vocales, jueces y fiscales.

  Extender a los jueces y fiscales el títul


 títuloo ooficial
ficial que llos
os ac
acredita.
redita.
Tras el Referéndum de 2018, se aprobó la reforma y disolución del CNM por el caso CNM-Audios, la
conformación de la Junta Nacional de Justicia JNJ, que asumirá sus funciones. La JNJ está en proceso de
organización. Hay una comisión para elegir a sus miembros que deben ser abogados. Serán elegidos por
CEPRE-UNI HUMANIDADES 106

CICLO INTENSIVO ESCOLAR NACIONAL Material de Estudio Nº 1 

meritocracia por 5 años y prohibida su reelección inmediata. La participación femenina es obligatoria en sus
miembros. A la fecha no presenta reglamento de funciones.
G.  Tribunal Constituci onal (TC
G.  ). Órgano supremo, autónomo e independiente de interpretación y
(TC).
control de la constitucionalidad. Se compone de siete miembros elegidos por cinco años. Sus siete
miembros son elegidos por el Congreso de la República por 5 años, sin reelección inmediata y gozan
de inmunidad. Sus principales funciones son las siguientes:

  Conocer, en última
hábeas data, y definitiva
y acción instancia, las resoluciones denegatorias de hábeas corpus, amparo,
de cumplimiento.
   Conocer en instancia úni única
ca la acción de inconstitu
inconstitucionalida
cionalidad.
d.
   Conocer conflict
conflictos
os de competenc
competencia, ia, o de atribuciones asignadas por la Constit
Constitución,
ución, conforme a
ley.
3.3.
3.3. O Organismos
rganismos autónomos de carácter ele elector
ctoral
al
H. Jurado Nacional de Elecciones (JNE). Su máxima autoridad es el Pleno, integrado por 5
miembros. Su presidente es elegido por la Corte Suprema. Los otros miembros son elegidos: uno por
la Junta de Fiscales Supremos, uno por el Colegio de Abogados de Lima, uno por decanos de las
Facultades de Derecho de universidades públicas y uno por decanos de Facultades de Derecho de
las universidades privadas. Entre sus principales funciones tenemos las siguientes:
- Fiscaliza la legalidad del ejercicio del sufragio y de la realización de procesos electorales.
- Entrega las credenciales a las autoridades elegidas en los procesos electorales.
- Se encarga de la elaboración de los padrones electorales. - Administra justicia en materia electoral.
- Mantiene el registro de las organizaciones
organizaciones políticas. - Proclama a los candidatos elegidos.
- Imparte la educación electoral
electoral..
I. Oficina Nacional de Procesos Electorales (ONPE). Autoridad que organiza y ejecuta distintos
procesos electorales,
elector ales, de referéndum y otros tipos de consulta popular. El jefe de la ONPE es nombrado
por el CNM por un período de 4 años mediante concurso público. Tiene como funciones:
   Encargado de la organización, ejecución de todos los procesos electorales y de consulta popular.
   Elabora y diseña la cédula de sufragio.
   Brinda información permanente sobre el cómputo desde el inicio del escrutinio en mesas de
sufragio.
   Se encarga de la verificación de firmas de adherentes de partidos políticos en proceso de
inscripción.
F. Registro Nacional de Identificación y Estado Civil (Reniec). Organismo autónomo encargado
de la identificación de los peruanos. El jefe del Reniec es nombrado por el Consejo Nacional de la
Magistratura, por un periodo de cuatro años. Entre sus principales funciones tenemos las siguientes:
- Se encarga de la inscripción de nacimientos, matrimonios
matrimonios,, divorcios, defunciones y otros actos que
modifican el estado civil.
- Proporciona al JNE y ONPE información necesaria para el cumplimiento de sus funciones en
procesos electorales.
- Otorga el documento nacional de identidad - Prepara y mantiene actualizado el padrón
electoral.
La ONPE, el JNE y el Reniec conforman el Sistema Electoral Peruano, de conformidad con lo establecido
por el artículo 177° de la Constitución Política del Perú. Mantienen coordinación según sus atribuciones.
4. Centralización y descentralización. El centralismo es un problema histórico del país presente
desde los inicios de la República. Se manifiesta no solo en la relación Lima-regiones, sino que se
reproduce en la relación entre capitales de regiones con sus provincias y distritos. Lima concentra las
principaless actividades económico-financieras del país; junto al poder político y administrativo.
principale
La descentralización es el proceso de transferir, dentro del Estado, autoridad y capacidad de decidir
recursos (dinero), poder y funciones del gobierno central hacia los gobiernos regionales y locales.
Estos últimos conocidos también como gobiernos subnacionales. El proceso de regionalización es un
CEPRE-UNI HUMANIDADES 107

CICLO INTENSIVO ESCOLAR NACIONAL Material de Estudio Nº 1 

proceso trunco en el país. Se inició en 2002. En 2005, se dio el primer paso democrático para la
integración de regiones en macrorregiones políticas; sin embargo, el referéndum fracasó por la
negativa de la mayoría de regiones (15 de 16). A la fecha, No hay resultados satisfactorios en este
proceso.
4.1.. Niveles de organización del pod
4.1 poder
er
Nivel nacional. Compuesto por el Poder Legislativo (Congreso de la República), el Poder Ejecutivo, el
Poder Judicial y los organismos autónomos con funciones especializadas (SBS, TC, BCR, JNE, etc.).
Nivel regional. Existente desde el 2003. Los 25 gobiernos regionales se han establecido en los
Callao.  
departamentos y la provincia constitucional del Callao. 
Nivel local. Conformado por los gobiernos locales o municipalidades, ya sean provinciales, distritales y
centros poblados. 
Gobiernos regionale
regionales.s. Poseen autonomía política, económica y administrativa; es decir, se
organizan y toman decisiones según sus propios criterios. Su función principal es fomentar el
desarrollo integral sostenible
sostenible en su jurisdicción, coordinar con el gobiern
gobiernoo central, con miras a
lograr la unidad política y económica. El presidente regional es elegido por sufragio directo por un
período de 4 años. 
Principales atrib
atrib uciones
uciones  
 Aprobar su organización interna y su presupuesto.
 Elaborar y aprobar el plan de desarrollo regional concertado con municipios y sociedad civil.
 Administrar sus bienes y rentas, promover y regular actividades económicas.
 Impulsar la competitividad, las inversiones y el financiamiento para la ejecución de los proyectos
de desarrollo y obras de infraestructura de alcance e impacto regional.
Gobiernos locales o municipales. Son la forma básica de organización territorial del Estado y
canales inmediatos de participación vecinal en asuntos públicos, que institucionalizan y gestionan
con autonomía. Sus autoridades, el alcalde y los regidores, son elegidos por sufragio directo por 4
años.  
años.
Principales atrib uciones  
atrib uciones
  Administrar sus bien
bienes
es y rentas.
 Crear, modificar y suprimir las contribuciones, tasas, arbitrios, licencias y derechos municipales,
conforme a ley.
  Aprobar el plan de desarroll
desarrolloo local concertado y presupuesto partici
participativo.
pativo.
  Promover, apoyar y reglamentar la participación vecinal.
vecinal.
  Aprobar el sistema de gestión ambiental loca
local.l.
  Organizar, reglamentar y administrar los servicios públicos locales de su responsabilidad.
responsabilidad.
 Planificar el desarrollo urbano y rural de sus circunscripciones.

 Brindar servicios de seguridad ciudadana, con la cooperación de la Policía Nacional del Perú.
 Aprobar, modificar o derogar las ordenanzas y dejar sin efecto los acuerdos.
6. Reforma del Estado. Es un proceso donde un Estado busca estructurarse política, administrativa
y económicamente. Esto lo hace con el objetivo de ser más eficiente y adecuarse a nuevas
conyunturas. Entre sus causas tenemos el contexto internacio
internacional
nal que se caracteriza por el avance de
la globalización económica, la crisis de la gobernabilida
gobernabilidadd y procesos democráticos.
7. Ética pública. Es la ética aplicada por servidores públicos, que en estos casos, son los
gobernantes, funcionarios públicos y todos los que prestan servicios en instituciones públicas.
Contribuye a la lucha para prevenir y evitar la corrupción. Además, persigue el objetivo de contar con
buenos gobiernos que afronten los retos de injusticia, pobreza, desconfianza o insatisfacción que
padecen los ciudadanos.

Evaluación

1. Señale la característica que corresponde al A) Presenta dos cámaras.


Congreso peruano. B) Personifica a la nación.
CEPRE-UNI HUMANIDADES 108

CICLO INTENSIVO ESCOLAR NACIONAL Material de Estudio Nº 1 

C) Es parte del Ejecutivo. D) las universidad


universidades
es públicas.
D) Autoriza empréstitos. E) el Consejo de Ministros.
E) Administra justicia.
4. Las municipalidades establecidas en distritos
2. Son institucion
instituciones
es que forman parte del Poder pertenecen al nivel _____ de organización de
Ejecutivo, a excepción del poder.
A) Ministerio de Educación. A) central B) local C) provincial D) regional
B) Ministerio Público. E) nacional
C) Ministerio de Justicia.
D) Ministerio de Salud. 5. Es el tribunal de más alto nivel de jerarquía
E) Ministerio del Ambiente. en el Poder Judicial.
A) Corte Superior B) Juzgado de Paz Letrado
3. Los siete miembros del Tribunal C) Corte Suprema D) Juzgado de Paz
Constitucional son elegidos por E) Juzgado Especializado
A) el Congreso de la República.
B) la Corte Suprema de Justicia. 1D 2B 3A 4B 5C
C) el presidente de la República.

Tema V. El poder político y su legitimación. Democracia y participación ciudadana.


Organizaciones civiles. Movimientos sociales, partidos políticos y ONG. Sistema de Defensa
Civil. Estado de Derecho. Convivencia en el Perú. 

1. El poder político y su legitimación. Es el poder al servicio de una idea, aceptado como válido
para la construcción social. Este es otorgado a las autoridades que representan a ciudadanos de un
Estado. Limita las libertades individuales para sobreponer el bien común y la convivencia
convivencia.. Asimismo,
consiste en el ejercicio de responsabilidades de funcionarios públicos. La legitimación del poder
político, en soberanía nacional, está dirigida y fundamentada en el pueblo a través de instituciones
que diseña el Estado para mejorar la calidad de vida. La ciudadanía elige gobernantes en condiciones
que considera adecuadas y decide el tiempo que deben gobernar. Esto se ejerce dentro de un sistema
democrático.
El Contrato Social. Jean-Jacques Rousseau (1712-1778) señala que los seres humanos acuerdan un
contrato social implícito, que otorga derechos a cambio de abandonar la libertad completa de la que
dispondrían en estado de naturaleza. El Estado es la entidad creada para hacer cumplir el contrato. Los
derechos y deberes de los individuos constituyen las cláusulas del contrato social. Del mismo modo, quienes
lo aceptan pueden cambiar los términos del contrato si así lo desean: los derechos y deberes no son
inmutables o naturales. Por otro lado, un mayor número de derechos implica mayores deberes; y menos
derechos, menos deberes. El término propone un orden político-ciudadano que se plantea entre lo que el
hombre hace, los derechos que demanda, las normas que se necesitan y que acepta cumplir para poder
convivir. De tal manera, esta relación está armonizada por el sentido de justicia. (Fuente:
http://www.filosofia.net/materiales/sofiafilia/hf/soff_mo_16_c.html)
2. Democracia
Democracia y parti cipación ciudadana
2.1.. Democ
2.1 Democracia.
racia. Es
 Es el régimen político donde el poder reside en el pueblo; es decir, se
s e autodetermina
y expresa su voluntad a través del sufragio y la participación política. Está basado en el respeto,
promoción y garantía de los derechos humanos (DD. HH.). Por ello, la democracia implica no solo una
estructura política sino abarca otros aspectos de la sociedad: prácticas sociales y valores para que la
sociedad logre desarrollar una mentalidad que la promueva.
Los principios de la democracia originan derechos y deberes recíprocos. Por ello, se observa:
  La persona exi
exige
ge al Estado la protección de las leyes, así como el reconocimiento de sus derechos.
  El Estado tie
tiene
ne el derecho
derecho de exigir a los ciudadanos el cumplimiento de las leyes.
Por lo señalado, la participación ciudadana es crucial para conseguir un nivel democrático que asegure el
cumplimiento de los derechos y libertades humanas.
Tipos de democracia
democr acia
CEPRE-UNI HUMANIDADES 109

CICLO INTENSIVO ESCOLAR NACIONAL Material de Estudio Nº 1 

Directa o pura. La soberanía reside en el pueblo y es ejercida por él sin necesidad de elegir
representantes:: delibera, toma las decisiones, sanciona leyes, etc.
representantes
Indirecta o representa
representativa.
tiva. El pueblo vota para elegir representantes que deliberen y tomen las
jerárquica. 
decisiones de forma jerárquica. 
Semidirecta o participativa. El pueblo se expresa directamente en ciertas circunstancias
particulares. Destacan tres mecanismos:
mecanismos: Referéndum, Plebiscito, Iniciativa popular.
popular.  
Líquida. El ciudadano tiene la posibilidad de votar por internet cada decisión del parlamento y
realizar propuestas. Puede delegar su voto a un representante para aquellas decisiones en las que
prefiere no participar. Esta delegatura puede anularse en cualquier momento.
2.2. Ciud
Ciudadanía.
adanía. Condición jurídico-política que establece una relación persona-Estado, con derechos
y deberes entre ambos. En el Perú, se adquiere al cumplir la mayoría de edad (18 años de edad) y
realizar el registro en Reniec. Todos los peruanos inscritos en el Registro Electoral reciben su
documento de identificación (DNI azul), que acredita su reconocimiento por el Estado como
ciudadanos.
El ciudadano se compromete, desde una reflexión autónoma y crítica, con la construcción de una
sociedad más justa; asimismo, con el respeto y valoración de la diversidad social y cultural.
Derechos ciudadanos: intervenir en asuntos públicos, ejercer libertad de pensamiento y expresión, los
ciudadanos pueden ejercer sus derechos de modo individual o a través de organizaciones políticas.
Deberes
De beres ciu dadanos: tienen
dadanos:  tienen relación con la participación de la vida política de la comunidad, de la nación y
el Estado. Honrar a la patria, proteger sus intereses, defender la Constitución y sus leyes; así como
com o los aportes
económicos que realizan los ciudadanos al Estado.
La suspensión de la ciudadanía. El artículo 33° de la Constitución Política indica por resolución
 judicial de interdi
interdicción,
cción, ppor
or sentenci
sentenciaa con pena pprivativa
rivativa de la libertad, ppor
or sen
sentencia
tencia con inhabilitac
inhabilitación
ión
de los derechos políticos.
El Estado de Derecho es el
¿Cuándo s e e
¿Cuándo ejerce
jerce una ciudadanía
ci udadanía plena? régimen de gobierno que
 Cuando se garantiza la autonomía de las personas. asegura la vigencia de la
 Cuando
 Cua ndo se reconoce los DD. HH. de cada ciudadano. Constitución, el cumplimiento de
 Cuando se ejerce el derecho a elegir y ser elegido para gobernar. la ley y el respeto a los derechos
 Cuando exist
existee un Esta
Estadodo de Derec
Derecho
ho que salvagu
salvaguarde
arde la vida ciudadanos.
política, social y familiar, etc.

2.3. Participación ciudadana. Es el ejercicio pleno del derecho de opinar, informarse y colaborar con
la toma de decisiones políticas. Cuando la ciudadanía organizada participa, de esta manera, en la vida
política, se habla de democracia participativa. Su finalidad es garantizar la consecución del bien
común.
En 2005, se modificó la ley sobre el voto de militares y policías. En las elecciones del 2006, participaron
activamente del proceso electoral. Sin embargo, no pueden postular a cargos de elección popular, participar
en actividades partidarias o manifestaciones ni realizar actos de proselitismo, mientras no hayan pasado a la
situación de retiro, de acuerdo a ley.

La participación ciudadana implica involucrarse de manera responsable y consciente en la tarea de


garantizar la plena vigencia y protección de los derechos humanos y la vida en democracia; por ello,
es importante desarrollar algunas capacidades ciudadanas. La Ley 26300 señala como derechos de
participación ciudadanos los siguientes:
Iniciativas legislativas. Derecho ciudadano para presentar al Congreso proyectos de ley. Se
presenta solicitud al JNE adjuntando un porcentaje de firmas. Si estas son convalidadas, se somete
la solicitud al veredicto del Congreso. El proyecto de ley rechazado en el Congreso puede ser
sometido a referéndum conforme a esta ley.
Iniciativa deRequiere
Constitución. reformala constitucional. Es el derecho
adhesión de un número para laequivalente
de ciudadanos reforma parcial o totaltres
al cero punto depor
la
ciento (0.3%) de la población electoral nacional. Es improcedente toda iniciativa de reforma
CEPRE-UNI HUMANIDADES 110

CICLO INTENSIVO ESCOLAR NACIONAL Material de Estudio Nº 1 

constitucional que recorte los derechos fundamentales consagrados en el artículo 1° y 2° de la


Constitución.
Referéndum. Derecho ciudadano para pronunciarse, conforme a la Constitución, en temas
normativos que se le consultan. Se somete a votación pública un asunto de trascendencia. Puede
ser planteado por uno de los órganos del Estado o la ciudadanía. Se solicita al JNE junto con el 10%
de firmas del electorado nacional; además, el respaldo de la mitad más uno de los congresistas en
el caso de reforma constitucional. Si el JNE aprueba el pedido envía la solicitud a la ONPE.
Derecho al sufragio. Es el derecho al voto que tienen los ciudadanos para elegir autoridades
políticas. Es personal, igual, libre, secreto y obligatorio hasta los setenta años, establecido en la
Constitución Política del Perú. Es facultativo después de esa edad.
El cabildo abierto. Instancia de consulta directa del gobierno local a la ciudadanía local, convocada
con un fin específico. El concejo provincial o distrital, por ordenanza, reglamentará su convocatoria.
Es potestad del alcalde para convocar a los pobladores de su distrito y consultar su opinión. Sin
embargo, la ley no exige que el alcalde cumpla con acuerdos tomados en el cabildo (no vinculante).
Otros mecanismos son las iniciativas en la formación de dispositivos municipales y regionales,
consejos de coordinación local y regional, y presupuestos participativos.
Los mecanismos de participación como el Referéndum fueron incluidos y aprobados en la Constitución de
1993. El primer referéndum del Perú se realizó en 2010, en el que se aceptó la devolución del Fonavi a los
aportantes. Asimismo, en 2018, el presidente Martín Vizcarra convocó a referéndum. Este último referéndum,
a la fecha, trató cuatro reformas constitucionales: la conformación de la Junta Nacional de Justicia (antes
CNM), la no reelección inmediata de congresistas, el retorno de la bicameralidad en el Parlamento y el
financiamiento privado a los partidos.
Consulta Previa a los pueblo s indígenas u o rigin arios, Ley 297 29785
85..  Mecanismo particular que fomenta la
participación ciudadana. Fue aprobada en 2011. Es criticada por su aplicación relacionada con temas de
extracción de recursos naturales. Permite a estos pueblos dialogar con el Estado para lograr acuerdos que
pueden afectar sus derechos colectivos, existencia física, identidad cultural, calidad de vida o desarrollo.
Participar en la toma de decisiones les permitirá acceder a mejores oportunidades para vivir de acuerdo a sus
prioridades. Se aplica cuando se explotan recursos naturales en su territorio, se emite una ley u ordenanza
que los afecta directamente, se realiza una obra de infraestructura que cambiará sus u modo de vida, se elaboran
políticas públicas que busquen atenderlos, entre otros. La Defensoría del Pueblo recomienda que, en
proyectos de inversión, la consulta debe realizarse durante la evaluación de impacto
im pacto ambiental. Las entidades
del Estado como ministerios, gobiernos regionales o Congreso de la República son los responsables de
implementarla. Los pueblos indígenas también pueden solicitar su inicio o ser incluidos en uno que se
encuentre en curso. El Ministerio de Cultura es el responsable de orientar sobre los requisitos y la forma de
solicitarlo.
2.4. Planificación concertada. Proceso donde participan representantes del gobierno, instituciones,
públicas, privadas, religiosas, etc., y organizaciones de la población. Busca establecer un conjunto
ordenado de acciones donde se manifiesten los intereses de la población y se recoja experiencias
existentes para elevar la calidad de vida a través del uso racional de los recursos tecnológicos,
naturales y del potencial humano.
Presupuesto participativo. 
participativo.  Mecanismo de asignación equitativa, racional y transparente de recursos
públicos. Fortalece la relación Estado-ciudadanía, que participa en la programación del presupuesto. Se
desarrolla en armonía con los de los gobiernos descentralizados y la fiscalización de la gestión. Para
Par a ello, los
gobiernos regionales y locales promueven mecanismos y estrategias de participación en los presupuestos,
así como en la vigilancia y fiscalización de la gestión de los recursos públicos. Los gobiernos regionales y
locales están encargados de convocar a organizaciones sociales a participar y deben disponer medidas para
identificar y acreditar a los participantes. Además, el MEF pública instructivos anuales que rigen el proceso.
Por otro lado, los consejos de coordinación regional y local (CCR y CCL) coordinan la programación del
presupuesto participativo con el apoyo del equipo técnico conformado en el gobierno regional o local.
2.5. Control ciudadano. Para mejorar el control del gobierno, la sociedad civil debe estar atenta a la
actuación de autoridades y usará mecanismos de participación para evaluar y constatar que realicen
cabalmente el trabajo encargado. El control se puede realizar ejerciendo los siguientes derechos
ciudadanos y a la vez estos son mecanismos de participación ciudadana.
CEPRE-UNI HUMANIDADES 111

CICLO INTENSIVO ESCOLAR NACIONAL Material de Estudio Nº 1 

Revocatoria. Permite a la ciudadanía decidir si las autoridades (alcaldes, regidores, presidentes


regionales, otras autoridades regionales y jueces de paz por elección popular) que fueron elegidas
por sufragio popular siguen en el cargo o son retirados. La solicitud de revocatoria se refiere a una
autoridad en particular. Debe ser fundamentada y no requiere presentar pruebas. El JNE convoca a
una consulta electoral y acepta la revocatoria si 50% de los electores hábiles asisten y la mitad más
uno vota a favor. De revocarse más de un tercio de los miembros del Concejo Municipal o del
Consejo
Remoción. Regional, se convoca
Sirve para destituir aautoridades
nuevas elecciones.
nombradas por el gobierno central o regional. Se tendrá
que presentar más del 50% de firmas de los ciudadanos de una jurisdicción electoral o judicial.
Rendici
Re ndici ón de cuentas. Mecanismo para interpelar autoridades nombradas por el gobierno central
o elegidas por el pueblo sobre la ejecución del presupuesto, de obras públicas y el uso de recursos.
Para solicitarlo se requiere el 10% con un máximo de 25 000 firmas de la población electoral y
presentarlos al JNE con un pliego de preguntas, las que se formularán a la autoridad cuestionada.
cuestionada.
Plebiscito es la consulta popular que permite conocer la opinión pública sobre la política oficial o un tema de
interés nacional. Su diferencia con el referéndum radica en que este se utiliza, en especial, para reformar de
leyes, normas e instituciones por un medio diferente al del Poder Legislativo.
3. El
El Acuerdo NaNacional
cional y toma de decisi ones
3.1.. El Acuerdo Na
3.1 Nacio
cio nal. Conjunto de políticas de Estado aprobadas sobre la base del diálogo y del
consenso, tras talleres y consultas a nivel nacional, para definir un rumbo para el desarrollo sostenible
del país y afirmar su gobernabilidad democrática. Asimismo, mantiene un foro político para conocer
sus avances y plantear sugerencias a su desarrollo. Sus acuerdos tienen carácter vinculante.
Foro del Acuerdo Nacional.
Nacional . Espacio de diálogo y concertación institucionalizado como instancia de
seguimiento y promoción del cumplimiento de las políticas de Estado del Acuerdo Nacional, ratificado por
Decreto Supremo en 2002. Su conformación es tripartita: participan Gobierno, partidos políticos con
representación en el Congreso de la República y organizaciones representativas de la sociedad civil a nivel
nacional. El presidente de la República es el presidente del Foro del Acuerdo Nacional, quien puede delegar
la conducción de sus sesiones en el Presidente del Consejo de Ministros.
Fue planteado por el gobierno de Alejandro Toledo y suscrito, en 2002, por representantes de
organizaciones políticas de la sociedad civil y del gobierno. Enrumbarán las políticas de Estado en los
próximos 20 años hasta el 2021. Su objetivo es acordar políticas de Estado que otorguen la estabilidad
que el país requiere para alcanzar un desarrollo sostenido. A la fecha, ha establecido 35 políticas de
Estado. Entre sus principales
principales metas tenemos las siguientes:
 Fortalecimiento de la democracia y del Estado de Derecho: busca consolidar el sistema democrático
y fortalecer los partidos políticos.


 Desarrol
 Desarrollo
 Promociónlo con equidad
del la y justiciadel
competitividad social:
país:construir consensos
que permita que
un mejor sustituyan
desarrollo los conflictos.
sostenible de la economía.
 Afirmación de un Estado eficiente, transparante y descentraliz
descentralizado:
ado: que permita una mayor
participación ciudadana
La sociedad civil también participa, mediante los medios de participación ciudadana.
4. Organizaciones
Organizaciones cciviles.
iviles. La participación ciudadana forma parte del proceso de descentralización
para acercar las decisiones de gobiernos regionales y locales a su población. Estos derechos y
deberes están desarrollados y garantizados por la Ley 26300, de Participación y Control
Ciudadanos. Las organizaciones sociales de base (OSB), en el Perú, defienden derechos a vivienda
y servicios urbanos, reproducen y adaptan la tradición de organizaciones comunitarias rurales, como
parte del proceso de migración y urbanización popular. Algunas organizaciones civiles son la
Confederación General de Trabajadores del Perú (CGTP), Central de Trabajadores del Perú (CTP),
Federación Textil del Perú, Sindicato Único de Trabajadores de la Educación (Sutep), Asociación Civil
Transparencia, Asociación de Damas de Ayuda al Instituto Nacional de Enfermedades Neoplásicas
(Adainen).
CEPRE-UNI HUMANIDADES 112

CICLO INTENSIVO ESCOLAR NACIONAL Material de Estudio Nº 1 

5. Movimi
Movimi entos ssociales.
ociales. Son agrupaciones informales de individuos y/o organizaciones dedicadas
a temas político-sociales para una transformación social. Surgen como modos de organización de
colectivos, de manera espontánea en una situación de crisis, fundamentalmente alternativos, que
luchan dentro de un campo político más o menos concreto. Estos movimientos suelen recibir apoyo
de partidos políticos, sindicatos y organizaciones no gubernamentales (ONG). Ejemplo: movimiento
feminista, movimiento obrero, movimiento ecologista, movimiento pacifista, etc. Tienen como finalidad
que la sociedad cambie en su estructura; pero también tienen dos fines específicos:

 Contribuyen
 Contribu yen a la formación de la opinión pública.
 Contribuyen en el adiestramiento de líderes que ocuparán puestos de gobierno.
Los movimientos sociales, al institucionalizarse, pueden llegar a formar partidos políticos si se organizan
formalmente, adoptan una estructura y participan en las contiendas electorales.
6. Partidos
Partidos po líticos. En 2003, se promulgó la Ley 28094 de Organizaciones Políticas. Señala a los
partidos políticos como asociaciones ciudadanas que constituyen personas jurídicas de derecho
privado cuyo objeto es participar, por medios lícitos, de modo democrático, en asuntos públicos del
país dentro del marco constitucional y de la presente ley. Poseen una estructura organizativa a nivel
local y nacional. La denominación partido se reserva a los reconocidos como tales por el Registro de
Organizaciones Políticas. Buscan apoyo popular a través de elecciones o de cualquier otro medio
(como movilizaciones
movilizaciones de masas). Las funciones que deberían cumplir son las siguientes:
 Asegurar la vigencia y defensa del sistema democrático. La Ley Orgánica de Elecciones
 Representan la voluntad de los ciudadanos y canalizar la opinión se ocupa de los partidos


pública. la participación de la población en las tareas públicas.
 Asegurar
políticos, indica los requisitos y
pasos para su inscripción.
 Realizar
 Realiz ar actividades de cooperació
cooperaciónn y proyección social. Sostiene que debe presentar una
 Legitiman el sistema político contribuyendo a su estabilidad. lista de adherentes ante el
Registro de Organizaciones
 Los partidos en el poder desempeñan la dirección de las acciones Pol Políti
íticas
cas JNE .
de gobierno.
 Los partidos, en la oposición, cumplen la función de control de la acción de gobierno a través de la
crítica y la movilización.
Según la Constitución Política, los ciudadanos pueden ejercer sus derechos individualmente o a través
de organizaciones políticas como partidos, movimientos o alianzas, conforme a ley. Tales
organizaciones concurren a la formación y manifestación de la voluntad popular.
- Princi
Princi pales funci ones de los partidos p olíticos
 Representación. Son intermediarios entre la sociedad y el régimen político, articulan las demandas
de diversos sectores para convertirlas políticas públicas.

Reducci
Re ducci
 social al ón de conflictos
formular ssociales.
ociales.
y gestionar medidasDeben
con canalizar
solucioneslasclaras
divisiones sociales.de
y específicas Intervenir en el conflictoy
modo democrático
pluralista entre los diversos actores.
 Ca
Canaliza
nalización
ción de la partici pación política. Motivan la inclusión ciudadana al sistema y fortalecen la
cultura política con hábitos de ciudadanía, socializando valores y movilizando grupos para como
elementos activos dentro de la vida pública.
 Solución del conflicto social. En el poder, deben producir políticas y suministrar personas
preparadas para los cargos en instituciones públicas que las pongan en práctica.

- Importancia. Expresan el pluralismo democrático, concurren a la formación y manifestación de la


voluntad popular, y a los procesos electorales. Asimismo, facilitan la concreción de compromisos entre
agrupaciones contrarias, se convierten en campos de aprendizaje para futuros políticos, y permiten
proponer políticas
políticas que de forma iindividual
ndividual sería difícil lograr. Por lo tanto, sin su ppresencia
resencia sería
complicado alcanzar el objetivo de crear un sistema político más estable y con mayor legitimidad.

7. Organizaciones no gubernamentales ONG. Son organizaciones no lucrativas de ciudadanos


voluntarios orientados a brindar servicios y funciones humanitarias. Están constituidos en forma
CEPRE-UNI HUMANIDADES 113

CICLO INTENSIVO ESCOLAR NACIONAL Material de Estudio Nº 1 

privada. Su objetivo es cubrir y ayudar en áreas en las cuales no existen políticas sociales o
económicas, no pretende reemplazar las acciones de los Estados u organismos internacionales.

8. Defensa Civil. Es el conjunto de medidas permanentes para prevenir, reducir riesgos, atender y
reparar los daños a personas y bienes que pudieran causar o causen desastres o calamidades. El
Estado, mediante el Sistema Nacional de Gestión del Riesgo de Desastres o Sinagerd, promueve y la
garantiza. Es el Instituto Nacional de Defensa Civil o Indeci su órgano central ejecutor. Relaciona
organismos públicos y privados, normas, recursos y doctrina orientados a la protección de la
población.

El Sinagerd está compuesto por los siguientes:


- Presidencia del Consejo de Ministros, que asume la función de ente rector.
- Consejo Nacional de Gestión del Riesgo de Desastres
- Centro Nacional de Estimación, Prevención y Reducción del Riesgo de Desastres (Cenepred)
- Instituto Nacional de Defensa Civil (Indeci) - Gobiernos regionales y gobiernos locales
- Centro Nacional de Planeamiento Estratégico (Ceplan)
- Entidades públicas, FF. AA. y PNP, las entidades privadas y la sociedad civil

- Instituto Nacional de Defensa Civil Indeci. Es un organismo público ejecutor que conforma el
Sinagerd, con calidad de pliego presupuestal, adscrito al Ministerio de Defensa. Es responsable
técnico de coordinar, facilitar y supervisar la formulación e implementación de la Política Nacional y el
Plan Nacional de Gestión del Riesgo de Desastres, en procesos de preparación, respuesta y
rehabilitación.
  Comités de Defensa Civil.
Civil . La población se organiza internamente en comisiones y se clasifican
en comités de Defensa Civil regionales,
r egionales, provin
provinciales
ciales y distritales.
  Bri gadas de D
Defensa
efensa C Civi
ivill . La población se organiza voluntariamente, para apoyar a los Comités
de Defensa Civil, antes, durante y después de que se produzca un desastre o emergencia.

 Al
 Algu
gunas
nas de ssus
us f un
unci
cion
ones:
es:  
  En prevención: organizar los comités de Defensa Civil en colegios y otros organismos, capacitar a
la población y promover los ejercicios de simulación de desastres.
  Durante el desastre o emergencia
emergencia:: contribuir al mantenimiento del orden, brindar primeros auxilios
y participar en la evacuación de la población.
  Después de la emergencia: apoyar las acciones de búsqueda y rescate, evaluar los daños, recibir,
almacenar y distribuir la ayuda.

Mediante la Ley Nº 29664, se creó el Sistema Nacional de Gestión del Riesgo de Desastres Sinagerd, como un
sistema interinstitucional, descentralizado, transversal y participativo, con la finalidad de identificar y reducir los
riesgos asociados a peligros o minimizar sus efectos, así como evitar la generación de nuevos riesgos, pre preparación
paración
y atención ante situaciones de desastre mediante el establecimiento de principios, lineamientos de política,
componentes, procesos e instrumentos de la Gestión del Riesgo de Desastres.

9. Seguridad ciudadana y cultura de paz


Seguridad ciudadana. Es el bienestar del ser humano, vinculado a derechos humanos, integridad
física, psíquica y moral de personas y su patrimonio; por ello, responde a la necesidad de estar libres
de temor y amenazas. Amparada en la Constitución
Constitució n y las leyes internacionales. La Ley 27933 la señala
como la acción integrada que desarrolla el Estado, con colaboración ciudadana, para asegurar su
convivencia pacífica, erradicación de la violencia y utilización pacífica de las vías y espacios públicos.
Asimismo, contribuir en la prevención de la comisión de delitos y faltas. En el Perú, se propone un
enfoque preventivo.
En 2003, se creó el Sistema Nacional de Seguridad Ciudadana. El Consejo Nacional de Seguridad
Nacional o Conasec es el máximo organismo encargado de la formulación, conducción y evaluación
CEPRE-UNI HUMANIDADES 114

CICLO INTENSIVO ESCOLAR NACIONAL Material de Estudio Nº 1 

de las políticas de seguridad ciudadana con autonomía funcional y técnica. Las instituciones que
integran el Conasec están encargadas de la prevención de delitos y faltas como robos, hurtos,
pandillaje, microcomercialización de drogas, entre otros males. Entre las instituciones integrantes del
Conasec tenemos las siguientes:
  Presidido por el MMinisteri
inisterioo del Interior, de su despacho depende la Policía Nacional del Perú
  Ministro de Justicia, de Educación, de Salud y de Economía o representante de cada uno
  Represent
Representante
ante de la Corte Suprema de Justicia
  Fiscal de llaa Nación o su represen

representante
tante
  Defensor del Pueblo o su represen
 representante
tante
  Dos presidente
 presidentess regionales o su suss representantes
representantes
  Alcalde Metropolitano de Lima o su represent
representante
ante
  Alcaldes de 2 provincias capitales de departamento con mayor número de electores o representantes
Cultu ra de paz. Es un conjunto de valores, actitudes y comportamientos que contribuyen a construir
la paz en base al respeto a la vida, a la dignidad de la persona y rechaza por completo la violencia en
cualquiera de sus formas (directa, estructural y cultural). Asimismo, promueve prácticas de no violencia
por medio de la educación, el diálogo y la solidaridad. Además, está alineada a los valores de la
democracia, los derechos humanos, la igualdad, la solidaridad, justicia, tolerancia y pluralismo.
Previene los conflictos tratando de atacar sus causas para solucionar los problemas mediante el
diálogo y la negociación entre las personas, los grupos y las naciones.
El Ministerio de la Mujer y Poblaciones
Poblac iones Vulnerables señ
señala
ala que para lograr una cultura de paz debemos

fomentar
 los siguientes
  El respeto a la vida, aspectos:
el fin de la violencia y la promoción y práctica de la no violencia por medio de
la educación, el diálogo y la cooperación.
  El respeto y el fomento de la igualdad de derechos y oportunidades de mujeres y hombres:
erradicación de la pobreza, analfabetismo, reducción de desigualdades entre las personas y
promoción del desarrollo económico y social con un enfoque sostenible.
  El compromi
compromiso so con el arreglo ppacífico
acífico de los confli
conflictos:
ctos: fomentar aptitudes pa
para
ra el diálogo
diálogo,, la
negociación y la solución pacífica de conflictos y problemas.
  El respeto pleno y la ppromoción
romoción de todos los derechos humanos y las libertades fundamentales:
eliminación de todas formas de discriminación y exclusión.
  El respeto y el fomento del derecho de todas las personas a la libertad de expresión, opinión e
información.
  Vivir con libertad, justicia, democracia, tolerancia
tolerancia,, solidaridad, cooperación, pluralismo, diversidad
cultural, diálogo y entendimiento a todos los niveles de la sociedad y entre las naciones; y
animados por un entorno nacional e internacional que favorezca a la paz.
El término “Cultura de Paz” fue empleado en la declaración del Congreso de Yamoussouko (1989) y fue un
aporte del sacerdote peruano Felipe Mac Gregor, quien conducía en el Perú la Comisión Nacional Permanente
de Educación para la Paz, adscrita al Ministerio de Educación. Mac Gregor publicó un libro Cultura de Paz
(1986) con descripciones completas de conflicto, violencia y paz.
10. Estado de Derecho. Es la organización político-social que se encuentra sujeta a mecanismos
regulados por ley donde los actos del Estado están limitados estrictamente por un marco jurídico,
guiados por el principio de legalidad y el respeto absoluto de los derechos fundamentales de las
personas. Asimismo, en este tipo de situación se respeta las libertades y derechos de la sociedad civil.
Sus autoridades se rigen, permanecen y están sometidas a un Derecho vigente conocido como Estado
de Derecho formal.
- Características del Estado de Derecho
  Imperio de la ley: respeto por las leyes y la Constit
 Constitución
ución como norma máxima.
  Respeto y gar
 garantía
antía de llos
os dere
derechos
chos humanos y llibertades
ibertades fundamentales.
  Separación de poderes del Estado (ejecutivo, legislativo y judicial) con relaciones de coordinación

y cooperación por un bien común para que no exista un abuso del poder.
  Control y fiscalizac
 fiscalización
ión de los poderes públicos
CEPRE-UNI HUMANIDADES 115

CICLO INTENSIVO ESCOLAR NACIONAL Material de Estudio Nº 1 

- Estado de excepción. Situación que declara un gobierno ante amenaza sobre la independencia o
seguridad del Estado. Suspende determinadas obligaciones contraídas y algunos derechos.
Tipos de Estado de excepción. Artículo 137 de la Constitución: El Presidente de la República, con
acuerdo del Consejo de Ministros, puede decretar, por plazo determinado, en todo el territorio
nacional, o en parte de él, y dando cuenta al Congreso o a la Comisión Permanente, los estados de
contemplan: 
excepción que en este artículo se contemplan: 
Estado de emergencia
 Declaratoria excepcional de carácter extraordinario generado por hechos que perturban la paz y

orden interno, situaciones catastróficas o graves circunstancias que afecten la vida de la Nación.
 Puede restringirse o suspenderse el ejercicio de los derechos constitucionales relativos a la libertad

y la seguridad personales, la inviolabilidad del domicilio, la libertad de reunión y de tránsito en el


territorio. En ninguna circunstancia se puede desterrar a nadie.
 Su plazo no excede de sesenta días y su prórroga requiere nuevo decreto. De la misma manera,

no se suspenden las garantías constitucionales.


 Si el presidente de la República considera adecuado, el control será tomado por las Fuerzas
Armadas.  
Armadas.
Estado de sitio
En caso de invasión, guerra exterior, guerra civil, o peligro inminente de que se produzcan, con
mención de los derechos fundamentales cuyo ejercicio no se restringe o suspende.
No excede de cuarenta y cinco días. La prórroga requiere la aprobación del Congreso.
11. Conviv
Convivencia
encia democrát
democrática.
ica. Es el conjunto de condiciones que permiten el desarrollo de relaciones
interpersonales
interpersonales que buscan la participación democrática y ciudadana al promover la construcción de
un entorno seguro, acogedor y colaborativo. Lograr una convivencia democrática demanda del respeto
hacia un conjunto de normas, un adecuado manejo de los conflictos y una comunicación efectiva como
una práctica oportuna y pertinente.
Las normas implican un conjunto de reglas o leyes que determinan el comportamiento y deben ser
cumplidas por un determinado individuo en un lugar y tiempo determinados. Existen distintos tipos de
normas, dependiendo de quién las promulgue, ante quién deben respetarse, quién es el encargado
de ejercer el castigo por incumplimiento. Los tipos de normas son las siguientes:
Morales. Son dictadas por el propio individuo; por lo tanto, son reglas de conducta intrapersonal. Es por esto
que están destinadas a la misma persona y responden a la propia conciencia.
Sociales. Estas normas emanan a partir de una serie de costumbres, tradiciones y modas que prevalecen
en una sociedad en particular y frente a la cual los sujetos deben responder con su cumplimiento. No son
 
obligatorias en su
Jurídicas. Son cumplimiento.
reglas de conducta que establece el Estado y se expresan en leyes. Son dirigidas para los
individuos de una comunidad política en particular, de quienes se espera su cumplimiento.

12. Convi
Convi venci
venciaa en el Pe
Perú.
rú. Se manifiesta en la capacidad que tienen los peruanos de compartir un
mismo espacio donde cada uno de involucrados participa libremente, respeta las diferencias, ejerce
sus derechos, cumple sus deberes y, de esta manera, benefician y fortalecen nuestra sociedad.

El Perú presenta sociedades que muestran diversas formas culturales y costumbres propias de cada
región. Con el objetivo de poder convivir en armonía entre las diferencias, se debe establecer lazos
que permitan el desarrollo de una cultura de paz.

Una convivencia democrática involucra respetar al que piensa de modo distinto, al que habla otra
lengua, al que presenta diferente cultura, religión o etnia: una convivencia en armonía sin violentar
ninguno de los derechos. En los dos primeros artículos de la Constitución Política peruana, se
garantiza
lograrlo selosnecesitan
derechosdeindividua
individuales,
les, locotidiana
la práctica que manifiesta
de unaelserie
interés
deen vivir bajo
valores: una cultura
Libertad, de paz.
igualdad, Para
justicia,
respeto, participación, pluralismo, tolerancia
t olerancia..
CEPRE-UNI HUMANIDADES 116

CICLO INTENSIVO ESCOLAR NACIONAL Material de Estudio Nº 1 

Principales probl ema


emass para la convi vencia democrática en el Pe
Perú

- Discri
Discri minación. Es cualquier tipo de distinción o restricción que, recibe un individuo o grupo social,
basada en el origen étnico, nacional, sexo, edad, físico, discapacidad, condición social o económica,
condiciones de salud, embarazo, lengua, religión, opinión, preferencias sexuales, estado civil o
cualquier otra tenga por efecto impedir o anular el reconocimiento o el ejercicio de los derechos y la
igualdad real de oportunidades de las personas. Implica un trato diferenciado por motivos irracionales,
prohibidos por la Constitución y conllevan prejuicios negativos hacia personas o grupos, que califican
como diferentes e inferiores. Tales actos vulneran la dignidad y niegan a individuos o colectivos su
condición misma de personas.
  En Perú, las distintas formas de discriminación social, según los sociólogos, han acentuado un
proceso de “naturalización de la discriminación” donde esta es negada por parte de las víctimas y
los victimarios; pues la consideran una conducta aceptable.
   Los más discriminados, en nuestra sociedad
sociedad,, son personas de rasgos autóctonos, a pesar de ser
el grupo mayoritario. Esta conclusión da una idea de lo complejo que resulta el racismo en el Perú.

- Exclusión social. Consiste en un conjunto de mecanismos que permiten observar el nivel de


presencia que una persona o grupo tiene o no en la sociedad; es decir, si se está dentro o está fuera
de ella y en qué distancia. Detectar la exclusión social permite diferenciar quienes participan y se
benefician de los que son excluidos e ignorados de la dinámica social.

Es usada para despojar a las personas o grupos de sus derechos de participación, titularidad de los
derechos sociales. Estos procesos excluyen a una parte de la población del disfrute de oportunidades
económicas, sociales. Su noción es multidimensional y abarca campos económico, social, político,
cultural; por ello, se señala la exclusión de mercados, de instituciones sociales y culturales.
- Racism
Racismoo Es una práctica discriminatoria
discriminat oria basada en atribuir un valor prioritario a las diferencias físicas
o somáticas (fenotipo) entre los grupos humanos. Busca fundamentar, a través de esas diferencias,
una supuesta superioridad de un grupo o comunidad étnica sobre las demás.
2.1.3. Violencia- Es la acción o conducta que causa la muerte, daño o sufrimiento físico, sexual o
psicológico tanto en el ámbito público como en el privado. Es un tipo de interacción social donde se
amenaza y/o provoca a otro ser humano o comunidad algún daño o sometimiento. La violencia puede
tener un origen individual o social. La violencia de origen social se produce cuando ciertos grupos
sufren algún tipo de violencia por su pertenencia a un determinado colectivo. En algunos casos, es
circunstancial;; en otros, estructural.
circunstancial
Una forma de tipificar la violencia más usada en instituciones del país es la clasificación de Galtung o Triángulo
de la Violencia de Galtung. Esta teoría establece que la violencia presenta tres aspectos:
- Violencia directa: visible, causa daño inmediato y se concreta en conductas de violencia física,
verbal y psicológica.
- Violencia estructural: se concreta en el conjunto elementos institucionalizados que componen la
sociedad, mantiene una situación de injusticia social e impide la satisfacción de las necesidades.
Se origina desde el sistema o estructura imperante.
- Violencia cultural: se genera en las ideas que ha desarrollado una sociedad (símbolos,
(símbolos, creencias,
normas, valores, etc.) y se concreta a través de las actitudes. Esta forma colectiva de pensar
legitima la violencia como un producto normal de una sociedad.

Violencia común. El robo y la delincuencia, en general, son caminos fáciles para la obtención de
calles 
bienes materiales. Ej.: Los asaltos en las calles 
Violencia soci al. Es la respuesta que da un grupo de pobladores que acumulan una mezcla de
frustración política.
Violencia por sentirse
polític excluidos
a. Hace delalprogreso.
referencia uso de laEj.: Reclamos
violencia paradellegar
las comunidades   cuenta
nativasen
al poder, sin tomar
subversiva  
los procesos democráticos que señala la Ley. Ej.: La violencia subversiva 
CEPRE-UNI HUMANIDADES 117

CICLO INTENSIVO ESCOLAR NACIONAL Material de Estudio Nº 1 

En 2015, se promulgó la Ley 30364, para prevenir, sancionar y erradicar la violencia contra las mujeres y los
integrantes del grupo familiar. En esta norma se contemplan cuatro tipos de violencia en estos casos: física,
psicológica, sexual y económica o patrimonial.
- Corrupc
Corrupción.
ión. C
Conjunto
onjunto de normas y acciones ilegales de individuos y/o grupos para ganar influencia
en las acciones de los gobernantes: formulación o implementación de políticas. Conlleva al uso
indebido del cargo público para obtener beneficios privados. Expresa una transacción entre corrupto
y corruptor, aprovechando su cargo público para obtener beneficio ilícito de la institución donde
participa y tiene poder. Sus principales manifestaciones son el soborno, cohecho, colusión, tráfico de
influencias, etc.
Entre las diversas formas de clasificar la corrupción podemos señalar la de Del Castillo (2003): la
política se da entre quienes realizan las normas y las políticas públicas; la burocrática, entre quienes
aplican las normas y políticas públicas; la funcional, que ayuda a las estructuras a moverse; la
disfuncional,, que obstaculiz
disfuncional obstaculizaa y vulnera la gestión de los recursos públicos.
En 2012, se aprobó el Decreto Supremo Nº 119-2012-PCM, que aprueba el Plan Nacional de Lucha Contra
la Corrupción 2012-2016. Asimismo, desde el año 2000, el gobierno ha emprendido un proceso constante de
lucha contra la corrupción como la aprobación, en abril del 2018, del Plan Nacional de Integridad y Lucha
contra la Corrupción 2018-2021.
- Pobreza.
Pobr eza. El INEI señala que es una condición
condició n donde una o más personas tienen un nivel de bienesta
bienestarr
inferior al mínimo socialmente aceptado. Se asocia con la incapacidad para satisfacer necesidades
básicas de alimentación. Luego, incluye salud, condiciones de vivienda, educación, empleo, ingresos,
gastos, y aspectos más extensos como la identidad, los derechos humanos, la participación popular,
entre otros. Su gasto per-cápita (por persona) mensual, no supera el costo de la canasta básica de
alimentos y no alimentos.
Pobreza extrema. Situación donde en un hogar no disponen de recursos que permitan satisfacer al menos
las necesidades básicas de alimentación; es decir, una canasta básica de alimentos.
Pobreza monetaria. Situación el gasto per cápita de un hogar es insuficiente para adquirir una canasta básica
de alimentos y no alimentos (vivienda, vestido, educación, salud, transporte, etc.).
En abril de 2019, el INEI publicó su último informe sobre la pobreza monetaria en el Perú para el 2018.
Donde indicó que la pobreza monetaria afecta al 20,5% de la población. La tendencia en el país, en
la última década, indica una disminución; sin embargo, entre 2016-2017 aumentó en 1 %.
En 2018, el gasto real promedio per cápita (por persona) mensual fue de 758 soles registrando un aumento
de 1,7% (equivalente a 12 soles per cápita) respecto al
a l nivel alcanzado el año 2017. La Línea de Pobreza,
Pobr eza, en
2018, fue de 344 soles por persona (338 en 2017); por lo tanto, la pobreza monetaria
m onetaria indica los que no logran
superar ese monto. Para el caso de pobreza extrema, se considera que no cubren el valor de la canasta
alimentaria de 183 soles per cápita.
Para 2018, INEI organizó las regiones según nivel de pobreza. En el nivel 1 (mayor índice de pobreza
monetaria) está Cajamarca entre 37% y 46%. En un segundo grupo están Amazonas, Apurímac,
Ayacucho, Huancavelica,
Huancavelica, Huánuco, Loreto, Pasco, Puno que presentan entre 33% a 36%. Se observa
como Huancavelica redujo su índice de pobreza. Entre las regiones con menor índice de pobreza
(nivel 5) destacan Ica y Madre de Dios que presentan entre 2% a 4%. Lima y Callao están en el nivel 4.
La población en extrema pobreza, en 2018, representa el 2,8% (4,1 en 2015 y 3,8 en 2017). En
Cajamarca presenta entre 9% a 14% de incidencia (Nivel 1). En el nivel 2 están Amazonas, Ayacucho,
Ayacucho,
Huancavelica,
Huancavelica, Huánuco, Loreto, Pasco, Puno presentan entre 6% a 8% de incidencia (en 2017, solo
Amazonas, Huancavelica, Loreto, Puno estaban en este nivel). En el grupo 4 están Arequipa, Ica,
Lambayeque, Madre de Dios, Moquegua, Provincia Constitucional del Callao, Provincia Lima, región
Lima, Tacna, Tumbes, Ucayali que están por debajo del 1%.
Los peruanos co nsideran como los pr oblemas más impo rtantes (porcentaje). Fuente: INE
INEII
CEPRE-UNI HUMANIDADES 118

CICLO INTENSIVO ESCOLAR NACIONAL Material de Estudio Nº 1 

2012 2017
Pobreza 34.5 Corrupción 48.1
Falta de empleo 26.5 Delincuencia 44.5
Delincuencia 26.3 Pobreza 19.8
Corrupción 25.0 Falta de seguridad ciudadana 19.1
Falta de seguridad ciudadana 18.0 Falta de empleo 13.4
Mala calidad de la educación estatal 14.7 Mala calidad de la educación estatal 12.2

Evaluación

1. Es la acción consistente en la supervisión a A) un partido político.


las autoridades elegidas democráticamente por B) una organización no gubernamental.
parte de los miembros de una sociedad. C) un frente
f rente de defensa regional.
A) Sociedad civil D) un movimiento social.
B) Seguridad ciudadana E) una organización social de base.
C) Control ciudadano
D) Contrato social 4. Es la sociedad jurídica y políticamente
E) Cultura de paz. organizada por medio de la estructuración del
poder público para su ejercicio dentro de un
2. El Acuerdo Nacional es un conjunto de territorio determina
determinado.
do.
políticas de _______ consensuadas que fue A) Ciudadanía
Ciudadanía B) Gobierno
planteado durante la presidencia de ________. C) Nación D) República
A) Estado – Alejandro Toledo E) Estado
B) gobierno – Valentín Paniagua
C) gobierno – Ollanta Humala 5. Está referido a los distintos mecanismos por
D) Estado – Alberto Fujimori los cuales un grupo social queda al margen de
E) Estado – Alan García la participación social y la igualdad de
oportunidades.
3. Amnistía Internacional es una entidad sin A) Racismo B) Violencia
fines de lucro que promueve la defensa de los C) Dictadura D) Exclusión
derechos humanos a nivel nacional e E) Etnocentrismo
internacional.l. Tomando en cuenta estas
internaciona 1C 2A 3B 4E 5D
características, esta institución es un ejemplo
de
CEPRE-UNI HUMANIDADES 119

CICLO INTENSIVO ESCOLAR NACIONAL Material de Estudio Nº 1 

Economía 
TEMA I. ECONOMÍA.
ECONOMÍA. DIVISIÓ
DIVISIÓN
N DE LA ECONOM
ECONOMÍA.
ÍA. PROBLEMA
PROBL EMA DE L
LAA ESCASEZ.
COSTO DE OPORTUNIDAD. NECESIDADES HUMANAS. BIENES Y SERVICIOS.

1. Definic
Definición
ión de Econo
Economía
mía
Etimológicamente,
oikonomos el término
 (oikos, casa economía
y nemein   provieneesdel
, administrador); latín
decir, unoeconomus , y ya
jefe de familia, este
seadel griego
el padre,
la madre o ambos, son los administradores que deben llevar un estricto régimen presupuesta
presupuestal.l.
1.
1.1.
1. D
Defini
efinicion
cion es de econom
economía
ía
“Es la ciencia social que se ocupa de estudiar de la manera como se administran
administ ran los recursos esc
escasos,
asos,
para poder hacer frente a nuestras necesidades que se presentan de manera ilimitada ”. Teoría
neoclásica  –  Lionel Robbins. La economía política es la ciencia que estudia las leyes que rigen la
producción, la distribución, la circulación y el consumo de los bienes materiales que satisfacen
humanas .  –  Karl Marx. A su vez, este autor señala que es la ciencia que estudia las
necesidades humanas.
relaciones sociales de producción.
1.
1.3
3 Objeto de estud
estudioio  
En tanto el hombre se enfrenta a objetos y técnicas para producir, la economía se encarga de
estudiar las actividades económicas, es decir, lo que realiza el humano para satisfacer sus
necesidades. La economía se denomina ciencia a partir de la publicación de la obra
Riqueza de las naciones, de Adam Smith (1776), a quien se considera el padre de la
moderna. 
economía moderna. 
2. Divisi
División
ón de la E
Econom ía 
conomía
Esta se planteó por Jhon Neville Keynes (padre de Jhon Maynard Keynes) en su libro
El alcance y método de economía política (1891).
2.
2.1.
1. Economía pos iti va. Trata de conocer y describir la realidad tal como es sin la
itiva.
intervención de juicios de valor o consideraciones morales. Se refiere a los hechos, lo que
es . Se divide en 
en 
Economía descript iva 
descriptiva Teoría
Te económi ca 
oría económica
Recopil ación de datos
Recopil datos que
 que describen la Es el conjunto de principios, leyes, teorías y
realidad económica. modelos  que permitan describir, explicar y predecir
modelos 
Ejemplo: el INEI (Instituto de Estadística e los fenómenos económicos. Se apoya en la
Informática). También, los procesa en información proporcionada por la Economía
cuadros estadísticos y los publica. descriptiva.
Ejemplo: la teoría del consumidor y del productor.
2.
2.1.
1.1
1 División de la Teoría Econó mica  
Econó
Microeconomía  Macroeconomía 
Estudia los agentes económicos individuales 
individuales   Estudia la economía en forma conjunta,
conjunta, a
y cómo las familias y las empresas toman través de los agregados económicoeconómicos.
s.
decisiones para determinar los precios y las Ejemplo: El PBI, la inflación, el desempleo, y
cantidades de producto de equilibrio.
equilibrio. el crecimiento económico.

2.2. Economía normativa. Conjunto de medidas adoptadas por el Gobierno con el fin
de alcanzar
alcanzar el bienestar general de lo que debería ser . Para ello aplica lo siguiente:
Política Fiscal   Política Monetaria 
CEPRE-UNI HUMANIDADES 120

CICLO INTENSIVO ESCOLAR NACIONAL Material de Estudio Nº 1 

A cargo del Ministerio de Econom Finanzas  (MEF).


Econom ía y Finanzas A cargo del Banco Central d e Reserva
Reserva  
Instrumentos: Impuestos y gasto público. (BCRP).
Puede ser: Expansiva (aumento de gasto) o Instrumentos: Tasa de encaje legal.
contractiva (aumento de impuestos). Objetivo: Preservar la estabilidad
monetaria.
3. El prob lema de la escasez 
Es el conflicto permanente entre necesidad es ilimitadas y los recursos escasos. 
necesidades
El problema de la escasez implica que los recursos deben asignarse alternativamente en la
producción, lo que significa que cuando se decide entre la producción de dos tipos de bienes también
se ha decidido la cantidad de recursos que serán destinados para cada uno.
Necesidades
Necesidades ilimi tadas 
il imitadas Recursos escasos  
Hoy me gustaría estudiar, ir al cine, jugar fulbito, Tengo suficiente tiempo, dinero. Puedo hacer
etc. todo lo que yo deseo. 
No puedo hacer todo lo que deseo porque no ttengo engo suficientes recursos.

4. El costo de oportunidad  
El costo de oportunidad se presenta siempre que tomamos alguna decisión y es la mejor
alternativa dejada de lado. Por ejemplo, al elegir entre estudiar o divertirse. Supongamos que
se elige ahorrar, entonces, la mejor alternativa dejada de lado es el de consumir; es decir,
dejar de consumir para ahorrar indica que aquel es el costo de oportunidad de este.

5. Los di
diez
ez pri
princip
ncipios
ios de
d e la Economía
  Las personas enfrentan
enfrentan disyuntivas.
  El costo de una cosa es aquello a lo que se renuncia para obtenerla.
  Las personas racionales piensan en términos
términos marginales.
  Las personas responden a los incentivos.
incentivos.
  El comercio puede mejorar el bienestar
bienestar de todos.
  Los mercados normalmente son un buen mecanismo para organizar la actividad
económica.
  El gobierno puede mejorar algunas veces los resultados del del mercado.
 El nivel de vida de un país depende de la capacidad que tenga para producir bienes y
servicios.
  Cuando el gobierno imprime demasiado dinero, los precios
precios se incrementan.
  La sociedad enfrenta a corto plazo una disyuntiva
disyuntiva entre inflación y desempleo.
6. Necesid
Necesidades
ades
Necesidad es la sensación de carencia o insuficiencia,
insuficiencia, ya sea material o inmaterial,
que el hombre experimenta como consecuencia de sus exigencias corporales o
espirituales. Esta sensación desagradable debe ser aplacada mediante el consumo de
bienes y servicios. 

6.
6.1.
1. P
Proc
roceso
eso de satisfacc
satisfacción
ión de las necesidades
CEPRE-UNI HUMANIDADES 121

CICLO INTENSIVO ESCOLAR NACIONAL Material de Estudio Nº 1 

6.
6.2.
2. C
Características
aracterísticas de las necesidades
Ilimitadas en El número de necesidades que experimenta el hombre es ilimitado y varían
número  
número de un lugar a otro y de época a época.
Conocida como la ley de la saturabilidad de las necesidades.
Limit ada
adass en Ejemplo:
gaseosa.Para
Ley calmar la sed,
de Gossen: “Eltal vez, tomaríamos
deseo de
decrece no mássede
crece conforme dosface
satis vasos
satisface unade
capacidad  
capacidad
necesidad”. Esto también es conocido como el principio de la utilidad marginal
decreciente
Depende de las circunstancias y de acuerdo al momento en que se dé dicha
Varían en
necesidad.
intensidad  
intensidad
Ejemplo: Tomar un helado en verano o tomarlo en invierno
Las necesidades se presentan de forma simultánea, y debido a la escasez de
Concurrentes   recursos hay que aplacar las más urgente o apremiantes.
Concurrentes
Ejemplo: Cuando tienes hambre puedes tener sed y frío f río al mismo tiempo.
Para la satisfacción de algunas necesidades es preciso antes poder satisfacer
otras necesidades.
Complementarias  
Complementarias
Ejemplo:  La necesidad de estudiar exige un centro de estudio, libros, pasajes,
Ejemplo:
fotocopias, etc.
Sustitutas  
Sustitutas Una misma
Ejemplo: necesidad
lápiz puede satisfacerse de distintas maneras.
o portaminas.
La forma como se satisface una necesidad tiende a repetirse constantemente,
Tienden a fijarse 
fijarse  entonces se fija o se hace un hábito.
Ejemplo: El fumar.
6.3. Clasificación  de  l as  necesidades  
Según su intensidad
Su satisfacción es indispensable para la conservación de la vida, por lo que
Necesidades
deben saciarse obligatoria y prioritariamente.
prioritariamente.
primarias  
primarias
Ejemplo: Alimentación
Son aquellas necesidades que deben satisfacerse para contribuir a mejorar el
Necesidades
nivel de vida del ser humano.
secundarias  
secundarias
Ejemplo: Estudio
Necesidades Llamadas necesidades suntuarias, superfluas o de lujo.
terciarias  
terciarias Ejemplo: Relojes Rolex.
CEPRE-UNI HUMANIDADES 122

CICLO INTENSIVO ESCOLAR NACIONAL Material de Estudio Nº 1 

7. Bienes
Se denomina bienes a todas las cosas u objetos materiales o inmateriales que, por sus
características, tienen la capacidad de satisfacer necesidades humanas .

7.1.. Clases de bienes


7.1 bi enes
Los bienes libres son provistos por la naturaleza.
naturaleza.

Bienes
(nolibres Son
costoabundantes debido a que
de oportunidad. no sevalor
Tienen incurre
deenuso
un
económicos) (satisfacen necesidades), pero no tienen valor de
 (no tiene capacidad de ser intercambiado).
cambio (no
cambio
Son aquellos que, para obtenerlos, requieren de la
intervención del hombre.
hombre. Son escasos, porque al
Bienes consumirlos y al producirlos se incurre en un costo
económicos de oportunidad. Tienen valor de uso (satisfacen
necesidades) y valor de cambio (tienen (tienen la
capacidad de ser intercambiados).
intercambiados ).

7.
7.2.
2. Clasificación
Clasificació n de bienes e
econó
conómico
mico s
Los bienes económicos pueden ser clasificados utilizando diversos criterios.

a. Por
Por su fu
funció
nció n o grado
Sonde elaboración
aquellos que para ser consumidos deben ser transformados
Bienes intermedios 
intermedios  
previamente; es decir, requieren de una transformación adicional.
Se encuentran listos para el consumo; es decir, se orientan a la
Bienes fin ale
aless   satisfacción de necesidades de manera directa y por lo tanto, no requieren
de alguna transformación adicional.
b. Por su utili za
zación
ción
Se utilizan para satisfacer las necesidades personales de los seres
Bienes de consumo 
consumo   humanos. Satisfacen las necesidades en forma inmediata y directa.
Se usan para crear otros bienes, es decir, se consumen en un nuevo
c apital   proceso productivo. Son utilizadas por las empresas (maquinarias,
Bienes de capital (maquinarias,
equipos, instalaciones, insumos, materias primas).
c. Por l a relación
relación q ue surge e
en
n su utili
utiliza
zación
ción
Dos bienes son complementarios si un bien de forma independiente no es
Bienes capaz de para
otro bien satisfacer unapueda
que así determinada necesidad
satisfacer y es necesario asociarlo con
esa necesidad.
complementarios  
complementarios 
Presentan demanda conjunta.
Ejemplo: Limón + Pescado = Ceviche.
Dos bienes son sustitutos si satisfacen la misma necesidad. También son
Bieness sustitutos
Biene sustitutos   llamados bienes sucedáneos.
Ejemplo: Pepsi y Agua Mineral.
d. Por su relación con el ingreso
Es aquel bien que, al aumentar el ingreso de las personas, la demanda de este
Bien inferior   disminuye y viceversa.
Ejemplo: La ropa de segunda mano.
Es aquel bien que, al aumentar el ingreso de las personas, se incrementa su
Bien normal 
normal   demandada, y viceversa.
Ejemplo: Alimentos o ropa de calidad estándar o de marca conocida.
Al igual que el bien normal, al aumentar el ingreso se incrementa su demandada,
Bien superior   con el detalle de que la variación porcentual de la demanda es mayor que la
variación porcentual del ingreso.
CEPRE-UNI HUMANIDADES 123

CICLO INTENSIVO ESCOLAR NACIONAL Material de Estudio Nº 1 

Es aquel bien que, al ocurrir una variación en el ingreso, la demanda del bien no se
Bien neutral 
neutral  
altera.
e. Por
Por su relación ccon
on el precio
Son los bienes que cumplen con la ley de la demanda, es decir, al aumentar el precio
Bien normal del bien, su cantidad demandada disminuye, ceteris paribus.
paribus. Su curva de demanda
es de pendien
pendiente
te negativa.

Es
del un bien inferior.
producto. Su cantidad
Presentan curva dedemandada tiene una
demanda positiva. Esrelación
un casopositiva con elpor
mencionado precio
Bien Giffen 
Giffen   Alfred Marshall en su libro Principios de economía.
economía. Sin embargo, en la actualidad los
datos recopilados muestran que hubo un error en la interpretación al confundirse
cantidad demandada con demanda.
Es un bien superior. La cantidad demandada tiene una relación positiva con el precio
positiva.  Esto se debe a
del producto. Por esa razón, presentan curva de demanda positiva.
Bien Veblen 
Veblen  
que un aumento del precio puede generar que la percepción de exclusividad
aumente haciéndolo más atractivo para el consumidor de altos ingresos.

8. SERVICIOS:  Son las actividades económicas que realizan las personas o instituciones para
satisfacer directamente necesidades de otras a través de su prestación, ayuda o auxilio.

8.1. Características
•  Son inmaterial
inmateriales
es (intangibl
(intangibles).
es).
•  Se consumen al mismo tiempo que se producen.
•  Su prestación requiere del uso de bienes que permitan a quien los produce
realizar adecuadamente su labor.
8.
8.2.
2. C
Clasifi
lasifi cación
Se tiene dos criterios para clasificarlos.
Servicios
Según privados Son administrados y organizados por la empresa privada.
quién los
brinda Servicios Son administrados y organizados por el Estado, a través del gobierno
públicos central, municipalidades
municipalidades y empresas públicas, entre otros.
Servicios El beneficio solo alcanza a una persona en un tiempo y espacio
Según a individuales determinado.
quiénes se
les brinda Servicios
colectivos El beneficio lo recibe simultáneamente un grupo de personas.

EVALUACIÓN 01
1. Adolfo Pérez está convencido de que la sus costos, se encuentra asociado a la
economía debe asegurar que todos tengan definición __.
alimento y que, además, el Estado cuente con A) neoclásica B) marxista C) keynesian
keynesianaa
los suficientes recursos para defender la D) positivista E) etimológica
frontera ante cualquier posible ataque. La
creencia de Adolfo está en el marco de la 3. Dado los siguientes enunciados sobre el
 __________.
 _________ _. presupuesto público señale la verdad (V) o
A) microeconomía B) economía descripti
descriptiva
va falsedad (F) de los mismos
C) macroecono
macroeconomía
mía D) teoría económica I. El problema de la escasez se asocia a las
E) economía normativa necesidadess ilimitadas.
necesidade
II. El costo de oportunidad se asocia
2. El estudiobusca
económico de la optimizar
forma en sus
que resultados,
un agente directamente con el problema de la escasez.
esto es, maximizar sus ganancias o minimizar
CEPRE-UNI HUMANIDADES 124

CICLO INTENSIVO ESCOLAR NACIONAL Material de Estudio Nº 1 

III. En una determinada época puede haber 5. Los padres de Paquito lo llevaron a un centro
baja inflación y bajo desempleo de diversiones y Paquito quedó deslumbrado
simultáneamente. con todos los juegos que veía alrededor suyo.
IV. El comercio siempre mejora el bienestar de Sobre todo, tuvo ganas de jugar Paintball, subir
todos. a la montaña rusa, al Tagadá y a los carros
A) VFFF B) FFFF C) VVFF D) VFFV E) FVFV chocones, esto implica que las necesidades de
Paquito son __________. Lamentablemente
4. Relacione correctamente: bien(es) - clase de solo podía elegir dos de ellos, pero un mes
bienes. después les pidió a sus padres que lo vuelvan
I. agua y jabón a. libre a llevar, dado que sus necesidades
II. luz solar b. intermedio  __________.
 _________ _.
III. motor c. complemen
complementarios
tarios A) concurrentes – son sustitutas
IV. menudencia d. inferior B) concurrentes – son limitadas en capacidad
A) Ib,IIa,IIIb,IVd B) Ic,IIa,IIId,IVb C) concurren tes – tienden a fijarse
concurrentes
C) Id,IIa,IIIb,IVa D) Ic,IIb,IIId,IVa D) ilimitadas en número – varían en intensidad
E) Ic,IIa,IIIb,IVd complementarias – varían en intensidad
E) complementarias
 
TEMA II. FACTORES PRODUCTIVOS. PROCESO ECONOMICO. LA EMPRESA
POBLACION LABORAL EN EL PERU Y EL MERCADO

1. FACTORES
Son PRODUCTIVOS
los diferentes   se utilizan para poder crear bienes y servicios, los cuales
elementos que
se combinan óptimamente para la producción. Los factores produc
productivos
tivos se cclasifican
lasifican en:
Está constituido por las fuentes naturales, con las cuales se inicia la cadena
Tierra  
Tierra productiva. Dentro de este factor se encuentran las tierras empleadas para la
agricultura y la ganadería, la flora y la ffauna,
auna, los yacimiento
yacimientos, s, los bosques, etc.
Se compone del esfuerzo y las capacidades
las  capacidades hhumanas
umanas del trabajador, que son tanto
Trabajo   físicas como intelectual
Trabajo intelectuales.
es.
Son bienes que sirven para producir otros bienes: maquinarias, herramientas,
Capital   instalaciones, infraestructura, materias primas, insumos. Es el factor productivo
Capital
generado por el hombre, tambi tambiénén se den
denomina
omina ccapital
apital físico o real.
Es la organización en sí, que realiza la producción del bien o del servicio, a
Empresa  cambio de un beneficio. El rendimiento de este factor depende de quien dirige la
Empresa 

Estado  
Estado Es el organismo
precios, el Estadofiscalizador de lacontrol
puede ejercer actividad productiva.
productiv
sobre ciertosa.productos
En cuantoestableci
se refiere
estableciendoa los
endo precios
máximos y precios mínimos.

2. PROCESO ECONÓMICO 
Es un conjunto de fases o etapas mutuamente dependient
dependientes
es que explican la conducta de la
actividad económica que desarrollan las sociedades para satisfacer sus necesidades.

2.1. Etapas
CEPRE-UNI HUMANIDADES 125

CICLO INTENSIVO ESCOLAR NACIONAL Material de Estudio Nº 1 

a) Producción: Es la actividad orientada a generar bienes y servicios útiles al hombre.


En esta etapa intervienen los siguientes factores productivos:

Factores prod uctivos  


Factores
Factor   Características 
Tierra (N) Pasivo, condicionante y originario.
Trabajo (L) Activo, determinante y originario.
Capital (K) Auxiliar y derivado. PRODUCE
Empresa (E) Organizador.
Estado (G) Estabilizador y regulador.

K , E, G) 
Función de producción: f (Q) = f (N, L, K,

b) Circulación: Fase del proceso económico que pone en relación estrecha las unidades
productivas (empresas) con las unidades de consumo (familias). En esta fase se fforman
orman los
precios.

b) Distribución: Es la asigna
asignación,
ción, retribución o pag
pagoo correspondiente a cada factor
productivo

Factor   Retribución
Tierra 
Tierra  Renta
Trabajo  
Trabajo Salario
Capital  
Capital Interés
Empresa  
Empresa Beneficio o ganancia
g anancia
Estado  
Estado Tributos

d) Consumo: Es el empleo o utilización de los bienes y servicios producidos para la


satisfacción de las necesidades.
e) Inversión: Es el gasto destinado a la adquisición de materias primas, insumos y
bienes de capital para mantener o incrementar la capacidad productiv
productivaa de las empresas y el
CEPRE-UNI HUMANIDADES 126

CICLO INTENSIVO ESCOLAR NACIONAL Material de Estudio Nº 1 

país. La inversión equivale al ahorro que se ha hecho en un periodo anterior bajo el supuesto
de una economía cerrada

AHORRO (t0) = INVERSIÓN(t1)

3. EMPRESA 
Surge como consecuencia de la actividad económica del ser humano, para la producción de bienes
y servicios. Es el factor
f actor organizador
organizador de la producción.

3.
3.1.
1. Cla
Clasif
sificació n 
icación
a) Según el tamaño de la empresa: 
De uno (1) a diez (10) trabajadores. Las ventas anuales no deben superar
Micro empresa 
empresa 
las 150 UIT.
De uno (1) a cien (100) trabajadores. Las ventas anuales no deben superar
Pequeña empr
empresa
esa  
las 1700 UIT.
De cien (100) a doscientos (200) traba
trabajadores
jadores.. Las ventas anuales superan
Mediana empresa 
empresa  las 1700 UIT.

Gran empresa 
empresa  Más de 200 trabajadores.
b) Según la legislación o aspecto jurídico
Empresa  
Empresa Es la forma más simple de organizar una empresa, ya que posee
Individual de 
de  un solo propietario, que es el ttitular.
itular. Tiene limitaciones para su
Responsabilidad  
Responsabilidad crecimiento ya que no hay manera de incorporar capital de otros socios.
Limitada (E.I.R.L.) 
(E.I.R.L.)  Suele ser usado para microempresas.
Se constituye con un mínimo de dos y un máximo de veinte personas,
Sociedad
Socieda d c omercial denominados socios, quienes aportan capital y participan en la
de responsabilidad administración de la empresa. Ante compromisos de la sociedad, los
Limititada
Lim ada ( S.R
S.R.L.)
.L.) socios no responden con su patrimonio personal. Suele ser usado para
empresas familiares.
El capital está dividido en acciones. Los propietarios de la empresa son
los accionistas, quienes reciben las ganancias (dividendos) en
Sociedad  
Sociedad
proporción a la cantidad de acciones que poseen. Las sociedades
 Anón
 An ónimima
a (S.A.)
(S.A.)   anónimas pueden ser abiertas (S.A.A.) o cerradas (S.A.C.). Suele ser
usado para grandes empresas.

4. POBLACIÓN LABORAL EN EL PERÚ  

Población en Edad de Trabajar (PET): Es el conjunto de personas que están aptas


para trabajar. En el Perú, se considera a partir de 14 años.
Población Económicamente Acti va (P(PEA)
EA):: Personas en edad de trabajar
tr abajar Población no 
no  
que se encuentran trabajando (ocupados) o buscando activamente trabajo Económicamen
(desocupados). te 
te  
PEA ocupada: Es el conjunto de la PEA que trabaja en PEA    Ac ti va
 Acti
desocupada:   (inactivos):
una actividad económica. En este grupo se encuentra las desocupada:
personas independientes
independientes y dependientes. Personas de
CEPRE-UNI HUMANIDADES 127

CICLO INTENSIVO ESCOLAR NACIONAL Material de Estudio Nº 1 

PEA adecuadamente PEA subempleada: Son los 14 años a más Son todas las
empleada: trabajadores cuya ocupación que no tienen personas que
Está conformada por aquellos no es adecuada cuantitativa trabajo, pertenecen a la
que laboran 35 horas o más ni cualitativamente. En el buscan población en
a la semana y reciben caso del Perú, se considera activamente y edad de trabajar
ingresos por encima del dos grupos de subempleo, no lo que no trabajan
ingreso mínimo referencial o por horas (menos de 35 horas encuentran. ni buscan
aquellos que laboran menos a la semana) y por ingresos Se puede trabajo y no
de 35 horas semanales y no (ingreso mensual menor al dividir a la desean trabajar.
desean trabajar más horas. ingreso mínimo de vez en Ejemplo: las
referencia). cesantes y amas de casa,
as ir
iran
ante
tes.
s. los estud
estudian
iantes
tes

5. EL MERCADO
Es el conjunto de mecanismos que permite la interacción de vendedores (ofertantes) y
compradores (demandantes) con el objetivo de realizar el intercambio de bienes, servicios y
factores productivos. 

Mercados Virtuales versus Mercados Espaciales. 


Espaciales.  Como consecuencia del progreso de la la
tecnología el mercado se ha desprendido de su carácter localista; hoy en día mercado es el conjunto
de actos de compra y venta sin ninguna referencia espacial concreta.

El mercado
mercado se estructura en dos fformas:
ormas:
Mercado d e competencia perfecta
Mercado perfecta   Mercado
Me rcado de comp etencia imperfecta
imperfecta  
El precio es determinado por la interacción de El precio es determinado por alguien o por
los ofertantes y los demandantes. algunos ofertantes y demandantes
demandantes..

6. ELEMENTOS DE MERCADO:

Productos(Q) Precio(P)
Mercancías, bienes, servicios, factores Es el valor monetario de los productos que serán
f inancieross que serán intercambiados. Este precio es determinado por la
productivos y activos financiero
intercambiados.  interacción de la oferta y la demanda, dependiendo
Ofertantes(O)   si es de competencia perfecta o imperfecta. El precio
es la principal información que nos brinda el mercado
Integrando por los aagentes
que irán al mercado económicos
vender los productos al tomar nuestras decisione
decisioness económicas.
teniendo una capacidad y disposición para
venderlos.  
Demandantes(D)
Integrado por los agentes económicos que
irán al mercado a adquirir los productos
teniendo capacidad y disposición para
adquirirlos.  

7. CLASIFICACIÓN
7.
7.1.
1. S
Según
egún el volumen
vol umen ttransado
ransado
CEPRE-UNI HUMANIDADES 128

CICLO INTENSIVO ESCOLAR NACIONAL Material de Estudio Nº 1 

MERCADO MAYORISTA MERCADO MINOR


MINORISTA
ISTA
Donde se pueden adquirir grandes cantidades Donde los productos se comercializan por
de un determinado producto. La ventaja de este unidades o cantidades pequeñas generalmente
mercado es que a medida que se adquiere está formada por comerciantes y
mayor cantidad el precio unitario disminuye. revendedores. El precio unitario es mayor.
Ejemplo: mercado mayorista de Santa Anita Ejemplo: Supermercados, bodegas.

7.2. Según el acceso al mercado


MERCADO AB ABIERTO
IERTO MERCADO CERRADO
Son los mercados donde no existen restricciones Son aquellos mercados donde existen
de ningún tipo, ni para los compradores ni restricciones económicas, legales,
vendedores. tecnológicas, etc.
Ejemplo: mercado cambiario en el Perú. Ejemplo: mercado de valores primario de Lima.
7.
7.3.
3. S
Según
egún el produc
pr oducto
to
MERCADO REAL MERCADO FINANCIERO
Es el mercado donde se producen y se pone a la Es el mercado donde existen intermediarios
venta de bienes y servicios. financieros y se venden acciones y bonos.
7.4. Según el aspecto legal
Donde los participantes, ofertantes y demandantes, cumplen las regulaciones
MERCADO
FORMAL propias de cada
inscripción país, licencia
en registros de RUC,
públicos, funcionamie
funcionamiento,
nto,impuestos.
pago de certificado de Defensa Civil,
Ejemplo: bancos, supermercad
supermercados. os.
Donde los participantes no cumplen las disposiciones legales tributarias por
razones de evasión y elusión fiscal, y falta de responsabilidad al no cumplir las
MERCADO disposiciones
disposicion es administrativas, regulatorias del Estado. El comercio no es
INFORMAL clandestino y el producto que se comercializa es legal.
Ejemplo, el trabajo
el trabajo doméstico no declarado, la venta ambulante espontánea, la
minería informal.
En este mercado se comercia productos prohibidos por ley o se producen
MERCADO
ILEGAL infringiendo leyes distintas a las tributarias. Su comercio es clandestino
clandestino..
Ejemplo: narcotráfico, contrabando, celulares robados, minería ilegal.
Se genera por la intervención del Estado al establecer control de precios y los
MERCADO
productores restringen su producción generando desabastecimiento. La escasez
NEGRO
origina un comercio clandestino a precios diferentes del precio oficial.
8. MODELO
MODELO DEL FLUJ
FLUJOO CIRC
CIRCULAR
ULAR DE L
LAA ECONOMÍA
Es un modelo económico que permite analizar de manera sencilla y resumida la interacción de las
unidades de consumo (familias), las unidades de producción (empresas) y el Estado, sin comercio
exterior.  
exterior.
CEPRE-UNI HUMANIDADES 129

CICLO INTENSIVO ESCOLAR NACIONAL Material de Estudio Nº 1 

 Agent
 Ag entes
es ec onóm
on ómii co
coss Merc ado
adoss
Están conformados por Se identifica dos tipos de mercados:
familias, empresas, Estado los de bienes y los de factores

Mediante este esquema se muestra el proceso de traslado o circulación de bienes, servicios


y factores productivos, que se da entre los agentes económicos y se ejecuta mediante el
mecanismo de mercado. El Estado brinda y adquiere factores, bienes y servicios en los
mercados
vez, cobra pagando o cobrando
los impuestos porlas
y entrega ellos al igual que ylas
transferencias familias o las Empresas. Pero a la
subsidios.
8.
8.1.
1. R
Rol
ol econó
económico
mico de los Agentes

Las familias deciden cuánto de su trabajo, tierra y capital venderán o alquilarán


en los mercados de factores 
factores   a cambio de salarios, renta, intereses y
FAMILIAS beneficios.
(Unidades de Su objetivo es satisfacer a plenitud sus necesidad
necesidades,es, al menor gasto posible.
consumo) Supuestos: Las familias son dueñas de los factores de producción
Maximizan utilidad.
EMPRESAS Es una unidad económica que decide las cantidades de factore factoress de la
(Unidades de producción que alquilarán, y vende su producción en los mercados de bienes.
producción) Supuestos: Maximizan ganancias
Su papel en la economía moderna, tiene tres funciones económicas principa
principales:
les:
Aumentar la eficiencia cuando promueven la competencia, reduce las
ESTADO externalidades negativas y provee bienes públicos.
Redistribución del ingreso (fomenta la equidad)
Estabilidad y el crecimiento macroeconómic
macroeconómicoo

8.
8.2.
2. T
Tipo
iposs de
d e mercados
MERCADO DE Son aquellos en los cuales se compran y venden bienes y servicios.
BIENES
MERCADOS DE Son aquellos en los cuales se compran y venden factores de la producción.
FACTORES

8.3. Flujos económicos


Son las corrientes de productos y dinero que circulan entre las familias y las empresas.
Permiten conocer las direcciones que siguen los bienes servicios, factores y el dinero,
identificando a los ofertantes y demandantes de cada mercado.
Flujo real
real o físico Flujo nominal o monetario 
CEPRE-UNI HUMANIDADES 130

CICLO INTENSIVO ESCOLAR NACIONAL Material de Estudio Nº 1 

Flujo monetario que parte de las familias a las


Flujo real de bienes y servicios que van de las
empresas por la adquisición de bienes y
empresas a las familias y distribuidos a través
servicios en el mercado de bienes.
del mercado de bienes.
Flujo monetario que parte de las empresas a las
real de los factores productivos que serán
Flujo real 
familias por la adquisición de factores
utilizadas en la producción de bienes y servicios,
productivos.
producti vos. Represent
Representaa el costo de producción
empresas. 
que van de las familias a las empresas. 
de las Empresas.

EVALUACIÓN 02 
1. El Organismo Supervisor de la Inversión en menor avance en 10 años, estuvo explicada
Telecomunicaciones (Osiptel) confirmó las sobre todo por la desaceleraci
desaceleración
ón de el/la
multas impuestas a Bitel y Entel por montos de  _____________,
 _________ ____, ante la dificultad de las
S/1 millón 79 mil y S/1 millón 176 mil personas para encontrar trabajo, originándose
respectivamente. Las sanciones corresponden a una caída de las compras, que no sólo se refleja
la prestación de servicios brindada a equipos en la categoría de centros comerciales, sino
móviles reportados como robados que se también en otras como en los productos
comercializan en los mercados _____________. alimenticios,
alimenticios, prendas de vestir y calzado.
Ambas empresas deberán adecuarse a la A) inversión B) consumo C) producción
normativa vigente implementando medidas que D) circulación E) distribución
distribución
garanticen
reportados la verificación
y que de los
eviten que equipos
la falta móviles
se repita. 4. La Cámara de Comercio de Lima (CCL) indicó
A) negros B) informales C) mayoristas que la edición XVIII de los Juegos
D) ilegales E) cerrados Panamericanos tendrá un efecto dinamizador en
2. El Joven Aldo Fernández., hasta julio del las ventas de diversos negocios. En ese sentido,
presente año se encontraba buscando trabajo; se beneficiarán las microempresas y las
luego de presentarse a varias convocatorias pequeñas empresas que generarán ventas por
durante el último mes; finalmente el día de US$30 millones en actividades vinculadas
mañana empezará a trabajar en un restaurant
restau rant de principalmente al alojamiento, servicios
comida rápida durante 4 horas diarias, lo cual le turísticos, transporte de pasajeros, gastronomía
gastronomía,,
dará el tiempo suficiente para continuar con la confecciones textiles y artesanías.
práctica del tenis, su deporte favorito. Tomando en cuenta el enunciado anterior,
Tomando en cuenta la información anterior, podemos afirmar que el factor productivo que se
podemos afirmar que Aldo Fernández ha pasado verá beneficiado tiene por característica ser
de ser A) organizador. B) auxiliar y derivado.
A) PEA Subempleada por Ingresos a Población C) pasivo y condicion
condicionante.
ante. D) regulador.
Económicamente
Económicamen te Inactiva. E) activo y determinante.
B) PEA desempleada a PEA subempleada por
horas. 5. La guerra comercial entre Estados Unidos y
C) PEA subempleada por horas a PEA China empeora por momentos con la última
desocupada. subida de aranceles mutuos, lo que está
D) PEA adecuadamente empleada a Población provocando pérdidas en las Bolsas de todo el
Económicamente
Económicamen te Inactiva. mundo. La de Wall Street registró la peor caída
E) Población Económicamente Inactiva a PEA del año y las bolsas asiáticas también han
subempleada por horas. registrado pérdidas. Esta más que demostrado
que este tipo de conflictos tienen mucha
3. Según las cifras reportadas por el Banco influencia en las fluctuaciones que se presentan
Central de Reserva (BCR); el menor crecimiento en estos mercados clasificados
clasificados como
de la economía peruana registrado en el A) reales. B) cerrados. C) abiertos.
segundo trimestre del 2019, que implicó su D) virtuales. E) minoristas.
CEPRE-UNI HUMANIDADES 131

CICLO INTENSIVO ESCOLAR NACIONAL Material de Estudio Nº 1 

TEMA III. LA DEMANDA Y LA OFER


OFERTA.
TA. ELASTICIDAD PRECIO DE DE L
LAA DEMANDA
EQUILIBRIO Y DESEQUILIBRIO DEL MERCADO. LEY LE Y DE SAY. CONTROL DE
PRECIOS. 
1. LA DEMAN
DEMANDA DA  
Se define como las cantidades de bienes y servicios que un consumidor desea y está en la
capacidad de comprar a diferentes precios en un determinado período de tiempo,
manteniendo los demás factores constantes.
1.1 La funci ón de demanda 
1.1
La función de demanda de un bien en el mercado proporciona la relación entre la cantidad
demandada del bien en un período determinado y su precio (mientras todo lo demás
permanece constante).

La expresión funcional o matemática de la demanda se representa de la siguiente manera:


Qd = f ( Px , I, G, Ps , Pc , otros)
Donde:
Qd = es la cantidad demandada G= gustos
Px = El precio del bien x Ps = precios
precios de lo
loss bienes sustitutos
I= el ingreso del consumidor   Pc =precios de bienes complementarios 
Aplicando
analizandolasolo
condición
el efecto“ceteris paribus
del precio ” (mantener
sobre la cantidad los demás tenemos
demandada, factores que:
constantes) y
paribus) 
Qd = f (P x, ceteris paribus)

1.2) La tabla de la demanda y curva de demanda 


Suponga que una función de la demanda del bien X es Qdx =5  –  Px, ceteris paribus. Al
sustituir diferentes precios, se obtiene la tabla de la demanda. Al representar cada par de
valores como un punto en una gráfica y unir los puntos resultantes se obtiene la curva de
demanda.
Tabla de la demand
demanda a Curva de demand
demandaa 
Precio  Cantidad demandada 
(Px)  (Qdx ) 
 A B C D 5  0 
4  1 
E F  3  2 
2  3 
1  4 
0  5 

Esta relación inversa entre el precio y la cantidad se refleja en la pendiente negativa de la


curva de demanda. Este hecho se conoce como ley de la demanda. 

1.
1.3)
3) Cambi
Cambios
os en demanda y cantidad
cantid ad demanda
demandada
da
Cambi
Ca o en la cantidad demandada  Ca
mbio Cambi
mbio en la demanda 
o en
Un cambio en su propio precio (Px) Cuando cualquiera de las condiciones ceteris
provocará un movimiento a lo largo paribus varía, toda la curva de demanda se
de la misma curva de demanda. El desplaza hacia la derecha o a la izquierda. En
término cantidad demandada se la figura se ilustra un aumento en la
refiere a un punto de la curva de demanda. Cuando la demanda aumenta, la
demanda. curva de demanda se desplaza hacia la derecha.
CEPRE-UNI HUMANIDADES 132

CICLO INTENSIVO ESCOLAR NACIONAL Material de Estudio Nº 1 

1.
1.4
4 Fa
Factor
ctor es que influyen
inf luyen en lo
loss despl
desplaz
azamientos
amientos d e la D
Demanda
emanda
La Demanda de un consumidor o comprador cualquiera depende de los siguientes
factores:
  El ingreso del consumidor  
  El precio de los bienes sustitutos 
  El precio de los bienes complementarios  
  Gustos, preferencias y costumbres 
  La publicidad 
  La población 
  El clima 
2) LA OFERTA
La cantidad ofrecida de un bien es la cantidad que los productores están dispuesto a vender
en un período dado a un precio determinado, manteniendo los demás factores constantes 

2.
2.1) ofert a 
1) La funci ón de oferta
La función de oferta de un bien en el mercado proporciona la relación entre la cantidad
ofrecida del bien en un período determinado y su precio (mientras todo lo demás permanece
constante).
La expresión funcional o matemática de la oferta de un bien con respecto a los factores
que la determinan. Se representan de la siguiente manera:
Qo = Q(Px , T, C, Pa, E, otros ) 
Dónde:
Qo = es la cantidad ofertada C= costos de producción
Px
Px== El precio del bien x Pa=
Pa= precio del bien alternativo
T= tecnología  E= expectativas 
expectativas
Aplicando el método “ceteris paribus”  (mantener los demás factores constantes) y
analizando solo el efecto del precio sobre la cantidad ofertada, tenemos que:

paribus) 
Qo = Q(Px , ceteris paribus)
Al variar el precio del bien, pero manteniendo
manteniendo constant
constantes
es los otros factor
factores
es (la condición
ceteris paribus), se obtiene la tabla de oferta del individuo.

2.
2.2.
2. La tabla de oferta y cur va de oferta  
curva
Tabla
Ta bla de oferta Curva de oferta
CEPRE-UNI HUMANIDADES 133

CICLO INTENSIVO ESCOLAR NACIONAL Material de Estudio Nº 1 

Precios   Cantidades a 
posibles   vender  
A 5  20 
B 4 15
C 3 10
D
E 21 5
0

La tabla de oferta relaciona la cantidad ofrecida de un bien a su precio de mercado, si todo lo


demás permanece constante. La curva de oferta tiene pendiente positiva. La relación
que existe entre el precio y la cantidad ofertada es directa, ya que a mayor nivel de
precio la cantidad ofertada se incrementa
incrementa.. A esta relación directa se conoce com
comoo la ley de
la oferta. 

2.
2.3)
3) C
Cambios
ambios en la canti
cantidad
dad ofertada y cambi
cambios of erta 
os de la oferta
Para representar el aumento (dismin
(disminución)
ución) de la cantidad ofertada basta tener en cuenta
que este aumento (disminución) consiste en un(a) incremento (disminución) en las
cantidades ofrecidas a cada precio distinto, dentro de la misma curva O.

Canti
Ca dad ofertada 
ntidad Cambi
Ca os en la oferta 
mbios
Un cambio en el precio (Px) Cuando las variaciones de factores distintos del precio
provocará un movimiento a lo largo del bien afectan a la cantidad que se suministra, estos
de la misma curva de oferta. cambios se denom
denominan
inan despl
desplazamientos
azamientos de la oferta.
El término cantidad ofertada se La oferta aumenta (o disminuye) cuando aumenta (o
refiere a un punto de la curva de oferta. disminuye) la cantidad ofrecida a cada uno de los precios
de mercado.

2.
2.4
4 Fa
Factor
ctor es que influyen
inf luyen en el despl
desplaz
azamiento
amiento de la Oferta
La oferta de un vendedor o productor cualquiera depende de los siguientes factores:
   Costo de las materias primas   Clima

   Costo de la mano de obra   Impuestos


   La tasa de interés   Tecnología


3) Elasti
Elasticid
cidad
ad precio d
de demanda::  
e la demanda

Mide el grado de sensibilidad de la cantidad demandada de un


bien ante cambios de su precio. Numéricamente
como el va
Numéricam ente se expresa
lor absoluto del cambio porcentual de la cantidad
valor
%
 = | |  

demandada dividido por el cambio porcentual del precio.


% 
CEPRE-UNI HUMANIDADES 134

CICLO INTENSIVO ESCOLAR NACIONAL Material de Estudio Nº 1 

Ti o de Dema
mand
ndaa  Valor
Valor de la Ela
Elasti
sticid ad 
cidad Número de Bienes
Elástica Mayor a 1 Muchos
Inelástica Menor a 1 Pocos
Unitaria Igual a 1 constante
Perfectamente elástica Infinito Infinitos
Perfectamente inelástic
inelástica
a Cero Ninguno

1. EQUILIBRIO DEL MERCADO 

1.1 DEFINICIÓN 
Es la
la sit
situación
uación en donde
donde las fuerzas del mercado se igualan. Esto ocurre cu cuan
ando
do se
establece un precio de equilibrio permitiendo que los planes de compra se igualen con los
planes de venta.

El precio de equilibrio es aquel que permite igualar la cantidad demandada con la cantidad
ofertada, esta cantidad es llamada cantidad de equilibrio. Las fuerzas de oferta y demanda
están balanceadas, no existe motivo alguno para que el precio suba o baje, siempre y
cuando el resto se mantenga constante o ceteris paribus.
4. GRÁFICA DEL EQUILIBRIO DE MERCADO
P
Qd = Qs = Qe
Elementos de la gráfica 
S
Punto de Equilibrio : Es el punto en el cual la
curva de la oferta y demanda se intersectan,
generandose asi el punto (QE;PE).
E
Cantidad de Equilibrio (Qe): Es la cantidad que
Pe muestra la coincidencia de la cantidad ofertada y
la cantidad demandada.( Qe = Qd = Qo )
D Precio
Pre cio d e E quilib rio (Pe): Es el precio de mercado
Equilib
que hace que la cantidad demandada sea igual a
Qe Q/t la cantidad ofrecida.

4.1 DETERMINACIÓN MATEMÁTICA DEL PUNTO DE EQUILIBRIO


CEPRE-UNI HUMANIDADES 135

CICLO INTENSIVO ESCOLAR NACIONAL Material de Estudio Nº 1 

Dadas las funciones de 2) Sustituir Pe en Qd o P Para Q = 0, tenemos:

demanda y oferta, Qspara determinar Qe: 0 = 100 – 2,5 P

determinar el precio y la P = - 100 / --2,5


2,5 = 40 S
cantidad de equilibrio: Qe = 100 a 2,5 (12,5) 40
Qd = Qs
Qs = Qe
Qe
= 50 + 1,5 (12,5) Para P = 0, tenemos:
Qd = 100 – 2,5 P = 68.75 Q = 100 – 2,5(0)
Qs = 50 + 1,5 P Así que tenemos el punto E Q = 100

de equilibrio dada por: 12,5


1) Hacer Qd = Qs para Para P=0, tenemos:

encontrar Pe: Qe= 68,75 Q = 50 + 1,5(0)

100 – 2,5 P = 50 + 1,5P Pe= 12,5  D


P = 12,5 = Pe 
50 68,75 100 Q/t

4.2 LEY DE LOS


L OS PREC
PRECIOS
IOS (OFERTA Y L A DEMANDA)  
(OFERTA
DEMAND A  
CONTRACCIÓN DE LA DEMANDA
CONTRACCIÓN L A DEMANDA  
EXPANSIÓN DE LA

Con la oferta invariable, existe una relación DIRECTA entre el precio y la demanda.
CONTRACCIÓN
CONTRAC CIÓN DE LA OFERTA  EXPANSIÓN
EXPANSIÓ N DE L A OFERTA  

Con la demanda invariable, existe una relación INVERSA entre el precio y la oferta.
La ley de precios, también llamada la ley de la oferta y la demanda, establece qué cambios de la
demanda u oferta generan cambios del precio de equilibrio.

En resumen:
Oferta no Oferta Oferta
varia aumenta disminuye
Demanda P no varía P baja P sube
no varia Q no varia Q sube Q baja
Demanda P sube Q P ¿? P sube
aumenta sube Q sube Q ¿?
CEPRE-UNI HUMANIDADES 136

CICLO INTENSIVO ESCOLAR NACIONAL Material de Estudio Nº 1 

Demanda P baja P baja Q P ¿?


disminuye Q baja ¿? Q baja

5. DESEQUILIBRIO DEL MERCADO 


El precio de un bien regula las cantidades demandadas y ofrecidas del mismo. Si el precio
es demasiado alto, la cantidad ofrecida excede a la cantidad demandada. Si el precio es
demasiado
se conocenbajo,
comoladesequilibrios,
cantidad demandada
uno de excede a lamás
l os casos
los cantidad ofrecida. Estas
representativos doscontroles
son los situaciones
de
precios.

5.1. CONTROL DE PRECIOS

Es una forma que tienen


tienen los gobiernos de modificar el precio de equilib
equilibrio.
rio. Generalmente se
establece cuando los gobiernos de turno consideran que el precio es injusto para los
consumidores (compradores) o productores (vendedores), estableciendo precios mínimos o
máximos. Estas intervenciones generan que el mercado pierda la capacidad de asignar
eficientemente los recursos, ocasionando sobreproducción o escasez
escasez..

Sobreproducc
Sobre exceso de oferta 
producc ión o exceso Escasez o exceso de demanda 
Se puede presentar debido al establecimiento de Se puede presentar debido al
un precio oficial mayor al de equilibrio establecimiento de un precio oficial menor
conocido como precio mínimo. al de equilibrio conocido como precio
Por ejemplo, un precio oficial mínimo de 12, máximo.
ocasionará que: Qo= 56 y Qd=40, presentán- Por ejemplo, un precio oficial máximo de 8,
dose una sobreproduccion de 16 unidades. ocasionará que la Qo= 44 y Qd=60

5.1.2 CONSECUENCIAS DEL CONTROL DE PRECIOS 


OFERENTE  DEMANDANTE 
CEPRE-UNI HUMANIDADES 137

CICLO INTENSIVO ESCOLAR NACIONAL Material de Estudio Nº 1 

No tiene incentivo a produ cir mmás


ás ni Incurre en mayores costo s para obtene
obtenerr
mejorar la calidad: Al caer el precio, el bien:
bien : Pierde tiempo en hacer largas
disminuye sus ingresos, hasta el colas, además de la pérdida de horas
punto de no poder cubrir sus costos, laborales. Así mismo, pagará mayores
Precio   desincentivando la producción. Del
Precio precios en el mercado negro, para
Máximo  mismo modo, no requiere mejorar la
Máximo  satisfacer sus necesidades. Se encuentra
calidad del producto, ya que toda la a merced del productor u oferente:
oferente: Este
producción se venderá. puede “discriminar”  si vender o no en
Tie ne incentivo a especular: Ofrecerá
Tiene función a afinidad, grupo social o étnico,
los productos en el mercado negro. etc.
Debe
De be incurri r en may
mayores
ores cost os: Tiene incentivos para conseguir
Para poder vender su producto, produ ctos en mercados ilegaleilegaless :
Precio   deberá emplear mayor tiempo y
Precio Debido a que los encontrará más
Mínimo   recursos.
Mínimo baratos, por ejemplo, productos de
Disminuye sus ingresos:
ingresos : Debido a contrabando.
la caída en la cantidad demandada.

EVALUACIÓN 03
1. Indicar V o F y marcar la respuesta correcta. D) disminuye la cantidad demanda y, en
( ) Los consumido
consumidores
res bajan el pprecio
recio y por consecuencia, los precios de este producto

(ende
) El aumenta la cantidad
comportami
comportamiento demandada.
ento del produ
productor
ctor es tal que disminuyen.
E) aumenta la oferta y, en consecuencia, los
cuando el precio sube la oferta aumenta, precios de este producto se incrementan.
cuando el precio baja la oferta disminuye
disminuye..
( ) La curva de la oferta tiene pendi
pendiente
ente 3. La demanda de Colchones es P=1200-Q,
positiva, ya que a un precio mayor los mientras que la oferta es P=Q. El Estado impone
productores están dispuestos a ofrecer más de el control de precios y fija el precio ______ en P=
este bien. S/. 300. Suponga que la demanda crece en el
( ) Los movimiento
movimientoss a lo largo de la curva de mercado a P=1400- Q, todo esto generara una
oferta ocurrirán si existe un cambio en el precio A) mínimo - sobreproducción de 100 colchones.
del bien. B) mínimo - sobreproducción de 200 colchones.
A) FFVV B) FFFV C) FVV FVVV V D) VVFF E) VFV
VFVFF C) máximo - sobreprodu
sobreproducción
cción de 600 colchones.
D) máximo - escasez de 200 colchones.
2. La tradición de consumir pescado en Semana E) mínimo - escasez de 100 colchones.
Santa, algunas especies registraron aumentos
4. Si un consumidor considera que determinados
en un 87%pesquero
Terminal mayor quedelosVilla
de un mes atrás.
María En el
del Triunfo bienes tienes muchos sustitutos, además
(VMT), el precio del bonito se incrementó en consideran que si el precio aumenta en un 2%,
92%, al pasar de S/ 3 por kilogramo el 15 de los ingresos de los empresarios
marzo, a S/ 5.8 por kilogramo el último lunes, de A) aumentarán. B) disminuirán.
acuerdo con las cifras que reporta Datero C) permanecen constantes.
Pesquero del Ministerio de la Producción D) alcanzan un máximo. E) alcanza un mínimo.
(Produce). 5. Si la oferta y la demanda tienen pendiente
Del texto se infiere que positiva pero la pendiente de la demanda es
A) aumenta la demanda y, en consecuencia, los menor a la oferta, entonces, existirá _________.
precios de este producto se incrementa. A) escases B) sobreproducci
sobreproducción
ón C) equilibrio
B) disminuye la demanda y, en consecuencia, D) desequilibrio E) exceso
los precios de este producto disminuyen.
C) aumenta la cantidad demanda y, en
consecuencia, los precios de este producto se
incrementan.
CEPRE-UNI HUMANIDADES 138

CICLO INTENSIVO ESCOLAR NACIONAL Material de Estudio Nº 1 

TEMA IV. MERCADO. COMPETENCIA PERFECTA, IMPERFECTA Y MONOPOLÍSTICA. 


FUNCIÓN DE PRODUCCIÓN. PRODUCTO MEDIO Y MARGINAL. COSTOS  

1. MERCADO 
Es el lugar o la condición donde se realizan los
intercambios entre bienes y servicios. Estos agentes
económicos podrán llegar a un acuerdo a través del
establecimiento del precio del producto.

2. MODELOS DE MERCADO 
Las distintas formas que puede adoptar la competencia describe el estado de un mercado
con respecto a los ofertantes y los demandantes. Se diferencian por el número y tamaño de
los productores y consumidores en el mercado, el tipo de bienes y servicios que se
comercian, el grado de transparencia en la información. También son relevantes las barreras
a la entrada de nuevos competidores.

3. PODER DE MERCADO  
Si una empresa puede influir en el precio de mercado del bien que vende, se dice que
tiene poder de mercado.
 Agent
 Ag ente
e Econ
Ec onóm ic o  
ómic Racionalidad  
Consumidores Familias   Maximizar su utilidad Minimizar Gasto
Producto res (E
(Empresas)
mpresas)   Maximizar su beneficio Minimizar Costos
4. PRINCIPALES MODELOS DE MERCADOS 
4.1. COMPETENCIA PERFECTA  
La competencia perfecta es la situación de un mercado donde existen muchos (ofertantes y
demandantes) estos carecen de poder para manipular el precio, y se da una maximización
del bienestar. Esto resulta en una situación ideal de los mercados de bienes y servicios,
donde la interacción de la oferta y demanda determina el precio.
Características 
Coexisten muchos ofertantes (vendedores) y demandantes (compradores) del mercado, que
están dispuestos a vender o comprar un determinado producto. (estructura atomizada).
atomizada).
El precio lo determina el mercado. 
mercado. 
Los productos que se ofrecen en este tipo de mercado son homogéneos o iguales.  
Los vendedores y compradores no tienen control distinguib le sobre el precio de venta. 
distinguible
simétrica). 
Los compradores y vendedores están bien informados (información simétrica). 
Los vendedores y compradores pueden vender o comprar libremente entre ellos; por tanto, tienen
libertad de movimiento (de entrada y salida).
Los vendedores y compradores son precios aceptantes. 

4.2. COMPETENCIA IMPERFECTA  


La competencia imperfecta es la situación de falla de mercado en la que, a diferencia de la
situación de competencia perfecta, un solo agente de los que funcionan en el mercado o unos
pocos pueden influir en diferentes grados sobre el precio del producto en el mercado.

  Los vendedores  pueden control


Características: controlar
ar en alguna medida el precio de su producto.
CEPRE-UNI HUMANIDADES 139

CICLO INTENSIVO ESCOLAR NACIONAL Material de Estudio Nº 1 

  Existe diferenciaci
diferenciación
ón del produ
producto.
cto. Es decir, que cada eempresa
mpresa ofrece
ofrece un producto que es al
menos algo diferente al de otras. Por ello, las características (diseño, usos, servicios, etc.).
  Existe información incompleta en el mercado
mercado.. Por tanto, los compradores no conocen las
características de todos los productos.
  Existen barreras de en
entrada,
trada, los vended
vendedores
ores y compradore
compradoress no pueden opera
operarr libremente en
ese mercado.

5. COMPETENCIA
COMPETENCIA IMPER FECTA POR EL LADO DE L A OFERTA 
IMPERFECTA
5.1. MONOPOLIO 
Un monopolio es una situación legal o fallo de
mercado, en el cual existe un productor
(monopolista) que posee un gran poder de
mercado y es el único en una industria dada
que posee un producto, bien, recurso o servicio
determinado diferenciado y no tiene sustitutos
cercanos.
proveedor. 
Único proveedor.  único. 
Producto único. 
asimétrica. Barreras para entrar al
Información asimétrica.
mercado.

5.1.1. TIPOS DE MONOPOLIO 


Monopolio Surge porque una única empresa puede ofrecer un bien o un servicio a todo un
Natural mercado con menos costos que dos o más empresas.
Ejemplo: Sedapal, Cálidda.
Monopolio Monopolio que surge ante un mandato de ley o por cuestiones estratégicas en
Legal términos económicos por parte del Estado.
Ejemplo: Telefónica (1994).

Cártel. Grupo de empresas que llegan a un acuerdo de colusión respecto a precios y niveles


Cártel. Grupo
de producción, en la que cada una conserva su autonomía administrativa. La legislación
prohíbe acuerdos expresos.

5.2 OLIGOPOLIO 
Cuando un mercado
un mercado es dominado por un pequeño número de
productores oferentes (los oligopolistas), se dice que existe un
oligopolio. Por eso oligopolio significa justamente “pocos
vendedores ”.
En una situación de oligopolio, el precio es menor que en el
monopolio y, por tanto, también las ganancias son menores
para cada vendedor.
Características: 
precios. 
Pocos vendedores dominan el mercado en cuanto a producción y precios. 
Hay muchos consumid ores desorganizados sin poder de mercado. 
consumidores
Los productos que se ofertan pueden ser diferenciados o estandarizados; por lo tanto
pueden existir sustitutos.
Existedeinterdependencia
través entremillonarias
la guerra de precios, los miembros del oligopolio
campañas que etc.
de publicidad se puede manifestar a
CEPRE-UNI HUMANIDADES 140

CICLO INTENSIVO ESCOLAR NACIONAL Material de Estudio Nº 1 

La información lo concentran los pocos vendedores, por lo que existe asimetría. 

Por ejemplo, la telefonía móvil, bancos, AFP, compañías de seguro, gaseosas, galletas,
entre otros.

La empresa Gloria, que es la única en comprar leche fresca al conjunto de ganaderos de la


región Arequipa, establece un precio bajo que le permite reducir sus costos de producción.
6. POR EL LADO D E LA DEMANDA 
DE
6.1 MONOPSONIO 
Mercado caracterizado por la existencia de un solo comprador enfrentándose a muchos
vendedores o productores desorganizados,
desorganizados, de tal manera que puede influir sobre el precio.

6.2. OLIGOPSONIO 
Es aquella situación en la cual existen pocos compradores que pueden influir sobre el
precio y muchos vendedores desorganiz
desorganizados.
ados.
Un ejemplo pueden ser los fabricantes de automóviles en un país como Japón. Para los
fabricantes de sillas para automóviles sólo existe un número reducido de compradores, que
son las pocas empresas ensambladoras de automóviles japonesas, quienes, por lo tanto,
podrán controlar las cantidades y precios de las sillas para automóviles, puesto que son los
únicos compradores en el país de ese producto.
7. COMPETENCIA
COMPETENCIA IMP
IMPERFEC
ERFECTA
TA POR EL L ADO DE LA OFERTA Y LA DEMANDA  
LADO

7.1. COMPETENCIA MONOPOLISTA

Este modelo tiene rasgos de la competencia perfecta y de monopolio, en este caso cada
empresa produce un bien que los compradores consideran diferente al de los otros
vendedores; sin embargo, como son muchos los vendedores, existe competencia entre ellos.

Características:

El mercado es atomizado. Existen muchos compradores y vendedores desorganizados. 


Existe diferenciación de productos en el mercado. 
mercado. 
En el corto plazo, los ofertantes tienen capacidad de influir en el precio debido a la
diferenciación,, lo que les da cierto poder de mercado.
diferenciación
mercado.  
Existen mínimas barreras para el ingreso a este mercado.
La información es asimétrica. 

8. FUNCION DE PRODUCCIÓN

DEFINICIÓN REPRESENTACIÓN
Es la relación existente entre los factores
f actores o
insumos utilizados en un proceso productivo = Cantidad de producción
(inputs) y la cantidad de producción obtenida = nivel tecnológico dado
(outputs) dado un cierto nivel tecnológico. Esta = Cantidad del factor naturaleza
relación establece la máxima producción que se = Cantidad del factor capital
obtiene con cada combinación
combinación de factores. = Cantidad del
del factor tra
trabajo
bajo = capital
humano
CEPRE-UNI HUMANIDADES 141

CICLO INTENSIVO ESCOLAR NACIONAL Material de Estudio Nº 1 

Cuando se analiza la producción se debe tomar en cuenta el corto y largo plazo.

8.
8.1.
1. Producc ión ded e corto plaz
pl azo
o
Se refiere al periodo de tiempo en el que no es posible alterar uno o más factores de
producción, dado que al menos uno permanece fijo y los otros variables.

 A.
SonInla
Indi
dicad
cador
ores
es d
dee efici
productividadefimedia
cienc nos indican
enc yia:marginal. el análisis
En el rendimiento de los
de corto factores
plazo o productividad.
se calcula la
productividad del factor variable. El factor que evaluaremos es el trabajo.

Producto medio: Producto marginal:


Es el rendimiento promedio del factor
trabajo Es la división entre el producto total
y la cantidad del factor trabajo( factor
variable):
∆
= ∆  

=   
Producción adicional del factor trabajo. Es
la división entre la variación del producto
total y la variación del factor trabajo:
Ejemplo:
Q= 200 T= 10
 Ejemplo:
Q= 5 T= 1

B. Etapas
Etapas de la prod ucci
ucción
ón en el cort
cortoo plazo

Primera Segunda Tercera


El producto marginal es El producto medio es mayor al
mayor al producto medio producto marginal hasta el punto El nivel de producción
hasta el punto en que que este último se iguala a cero, comienza a caer, dado que
(B´) 
ambos se igualan (B´)  donde se obtiene el máximo nivel el producto marginal se
C´). 
de producción (C y C´).  negativo. 
vuelve negativo. 

Primera etapa: el
Producto marginal
supera al producto
medio 

Segunda
etapa: el producto
medio supera al
marginal 
CEPRE-UNI HUMANIDADES 142

CICLO INTENSIVO ESCOLAR NACIONAL Material de Estudio Nº 1 

Tercera etapa: el
producto marginal es
negativo 

B. La ley de rendim
rendimientos
ientos marg
marginales
inales decrecientes:
Establece que cuando aumenta el uso de un factor (y los demás se m mantienen
antienen constant
constantes),
es),
se alcanza un punto en el que son cada vez menos los incrementos de la producción. Hasta
llegar a ser cero y a partir de ahí negativo. Se aplica al corto plazo.
8.
8.2.
2. Produc
Producció
ción
n en el largo pl
plaz
azo:
o:
Se refiere al periodo de tiempo necesario para que todos los
factores de producción sean variables (variación a escala).

El largo plazo no tiene un periodo de tiempo determinado,


puede ser días, meses o años. Las empresas transitan entre el
corto y el largo plazo conforme combinan la cantidad de
factores productivos
productivos en el proceso productivo. En el largo plazo se analizan tres casos.
Rendimi entos a escala crecientes
Rendimi
La variación porcentual de la producción es mayor
a la variación porcentual de los factores.
f actores.
Rendimi
Re
La ndimi entos
variación a escaladeconstantes
porcentual la producción es igual a
la variación porcentual de los factores.
Rendimi
Re ndimi entos a escala decreciente
decrecientess
La variación porcentual de la producción es menor
a la variación porcentual de los factores.
f actores.

9. COSTOS DE PRODUCCIÓN

COSTO FIJO Costo que no varía con respecto a la producción


COSTO
COSTO VARIABL E Costo que varía con respecto al producción
COSTO TOTAL
TOTAL = COSTO FIJOS + COSTO VARIABLE
CEPRE-UNI HUMANIDADES 143

CICLO INTENSIVO ESCOLAR NACIONAL Material de Estudio Nº 1 

COSTO MEDIO:  Es el costo promedio por MARGINAL :  Es la variación del
COSTO MARGINAL:
COSTO
unidad de producto. costo total con respecto a la variación de la
CME=CT producción
Q CMA= CT △△
 Q

Los costos de la empresa textil “GAMARRIN” en el corto plazo


Nivel de Costo fijo Costo Costo Costo Costo Medio
producción (CF) variable total marginal (CMe)
(Q) (CV) (CT) (CMg)
0 50 0 50 - -
1 50 50 100 50 100
2 50 78 128 28 64
3 50 98 148 20 49.3
4 50 112 162 14 40.5
5 50 130 180 18 36
6 50 150 200 20 33.3
7 50 175 225 25 32.1
8 50 204 254 29 31.8
9 50 242 292 38 32.4
10 50 300 350 58 35
11 50 385 435 85 39.5

EVALUACIÓN 04
1. Julio, quien es un estudiante muy aplicado, mencionada y otro con una marca local; por
observó que, en su escuela, solamente existe ejemplo “Gamarrita”, ambos hechos con las
una cafetería, además de estar prohibida la mismas máquinas, materia prima, personal,
venta en los exteriores de esta institución. Así entre otros, es decir, dos polos “iguales”. Por lo
mismo, pudo notar que los precios de los tanto, podemos afirmar que ambos son

productos,
mayores quepor
por suejemplo, la donde
casa, lugar gaseosa, son
hay varias productos.
A) homogéneos.
homogéneos. B) diferenciad
diferenciados.
os.
tiendas que venden este bien y otros C) complementa
complementarios.
rios. D) únicos.
alternativos. Dado que julio ya estudio las E) sustitutos perfectos.
diferentes estructuras de competencia perfecta,
puede considerar que la cafetería se aproxima a 3. José y Karla debaten sobre la conveniencia o
un (una) no de los monopolios. José argumenta que
A) monopolio. B) monopsonio. C) oligopolio. estos, en algunos casos, son necesarios, ya que
D) oligopsoni
oligopsonio.
o. E) competencia
competencia monopolístic
monopolística.
a. permiten disminuir los costos unitarios del
2. Según declaraciones Andrés Oppenheimer a producto, sin embargo, Karla señala que son
RPP noticias: "La marca de ropa Ralph Lauren malos ya que determinan tanto el precio como la
se produce en el Perú. Un polo de esa marca cantidad a producir, siendo los precios altos y las
cuesta US$ 85 en Miami (Estados Unidos). Por cantidades bajas. Con esta información, escoja
curiosidad, hice un par de llamadas y logré la alternativa correcta respecto a quién tiene la
ubicar al fabricante en Perú. Le pregunté: razón.

¿Cuánto
solo el 8%de esa8.5).”
(US$ cifra Esto
recibía? Me dijo
significa que que tan
en Perú A) José, legal.
monopolio ya que Karla está hablando de
se pueden producir dos polos, uno con la marca
CEPRE-UNI HUMANIDADES 144

CICLO INTENSIVO ESCOLAR NACIONAL Material de Estudio Nº 1 

B) Karla, ya que José no está considerando el I. El PmgL será cero.


equilibrio del mercado. II. Se cumple que PmgL es mayor que PmeL.
C) Ambos, ya que los argumentos en ambos III. La empresa se encuentra en la etapa I de la
casos son verdaderos. producción.
D) José, ya que, en los monopolios naturales A Solo I B) I y II C) II y III D) I y III
existe economía de escala.
E) Karla, ya que efectivamente los precios son 5. Teresa desea saber hasta que momento debe
elevados y las cantidades bajas. de seguir contratando personal para su
empresa. Al consultarle a su amigo, que es
4. Marcos ha constituido una empresa, la cual ingeniero económico, recibe las siguientes
cuenta con un taller dotado de una gran cantidad indicaciones. (Siendo Q: Producción total,
de maquinarias y equipos, sin embargo, solo PmgL: El producto marginal del trabajo, PmeL:
cuenta por el momento con muy pocos Producto medio del trabajo, Cmg: El costo
trabajadores. Dado esto ¿cuáles de las marginal del trabajo y Cme: Producto medio del
siguientes proposiciones sería verdaderas en el trabajo).
corto plazo? (Siendo PmgL: El producto marginal A) Cmg = 0, PmgL máximo B) Cme = Cmg,
Cmg, PmgL= 0
del trabajo y PmeL: Producto medio del trabajo). C) PmgL = 0, Q máximo D) PmgL = PmeL 
E) PmeL = 0, Q máximo

TEMA V. DINERO. TEORÍA CUANTITATIVA DEL DINERO.INFLACIÓN. SISTEMA


DINERO.INFLACIÓN.
O: CLASIFICACIÓN. ENTIDADES REGULADORAS. 
FINANCIERO:
FINANCIER

1. DINERO 
El dinero es todo medio de pago que la sociedad utiliza para intercambiar bienes y servicios,
además cuenta con respaldo legal, que se traduce al final en la aceptación de los billetes y monedas
que el Banco Central pone en circulación; es decir, es de aceptación forzosa. 

1.1. Características  
 Ac eptabi
 Acept abilili dad El dinero tiene curso legal, son de aceptación forzosa.
Estabilidad  
Estabilidad Debe mantener su poder adquisitivo a través del tiempo.
Fácil transporte 
transporte 
Durabilidad  
Durabilidad El dinero,
Debe ser en sus diversas
capaz formas,
de circular debe
en la ser fácil de
economía sin ttrasladar.
rasladar.
deteriorarse por un tiempo
Homogeneidad  
Homogeneidad Que reúna características uniformes de peso, tamaño, color, entre otras.
El dinero debe poder subdividirse en pequeñas partes (múltiplos y submúltiplos)
Divisibilidad  
Divisibilidad
sin que pierda su valor.

1.2. Funciones  

Medio d e 
Medio Las personas utilizan y aceptan dinero para la compra y venta de bienes,
cambio  
cambio servicios
servicios y factores.
cuenta Permite cuantifica
Unidad de cuenta  cuantificarr la riqueza o el valor de las mercancías en una sola
Las personas guardan dinero porque mantiene su valor en el tiempo. Facilita el
Reserva de valor  
ahorro.

1.3. Valores del dinero  


CEPRE-UNI HUMANIDADES 145

CICLO INTENSIVO ESCOLAR NACIONAL Material de Estudio Nº 1 

Valor legal o Es aquel valor del dinero que el Estado le asigna, y que aparece
nominal figurando en el billete o moneda.
Valor   Es el valor de la composición material del dinero; es decir, es el
Intrínseco  
Intrínseco
Valor real valor de los materiales utilizados para la elaboración de los billetes y
Es el valor que viene representado por el poder de compra que el dinero posee, es
Valor   decir esta expresado en bienes y/o servicios. El valor extrínseco del dinero se ve
Extrínseco   seriamente afectado cuando existen en la economía continuos procesos
Extrínseco

1.4. Ley de Gresh am  


1.4.
Fue planteado por Thomas Gresham (Gran Bretaña, 1519 - 1579). La ley señala lo siguiente:
"Cuando en una economía circulan dos monedas de igual valor legal, pero de distinto valor
real; entonces la moneda mala desplaza de la circulación a la buena". 
buena". 

1.
1.5.
5. T
Teoría
eoría cuantit ativa del di nero  
cuantitativa
Planteada por Irving Fisher, esta ecuación relaciona al sector monetario de la economía con
el sector real. También se le denomina ecuación de cambio o ecuación de Fisher y consiste
en lo siguiente:

MV = PQ 
Donde monetaria. 
M: Masa monetaria. 
V: Velocidad de circulación del dinero. 
dinero. 
promedio). 
P: Nivel general de precios en una economía (nivel promedio). 
Q: Producción total de bienes y servicios en una economía. 
  El sector monetario está dado por M.V y el sector real está representado por P. P.Q
Q
  En el ccorto
orto plaz
plazo,
o, V y Q permanecen cons
constantes.
tantes. La producción es de pleno emple
empleo.
o.
  Por lo tant
tantoo se puede llegar a la siguient
siguientee conclusión:

El Nivel general de precios (P) está en relación directa a la Masa monetaria (M).
El poder de compra del dinero está en relación inversa a Masa monetaria. 

2. INFLACIÓN

Fenómeno
nivel generaleconómico
de preciosque consiste
de una en el
economí a. incremento continuo, persistente y sostenido del
economía.

Lo con trari
trarioo a la inflación s e conoce como DEFLACIÓ
DEFLACIÓN
N

2.
2.1.
1. T
Tipos
ipos de infl ación  
inflación
De acuerdo a la tasa de inflación anual
anual que posee cada ec
economía,
onomía, se puede clasificar al
fenómeno inflacionario del siguiente modo:

Inflación Su tasa de inflación anual es de un solo dígito; es decir, es mayor o igual


Moderada a 0% y menor a 10 %.
Inflación Cuando la inflación anual en una economía es mayor o igual a 10 % y
Galopante menor a 1000 %.
Se produce cuando el nivel general de precios en una economía es
Hiperinflación mayor o igual a 1000 % anual.
CEPRE-UNI HUMANIDADES 146

CICLO INTENSIVO ESCOLAR NACIONAL Material de Estudio Nº 1 

2.
2.2.
2. C
Causas
ausas de la i nflació
nflaciónn 

a) Explicación Keynesiana 
Inflación por   Cuando la demanda excede a la oferta disponible de bienes y servicios,
la demanda 
demanda  el nivel general de precios se incrementa.
En este caso el aumento del nivel general de precios tiene su origen en el
Inflación
Costos
Costos    por   incremento de los precios
intereses, tributos, costos dede producción
los insumos,tales como: los entre
combustibles, salarios, alquileres,
otros.
Se debe a diversas circunstancias que afectan a la estructura económica
Inflación  
Inflación  de un país, por ejemplo
ejemplo,, lla
a existencia de mercados imperfectos o los
Estructural  
Estructural conflictos entre agentes económicos.

b) Explicación Monetarista
Masa Monetaria 
Por inc remento de la Masa
Se produce cuando las autoridades monetarias de un país recurren a la emisión de papel
moneda inconvertible. La inflación se refleja entonces en un aumento general de los
precios, en la depreciación monetaria, el éxodo de los capitales y la destrucción del ahorro.

2.
2.3.
3. C
Consecuenci
onsecuencias infl ación  
as de la inflación
Todo
las proceso inflacionario
economías, tales como: genera un conjunto de efectos negativos en el desarrollo de

Disminución del salario real: Lo cual reduce la capacidad de compra de bienes y servicios. El dinero
nacional se desvaloriza.
Empobrecimiento de la población.
Sustitución de la moneda nacional: Dolarización
Dolarización,, en otras palabras, aumenta el ahorro en
moneda extranjera (dólares), para poder mantener cierta estabilidad.
Disminución de las inversiones productivas: Debido a que existe incertidumbre por parte de las
empresas respecto al desarrollo de la economía, existen condiciones desfavorables para las
inversiones;; pues resulta difícil planificar los resultados futuros.
inversiones
Existencia de malestar social: Huelgas y protestas.
Encarecimiento del crédito.

3. SISTEMA FINANCIERO
Conjunto de instituciones y mercados, cuya función
básica es la transferencia de fondos de los ahorristas
(agentes ofertantes de fondos) hacia los inversionistas
(agentes demandantes de fondos) a través de dos
alternativas. En primer lugar, los intermediarios
financieros, como un banco. En segundo lugar, a través
de los mercados de capitales, como los mercados de
bonos, acciones, y derivados financieros.

3.
3.1
1 Mercado Financi
Financiero
ero
Surge de la interacción de los agentes que son ofertantes de fondos con los agentes que
son demandantes de fondos.
3.2. CLASIFICACIÓN DEL SISTEMA
SISTEMA FINANCIERO PERUANO
CEPRE-UNI HUMANIDADES 147

CICLO INTENSIVO ESCOLAR NACIONAL Material de Estudio Nº 1 

3.2.1 Mercado indirecto:  el ofertante no interactúa con el demandante de fondos, el que
interactúa entre ambos es un tercero, al que se le denomina intermediario financiero. Dentro
de ella encontramos la siguiente clasificación.

3.2.1.1 Sistema Financiero Bancario  


Comprende todas las instituciones bancarias que operan en un país. Integrado por:
Es el agente financiero del Estado, encargada de las operaciones
bancarias del sector público. Recibe depósitos de ahorros en lugares
donde la banca privada no tiene oficinas.
Banca Su negocio principal consiste en captar dinero del público en depósito
privada  y en utilizar ese dinero, su propio capital y el que obtenga de otras
cuentas de financiación para conceder créditos.

3.2.1.1.1 Clases de operaciones bancarias


a) Activas. Operaciones por medio de las l as cuales las instituciones financieras prestan dinero
y realizan inversiones, asumiendo la calidad de acreedoras frente a sus clientes. Las
diferentes operaciones financieras. Ejemplos: préstamos directos, descuento de letras,
pagarés, préstamos para capital de trabajo, financiamiento para operaciones de exportación
e importación.
Tasa
Ta sa costo
cos to efectivo anual (TC EA)::  A la tasa de interes activa que el banco cobra por
(TCEA)
un crédito, se le suma el cobro de comisisones y gastos extras. Es decir, la TCEA es la
tasa que te permite saber cuál será el costo total que deberás pagar al banco.

b) Pasivas. Operaciones por medio de las cuales las instituciones financieras se constituye
constituyenn
en deudoras, sobre todo respecto a los depósitos que reciben de otros agentes económicos
(clientes). Diferentes tipos de depósitos. Ejemplos: depósitos de ahorros, cuenta corriente,
depósito a plazo, certificado bancario, depósitos CTS.
Tasa de rendimiento efectivo anual (TREA): A la tasa de interes pasiva se desuenta el
cobro de comiciones y otros gastos de mantenimiento que reducen el rendimiento de tus
ahorros. La TREA indica el monto que efectivamente se gana por un depósito en el sistema
financiero.

3.2.2. Mercado directo:  es aquella parte del mercado financiero en el cual los ofertantes de
fondos (agentes
(agentes superavitarios)
deficitarios); tienen
n secontacto
esta interacción
interacció directodel
realiza a través conmercado
los demandantes
de valores. de fondos
MERCADO DE VALORES
Mercado
Me rcado primario
pri mario Me
Mercado
rcado secundario
Las empresas realizan la colocación de la primera En el mercado secundario se transan
emisión de valores con el objeto de obtener operaciones de valores ya emitidos en
financiamiento para la ejecución de sus proyectos. primera colocación y cuyos precios están en
función a la oferta y demanda.

Título que representa derechos de propiedad en una determinada sociedad y, por lo


 Acci
 Ac ci ón
tanto, derecho a participar en las utilidades de la misma. Genera renta variable.
Bonos Título emitido por una institución que da al titular el derecho a una renta monetaria fija
estipuladas.  
y a una suma fija declarada en la fecha o fechas estipuladas.

3.3. INSTITUCIONES SUPERVISORAS DEL SISTEMA FINANCIERO PERUANO


CEPRE-UNI HUMANIDADES 148

CICLO INTENSIVO ESCOLAR NACIONAL Material de Estudio Nº 1 

Son entes representativos de la estructura oficial, cuya misión fundamental es proteger los
intereses del público, regulan la participación de las personas naturales y jurídicas que concurren
al mercado financiero, formulan y ejecutan políticas económicas. 

3.3.1
3.3.1 Superi
Superint
ntendenci
endenciaa de Banca y Seguro
Seguross y AF
AFPs
Ps (SBS)
Entidad encargada de regular y supervisar el sistema financiero, de
seguros yesAFPs,
primordial así el
preservar como detectar
interés lavado de activos.
de los depositantes, Su objetivo
objetivo 
de los asegurados y 
de los afiliados al SPP, controlando en representación del Estado, al
sistema financiero que operan con fondos del público. 

3.3.2
3.3.2 Superi
Superint
ntendenci
endenciaa de Mercado
Mercado de Valor
Valores
es (SMV)
(SMV)
Organismo especializado adscrito al Ministerio de Economía y
Finanzas, cuyas funciones son las de supervisar e inspeccionar los
mercados de valores, velar por la transparencia de los mercados, la
correcta formación de precios y la protección de los inversores.

EVALUACIÓN 05

1. Con respecto
verdadero al sistema
(V) o falso monetario,
(F) según señale
corresponda A)
C) divisibilidad
divisibil
duración.idad. B)
D) estabilid
estabilidad.
durabilidad.
durabilidad.
ad.
I. La ininflación
flación afecta la durabilidad del dinero. E) homogeneidad.
II. La estabilidad se logra garantizando la
homogeneidad del dinero. 4. La empresa JULITO desea financiar sus
III. El ahorro se relaciona con la función medio actividades productivas. Acude a un banco
de pago. reconocido a solicitar un préstamo. El crédito se
IV. La homogeneidad se relaciona con el logra concretar después de la respectiva
material de fabricación del dinero. evaluación. La entidad pública que supervisa a
A) FVFF B) FFFF C) FFVF estas instituciones financieras es la
D) VFVF E) FFVV A) BCR B) SBS C) ONP D) SMV E) MEF

2. La señora Daniela Barrientos el fin de semana 5. Sobre el sistema financiero y sus


realiza sus compras semanales. En el mercado características, relacione según corresponda y
noto que necesita el 25% más de dinero para marque la respuesta correcta
realizar sus compras habituales. Por lo señalado I. Supervisa a los bancos.
se está presentando una inflación II. Reciben depósitos.
A) moderada. B) controlada. C) devaluada. III. Se venden acciones y bonos.
D) galopante. E) hiperinflación.
hiperinflación. IV. Supervisa el mercado de valores.
a. Bancos
3. El comerciante Sandro Huamán, dedicado a b. SBS
la venta de golosinas al por mayor y menor, en c. Mercado de valores
su caja registradora tiene billetes y monedas de d. SMV
distinta denominación, incluyendo los céntimos. A) Ib,IIa,IIIc,IVd B) Ic,IIa,IIId,IVb
Lo que busca es realizar fácilmente sus C) Id,IIa,IIIb,Iva D) Ib,IIa,IIId,IVc
transacciones. La característica del dinero que E) Ic,IIa,IIIb,IV
se está cumpliendo es
 
CEPRE-UNI HUMANIDADES 149

CICLO INTENSIVO ESCOLAR NACIONAL Material de Estudio Nº 1 

Filosofía 
TEMA I. FILOSOFÍA ANTIGUA. PERIODO COSMOLÓGICO. PERIODO
 ANTROPOLÓGICO.
 ANTROPOL ÓGICO.

1. LA FILOSOFÍA: Conjunto de saberes que estudian los conceptos más generales como
vida, origen y justicia, a la vez, problematiza las hipótesis más generales como la que afirma
que este mundo y que es cognoscible. 

 A. L AS PREGUNTAS FIL OSÓFICAS


Para entender lo que es el quehacer de la filosofía compararemos dos tipos de preguntas.
Hay preguntas que constituyen problemas solubles dentro
dentro de la ciencia. Por ejemplo:
1. ¿Qué diferencia hay entre los electrones, los protones y los neutrones?
2. ¿Pueden desaparecer los elementos que interviene
intervienenn en una reacción química?
3. ¿Existieron formas de vida anteriores a la aparición del ser humano en la Tierra?  

Sin embargo, también existen otro tipo de preguntas que son muy distintas. Por ejemplo:
1. ¿Tiene sentido toda la vida en general?
2. ¿Qué define al ser humano específicamente?
3. ¿Es malo matar a otro ser vivo por algún motivo?
4. ¿Cuándo hablas de “Dios” a qué te refieres?
5. ¿Cómo debemos vivir en esta sociedad? 
Las cinco anteriores son preguntas filosóficas y, en comparación con las tres primeras, no
tienen ni una respuesta definitiva ni un método único de resolución. Por ello, este tipo de
preguntas genera polémica y origina intensos debates que pretenden intentar responderlas
de modo adecuado y conveniente.

B. LA ACTITUD FILOSÓFICA
La explicación psicológica de la filosofía la considera como una actitud. La actitud es la
disposición subjetiva que lleva al hombre a reaccionar frente al mundo de distintas maneras.
Existen diversas actitudes: pragmática, moral, estética, religiosa, científica y filosófica.
Particularmente, la actitud filosófica tiene las siguientes características.

Caracterí
Características
sticas Explica
de lla
a actitud ffilosóf
la realidadilosófica  
icaconjunto.
en su Además, el campo de sus reflexiones abarca
Totalizadora
aspectos de máxima generalidad.
Tiene por objetivo indagar sobre los principios y fundamentos de la realidad, es
Radical
decir, sobre la raíz de los problemas más fundamentales.
Plantea argumentos lógicamente constituidos. Sus teorías no admiten criterios
Racional
de autoridad o creencias místicas inverosímiles.
Cuestiona las teorías que explican la realidad. Busca refutar las razones que
Crítica  
Crítica
sustentan una tesis.
Encuentra contradicciones
contradicciones donde se pensaba
pensaba que solo habí
habíaa certezas. Así, se
Problemática
aleja del sentido común y se pregunta sobre lo considerado obvio.
En base a lo anterior, podemos decir que la filosofía es una actividad intelectual que es
reflexiva pues se vale de la argumentación elaborando y evaluando razonamientos; además,
es crítica ya que propicia la incertidumbre que libera de dogmatismos y prejuicios.

2. FILOSOFÍA ANTIGUA
CEPRE-UNI HUMANIDADES 150

CICLO INTENSIVO ESCOLAR NACIONAL Material de Estudio Nº 1 

La filosofía surge en Grecia mediante un paso del mito al logos, es decir, la filosofía inició al
criticar los mitos griegos acerca del origen de todo lo existente. En vez de aceptar la
explicación religiosa, los primeros
pr imeros filósofos sostuvieron otras explicaciones más coherentes,
racionales, naturalistas y lógicas.
I. PERIODO COSMOLÓGICO
Tema: arje (principio)
LOS PRESOCRÁTICOS
Los primeros pensadores postularon una primera sustancia como aquella que permitía
explicar todo lo existente.
 A) MONISTAS
MONISTA S
-Tales--------------Agua -Anaximandro----Apeirón
-Anaxímenes-----Aire -Pitágoras---------Número
-Heráclito----------Fuego -Parménides------Ser
Después de comparar las propuestas de Heráclito y Parménides, los filósofos consideraron varios
principios que explican la realidad. Pero, además, estos pensadores añadieron la explicación del
proceso del movimiento del todo.
B) PLURALISTAS
-Empédocles-----4 elementos -Anaxágoras------Semillas
-Demócrito--------Átomos
II. PERIODO ANTROPOLÓGICO
Tema: areté (virtud)
 A) SOFISTAS
SOFISTA S
Estos eran maestros ambulantes que cobraban por enseñar sus conocimientos. Enseñaban
retórica, oratoria, etc. Esto, según ellos, servía para lograr la virtud política que consiste en
poder convencer en cuanto a los asuntos públicos.
-PROTÁGORAS
Afirmó que el hombre es la l a medida de todas las cosas. Sostuvo que sobre una misma
cuestión existen dos puntos de vista igualmente válidos.
B) SÓCRATE
SÓCRATES S
Mientras que los sofistas afirmaban que la verdad dependía de lo que a cada quien le
pareciera, Sócrates afirmaba que el conocimiento que ofrecen los sofistas es solo un saber
aparente:
lugar; y si el
noconocimiento verdadero
es así, es sólo lo esSócrates
una opinión. de lo universal y es
describió válidocomo
el alma en cualquier tiempo
aquello en virtudy
de lo cual se nos califica de sabios o de locos, buenos o malos, una combinación de
inteligencia y carácter. Por ello, hay una verdad absoluta que está en el alma de cada ser
humano y que tenemos que dejar salir.

PRÁCTICA

1. Según Kant, las características del objeto son mostrando el rasgo de ___________ propio de
captadas por los sentidos, pero todo ello es la actitud filosófica, ya que buscan explicar el
determinado por la razón. De acuerdo a las principio de todo lo existente.
disciplinas
disciplinas filosóficas, ¿cuál de todas estas A) radicalidad B) problematic
problematicidad
idad
apreciamos en el argumento de Kant? C) universali
universalidad
dad D) religiosida
religiosidadd
A) Axiología B) Ontología C) Ética E) racionalidad
D) Estética E) Gnoseología 3. Los pitagóricos suponían que los números
2. Cuando los filósofos presocráticos plantean
constituían la esencia de todo lo existente. De
que el origen de todo es el arjé
arjé,, estarían

CEPRE-UNI HUMANIDADES 151

CICLO INTENSIVO ESCOLAR NACIONAL Material de Estudio Nº 1 

acuerdo con esta idea, se puede inferir que, para B) conoce de todo un poco
los pitagóricos, C) conoce bastante de todo
A) solo existen números enteros racionales. D) sabe que no sabe
B) el universo es ordenado y armonioso. E) sabe que sabe
C) la filosofía se reduce a las matemáticas. 5. Señale aquello que sea compatible con los
D) toda la realidad resulta incognoscible. sofistas:
E) el fenómeno del movimiento resulta una A) El relativismo sobre la moral.
ilusión. B) El estudio de la naturaleza.
C) Este no es el único mundo.
4. La sabiduría, según Sócrates, se da cuando el D) La mayéutica y la ironía.
ciudadano E) La búsqueda de la verdad.
A) no sabe que sabe

TEMA II. PERIODO ONTOLÓGICO. PERIODO HELENÍSTICO-ROMANO. FILOSOFIA


RENACENTISTA.
RENACENTISTA. FILOSOFÍA MODERNA.

I. PERIODO ONTOLÓGICO
Tema: ontos
ontos (ser)
 (ser)
-PLATÓN
La metafísica de Platón divide al mundo en dos distintos aspectos; el mundo inteligible (el
mundo del auténtico ser) —, y el mundo que vemos alrededor nuestro en forma perceptiva —
el mundo de la mera apariencia—. El mundo perceptible consiste en una copia de las formas
inteligibles o Ideas. Estas formas (o ideas) no cambian y sólo son comprensibles a través del
intelecto o entendimiento  – es decir, la capacidad de pensar las cosas abstrayéndolas de
cómo se nos dan a los sentidos.
-ARISTÓTELES
Para Aristóteles, la filosofía primera o metafísica es la ciencia que estudia la causa y principio
primero de todas las cosas. Así, considera que el ser es la causa y el principio primero de
todas las cosas. El ser se dice de muchas maneras y la principal manera de decir el ser es
como sustancia.

 A. Teorí
Teo ría
a hi
hilem
lemór
órfifi ca
A diferencia de Platón, Aristóteles considera que la idea o forma de algo individual no se
encuentra fuera de la realidad concreta, sino que existe unida a la materia. Esta tesis
aristotélica recibe el nombre de hilemorfismo; es decir, las cosas reales se encuentran
compuestas por una materia (hyle) y una forma (morphé).
B. Teoría
Teoría dedell movimi
mo vimiento
ento
Podemos hablar también del ser como acto y potencia. Estas nociones le permiten, a su vez,
explicar la generación y el devenir, es decir, el movimiento en el mundo sensible.
Específicamente,
Específicament e, el movimiento consiste en la actualización de las potencias de algo, por ello,
el movimiento es definido como el paso de la potencia al acto, es decir, del poder ser a ser.
C. Teoría
Teoría de las cuatro causas
Solo se conoce algo cuando se conoce su causa, ya que en ella yace la esencia, en otras
palabras, lo que es. La causa de algo debe exponer lo que es la cosa, de qué está hecha,
quién o qué la hizo y para qué la hizo. Aristóteles llamó a estas categorías de diferentes
formas: causa material, causa formal, causa eficiente y causa final.

II. PERIODO HELENÍSTICO-ROMANO


Tema: ataraxia
ataraxia (tranquil
 (tranquilidad
idad del alma)
CEPRE-UNI HUMANIDADES 152

CICLO INTENSIVO ESCOLAR NACIONAL Material de Estudio Nº 1 

-CINISMO
-CINIS MO (Diógenes de Sínop e)
El hombre solo es feliz cuando vive de acuerdo con su naturaleza. Esto requiere que el
hombre asuma los deseos de sus sentidos y su intelecto. Solo puede hacerlo si es libre de
hacer y decir sin someterse a la presión de la sociedad.
-ESCEP
-ES CEPTIC
TICISM
ISMOO (Pirrón
(Pirr ón de Elis)
El is)
Esta tendencia también buscaba la felicidad y afirmó que el hombre es infeliz porque se
perturba y mortifica tratando de saber cosas que no están al alcance de su conocimiento, ni
al alcance de sus posi
posibilidades.
bilidades. Ya que los sent
sentidos
idos o la razón no pueden aafirmar
firmar ni la verdad
ni la falsedad de las cosas. De este modo, la actitud del hombre debe ser la abstención de
opinar sobre la naturaleza y de juzgar los hechos de los hombres. Si se evita emitir juicios se
logrará la tranquilidad del alma (la ataraxia) y con ello la felicidad.
-ESTOICISMO (Zenón de Citio)
Para los filósofos estoicos, la vida del hombre es el resultado de la oposición entre la
necesidad y la libertad. Al final la necesidad del cuerpo y del intelecto se impone; por lo tanto,
el hombre debe resignarse y adoptar una actitud de aceptación de su destino. Es por ello que
la actitud frente a la vida debe ser resignada. La vida virtuosa se logra cuando se es conducida
por la razón.
-HEDON
-HE DONISM
ISMO O (Epicuro
(Epic uro de Samos)
Samos )
Para el epicureísmo, la felicidad consiste en la búsqueda de placer (hedonista) pero no de
cualquier
cuerpo tipo deenplacer,
consiste sinoy el
no sufrir en administrado racionalmente.
el caso del alma Así, el placer
en no ser perturbada. Deen el caso
este modo,,del
modo el
placer es entendido en la moral epicúrea no de manera positiva, sino negativa, como ausencia
de dolor.

III. FILOSOFIA RENACENTISTA


El renacimiento fue un movimiento cultural de origen italiano que surge en los siglos XV y XVI
en Europa. El renacimiento marca una ruptura con el pensamiento del mundo medieval e
inaugura un proceso de cambio en la civilización occidental. La reflexión se centra en el
hombre y en la naturaleza. La nueva imagen del hombre se basa en el reconocimiento de su
propio ser como hombre, acentuándose
acentuándose la separación entre él y Dios. Así, la divinidad pierde
su papel central y la iglesia empieza a perder su poder político.

 A. Caract
Car acterís
erís ti cas
  Un individualismo teórico y práctico.
  Una exaltación de la vida
vida mundana.
  Una crítica a la iglesia católica.
católica.
  Uso de la razón y del método
método experimental.
  Humanismo filosófico y artístico.
artístico.
  Surge una nueva clase social: la burguesía.
burguesía.
C. Importancia
Import ancia de la fil
filosof
osofía
ía renacenti
renacentista
sta
El pensamiento renacentista marcó un momento de transición. Da inicio a la modernidad con
sus ideas de individualismo, naturalismo, laicismo y su necesidad de emplear el método
experimental que hace posible el desarrollo de la ciencia moderna.

Práctica
1. Los filósofos estoicos consideraron tres conseguir la vida feliz del sabio partiendo de la
aspectos como los más importantes en la idea según la cual existe una conformidad entre
filosofía: la lógica, la física y la ética. En el campo A) el intelecto y la experiencia.
ético, asumieron que la libertad consistía en B) la naturaleza y la razón.
aceptar nuestro destino y que la finalidad era C) los sentidos y la educación.

CEPRE-UNI HUMANIDADES 153

CICLO INTENSIVO ESCOLAR NACIONAL Material de Estudio Nº 1 

D) la teoría y la vida pública.


E) la felicidad y el deber. 4. En su teoría ______, Platón postuló la
existencia del mundo de las esencias
2. Señale la proposición correcta con respecto al distinguiéndolo del conjunto de las cosas
Renacimiento. sensibles
A) Inicio de la teoría del conocimiento A) del bien supremo B) de la sustancia
B) Predominio de la teología. C) de las ideas D) de la reminiscenc
reminiscencia
ia
C) Ruptura con el pensamiento medieva
medieval.l. E) del devenir
D) Unidad de empirismo y racionalis
racionalismo.
mo.
E) Interés por crear una ciencia universal. 5. La física de Aristóteles explica el fenómeno
del movimiento natural de acuerdo a cuatro
3. ¿Cuál es la importancia de la filosofía del causas. Según dicho enfoque, la causa eficiente
renacimiento? se refiere
A) Consolidó el poder de la iglesia. A) a aquello de lo que está hecho un objeto
B) Introdujo el tema del antropocentrismo. B) a aquello que un objeto es en esencia
C) Centró su interés por la teología. C) a la finalidad a la que tiende un objeto
D) Estableció las bases de lo moderno. D) al agente de quien procede el movimiento
E) Debatió sobre el conocimiento. E) al primer principio de todo lo existente
 
TEMA III. RACIONALISMO. EMPIRISMO.

FILOSOFÍA MODERNA
I. RACIONALISMO
Corriente filosófica que afirma que es posible
posible conocer la estructura de la realidad a parti
partirr de
los principios puros del pensar. Considera que la ordenación lógica del mundo permite
comprender esta estructura de manera deductiva. Se toma como modelo el método
matemático deductivo
deductivo que parte de pocos y seguros axiomas para llegar a la verdad.
Rep: Descartes, Malebranche, Pascal, Spinoza, Leibniz.

RENATO DESCARTES
Filosofía: Ciencia de los 1ros principios y causas para fundamentar las ciencias.
Obras : Discurso del Método, Meditaciones metafís
metafísicas,
icas, Principios de la filosofía, Tratado de
las pasiones.

1. Evidencia: Evitar: todos


METODOLOGÍA Propone 4 reglas ydel
los prejuicios método
admitir como verdadero solo aquello que pueda ser conocido
clara y distintamente
2. Análisis: Descomponer los problemas en tantas partes como sea posible
3. Síntesis: Avanzar paso a paso desde el objeto más simple, como por grados, hacia el más complejo
4. Enumeración: Asegurar la integridad del sistema mediante la revisión o enumeración
La finalidad es llegar a ideas simples que sean intuidas de modo inmediato y evidente. De estas
tendrán que ser deducidas o derivarse proposiciones verdaderas.
GNOSEOLOGÍA
Hay tres tipos de ideas:
1. Idea
Ideass inn
innatas:
atas: 2. Idea
Ideass adventicias: 3. Idea
Ideass fict
fictici
icias:
as:
son evidentes y ya nacen con el vienen del exterior a través de son fabricados por el
hombre. los sentidos. Las cosas nos pensamiento y se producen en
Ej: yo, causalidad, sustancia, causan sensaciones
sensaciones.. la imaginación
sustancia, Dios Ej: color, sabor, textura, sonido, Ej: dragón, sirena, pegaso,
Plantea el innatismo cognitivo aroma minotauro, centauro
CEPRE-UNI HUMANIDADES 154

CICLO INTENSIVO ESCOLAR NACIONAL Material de Estudio Nº 1 

II. EMPIRISMO
Corriente filosófica que considera a la experiencia sensorial como el fundamento del
conocimiento. Para esta postura solo son reales los objetos y fenómenos particulares. El uso
correcto de la razón puede ordenarlos y obtener conclusiones inductivas a partir de ellos.
Como consecuencia, el empirismo influirá en el nacimiento de la ciencia natural y en política
enfatizará en el indiv
individuo.
iduo. Representant
Representantes:
es: Bacon
Bacon,, Hobbes, Locke, Berk
Berkeley,
eley, Hume. 
DAVID HUME
Filosofía: Es el estudio de la naturaleza humana 
Obras: Tratado de la naturaleza humana, Ensay
Ensayos
os morales y políticos, Investigación sobre el
entendimiento humano.
GNOSEOLOGÍA
Hay dos tipos de contenidos de la conciencia (percepciones). Entre los cuales hay dos clases:
a) impresiones:
Son percepciones sensoriales y percepciones internas (afectos, emociones, deseos). Son de
alta intensidad
b) pensamientos (idea
(id eas):
s):
Son imágenes de las impresiones que tenemos cuando reflexionamos sobre ellas, las
recordamos o las imaginamos. Son de baja intensidad
A partir de laselimpresiones
imaginación, surgen
hombre tiene las ideas simples.
la capacidad de formarA partir
ideasde esas ideasEstas
complejas. y mediante la
ideas se
conectan mediante la ley de la asociación siguiendo los principios de: Semejanza, contigüidad
en tiempo o lugar, y causa-efecto. Un concepto solo tiene significado si los componentes de
su correspondiente idea derivan de las impresiones

PRÁCTICA

1. Después de aplicar rigurosamente la duda B) uno fue matemático y el otro no


metódica, Descartes llegó al siguiente principio: B) parten de principios metodológicos distintos
A) Tan solo existe Dios. D) uno es racionalista y el otro empirista
B) La ciencia es demostrable. E) sostienen distinta definición de “idea” 
C) El mundo es demostrable.
D) La mente es extensa. 4. Para Hume, la relación causa-efecto que
E) Pienso, luego existo. creemos
 A) observar en los
la inexistencia de hechos es debido
nuestro a
“yo” como
2. Según las reglas de Descartes, la acción que sustancia.
implica atender los problemas más simples para B) la inexistencia del principio de causalidad.
luego tener la seguridad de la resolución de los C) la costumbre de haber percibido hechos
más complejos, se relaciona con la regla sucesivos.
A) de la revisión. B) de la causalidad. D) las ideas que tienen referentes en sus
C) de la síntesis. D) de la evidencia. impresiones.
E) del análisis. E) las impresiones cuyo basamento son las
ideas.
3. El sistema de Descartes demuestra la
existencia del yo, mientras
mien tras que el de Hume niega 5. Señale el tema estudiado por la filosofía
tal existencia. Ambos pensadores se moderna.
contraponen debido a que ________________. A) el ser B) lo divino C) el sujeto
A) uno afirma la relación causal y el otro la niega D) el lenguaje E) la realidad virtual

TEMA IV. LA ILUSTRACIÓN. IDEALISMO TRASC


TRASCENDEN
ENDENTAL.
TAL.
CEPRE-UNI HUMANIDADES 155

CICLO INTENSIVO ESCOLAR NACIONAL Material de Estudio Nº 1 

I. LA ILUSTRACIÓN
La Ilustración fue un movimiento que desarrolló ideas filosóficas y políticas que se extendieron
por Europa (Inglaterra,
(Inglaterra, Alemania, Italia y Francia) durante el sig
siglo
lo XVIII. El pens
pensamiento
amiento
ilustrado expresa el conjunto de ideas liberal-democráticas que las clases m medias
edias en ascenso
oponen al viejo régimen
r égimen de las monarquías absolutas.

1.
1.1.
El 1.siglo
C
Características
aracterísticas de lla
de las luces tuvo a Ilustración
una Ilustr
granación
influencia en tres aspectos vitales: económico, político y social.
Entre las principales características de la Ilustración tenemos:
- Confianza en la razón por ser autónoma y crítica.
- Rechazo al dogmatismo y confianza en la ciencia.
- Exaltación de la idea de progreso. (Optimismo respecto al futuro)
- Defensa de la libertad y de los derechos del hombre.
- Fomento de lla a tolerancia política y rel
religiosa.
igiosa. (Oposición a tod
toda
a superstició
superstición)
n)
- Énfasis en la educación para hacer mejores ciudadanos.
JEAN JACQUES ROUSSEAU
Obra: El contrato social 
En el estado de naturaleza los humanos son primates inofensivos y aislados que evitan
dañarse entre sí. Son hombres íntegros, sanos, rectos, justos, sin industria, sin lenguaje, sin
domicilio, sin guerra, sin sociedad.
La agricultura
“esto es mío”genera
y lo s la idea de
demás le propiedad. Según
creyeron. Así Rousseau,
surgió la ideaalguien delimitó la
de propiedad tierra y Los
privada. dijo
terratenientes comprendieron que deben legitimar y hacer respetar sus derechos de
propiedad. Así surgieron los contratos ppolíticos
olíticos y ssociales
ociales que se basan en el consenti
consentimiento
miento
de quienes desean paz y seguridad. Pero en realidad, se busca constituir gobiernos coactivos
que priven a la mayoría de su libertad. Así la civilización surge y si bien es más sofisticada,
resulta más hipócrita y egoísta.

Alguna institución debe concentrar la autoridad soberana para dictar leyes necesarias y velar
por su cumplimiento. La voluntad general en una asamblea diseña las leyes del Estado
colectivista que busca el beneficio de la sociedad en su conjunto. Los ciudadanos deben
obedecer la ley porque ellos mismos las han dictado en una asamblea y si bien no benefician
a cada uno en particular beneficia a la sociedad en su conjunto. Evitamos el malogro del
hombre con un nuevo contrato social que es el acto de fundar una sociedad basada en la
voluntad general hacia el bien común.
II. IDEALISMO TRASCENDENTAL
TRASCENDENTAL
Este consiste en poner de relieve la función de lo “puesto” en el conocimiento. Así pues, no
afirma que los objetos externos no existen, o que su existencia es problemática; afirma
únicamente que la existencia de los objetos externos no es cognoscible mediante percepción
inmediata. 

1. IMMANUEL KANT (1724-1804)


Nació en la Königsberg. Sentó las bases del idealismo alemán y las Críticas constituyen el
centro de su obra. Estas son Crítica de la razón pura  (1781), Crítica de la razón práctica  
(1788), Crítica del juicio (1790).

 A. Cr comienza
Cri
Kant i tici
ti cism
smoo   por considerar que poseemos condiciones a priori para conocer: las formas a
priori del espacio y el tiempo, dadas por la sensibilidad; y las categorías a priori dadas por el

CEPRE-UNI HUMANIDADES 156

CICLO INTENSIVO ESCOLAR NACIONAL Material de Estudio Nº 1 

entendimiento. Estas condiciones nos permiten ordenar y organizar nuestro conocimiento.


Este conocimiento resultante es lo que se llama fenómeno , es decir, la cosa-para-mí, la
apariencia. Pero, la cosa-en-sí, la esencia, a saber, el noúmeno , no puede conocerse. Es por
ello, que Kant afirma que solo podemos conocer el mundo fenomenal, ya que el mundo
noumenal solo puede ser pensado.

B. Lógica: Los juicios y el conocimiento


Todo conocimiento se expresa a través de juicios. Mediante ellos se establecen ciertas
relaciones entre dos conceptos: el sujeto y el predicado.
Los juicios se diferencian por su forma en analítico y sintético; y por su validez en relación con
la experiencia en a priori y a posteriori.
Tipos de Juicio Ejemplos
Eje mplos
 An alític
 Analít icoo   A pri
p ri or
orii   “El triángulo tiene tres lados” es un
El predicado está contenido en  juicio analítico
Expresa algo universal, analítico ya que el predi
predicado
cado
el sujeto. No hay aumento de “tener tres lados” está contenido
necesario y que es previo a la
información.  
información. experiencia. en el sujeto “triángulo”. También,
es a priori.
Sintético  
Sintético  A pos
p oster
terio
io ri
ri   “Una pizarra es verde” es un juicio
El predicado no está contenido Expresa algo particular, sintétic
sintéticoo ya que el predicado “ser
en el sujeto. Por ello, es contingente y que proviene verde” no está contenido en el
informativo. de la experiencia
experiencia.. sujeto “pizarra”. Además, se
verifica a posteriori.
Los juicios sintéticos a priori  son los juicios científicos. Comunican la experiencia de los
fenómenos mediada por los conceptos puros del entendimiento. Los juicios sintéticos a priori 
forman parte de todas las ciencias teóricas como principios.

Práctica
1. En la Crítica de la Razón Pura,
Pura , Kant aborda D) Difunde el mito del progreso.
un problema de índole E) Surge con el declive de la burguesía.
A) ética B) antropológica
antropológica C) gnoseológica
D) estética E) axiológica
axiológica 4. El programa ilustrado se expresa en la idea
del progreso humano basado en la confianza en
2. El giro copernicano
copernicano,, según Kant, significa que la razón humana. Ello no implica
A) el conocimiento es un reflejo del objeto en la A) liberación del hombre de la ignorancia y la
mente del sujeto. opresión.
B) el sujeto es un elemento pasivo en el B) defensa de la ciencia y la técnica.
conocimiento. C) cuestionamiento de la tradición, los prejuicios
C) es la percepción del sujeto la que crea al y los dogmas.
objeto. D) sustituir las supersticiones y dogmas por la
D) el sujeto es un factor activo en la organización ciencia.
del conocimiento. E) la defensa del sentimiento y la tradición.
E) existe una realidad objetiva independiente del
sujeto. 5. Señale un ejemplo de juicio analítico:
A) La pera es verde y grande.
3. Señale el enunciado incompati
incompatible
ble con la B) La madre de Juan es Ana.
Ilustración. C) Un kilo pesa 1000 gramos.
A) Afirmó la autonomía de la razón. D) El azúcar pesa un kilo.
B) Se opone a las supersticiones. E) El automóvil es caro.
C) Es optimista respecto al futuro.
CEPRE-UNI HUMANIDADES 157

CICLO INTENSIVO ESCOLAR NACIONAL Material de Estudio Nº 1 

TEMA V. G. W. F. HEGEL.
KARL
KA RL MARX.
MA RX. AUGUSTO C COMTE.
OMTE.

1. G. W. F. HEGEL (1770 -1831)


Nació en Stuttgart, Alemania. Es el último filósofo
f ilósofo que planteó un sistema de la ciencia o del
saber con el objetivo de contener todo el conocimiento existente. Sus obras más importantes
son Fenomenología del espíritu  (1807), Ciencia de la lógica (1817) y La Filosofía del Derecho
(1820).

 A. MÉTODO: La d ialéct


ial éctic
ic a
La dialéctica es el método que utiliza Hegel para conocer el fundamento de todo lo existente.
Con la dialéctica se explica cómo la realidad puede superarse a sí misma. Este método tiene
una cualidad particular: el movimiento. De acuerdo con la dialéctica, la verdad no es algo
estático, sino algo cambiante. Sin embargo, esta no rechaza totalmente las verdades
anteriores sino que, por el contrario, las asume. Así pues, el movimiento dialéctico se
caracteriza por asumir todas las verdades anteriores, sintetizándolas con las nuevas. Este
movimiento racional es infinito.

B. METAF
METAFÍS
ÍSICA:
ICA: El Espíritu Absol
Absoluto
uto
Para Hegel, la
movimiento, realidad
que debe
se supera entenderse como
constantemente. unsujeto
Este sujetoesque se encuentra
el Espíritu en proceso,
que genera en
su propia
determinación y se actualiza continuamente. La realidad como sujeto es conciencia,
capacidad de interiorización, espíritu que conoce, vuelta a sí mismo. Asimismo, Hegel
consideró que la verdad no surge a partir de un conocimiento de algo especíespecífico
fico sino de una
búsqueda inacabada y de la totalidad del conjunto y de cada una de sus partes. Así pues, el
conocimiento que él busca tener de la realidad no es un proceso fijo ni estático sino que en
cada etapa se amplia y progresa. Esto ocurre porque el Absoluto sólo existe concretándose
y encarnándose en todas las cosas. Por ende, decir que todo es Espíritu Absoluto quiere decir
que nada tiene ser, ni es por tanto verdaderamente conocido, si no es entendido como un
momento de la vida infinita. Así, el Espíritu, como la racionalidad que gobierna al universo en
su totalidad, se manifiesta a la conciencia bajo formas diferentes que van de lo simple y lo
sensible a lo complejo y lo abstracto. De este modo, tenemos en orden ascendente al Espíritu
Subjetivo, Espíritu Objetivo y Espíritu Absoluto.

2. KARL
KA RL MARX
MA RX (1818-
(1818-1883)
1883)
Nació en Tréveris (actual Alemania) en 1818. Estudió en Bonn y Berlín. Su tesis doctoral fue
Diferencia entre la filosofía de la naturaleza de Demócrito y la de Epicuro  (1841), donde ya
muestra su cercanía al Materialismo. Criticó a los hegelianos revolucionarios, pues consideró
que la filosofía debe dejar de interpretar la realidad, ya que de lo que se trata es de
transformarla.

 A. FILOSOFIA
FIL OSOFIA DE L A HISTORIA: Mater Materii alism
ali smo
ohhii stór
st óric
ic o
Esta teoría explica la relación entre la estructura económica y la superestructura ideológica.
Los hombres crean sus representaciones
representaciones e ideas, pero están con condicionados
dicionados por el modo de
producción de la vida material. Por lo tanto, la superestructura ideológica está determinada
por la estructura económica. Así, si se transforma la estructura, entonces se transforma la
superestructura.
B. ANTROPOLOGÍA FILOSÓFICA: Alienación del trabajo

CEPRE-UNI HUMANIDADES 158

CICLO INTENSIVO ESCOLAR NACIONAL Material de Estudio Nº 1 

En Manuscritos económicos-filosóficos de 1844 explica que el hombre puede vivir acorde a


su humanidad, es decir, apropiándose y modificando la naturaleza de acuerdo a sus
necesidades y a sus ideas. Esto es, vivir trabajando. Pero, actualmente el hombre no trabaja
por la necesidad de apropiarse de la naturaleza, sino para su subsistencia. Así, el trabajo se
ha vuelto externo al obrero, ya no le pertenece, pues ahora es una actividad constrictiva. Y,
es constrictiva porque la propiedad privada separa al trabajo del obrero. La materia prima, los
medios de producción y el producto son alienados del trabajo, no le pertenecen al obrero.
Incluso, el obrero ya no se pertenece a sí mismo, es una mercancía administrada por el
capitalista. Y, así, termina deshumanizado,
deshumanizado, convertido en una cosa que se compra o vende.

3. AUGUSTO COMTE (1798-1857)


Fue un filósofo francés, miembro de una familia católica, fe a la cual renunció desde joven.
Fundó el Positivismo
Positivismo y es consid
considerado
erado el padre de la So
Sociología.
ciología. Obras
Obras:: Curso de filosofía
positiva (1842), Discurso sobre el espíritu positivo  (1844) y Sistema de política positiva que
instituye la religión de la humanidad (1854).

 A. Posi
Po sititivi
vism o: Marx interpreta la revolución industrial a partir de sus luchas de clases y sus
smo:
perjuicios, pero los positivistas lo interpretan a partir de sus beneficios, ya que confían en que
la ciencia y el espíritu científico restablec
restablecerán
erán la sociedad. El positivismo es una filosofía social
que busca
Algunas defundar una ética a partir
sus características de una
generales base científica. Su lema es: “Orden y progreso”.
son:
*Atención a los hechos. Por ‘hecho’ se entiende todo aquello que puede ser observado. 
*Aversión a la metafísica. Rechazo hacia entidades abstractas (esencias o sustancias).
*Exaltación de la ciencia. El único conocimiento válido es el científico. Asimismo, el único
método para la adquisición de conocimiento es el que usanlas ciencias naturales para
formular leyes. Cuando este método se aplica a la sociedad, surge la Sociología.
*La idea de progreso. Se entiende progreso como un mejoramiento constant
constantee y gradual de la
realidad.

PRÁCTICA
1. En la lógica dialéctica hegeliana, los E) Es el relato de personajes independientes a
momentos iniciales corresponden a la voluntad del Espíritu.
A) la antítesis
B) la afirmación y la
y la negación.
síntesis. 3. Para yHegel,
Religión el _______ se divide en Arte,
Filosofía
C) la negación de la negación. A) espíritu subjetivo B) espíritu objetivo
D) la superación de la negación. C) espíritu absoluto D) espíritu santo
E) la tesis y la síntesis. E) Derecho

2. Señale lo correcto sobre la filosofía de la 4. Sobre la alienación del trabajo del obrero, es
historia en Hegel incorrecto sostener, con Marx, que su trabajo
A) Es la narración progresiva y evolutiva del A) le pertenece B) le es ajeno
Espíritu. C) es externo D) es antinatural
B) Es el relato de hechos históricos eventuales y E) le deshumaniza
azarosos.
C) Es el registro de sucesos y personajes 5. Es falso
f also que el positivismo se fundamente en
anecdóticos del Espíritu. el conocimiento que procede de
D) Es la narración de eventos y circunstancias A) la metafísica B) la ciencia
causales y contingentes. C) la experiencia D) la naturaleza
E) los fenómenos
CEPRE-UNI HUMANIDADES 159

CICLO INTENSIVO ESCOLAR NACIONAL Material de Estudio Nº 1 

Psicología 
TEMA I: LA PSICOLOGÍA. MÉTOD
MÉTODOS:
OS: OBSERV
OB SERVACIÓN,
ACIÓN, EXPERIMEN
EXPERIMENTAL
TAL Y
CORRELACIONAL

1. LA PSICOLOGÍA
Es el estudio científico de la mente,
m ente, cerebro y comportamiento. Abarca su explicación desde
las moléculas, las estructuras cerebrales, los pensamientos, los sentimientos, las emociones,
hasta las influencias sociales y culturales, con muchos escalones intermedios.
2. OBJETIV
OBJ ETIVOS OS CIENT
CIENTÍFICO
ÍFICOSS DE LA
L A PSICOLOGÍA
1. Describir: Consiste en definir con mayor precisión los fenómenos psicológicos, para ello
se reúnen datos sobre un fenómeno en particular.
2. Expl icar:  Consiste en establecer relaciones de causa y efecto entre los factores implicados.
Explicar:
Se propone un nivel de explicación tentativ
tentativaa (hipótesis) que se somete a comprobación.
3. Predecir:  Si encuentra apoyo empírico de las hipótesis
hi pótesis antes propuestas, estas tendrán el
valor de predecir la conducta en otras situaciones.
4. Controlar: Las condiciones que se suponen causan la conducta, se pueden alterar o
controlar. Así por medio de la práctica psicológica se influencia en un fenómeno conductual. 
3. MÉTODOS DE LA PSICOLOGÍA
3.1. Observación
Consiste   de manera precisa y sistemática un conjunto de hechos relacionados
en describir
con el objeto estudiado. Estos hechos se pueden dar de manera espontánea y el investigador
interviene mínimamente para que esos hechos se manifiesten así. Se presenta dos formas:
extrospección e introspección.
3.
3.2
2 Método experi
experimental
mental
También conocido como el método científico por su carácter demostrativo, consiste en la
provocación de una conducta bajo situaciones controladas con la finalidad de comprobar una
hipótesis.
Pasos:
1. Formulación del problema 5. Elaboración de instrumentos: test,
2. Planteamiento de la hipótesis cuestionarios, pruebas de rendimiento, etc.
3. Determinar el marco teórico conceptual 6. Selección de sujetos
4. Determinar las variables (constructos), 7. Establecimiento del grupo experimental
variable dependiente (VD), independiente (GE) y control (GC)
(VI) y de control 8. Ejecución del experimento
9. Presentar los resultados

Objetivos:
  Permite verificar una hipótesis.
hipótesis.
  Permite reproducir situaciones experimentales
experimentales para una mejor observación del fenómeno. 
Ventajas:
 Es posible realizar observaciones controladas, establecer claramente relaciones causa  – 
efecto.
  Hay control estricto de
de las variables.
  Se trabaja con dos grupos de sujetos: control
control y experimental. 
Desventajas:
 Lo artificial del ambiente puede influir en el comportamiento de los sujetos, las variables
inesperadas y no controladas pueden confundir los resultados. 
3.
3.3.
3. M
Método
étodo ccorr
orrelacional
elacional

CEPRE-UNI HUMANIDADES 160

CICLO INTENSIVO ESCOLAR NACIONAL Material de Estudio Nº 1 

Este método busca establecer el grado de relación existente entre fenómenos psicológicos
en un sujeto o muestras de sujetos. Por ejemplo, establecer el grado de relación existente
entre el estrés y el rendimiento
r endimiento académico en alumnos del CEPREUNI.

Ventajas:
  Emplea métodos estadísticos
estadísticos para examinar la relación entre dos o más variables.
  Permite la predicción y puede utilizarse en el laboratorio, la clínica o los escenarios
naturales. 
Desventajas:
  Tal vez las relaciones se deban a coincidencias, no es posible establecer relaciones causa
 – efecto. 

EVALUACIÓN
1. Según los objetivos de la psicología, marque A) Observación B) Experimental C)
la alternativa que presenta una afirmación Correlacional D) Aplicativo E) Mixto
incompatible.  
incompatible.
A) Establece relaciones de causa y efecto. 4. Rosalía tiene como objetivo general
B) Evita proponer una explicación tentativa. relacionar el aprendizaje autorregulado con el
C) Predice la conducta ante situaciones. rendimiento académico de los estudiantes de
D) Describe los procesos psicológicos
psicológicos.. CEPRE-UNI. ¿Qué método de la psicología se
E) Controla las causas de la conducta. empleará?
A) Mixto B) Experimental
Experimental C) Correlaciona
Correlacionall
2. En un caso de violencia familiar, el psicólogo D) Cuasi experimental E) Observaciona
Observacionall
forense Fontana, busca determinar las causas
que hicieron que el asesino acometiera de tal 5. La investigación científica que gira en torno
t orno a
modo. ¿Qué objetivo científico de la psicología la influencia de la motivación lectora en la
se presenta? comprensión de textos expositivos. Se logra
A) Describir B) Explicar C) Controlar identificar los constructos que se
D) Predecir E) Aplicar operacionalizarán. ¿Qué paso del método
experimental hace referencia la identificaci
identificación?
ón?
3. ¿Cuál es el método de la psicología que A) Formulación del problema
describe un conjunto de hechos relacionados B) Determinar las variables
con el objeto de estudio, que presenta la C) Selección de sujetos
extrospección y la introspecci
introspección?
ón? D) Planteamiento de la hipótesis
E) Determinar el marco teórico

1B 2B 3A 4C 5B

TEMA II: ESCU


ESCUELAS
ELAS PSICO
PSICOLÓGICAS:
LÓGICAS: CIENT
CIENTÍFICA
ÍFICA Y CONTEMPO
CONTEMPORÁNEA
RÁNEA

1. ESCUELAS PSICOLÓGICAS
1. 1. ESTRUCTUR
ESTRUCTURALISMO
ALISMO (Al emani
emania,
a, 1920)
Fundadores: Wilhelm Wundt, creador del primer sistema psicológico y su discípulo Titchener,
desarrolló y sistematizó la psicología científic
científicaa establecida por Wundt.
Objeto
Obje to d e estudio: La conciencia.
Método: Introspección bajo un control experimental e ncluye la l a imaginación.
Postulado principal: Los elementos simples e irreductibles donde descansa toda actividad
mental son:
(base del sensación (base
pensamiento
pensamiento).
). de la percepción), sentimiento (base de las emociones) e imagen
CEPRE-UNI HUMANIDADES 161

CICLO INTENSIVO ESCOLAR NACIONAL Material de Estudio Nº 1 

tes:: Introdujo la medición y el experimento, marcando con ello el nacimiento de la


 Ap or tes
 Apor
psicología como ciencia.
1. 2. FUNCIONALISMO (Estados Unidos, 1986) 
Fundadores: William James, John Dewey, James Angell.
Método: Experimental, psicometría, observación (introspección y extrospección).
Obje
Objeto principal:Función
to d e estudio:
Postulado Se debeadaptativa de lacómo
comprender conciencia.
las distintas propiedades y característic
características
as
de la mente permiten al individuo la adaptación a su medio.
 Apor
 Ap tes:: Inicio de la psicología aplicada, antecedente teórico para el conductismo,
or tes
elaboración de pruebas mentales, investigación en aprendizaje, emociones y psicología
infantil.
1. 3. REFLEXOLOGÍA
REFLEXOLOGÍA (Rus ia, 1904)
Fundadores: Ivan Pavlov, Bechterev.
Método: Experimental. 
Objeto
Obje to d e estudio: Actividad del sistema nervioso y su relación con la conducta.
Postulado principal: La mayoría de conductas animales se fundamentan en un pensamiento
basado en un sistema
sistema sustituciones refl
reflejas,
ejas, es decir, por as
asociación.
ociación.
 Apor
 Ap tes:: Descubrió los principios que gobiernan el sistema nervioso: excitación – inhibición,
or tes
teoría sobre el aprendizaje: Condicionamien
Condicionamiento to clásico.
1. 4. CONDUCTISMO (Estados Unidos, 1908)
Fundadores: Watson, Thorndike, Weiss, Holt, Hunter y Lashley.
Método: Experimental y observación.
Objeto
Obje to d e estudio: Conducta observable y aprendida.
Postulado principal: El mecanismo de aprendizaje de una conducta humana es similar al de
los animales, se basa en un esquema E-R (Estímulo-Respu
(Estímulo-Respuesta).
esta).
 Apor
 Ap tes:: Los principios de aprendizaje, en psicología educativa: tecnología educativa y
or tes
problemas de aprendizaje, terapia centrada en la modificación de la conducta.
1. 5. GESTALTISMO
GESTALTISMO (Alem (Alemani
ania,
a, 191
1912)
2)
Fundadores: Max Wertheimer, Wolfgang Kolher, Kurt Koffka.
Método: Experimental y observación.
Objeto: Experiencia total inmediata: Percepción
Postulado principal: La mente es activa, dinámica y asociativa, por lo que no acepta
pasivamente lo que percibe, sino, que constant
constantemente
emente busca significado entre los estímulos
externos.
 Apor
 Ap tes:: Principios de la percepción, Rubín introdujo el concepto “figura - fondo”, dinámica
or tes
de grupo “teoría de campo”, psicoterapia gestáltica. 
1. 6. PSICOANÁLISIS (Austria, 1895)
Fundador: Sigmund Freud.
Método: Asociaciones libres, interpretación
interpretación de los sueños y la técnica de la catarsis.
Objeto
Obje to d e estudio: El inconsciente sobre la base de la sexualidad.
Postulado principal: El contenido inconsciente estructura la personalidad y explica
conductas actuales, además este contenido ha sido formado principalmente en la primera
infancia.
 Apor
 Ap tes::  Teoría sobre la personalidad y sus trastornos, teoría sobre la sexualidad (origen y
or tes
desarrollo),
sociales. psicoterapia de trastornos mentales, explicación psicoanalítica de fenómenos
CEPRE-UNI HUMANIDADES 162

CICLO INTENSIVO ESCOLAR NACIONAL Material de Estudio Nº 1 

2. CORRIENTES PSICOLÓGICAS CONTEMPORANEAS


2.1 PSICOLOGÍA GENÉTICA
GENÉTICA O EVOLUTIVA
Representante:  Jean Piaget 
Postulado: El comportamiento debe ser comprendido como una evolución en la que cada
etapa se basa en la anterior, y es seguida por otra avanzada hasta llegar a su culminación.  
or te: Sistematiza en etapas el desarrollo humano, planteando criterios de evaluación del
 Apor
 Ap
desarrollo normal.
2.2 NEOCONDUCTISMO
Representantes:  B. F. Skinner, Clark Hull y Edwar Toman 
Representantes:
Postulados: Para Skinner, la psicología debería estar centrada en la conducta observable
(conducta operante). 
or te: El ser humano asume un papel activo en el aprendizaje de su conducta, las
 Apor
 Ap
asociaciones de agrado o desagrado que la persona realiza acerca de las consecuencias de
sus actos condicionan su actuar futuro.
2.3 COGNITIVISMO
Representantes:  Ausbel, Piaget, Brunner y N. Wiener. 
Representantes:
Postulados: La psicología debe estudiar los procesos cognitivos: sensaciones,
percepciones, inteligencia, pensamiento – lenguaje e imaginación. 
or te: Centra su interés en la forma cómo se procesa la información y qué estrategias
 Apor
 Ap
utilizamos para organizar esa información.
2.4 HUMANISMO
Representantes:  Maslow, Rogers, Allport, Buhler y Bugental. 
Representantes:
Postulados:   El yo es el único centro unificador de su mundo privado y de su conducta
Postulados:
(autoidentidad, autoevaluación y autorrealización). 
 Apor
 Ap or te: Persigue la renovación del pensamiento psicológico, postula que este debe dejar de
lado aspectos patológicos para dedicarse a la consideración global de la persona y la
acentuación en sus aspectos existenciales (la libertad, el conocimiento, la rresponsabilida
esponsabilidad,
d, la
historicidad).

EVALUACIÓN
1. _______ es una escuela psicológica científica tuvo como objeto de estudio la actividad del
que utilizó métodos como las asociaciones libres sistema nervioso y su relación con la conducta.
yA)laElinterpretación
humanismo de losconductismo
B) El sueños. A)
C) Conductismo
Gestaltismo B)D)Reflexologí
Reflexologíaa
Cognitivismo
C) ElEl psicoanáli
psicoanálisis
sis D) E Ell funcionalismo E) Funcionalismo
E) El estructuralis
estructuralismo
mo
4. Los principios del aprendizaje en psicología
2. Según la escuela psicológica estructurali
estructuralista
sta educativa como la tecnología, problemas de
podemos afirmar que aprendizaje y terapia centrada en la modificación
A) propuso el uso de la técnica experimental. de la conducta son los aportes del Conductismo,
B) aportó a los trastornos de la personalidad. que tuvo como uno de sus fundadores a
C) estudió los sueños como base de la A) Ausbel. B) Piaget. C) Watson.
sexualidad. D) Brunner. E) Ivan Pavlov.
D) Maslow y Rogers son destacados
fundadores. 5. El yo es
yo es el único centro unificador de su
E) presentó como objeto de estudio la mundo y de su conducta (autoidentidad,
conciencia. autoevaluación y autorrealización) fue un

3. Ivan Pavlov fue uno de los fundadores de la postulado deta.la corriente


A) cognitivista.
cognitivis B) neocondu
neoconductista.
ctista.
escuela psicológica denominada ________, que C) genética. D) humanista
humanista..

CEPRE-UNI HUMANIDADES 163

CICLO INTENSIVO ESCOLAR NACIONAL Material de Estudio Nº 1 

E) evolutiva.
1C 2E 3B 4C 5D

TEMA III: FACTORE


FACTORES
SBBIOLÓGICOS
IOLÓGICOS DEL COMPORTAMIE
COMPORTAMIENTO
NTO HUMANO: SISTE
SISTEMA
MA
NERVIOSO PERIFÉRICO Y SISTEMA ENDOCRINO 

1. SISTEMA NERVIOSO PERIFERICO


Se encarga de enviar la información sobre las entradas sensoriales y recibe instrucciones
para accionar en músculos y glándulas. Está formado por los nervios que parten o llegan al
sistema nervioso central (SNC). Se clasifica en sistema somático y autónomo.
1.1. Sistema somático: Controla los cambios que provienen del mundo exterior. Está
formado por las fibras nerviosas que inervan la musculatura esquelética; es decir, los
músculos que pueden moverse voluntariamente. Participan en este sistema las neuronas
sensoriales (aferentes), que transmiten mensajes al SNC, y las motoras (eferentes) que llevan
mensajes del SNC a los músculos esqueléticos del cuerpo.
1.
1.2
2 Sistema a utón omo: Regula los cambios producidos en el medio interno del organismo.
autón
Está constituido por fibras nerviosas que controlan el músculo estriado cardíaco, las glándulas
y el músculo liso; es decir, los órganos que realizan acciones involuntarias. Se subdivide en
sistema simpático y parasimpático.

Sistema parasimpátic o Sistema sim


simpático
pático
Contrae la pupila. Dilata la pupila.
Estimula la salivación
salivación.. Inhibe la salivación
salivación..
Reduce el latido cardiaco. Acelera el impulso cardiaco.
Contrae los bronquios. Relaja los bronquios.
Estimula la actividad digestiva. Inhibe la actividad digestiva.
Contrae la vejiga. Relaja la vejiga.
Relaja el recto. Contrae el recto.
Estimula la vesícula biliar. Estimula la liberación de glucosa por el hígado.
2. SISTEMA ENDOCRINO
Regula las actividades y funcionamiento del organismo, los órganos especializados de este
sistema son las glándulas endocrinas. Estas se caracterizan por no tener conductos de salida
hacia el exterior de tal forma que los productos que fabrican (hormonas) los evacúan
directamente a la sangre o a los líquidos circulantes para ser distribuidos por el organismo.

Órgano
Órgano Hormona Función hormonal
Presenta dos partes que trabajan de manera independiente.
ipófisiss ante
ipófisi anterior
rior o adenohipófi sis: Segrega hormonas
adenohipófisis:
Glándula pituitaria o estimulantes
hipófisis En exceso, puede generar gigantismo;
Es una estructura del Trófica u hormona en escasez, enanismo. En los adultos,
tamaño de un frijol que del crecimiento.  genera acromegalia (crecimiento
cuelga de la base del deformado de los maxilares, huesos
encéfalo y está conectada nasales, manos, pies).
al hipotálamo
hipotálamo a trav
través
és de Controla la producción y secreción de
las hormonas de la corteza adrenal
fibras nerviosas.  Adenocorticotrofina
Estimula la producción y secreción de
Tirotrofina hormonas de la tiroides.

CEPRE-UNI HUMANIDADES 164

CICLO INTENSIVO ESCOLAR NACIONAL Material de Estudio Nº 1 

Inicia y mantiene la producción láctea


Prolactina en los senos.
Estimula la producción del cuerpo lúteo
Hormona o amarillo en la mujer, y la producción de
luteinizante testosterona en el hombre.
Actúa sobre los ovarios (a partir de la
Hormona pubertad), para que maduren los
folículoestimulante  folículos; y sobre los testículos para que
se produzcan los espermatozoides
2. Hipófisis posterior o neurohipófi sis
Vasopresina o Afecta la retención de agua en los
antidiurética riñones, controla la presión
sanguínea.

Oxitocina Estimula las contracciones uterinas


y de los conduc
conductos
tos lácteos en los
senos.

Órgano
Órgano Hormona Función hormonal
Glándula pineal
Está ubicada El nivel de esta hormona se eleva al
aproximadamente en el atardecer y alcanza su máximo hacia la
centro del cerebro. Melatonina  medianoche, para luego disminuir
cuando se apraproxima
oxima eell amanec
amanecer.
er. Este
ciclo dirigido por la luz ayuda a controlar
los ritmos corporales y los ciclos de
sueño.
Glándula tiroi
Glándula tiroides
des Regula el metabolismo (proceso de
Está localizada delante desintegración de los alimentos ingeridos
de la tráquea y debajo para almacenarlos en forma de energía).
de la laringe. Un exceso producirá ansiedad, excitación,
ritmo acelerado del corazón, tensión,
nerviosismo, diarrea y pérdida de peso.
Tiroxina Un déficit generará pérdida de energía,
reducción del ritmo cardíaco, estreñimiento y
sensación de frío en todo
t odo momento.
En la infancia, puede conllevar al retardo
mental; y en la adultez, puede causar
inactividad, somnolencia, lentitud y
sobrepeso. 
Glándula paratiroides Controla y equilibra los niveles de calcio y
Son cuatro minúsculas Paratiroidea o fósforo en los fluidos
f luidos tisulares y sanguíneos
sanguíneos,,
glándulas incrustadas Parathormona importantes para el desarrollo de huesos y
en la tiroides.  dientes. Su escasez causa que los huesos
se fracturen fácilmente y genera espasmos
musculares. Por el contrario, su exceso
genera pérdida de tono muscular y cálculos
renales 
CEPRE-UNI HUMANIDADES 165

CICLO INTENSIVO ESCOLAR NACIONAL Material de Estudio Nº 1 

El páncreas Disminuye el nivel de azúcar en la


Además de sus funciones Insulina sangre; estimula el metabolismo de la
digestivas (porción glucosa, las proteínas y las grasas. Un
exocrina), posee, por su déficit provocará diabetes.
porción endocrina, la
capacidad de secretar
Glucagón Aumenta el nivel de azúcar en la sangre
hormonas. 
EVALUACIÓN
1. Marque la alternativa que presenta 3. En situaciones de sismos, a Rafael se le
afirmación incongruente sobre el Sistema acelera el ritmo cardiaco. ¿Qué sistema estimula
Nervioso Periférico (SNP). a los órganos para asegurar respuestas ante
A) Está formado por los nervios que llegan al situaciones de peligro?
sistema nervioso central. A) Simpático B) Autónomo
B) Se clasifica en dos sistemas importantes: el C) Vegetativo D) Parasimpático
somático y el autónomo. E) Somático
C) Se encarga de enviar la información sobre
las entradas sensoriales. 4. Marque la alternativa que presenta
D) El sistema autónomo presenta el sistema afirmación incorrecta sobre el sistema
simpático y parasimpá
parasimpático.
tico. parasimpático.
E) El sistema autónomo controla los cambios A) Estimula la salivación.
que deviene del exterior. B) Reduce el latido cardiaco.
C) Estimula la vesícula biliar.
2. La función de estimular la producción del D) Inhibe la actividad digestiv
digestiva.
a.
cuerpo lúteo o amarillo en la mujer, y la E) Estimula la actividad digestiva.
producción de testosterona en el hombre
corresponde a la hormona _______ . 5. La _______ estimula las contracciones
A) tirotrofina B) luteiniza
luteinizante
nte C) prolactina uterinas y de los conductos lácteos en los
D) oxitocina E) melatonina
melatonina senos.
A) insulina B) progestero
progesterona
na
C) melatonina D) adenohi
adenohipófisis
pófisis E) oxcitocina
1E 2B 3A 4D 5E

TEMA IV: MEMOR


MEMORIA,
IA, ETAPA
ETAPASS Y TIPOS DE MEM
MEMORIA
ORIA

1. MEMORIA
La memoria es un proceso psicológico que sirve para almacenar información codificada.
Dicha información puede ser recuperada, unas veces de forma voluntaria y consciente y otras
de manera involuntaria.
1.1. Etapas de la memoria
FIJACIÓN Es el registr
registr o de la información y experiencias vividas.
CONSERVACIÓN  Es la retención y consolidación de la información y experiencias vividas.
Es un proceso activo de reproducción o reconstrucción de la información
que fue almacenada a corto o largo plazo.
EVOCACIÓN Espontánea:   Inusitada:
Espontánea: Inusitada:   La información que se reactualiza no tiene
En ella los relación alguna con los actuales contenidos de la
contenidos conciencia del sujeto.
CEPRE-UNI HUMANIDADES 166

CICLO INTENSIVO ESCOLAR NACIONAL Material de Estudio Nº 1 

son evocados  As  Asoc


ociat
iativ
iva:
a:   Los contenidos que se reactualizan son
involuntariam generados por la relación con la información existente en la
ente. conciencia en un momento dado.
Voluntaria: Se
Voluntaria:  Se da cuando de manera deliberada se hace el esfuerzo por
recordar algún hecho, cosa, situación o persona.
Consiste en ser consciente de que la experiencia recordada forma parte de
RECONOCI- nuestro pasado. Cuando las nuevas impresiones coincidan con las
anteriores, el reconocimiento se produce casi instantáneamente; cuando no
MIENTO
existe conciencia total, solo aparece el sentimiento de que el objeto no es
conocido.
LOCALIZACIÓN Es la ubicación del recuerdo en el tiempo y el espacio.

1.
1.2.
2. T
Tipo
iposs de
d e memori
memoriaa

Tipos  
Procesos de
Definición Características
control
Registra y almacena información tal cual llega a
 At enció
 Atenc ión
n
Suele almacenar los receptores.
Filtro de datos
características de Tiene como función, mantener información
contenidos en un
L los objetos, sensorial por más tiempo luego de la presencia
IA percibidas por patrón de física del estímulo para que se reconozca sus
ingreso sensorial
R alguno de los características y formas.
O que se da ante la
S sentidos. Necesita un tiempo para registrar el estímulo. Si
N limitación de
E Tiene capacidad en ese momento se presenta otro, se
procesamiento
S limitada (0.1 interrumpiría la actividad sensorial
de información
segundos – 0.5 desencadenada por el primero y el sujeto no
de nuestro
segundos) recordará al primer estímulo, lo olvidará.
cerebro
Conocida como
memoria activa o
de trabajo.
Recibe la información proveniente de la memoria
Almacena los Repetición
sensorial.
recuerdos, el Repaso, para
Almacena ítems significativos; por tanto,
O conocimiento, las fortalecer la
Z conserva información lingüística. Así, retiene
LA imágenes, los
conceptos, codificación,
la de interpretaciones de los estímulos. Por ejemplo,
información
P cuando escuchamos un argumento, no
estrategias de que se desea
O almacenamos los sonidos de palabras sino
T actuación, que que pase de la
R desciframos sentidos e intenciones involucrados
estamos utilizando memoria a corto
O en dicho argumento.
C en este momento. plazo a la
Su efectividad en el almacenamiento se basa en
Tiene capacidad memoria de
la repetición que permite una efectiva
limitada de 7 más largo plazo.
codificación.
o menos 2 ítems
de información. (15
 – 30 segundos)
CEPRE-UNI HUMANIDADES 167

CICLO INTENSIVO ESCOLAR NACIONAL Material de Estudio Nº 1 

Codificación
Implica organizar
Depósito de toda la información, habilidades,
la información de
capacidades y que no se utilizan en el momento,
forma
pero que potencialmente pueden recuperarse.
O significativa para Tipos:
Z
A su
L Explicita:  Verbal, consciente.
Explicita: Verbal,
P Tiene capacidad almacenamiento
. Semántica. Almacena datos generales. Memoria
O ilimitada. tipo diccionario
diccionario..
G La codificación
R
Episódica. Referida a eventos experimentados
puede ser
A personalmente
personalmen te (biografía). Memoria tipo diario.
L enactiva
Implícita. No
Implícita.  No verbal, automática.
(acción), icónica
Procedimental:: hábitos
Procedimental
(viso-espacial) y
Emocional.
sobre todo
verbal.

EVALUACIÓN

1. La _______ es una etapa de la memoria que D) Almacena una información permanente.


registra la información y las experiencias E) Organiza una información significativa.
vividas.
A) evocación B) atención C) conservación 4.
de La memoria
forma ______para
significativa organiza la información
un almacenamiento
D) fijación E) localización
localización
permanente.
2. Rodrigo recuerda con detalles su fiesta
f iesta de A) sensorial B) activa C) trabajo
cumpleaños que lo celebró junto a su padre, D) corto plazo E) largo plazo
quien falleció unas semanas después. Rodrigo
retiene aquellas experiencias que compartió 5. Relacione etapa de la memoria y su
con su padre. ¿Qué etapa de la memoria se descripción.
presenta? I. Fijación
A) Conservación B) Fijación II. Conservación
Conservación
C) Reconocimie
Reconocimiento
nto D) Recuerdo III. Reconocimiento
Reconocimiento
E) Localización a. Es el registro de la información y experiencias
vividas.
3. Marque la alternativa que presenta una b. Ser consciente de que la experiencia es parte
afirmación correcta sobre la memoria de corto del pasado.
plazo.
A) Fija los datos desde un patrón sensorial. c. Es laas vividas.
experiencias
experienci retención y consolidación de
B) Es conocida como la memoria de trabajo. A) I b, II a y III c B) I a, II c y III b C) I c, II b
C) Presenta la memoria semántica e implíci
implícita.
ta. y III a D) I b, II c y III a E) I a, II b y III c
1D 2A 3B 4E 5B
TEMA V: IMAGINACIÓN: DEFINICIÓN Y TIPOS. CREATIVIDAD: DEFINICIÓN Y FASES

1. LA IMAGINACIÓN
Es el proceso cognitivo que permite al individuo manipular iinformación
nformación generada internament
internamentee
con el fin de crear una representación a través de los sentidos de la mente. Algo más
desafiante es definir los ‘sentidos de la mente’. El ‘ojo de la mente’ es un término
frecuentementee utilizado (pero raramente definido) hace referencia a un mecanismo cognitivo
frecuentement
que ‘ve’ un objeto que se había visualizado previamente pero que no se encuentra presente
en el ambiente.
Tipos de imaginación
  Imaginación reproductora: Representa, en nuestra mente, aquello que hayamos podido
percibir con nuestros sentidos (sonidos, dibujos) con mayor o menos exactitud. Tiene un

CEPRE-UNI HUMANIDADES 168

CICLO INTENSIVO ESCOLAR NACIONAL Material de Estudio Nº 1 

carácter pasivo. Ejemplo: reproducción de escenas de la obra Romeo y Julieta al escuchar al


profesor de Literatura. 
  Imaginación creadora: Crea imágenes reales o ficticias. Es impulsada por nuestra
motivación e inspiración, y es dirigida por la inteligencia. Así, un individuo es capaz de reclutar
y combinar selectivamente ‘porciones de datos’ almacenados en la memoria para crear
nuevas imágenes o eventos que no se han experimentado con anterioridad Ejemplo: imaginar
las sirenas. 
2. CREATIVIDAD
Es un sistema flexible de signos vocales, gráficos y gestos y reglas formales que nos permite
representar y expresar nuestras ideas, pensamientos y sentimientos.
2.
2.1.
1. E
Ell pr oceso cr
crea
eativo
tivo
El psicólogo inglés Graham Wallas en El arte del pensamiento (1926) presentó cuatro etapas
del proceso creativo:
PREPARACIÓN

INCUBACIÓN

ILUMINACIÓN (INSIGHT)

VERIFICACIÓN

2.
2.2.
2. C
Características
aracterísticas d dee la perso
personana creativa
o  Capacidad de problema
problematizar
tizar los contenidos conc
concretos
retos que se vincula
vinculann con las necesidades
necesidades,,
actitudes y sentimientos de las personas.
o  Capacidad de la persona pa para
ra producir una gran cantidad de ideideas
as respecto a un tem
temaa
determinado.
o  Capacidad del traslado de ideas de un cont contexto
exto a otro con gra
grann rapidez y frecuencia.
o  Habilidad para producir respues
respuestas
tas no comune
comuness a las situaciones pr
problemáticas.
oblemáticas.
o  Capacidad de utiliz
utilizar
ar de modo diferente lo común en base a la reorganiz
reorganización.
ación.
2.
2.3.
3. Nivele
Niveless de creatividad
creativid ad
Según Irving Taylor, se pueden distinguir cinco niveles de creatividad (desde el más elemental
al más complejo):
o  Creatividad expresiva: Se apoya en una actividad espontánea y libre, sin relación a

técnicas o aptitudes. Por ejemplo: los dibujos infantiles precoces.


o  Creatividad productiva: Dispone de técnicas y habilidades, aparecen restricciones

impuestas por el saber y se actúa con un propósito. El producto final posee un mayor
contenido comunicativo.
o  Creatividad inventiva: Se logran inventos y descubrimientos gracias al desarrollo de

relaciones novedosas
provocan sorpresa. Es ele nivel
interpretaciones
propio de la simbólicas,
l a ciencia que evidencian gran flexibilidad y
y el arte.
CEPRE-UNI HUMANIDADES 169

CICLO INTENSIVO ESCOLAR NACIONAL Material de Estudio Nº 1 

o   Creatividad innovadora: Se produce una modificación de principios, la cual refleja una


Creatividad
comprensión profunda del campo problemático. Los productos ya no se miden por el mundo
experimental del individuo, sino por muchos otros campos de la cultura.
o  Crea
Creativi dad emergente: Se crean nuevos principios. Es el nivel de mayor complejidad, ya
tividad
que no se modifican principios existentes, sino que se plantean nuevos parámetros, nuev
nuevas
as
formas de pensar. Algunos ejemplos: Freud en Psicología, Picasso en pintura, Wright en
Arquitectura y Einstein en Física.
EVALUACIÓN

1. Marque la alternativa que presenta una A) innovadora. B) productiva.


afirmación incompatible con respecto a la C) inventiva. D) emergente. E) experimental
experimental..
imaginación.
A) Es un proceso cognitivo que manipula la 4. Wilhelm Wundt es el creador del primer
información. laboratorio de psicología
de  experimenta
experimentall en
B) Crea una representación por los sentidos la 
la Universidad de Leipzig
Leipzig,,  y su discípulo
mentales. Titchener, desarrolló y sistematizó la psicología
C) Constituye un sistema inflexible que evita científica establecida por Wundt. ¿Qué nivel de
expresarnos. creatividad presentó Titchener?
T itchener?
D) Representa en la mente aquello que se ha A) Creatividad inventiva
percibido.
E) Crea representaciones del mundo real y de B)
C) Creatividad
Creatividadd productiva
Creativida emergente
la ficción. D) Creativida
Creatividadd innovadora
E) Creativida
Creatividadd expresiva
2. Marque la alternativa que presente un caso de
creatividad. 5. Matías es un niños de 6 años que le apasiona
A) Ante un problema, Elena presenta dibujar y usa lápices de colores, sus dibujos
propuestas no comunes. tiene un matiz artístico muy particular. ¿Qué
B) Daniela Quiroz cocina según el recetario de nivel de creatividad presenta Matías?
su progenitora. A) Creatividad inventiva
C) La imitación del baile no fue aceptada por el B) Creatividad emergente
riguroso jurado. C) Creativida
Cr eatividadd innovadora
D) Algunos psicólogos ppresentan
resentan postulados D) Creativida
Cr eatividadd productiva
ya establecidos. E) Creatividad expresiva
E) Diego decodifica la lección impartida por el
especialista. 1C 2A 3A 4C 5E
3. El arte varguardista modifica principios
establecidos, es decir, presenta creatividad
CEPRE-UNI HUMANIDADES 170

CICLO INTENSIVO ESCOLAR NACIONAL Material de Estudio Nº 1 

Razonamiento Verbal
SEMANA I: Definiciones / Analogías

Método de solución
Método sol ución
  Leer atentament
atentamentee la definición propuest
propuestaa identificando el género próximo y la di
diferencia
ferencia
específica.
  Leer cuidadosament
cuidadosamentee las opciones y selec
seleccionar
cionar aquella palabra que cum
cumpla
pla con los
componentes hallados en la definición.

Ejercicio
Ejercici o aplicativo: Coloque en los paréntesis la letra que corresponde a la definición de
cada palabra.
CEPRE-UNI HUMANIDADES 171

CICLO INTENSIVO ESCOLAR NACIONAL Material de Estudio Nº 1 

Método de solución
Método sol ución
  leer ate
atentamente
ntamente la analo
analogía
gía de la premisa o par base y hallar su rrelación
elación o vínculo.
  leer cuid
cuidadosamente
adosamente las oopciones
pciones de rela
relación
ción aanalógica
nalógica de la parte inferior y det
determinar
erminar
la que presenta el vínculo analógico similar al de la premisa o par base.

Ejercicio aplicativo: Entre las siguientes palabras halle rasgos afines y forme pares
analógicos.
Fórceps, comer, castidad, tenista, quirófano, cónclave, mar, hambre,
dentista, cirujano, sastre, bucanero, cardenal, poeta, pirata, doncella,
parnaso, dormir, oficina, fragata, aire, sueño, soldado, secretaria,
CEPRE-UNI HUMANIDADES 172

CICLO INTENSIVO ESCOLAR NACIONAL Material de Estudio Nº 1 

Tipo de relación analógica Pares


Pares analógico
analógicoss
1. _______________________
__________________________
___ _________: ________ :: ________ : ________
2. __________
______________________
________________
____ _________: ________ :: ________ : ________
3. __________
______________________
________________
____ _________: ________ :: ________ : ________
4. __________
______________________
________________
____ _________: ________ :: ________ : ________
5. __________
______________________
________________
____ _________: ________ :: ________ : ________
6. __________
______________________
________________
____ _________: ________ :: ________ : ________
7. __________
______________________
________________
____ _________: ________ :: ________ : ________

PRÁCTICA
I. DEFI
DEFINIC
NICIÓN
IÓN - Elija la palabr
palabraa que se defina
defin a con la
l a premi
premisa
sa plant
planteada.
eada.
1. _____: Hacer algo de pronto, sin estudio ni preparación.
A) Improvisar B) Estafar C) Innovar D) Repentizar E) Sublimar

2._____:Actitud
ofrecen y sacar de queellas
consiste en aprovechar
el mayor al máximo las circunstancias que se
beneficio posible.
A) Arribismo B) Avaricia C) Progreso D) Oportunismo E) Viveza
II. ANALOG
ANALOGÍA ÍA - Elija el par análogo al ppar
ar base escrito en mayúscul as.
3. LINGÜÍSTICA : LENGUAJE ::
A) citología: biología B) psicología: conducta C) ornitología : ornitorrinco
D) taxonomía: plantas E) axiología: moral
4. DESESPERACIÓN : ESTOICISMO ::
A) belleza: escatología B) placer: hedonismo C) religión : ateísmo
D) color: acromatismo E) decoro: cinismo

III. PRECISIÓN LÉXICA - Elija el término que, al sustituir la palabra subrayada, resulte
el más adecuado
adecuado para el contexto
context o plantea
plant eado.
do.
5. El apoyo del Estado puede incrementar el interés para iniciar más proyectos de gente
emprendedora.
A) dinamizar B) propiciar C) incrementar D) incentivar E) acelerar
6. Era muy lento para comprender algo o para resolver acertijos debido a la falta de práctica.
A) tozudo B) parsimonioso C) remolón D) aletargado E) obtuso
IV. ANTONIMIA CONTEXTUAL - Elija el término que, al sustituir la palabra subrayada,
exprese el significado opuesto de la oración.
7. Durante una exposición, el ssarcasmo
arcasmo de los concurrentes ppuede
uede infundir miedo en el
expositor.
A) seguridad B) furor C) temeridad D) aherrojo E) garbo
8. No fueron adecuadas las respuestas que dio en la entrevista, sino totalmente oportunas a
la coyuntura política.
A) malas B) falaces C) erradas D) impertinentes E) inexactas
CEPRE-UNI HUMANIDADES 173

CICLO INTENSIVO ESCOLAR NACIONAL Material de Estudio Nº 1 

V. CONECTORES LÓGICO TEXTUALES - Elija la alternativa que, al insertarse en los


espacios en bl anco, una adecuadamente
adecuadamente las ideas del texto.
9. ______ la exigencia profesional se incrementara, ______ hay un mayor desarrollo
cualitativo en las empresas; ______, urgiría replantear los programas de estudios
universitarios.
A) Si – dado que – entonces B) Aunque  –  pero  –  pues C) Porque –  si  –  pero
D) Ya que – y – aunque E) Para que – siempre que – además
10. El despecho es el resentimiento
resentimiento ______ responde a un disgusto que se sient
sientee por alguien
 ______ este le ccausó
ausó un desengaño u ofensa; ______, el despechado siente impulso de obrar
vengativamente.
A) o –ya que –verbigracia B) porque –aunque –sin embargo C) incluso –y –asimismo
D) y – a causa de que –  por eso E) solo si – es decir –  entonces

VI. INFORMACIÓN
INFORMACIÓN ELIMINADA
EL IMINADA  – Elija la oración que no es pertinente o es redundante
con el contenido
cont enido global del texto.
11. I. La preocupación de Gabriela Mistral por la maternidad y el reconocimiento de la
diferencia de los sexos le costaron la crítica de las feministas de su época. II. Las llamadas
feministas le atribuyeron una posición favorable al sistema patriarcal. III. La Mistral decía: "Yo
no
de creo
estashasta hoy enfeministas.
ʹderechasʹ la sonada IV.
igualdad
“Vacilomental
muchodeenlos sexos; suelo
contestar sentirme
con una por debajo
afirm ativa cuandoaún
se
me hace por la milésima vez la pregunta: ¿es Ud. feminista? Me parece más honrado
contestar un no escueto”, sostenía Gabriela Mistral. V. La Mistral afirmaba que hace años se
le invitó a pertenecer al feminismo y contestó: “Con mucho gusto,  cuando en el Consejo tomen
parte las sociedades de obreras, y sea así, verdaderamente nacional”. 
A) I B) II C) III D) IV E) V
12. I. El modo de proproducción
ducción esclav
esclavista
ista fue el primer modo basado en la explotac
explotación
ión que
aparece en la historia. II. El modo escl esclavista
avista alcanzó su máximo desarrollo en la Grecia
antigua y, sobre todo, en la Roma clásica. III. En el régimen esclavista las relaciones de
producción se basaban no solo en la propiedad de los dueños sobre los medios de
producción, sino también sobre los esclavos considerados como “instrumentos”. IV. Esta
sociedad se dividió en dos clases, los señores esclavistas y los esclavos, en la cual los
esclavos eran tratados como medios parlantes, sin derecho alguno. V. Para obtener el
dominio de los esclavos, este sistema de producción estructuró un aparato de violencia y
coerción, mediante el cual los contingente
contingentess de esclavos se abas
abastecían
tecían de las guerra
guerrass y con
los campesinos y artesanos arruinados.
A) I B) II C) III D) IV E) V

VII. PLAN DE REDACCIÓN - Elija la alternativa que contenga la secuencia correcta que
deben seguir loslo s enunciados para que la e estr
structu
uctura
ra del texto sea aadecuada.
decuada.
13. EL FIN DEL MONOPOLIO Y LOS PIRATAS
I. El monopolio comercial propició contrabandos y piraterías por parte de Inglaterra y Francia.
II. Los piratas más conocidos fueron los corsarios ingleses durante el reinado de Isabel I.
III. España permitía a Inglaterra compartir el mercado una vez al año en el tratado de Utrecht.
IV. Los piratas solían ser contratados por sus propios gobiernos en algunas circunstancias.
V. La amenaza al contrabando
contrabando surgió en 11713
713 cuando Españ
España a firmó el tratado de Utrecht
Utrecht..
A) II  ––IV –III  ––  –
I–V B) II  ––IV –III  ––  –
I–II C) I  ––IV –II  ––V –III D) I  ––II  ––IV –V –III E) II  ––IV –V –I  ––III
14. CONSECUENCIAS DE LA BLOBALIZACIÓN

CEPRE-UNI HUMANIDADES 174

CICLO INTENSIVO ESCOLAR NACIONAL Material de Estudio Nº 1 

I. La liberación del comercio y el libre tránsito del flujo de capitales ocurren, pues el Estado no
interviene como agente económico.
II. Los defensores de la globalización consideran que es sumamente beneficiosa, inevitable e
irreversible.
III. Las naciones menos desarrolladas, al estar en permanente contacto con el “primer mundo”
verán crecer sus economías.
IV. Beneficiosa es porque el desarrollo económico de los países industrializados se debe
propagar al resto del mundo.
V. Bajo las mismas reglas de libre mercado, los bienes y servicios deben ser producidos por
empresas privadas.
A) II  ––IV –V –III  ––I B) II  ––IV –I  ––III  ––V C) II  ––IV –III  ––I  ––V D) II  ––IV –III  ––V –I E) II  ––IV –I  ––V –III

VIII. INCLUSIÓ
VIII. INCLUSIÓN N DE IINF
NFORMAC
ORMACIÓN IÓN - Eli
Elija
ja la
l a oración
oraci ón que, al iinser
nsertars
tarse
e en el espaci
espacioo en
blanco, complete de manera
manera lógica la información global del texto.
15. I. La revolución indust
industrial
rial es el periodo comprendid
comprendidoo entre los años 1760 y 1840, en el
cual empezaron a utilizarse nuevas máquinas para la producción. II. Entre estas máquinas, la
de vapor fue considerada como principal fuente de energía. III. Esta máquina a vapor se usó
al principio en las fábricas textiles y siderúrgicas y, luego, se empleó para la navegación y el
transporte en ferrocarriles. IV. ________________. Fue revolucionario en el sentido de que
la política, la que
económicos sociedad y la vida de las personas evolucionaron a la par de los grandes cambios
trajo consigo.
A) Entre los cambios producidos por la revolución industrial se puede señalar la disminución
de la población agrícola.
B) Este fue un cambio tecnológico trascendental, que tuvo sus inicios en Inglaterra, y , más
tarde, se difundió por Europa y EE.UU.
C) En este periodo, hubo cambios significativos, como el aumento de la población urbana y
la mejora de la educación.
D) Esta máquina a vapor aceleró la economía en diferentes países industrializados o no
industrializados.
E) Esta tecnología compleja fue secedánea de una tecnología simple gracias al uso de
grandes máquinas.

16. I. Después de la fecundación interna, las hembras de los insectos depositan los huevos
en lugares como el agua. II. También los dejan en el suelo, plantas u otros animales donde
se desarrollan y crecen hasta la madurez. III. Los embriones jóvenes encerrados en los
huevos obtiene alimento del vitelo, intercambian gases y liberan deshechos. IV.
 ______________.
 ____________ __. V. La may
mayoría
oría de los ins
insectos
ectos experimenta metamorfosis completa: huevo,
larva, pupa y adulto.
A) Con el tiempo, el embrión sale del huevo, ya se alimenta y experimenta metamorfosis.
B) La metamorfosis
m etamorfosis presenta cuatro fases o etapas que se suceden progresivamente.
C) En la metamorfosis
m etamorfosis complet
completa,
a, un huevo fecundado se desarrolla primero en una larva.
D) Por una serie de cambios atraviesan las crías que se desarrollan fuera de la madre.
E) Para proteger a las crías, la metamorfosis en insectos es un proceso lento y gradual.

17. I. La penicilina comenzó a utilizarse de forma masiva durante la Segunda Guerra Mundial.
II. Eran circunstancias donde se hizo evidente su valor terapéutico. III. ____________
_______________.
___. IV.
En este sentido, se ha mostrado sumamente útil para combatir enfermedades como la
gonorrea y la sífilis. V. Sin embargo, ninguna de las penicilinas es efectiva para resfriados,
gripe u otras infecciones virales.

CEPRE-UNI HUMANIDADES 175

CICLO INTENSIVO ESCOLAR NACIONAL Material de Estudio Nº 1 

A) En ese momento, el uso de la penicilina fue crucial para evitar la muerte de los soldados.
B) Permite tratar, efectiva
efectivamente,
mente, muchas enfermedades relacionadas con la influenza.
C) En realidad, la penicilina ha permitido incrementar los índices de esperanza de vida.
D) De hecho, el modelo de la preparación de los antibióticos proviene de la penicilina.
E) Desde entonces, se usa eficazmente en el tratamiento contra gérmenes infecciosos.

IX. COHE
COHERENRENCIA CIA Y C COHE
OHESIÓ SIÓN N TE
TEXTU
XTUAL AL - Eli ja el or den co rrecto rr ecto que d deben
eben seguirsegu ir los
enunciados p ara que el texto sea coherente y cohesivo.
18. I. En la agricultura, los incas lograron organizar sus cultivos en zonas ecológicas. II. De
esta manera, su producción estaba controlada, era eficiente y de calidad, lo cual permitía la
domesticación de una mayor cantidad de plantas. III. En consecuenci consecuencia, a, los incas desarrollaron
un control vertical de pisos ecológicos”. IV. Es decir, sembraban sus cultivos en función del
clima que existía en cada piso ecológico según su altitud. V. Esta técnica de sembrío les
permitió no solo habilitar tierras, sino también aprovechar el agua que discurría por medio de
canales que conectaban los pisos.
A)   –
I–IV –V –III  ––II B) I  ––V –III  ––IV –II C) I  ––V –III  ––II  ––IV D) I-III  ––IV –II  ––V E) I  ––III  ––V –IV –II

19. I. En consecuencia, la identidad política supone la cuestión de la incertidumbre relativa a


los criterios de solución de conflictos. II
II.. En tal sentido, el tipo de solución de conflictos tiene
tiene
fines colectivos
permiten definiratérminos
largo plazo y de divergencia
fundamentales relativa.
en una III. LosIV.
sociedad. fines
Lascolectivos a largo
teorías de plazo
la justicia
presuponen siempre un reparto equitativo de la identidad política. V. Para ello, la solución de
conflictos entre intereses
intereses mensurables ssolo
olo es factible por una justi
justicia
cia negociable.
A) IV- II- I- V-III B) III-I-II-IV-V C) III-V-I-II-IV D) IV- I- II- V-III E) IV- I- V- II-III

X. COMPRENSIÓN DE LECTURA - Luego de analizar el texto, elija la alternativa que


contenga
cont enga la respuesta a cada pregunt
pregunta.a.
TEXTO 1
La enfermedad de Huntington es un desorden genético letal causado por un alelo autosómico
dominante poco común. Un desorden letal es una enfermedad que causa la muerte a la
mayoría de los individuos afectados. El sistema nervioso de una persona afectada de
Huntington se degenera progresivamente, lo cual resulta en movimientos incontrolados y
entrecortados de la cabeza y las extremidades y en deterioro mental. Actualmente, no existe
un tratamiento efectivo contra esta enfermedad.
20. Resulta compatible con el texto sostener que la enfermedad de Huntington
A) carece de un tratamiento para el paciente con este desorden.
B) presenta una gran incidencia de casos en la población.
C) es causado por un alelo autosomátic
autosomáticoo recesivo poco común.
D) causa la muerte de un minúsculo número de quienes la padecen.
E) deviene en la pérdida paulatina de las facultades mentales.

21. Es posible inferir que los pacientes del mal de Huntington


A) se mantendrán lúcidos siempre, a pesar de este mal.
B) no podrán ser curados, pues solo recibirán una terapia.
C) están exentos de padecer síntomas o sucumbir a la muerte.
D) carecen de un alelo autosomático dominante común.
E) siempre tendrán control de sus movimientos corporales.
22. Si el sistema nervioso de una persona afectada de Huntington no se degenerara, entonces

CEPRE-UNI HUMANIDADES 176

CICLO INTENSIVO ESCOLAR NACIONAL Material de Estudio Nº 1 

A) actualmente, no existiría un tratamiento efectivo para esta enfermedad.


B) esta enfermedad distaría de ser considerada un desorden genético letal.
C) el paciente no se experimentaría movimien
movimientostos involuntarios de la cabeza.
D) el cerebro del paciente Huntingt
Huntington
on sufriría daños progresivos e irreversibles.
E) es seguro que el deterioro mental se agudice en quien padece Huntington.

TEXTO 2
La Conquista significó también una agresión biológica que produjo una violenta caída
demográfica, un verdadero exterminio de la población aborigen, tanto por las condiciones de
trabajo y, en general, de vida, a las que fueron sometidos los indígenas. Así como por las
epidemias caudadas por las enfermedades europeas que alcanzaron proporciones
devastadoras y contra las cuales los indios no tenían resistencias biológicas específicas.
Los datos censales resultan dramáticamente elocuentes. Según estimaciones serias (N.D.
Cook), la población del Perú al momento del contacto debió ser aprox. 6 millones de
habitantes. Doscientos cincuenta años después, en la época del virrey Gil de Taboada y
Lemos (1795), se contaron 1076,122 habitantes y solo 608,712 indígenas, sin considerar la
audiencia de Quito. Si el crecimiento de la población hubiese sido normal, en esta última fecha
debió haber sido alrededor de 20 millones.
23. Resulta incompatible con el texto sostener que la Conquista
A)
B) fue un periodo
estuvo signadasangriento para la población
por tratos inhumanos indígena.
hacia los indígenas.
C) supuso la disminución significativa de la población aborigen.
D) conllevó una serie de situaciones adversas para los nativos.
E) significó una ganancia para colonizadores y colonizados.

24. Es posible inferir que la población nativa durante la conquista


A) careció de condiciones propicias para su desarrollo.
B) no se vio mermada por el trato de los colonizadores
colonizadores..
C) no contaba con las resistencias biológicas globales.
D) registraba aproximadamente 6 millones de habitantes.
E) fue la responsable de las enfermedades europeas
europeas..

25. Si los indios hubiesen contado con resistencias biológicas específicas, es probable que
A) la población aborigen se habría incrementado gradualmente durante la Conquista.
B) ningún tipo de epidemia se hubiese generado en el Perú de aquella época de conquista.
C) la Conquista no hubiese significado una agresión biológica para el pueblo colonizado.
D) no hubieran sucumbido con facilidad a las enfermedades europeas de los colonizadores.
E) sería implausible sostener la celeridad del descenso demográfico de los indios.

TEXTO 3
La filosofía es un conocimiento de la realidad diferente al de la ciencia. El filósofo contempla
la naturaleza, el hombre, el hecho mismo de conocer, la belleza, el bien y el mal, Dios, y se
pregunta sobre el sentido de todas estas realidades. La ciencia busca las causas de los
hechos y acontecimientos, pero para encontrar las leyes que los producen. La filosofía busca
las causas más lejanas, los fundamentos últimos de todas las cosas, y no lo hace para usar
esos conocimientos, para aplicarlos, como la ciencia y la técnica, sino para admirar y entender
al mundo y al propio hombre. 
26. En relación al conocimiento, según el texto, se puede afirmar que
A) suele presentar fines pragmáticos en la filosofía.

CEPRE-UNI HUMANIDADES 177

CICLO INTENSIVO ESCOLAR NACIONAL Material de Estudio Nº 1 

B) el filósofo y el científico tienen intereses afines.


C) podría
podría rev
revelar
elar la raz
razónón del ser en la realidad.
D) la técnica descubre las leyes de la naturaleza.
E) es imposible entender al hombre y al mundo.

27. Si la filosofía buscara los fundamentos últimos de todas las cosas para aplicarlos,
entonces
A) la ciencia se encargaría
encargaría de admirar el m mundo.
undo. B) el filósofo se centraría en ent
entender
ender al
hombre.
C) ya no contemplaría la naturaleza ni al hombre. D) la ciencia y la técnica habrían perdido
su fin.
E) su labor cognitiva sería igual al de la ciencia.

SEMANA II: Precisión léxica / Antonimia contextual

Emplear el lenguaje usando palabras


con su significado preciso no solo
cobra importancia sino que se hace
fundamental para que el mensaje
expresado sea completamente
comprendido por las demás
personas. Los ejercicios de
precisión léxica exigen el uso
puntual de los términos en el
contexto de la oración . No solo
debemos utilizar
utilizar palabras con sig
significado
nificado apropiado, sino también esp
específico,
ecífico, es decir,
palabras concisas y exactas que definan con la mayor concreción la idea que deseamos
expresar. Por ejemplo, veamos las siguientes dos oraciones:
  Carlos se rompió  el fémur jugando futbol. (El término «rompió» debe ser reemplaz
 reemplazado
ado por
«fracturó»).
  Diana rompió  la promesa que hizo a Javier. (El término «rompió» debe ser rreemplazado
 eemplazado
por «incumplió»).
Método
Mé todo de soluc ión
  Leer con atención la oración dada y entend
 entender
er de mane
manera
ra clara el context
contextoo en el cual está
inmersa la palabra subrayada.
  Elegir entre las opciones la palabra que vay
 vayaa más acord
acordee con el context
contextoo entendido y que
precise mejor el significado de lo que se desea comunicar.

Ejercicio
Ejercici o aplicativo: Sustituya las palabras subrayadas con otras que sean más precisas
según el contexto. 
1. Brasil venció a su oponente con gran facilidad. _____________
2. El director fue sacado de su cargo por inmoral. _____________
3. No diré el nombre del asesino. _____________
4. El profesor hizo los ejercicios en la pizarra. _____________
CEPRE-UNI HUMANIDADES 178

CICLO INTENSIVO ESCOLAR NACIONAL Material de Estudio Nº 1 

La antonimia es un caso de
vinculación semántica donde las
palabras presentan significado
opuesto o contrario. Esta relación de
oposición que se produce entre dos
palabras deben corresponder a una
misma categoría gramatical. Así
también, las palabras por sí mismas
no determinan su antónimo; por eso,
es necesario tomar en cuenta el
contexto. El objetivo de este tipo de
ejercicios es que el alumno desarrolle
su capacidad de abstracción, logre
una mayor precisión en el manejo de
vocablos y mejore su comprensión.

Método de solución
Método sol ución
• leer la oración en función de la palabra subraya
subrayada.
da. 
• pensar en un antónimo de dicho término que permita obtener una oración opuesta a la
planteada.
ontextual. 
• leer cada oración y elegir aquella que construya mejor una relación de antonimia ccontextual.
Ejercicio aplicativo: Elija el antónimo de la palabra en negrita que permita expresar el
significado opuesto de cada oración. 

atendió - derogó – leal - lúcida - mezquina

1. El nuevo presidente promulgó  muchas leyes. ____________


_____________________
_________
2. Su mente ofuscada lo llevaba a situaciones increíbles. __________
_____________________
___________
3. El pueblo ssabía
abía que se tratab
tratabaa de una familia generosa. _____________________
4. El tutor soslayó los reclamos de los alumnos. ____________
_____________________
_________
5. Fidel se caracterizaba por ser uunn hombre pérfido . _____________________

PRÁCTICA
I. DEFI
DEFINIC
NICIÓN
IÓN - Elija la palabr
palabraa que se defina
defin a con lla
a premi
premisa
sa plant
planteada.
eada.
1. _________: Que guarda modo y compostura en su conducta y ademanes.
A) Zalamero B) Prudente C) Circunspecto D) Modoso E) Cauteloso

2. _________: Disponer o preparar con astucia o dolo un enredo, engaño o traición.


A) Enzarzar B) Planificar C) Improvisar D) Tramar E) Conspirar

II. ANALOG
ANALOGÍA ÍA - Elija el par análogo al par base escrito en mayúscul
mayúsculas.
as.
3. DESACATO
DESACATO : SAN SANCIÓN
CIÓN ::::
A) suicidio: conmoción B) infracción: multa C) homenaje: aniversario
D) litigio: juicio E) muerte: luto
4. URÓLOGO: CISTITIS ::
A) dermatólogo: piel B) geriatra : vejez C) Veterinario: animal
D) psicólogo: bipolaridad E) oncólogo : melanoma

CEPRE-UNI HUMANIDADES 179

CICLO INTENSIVO ESCOLAR NACIONAL Material de Estudio Nº 1 

III. PRECISIÓN LÉXICA - Elija el término que, al sustituir la palabra subrayada, resulte
el más adecuado
adecuado para el contexto
context o plantea
plant eado.
do.
5. En el registro de Inforcorp, a pesar del transcurso del tiempo, hay deudas que no terminan.
A) finalizan B) vencen C) acaban D) rescinden E) prescriben

6. Muchos reos de cadena perpetua suelen recordar la vida en libertad junto a sus familiares
y amigos.
A) ansiar B) rememorar C) idealizar D) añorar E) soñar

IV. ANTONIMIA CONTEXTUAL - Elija el término que, al sustituir la palabra subrayada,


exprese el significado opuesto de la oración.
7. En el editorial del diario oficial se defendió al periodista honesto sindicado de extorsión por
funcionarios corruptos.
A) protegió B) vindicó C) ensalzó D) avaló E) amparó

8. Las condiciones climáticas pueden ser perjudiciales para viajar vía aérea.
A) malas B) benévolas C) buenas D) propicias E) promisorias

V. CONECTORES LÓGICO TEXTUALES - Elija la alternativa que, al insertarse en los

9. espacios
______ elenapoyo
b lanco,
blanco,
erauna adecuadame
adecuadamente
total, ______ su nte las idea
carismai deas
eras evidente,
del ttexto.
exto. se sentía aislado; ______,
renunció a su cargo.
A) Aunque – porque – por eso B) Si  –  aunque  –  pero
pero C) A pesar de que –  y  –  y
D) Puesto que – pero – es decir E) Ya que – y – aunque

10. La avenida principal estaba libre, ______ todos los ambulantes habían sido desalojados.
 ______, muchos querían retomarla a la fuerza, ______ pusieron a un grupo policial para
vigilarlos.
A) pues –No obstante –ya que B) porque –Sin embargo –ergo C) ya que –Pero –asimismo
D) conque – Además –  por eso E) a causa de que – Incluso –  y

VI. INFORMACIÓN EL IMINADA  – Elija la oración que no es pertinente o es redundante


INFORMACIÓN ELIMINADA
con el contenido
cont enido global del texto.
11. I. El Premio Nobel se otorga cada año a personas que realicen descubrimientos
sobresalientes. II. También
También se otorga en caso de una contribución notable a la sociedad en el
año anterior. III. El premio no puede ser otorgado póstumamente, ni ser compartido por más
de tres persona. IV. Con este reconocimiento se recibe una medalla de oro, un diploma y una
suma de dineros. V. El Premio Nobel se concede a científicos o personalidades vivos, nunca
post mortem.
A) III B) I C) IV D) I E) V

12. I. El hemisferio izquierdo se separa ddel


el hemisferio derecho median
mediantete una cisura sagital
profunda localizada
localizada en la línea media del cerebro. II. El hemisfe
hemisferio
rio cerebral izquierdo se
encarga de la planificación y ejecución de nuestras actividades. III. El hemisferio izquierdo se
relaciona directamente con el manejo de información lógica para la toma de decisiones. IV.
Este hemisferio almacena ideas en un lenguaje no verbal para traducirlas a un idioma
aprendido por el individuo. V. El hemisferio cerebral izquierdo permite la comprensión de
aspectos lógicos gramaticales del lenguaje y la organización de la sintaxis.
A) V B) II C) I D) III E) IV

CEPRE-UNI HUMANIDADES 180

CICLO INTENSIVO ESCOLAR NACIONAL Material de Estudio Nº 1 

13. I. El Estado de Emergencia está regulado por la Constitución Política del Perú; y es


oficializado por el presidente de la República,
República, en acuerdo con los minist
ministros
ros de Estado. II. Una
solicitud de declaratoria de Estado de Emergencia debe seguir el procedimiento establecido
por el decreto supremo 058-2001-PCM. III. La autoridad del Comité de Defensa Civil del
gobierno local (provincial o distrital) canaliza su pedido mediante el Comité Regional de
Defensa Civil. IV. Luego, el Comité Regional de Defensa Civil evalúa la viabilidad del pedido
y, si lo aprueba, este es derivado al Instituto Nacional de Defensa Civil (Indeci). V. La
declaración oficial de Estado de Emergencia conlleva a la restricción de algunos derechos,
como libertad de tránsito, inviolabilidad
i nviolabilidad de las comunicaciones
A) II B) IV C) I D) V E) III

VII. PLAN DE REDACCIÓN - Elija la alternativa que contenga la secuencia correcta que
deben seguir lo s enunciados para que la e estr
struct
uctura
ura del texto sea a adecuada.
decuada.
14. EL ORIGEN DE LOS SISMOS
I. Se pueden evitar consecuencias fatales a través de la participación en simulacr
simulacros
os de
sismo.
II. Los sismos son fue
fuertes
rtes perturbac
perturbaciones
iones en el interior de
dell planeta y originan movimientos.
III. Grandes ciudades fueron devastadas por los sismos de alta intensidad en algunas
regiones.
IV. La intensidad del rompimiento es directamente proporcional a los daños ocasionados ocasionados..
V. Las alteraciones se se producen por la ruptura de rocas en las capas interiores de la tierra.
A)   –
I–II  ––IV –III  ––V B) II  ––V –IV –III  ––I C) II  ––IV –V –III  ––I D) I  ––II  ––V –IV –III E)   –
I–II  ––III  ––IV –V

15. EL MARAVILLOSO TEMPLO DE TINTIRI


I. El templo está situado en Puno y es una de las edificaciones más imponentes de la región.
II. Construido en 1860 por creativas manos indígenas, Tintiri ostenta una verdadera belleza.
III. Desde afuera, a pesar de los mordiscos del tiempo refleja una impactante im pactante majestuos
majestuosidad. idad.
IV. A diferencia de las iglesias de esta región, no usaron piedra pulida para su construcción construcción..
V. El interior, sin sin embarg
embargo, o, está casi en ruinas, con vizcachas profanando este lugar sacro.
A) II  ––I  ––IV –III  ––V B) I  ––V –III  ––II  ––IV C) I  ––IV –II  ––III  ––V D) II  ––  –
I–V –III  ––IV E) I  ––IV –III  ––II  ––V

16. CAUSAS DE LA MIOPÍA


I. Esto sucede debido a dos condiciones físicas que pueden ser de origen hereditario.
II. Una visión nítida requiere que estos rayos se enfoquen exactamente sobre la retina.
III. Se observa, generalmente, una córnea demasiado curvada curvada o un globo ocular grande.
IV. Los rayos de luz que penetran en el ojo son refractados por la córnea y el cristalino.
V. Lo que sucede, sin embargo, en el ojo miope es que la luz se enfoca delante de ella.
A) II  ––III  ––  –
I–V –IV B) IV –II  ––V –I  ––III C) II  ––IV –V –I  ––III D) IV-II  ––V –III  ––I E) IV –II  ––III  ––I  ––V

VIII. INCLUSIÓ
VIII. INCLUSIÓN N DE IINF
NFORMAC
ORMACIÓNIÓN - Eli
Elija
ja la
l a oración
oraci ón que, al iinser
nsertars
tarse
e en el espaci
espacioo en
blanco, complete de manera
manera lógica la información global del texto.
17. I. Previo a realizar una toma, se configura la cámara para ajustar la calidad de la imagen
que proyectará sobre el material fotosensible. II. Al dispararse el obturador, dicho material es
finalmente expuesto, provocando en él alteraciones químicas o físicas. III. Estas reacciones
dan como resultado una «imagen latente», aún no visible pero presente en su estructura
interna. IV. V. Completada esta etapa, se procede a un
lavado de la película para eliminar los res
restos
tos de product
productos
os químicos.
A) La imagen latente de la película muestra un bosquejo de las característic
características
as de la foto.

CEPRE-UNI HUMANIDADES 181

CICLO INTENSIVO ESCOLAR NACIONAL Material de Estudio Nº 1 

B) En esta etapa, se tomarán en cuenta las condiciones ambientale


ambientaless que puedan influir.
C) Esta debilísima imagen se somete a un proceso de intensificació
intensificaciónn química o revelado.
D) La imagen puede no tener las propiedades deseadas, ento entonces,
nces, se retomará el proceso.
E) En esta estructura,
estructura, se deben realiza
realizarr varias tomas a fin de obt
obtener
ener imágenes latente
latentes.
s.

18. I. La saponificación es una reacción química entre un éster de ácido graso y una base o
alcalino. II. ______________________________. III. Es decir, tienen una parte polar y otra
apolar (o no polar), con la cual pueden reaccionar. IV. Como producto de este proceso se
obtiene como principal producto la sal de dicho ácido. V. Por ejemplo, los jjabones
abones son sales
de ácidos grasos que se obtienen mediante esta reacción.
A) Estos pueden interactuar con sustancias de propiedades dispares pues son anfipáticos.
B) Este método consiste en hervir la grasa, añadiendo lentamente soda cáustica (NaOH).
C) La reacción química industrialmente obtiene dos conocidos productos: jabón y glicerina.
D) Los ácidos grasos son biomoléculas lipídicas formadas por cadenas de hidrocarburos.
E) La base química es una sustancia que en disolución acuosa aporta iones OH− al medio. 

IX. COHE
COHERENRENCIA
CIA Y C
COHE
OHESIÓ
SIÓN N TE
TEXTU
XTUAL
AL - Eli ja el or den co rrecto
rr ecto que d
deben
eben seguir
segu ir los
enunciados p ara que el texto sea coherente y cohesivo.
coh esivo.
19. I. Por este motivo, la cantidad de combinaciones es inmensa y aún no se conocen todas.
II. Se pueden
componen. agruparreaccionan
III. Estos teniendo entre
en cuenta los elementos
sí al interior químicos
de la misma, en da
lo que común
lugarque
a estalos
formación. IV. Sin embargo, como no todos son susceptibles de tal agrupación existen otros
criterios. V. Debido a que constituyen parte de la corteza terrestre, algunos minerales se han
formado a partir de elementos químicos.
A) V-III- I-II-IV B) V-I-II-IV-III C) II-IV-I-V-III D) II- IV- III-V-I E) V-II-I-IV-III

20. I. Para ello, existen diversas técnicas de ingeniería genética utilizadas para aislar
segmentos de ADN. II Es decir, son manipulados en laboratorios para otorgarle una
característica específica.
específica. III. Otras se basan en el bombardeo de células con micropartículas
recubiertas de ADN. IV. Los transgénicos son organismos creados artificialmente y
modificados a nivel genético.V. Entre ellas, es común usar como vector otro ser vivo capaz
de inocular fragmentos de ADN.
A) IV-I-III-V-II B) IV-II-I-V-III C) IV-III-II-I-V D) IV-II-I-III-V E) IV-II-V-I-III

X. COMPRENSIÓN DE LECTURA - Luego de analizar el texto, elija la alternativa que


contenga
cont enga la respuesta a cada pregunt
pregunta. a.
TEXTO 1
Que el sistema capitalista mundial atraviesa una crisis profunda no debería ser una conclusión
que, quienes creemos en él, debamos negar como si en ello se nos fuera la vida.
Pero lo que hay que tener claro es qué está causando esa crisis. ¿Parte de la evolución
intrínseca al capitalismo?, como señalaba Carlos Marx ¿Crisis terminal? Sí, pero no del
capitalismo a secas.
Es la perversión mercantilista del capitalismo, el alejamiento de sus fuentes libertarias, su
mirada de soslayo de las formas democráticas, su consecuente sometimiento a los intereses
de las grandes corporaciones. Ese modelo está muriendo. Por sí solo y por obra de la protesta
social que, quizás no sea plenamente consciente de los orígenes de su malestar, pero que
en el fondo siente que se le ha arrebatado la soberanía, que ya no es el pueblo el que manda
en el el
si no fondo
gransiente que se le ha arrebatado la soberanía, que ya no es el pueblo el que manda
capital.
TAFUR, J. C. ¿Qué está en crisis en el mundo?

CEPRE-UNI HUMANIDADES 182

CICLO INTENSIVO ESCOLAR NACIONAL Material de Estudio Nº 1 

21. Es incongruente con el texto afirmar que


A) el capitalismo no está dirigido a colaborar activamente con la democracia.
B) las grandes corporaciones han sometido al capitalismo bajo sus intereses.
C) las protestas sociales manifiestan una conciencia absoluta de su malestar.
D) actualmente, el gran capital es el que manda en el gobierno de los países.
E) el capitalismo está muriendo por sí solo y a causa de las protestas sociales.

22. Se infiere del texto que, según el autor, el sistema capitalista


A) ya no posee la soberanía de su esencia ideológica como tal.
B) es indiferente a las pretensiones de grandes corporaciones.
C) forma parte del proceso económico de evolución mundial.
D) experimenta una crisis innegable al despojarse de sus pilares.
E) presenta una brecha abismal con el mercantilismo liberal.

23. Si los partidarios del sistema capitalista reconocieran la profunda crisis que atraviesa este
sistema, entonces
A) la postura de Carlos Marx resultaría contradictoria.
B) estos podrían identificar las causas de esta crisis.
C) el capitalismo mercantilista no se habría pervertido.
D) estos
E) las partidarios
grandes se aferrarían
corporaciones al progreso
no serían capitalista.
responsables.

TEXTO 2
No toda investigación científica procura el conocimiento objetivo. Así, la lógica y la matemática
-esto es, los diversos sistemas de lógica formal y los diferentes capítulos de la matemática
pura- son racionales, sistemáticos y verificables, pero no son objetivos, no nos dan
informaciones acerca de la realidad: simplemente, no se ocupan de los hechos. La lógica y la
matemática tratan de entes ideales; estos entes, tanto los abstractos como los interpretados,
i nterpretados,
solo existen en la mente humana. A los lógicos y a los matemáticos no se les da objetos de
estudio: ellos construyen sus propios objetos. Es verdad que a menudo lo hacen por
abstracción de objetos reales (naturales y sociales); más aún, el trabajo del lógico o del
matemático satisface a menudo las necesidades del naturalista, del sociólogo o del tecnólogo,
y es por eso que la sociedad los tolera y, ahora, hasta los estimula. Pero la materia prima que
emplean los lógicos y los matemáticos no es fáctica sino ideal.
BUNGE, M. La ciencia, su método y su filosofía.
24. Una idea compatible con el texto es que
A) los objetos de estudio de los
l os lógicos pueden ser de naturaleza concreta.
B) los resultados logrados por los matemáticos no pueden ser interpretados.
int erpretados.
C) los estudios de los lógicos
l ógicos contribuyen a los realizados por el sociólogo.
D) los sistemas de lógica formal son sistemáticos, verificables y objetivos.
E) las investigaciones cientí
científicas
ficas se enfocan en los conocimientos objetivos.

25. Se infiere del texto que los matemáticos y los lógicos


A) están orientados a la creación de sistemas lógicos para conocer la realidad circundante.
B) se dedican a la investigación de lo que su ciencia predetermine como objeto de estudio.
C) se caracterizan por poseer un pensamiento formal que les permite operar con símbolos.
D) satisfacen
E) debido a lalas necesidades
necesidades
complejidad de prof
profesionales
científica esionales
de como
su método, natu
naturistas,
selecc ristas,objetos
seleccionan
ionan sociólogos y tecnólogos.
de estudio fácticos.

CEPRE-UNI HUMANIDADES 183

CICLO INTENSIVO ESCOLAR NACIONAL Material de Estudio Nº 1 

26. Es posible colegir que el autor del texto tiene la intención de


A) informar la manera como se produce el conocimiento objetivo a través de hechos.
B) describir las características esenciales de los diversos sistemas de la lógica formal.
C) identificar la trascendencia del proceso de abstracción en las ciencias formales.
D) refutar los modelos teóricos utilizados por investigadores de las ciencias fácticas.
E) aclarar cuál es la naturaleza del conocimiento procedente de la lógica y la mmatemática.
atemática.

SEMANA
SEMANA II
III:
I: Conectores ló
lógico
gico -textuales / Información
Inform ación elim
eliminada
inada

La coherencia lógica y la concatenación de las oraciones que forman parte de un texto se


logran a través de un uso adecuado de los conectores lógicos . En este sentido, estos son
palabras o expresiones que marcan relaciones entre ideas. Existe, así, una gran variedad de
relaciones lógicas y, en esa medida, también existen conectores lógicos que las expresan. A
continuación, se presenta una clasificación de los conectores más importantes.
Conectores causales   Preceden al motivo o a la razón de lo
expuesto antes del conector.
porque, pues, ya que, puesto que, debido a, como consecuencia de, en vista de (que)
El congresista no asistió a la reunión, puesto que no estaba de acuerdo con el presidente.

ctores consecutivos  
Conectores
Cone Anuncian, después de ellos, un resultado de
la idea anterior.
entonces, en conclusión, por esta razón, por (lo) tanto, por eso, por ello, en consecuencia
Usted ha seguido una dieta inadecuada,
i nadecuada, por esa raz ón  deberá someterse a un tratamiento.
razón

Conectores adversativos   Señalan relaciones de oposición total.


más
Está bien, sin embargo,
arrepentido nolos
de todos obstante, por el contrario,
delitos cometidos; en cambio,
sin embargo,  nopero, sinode actitud.
cambia

ctores concesivos  
Conectores
Cone Expresan una dificultad que no impide el
cumplimiento de un hecho.
aunque, aun cuando, aun si, si bien, a pesar de que, pese a que
La prensa no colaborará con la difusión del evento, a pesar
pesar de qu
quee este sea benéfico.

ctores copulativos y aditivos  


Conectores
Cone Se usan para aumentar información del
mismo nivel.
y (e), ni, también, además, asimismo, incluso, inclusive, es más
Los alumnos del Cepreuni obtendrán una vacante, además , serán premiados por su
ingreso.
Conectores
Cone disyunt ivos  
ctores disyuntivos Se utilizan para señalar la posibilidad de
elegir entre dos o más opciones.

CEPRE-UNI HUMANIDADES 184

CICLO INTENSIVO ESCOLAR NACIONAL Material de Estudio Nº 1 

o (u), o bien, ya…ya , sea… sea 


El mandatario abrazará la postura idealista o  se alejará de la promesa del cambio.

Conectores explicativos   Explican la idea planteada en la primera


proposición.
en otras palabras, en otros términos, es decir, esto es, o sea
Es un hombre misántropo; es decir  no
 no le gusta la humanidad ni relacionarse con ella.

Conectores de ejemplificación   Se usan para señalar que se está


presentando un caso particular de una idea
general.
por ejemplo, a modo de ilustración, pongo por caso, verbigracia, como
Las aves de gran tamaño suelen haber perdido la capacidad de volar; por ejemplo, los
avestruces son buenos corredores, pero no vuelan.

Expresan el fin o la intención con que se


Conectores de finalidad   produce la acción del verbo principal.
para que, a fin de que, con el objeto de que, con la finalidad de (que)
Todas las ONG inscritas organizaron galas benéficas con la finalida d de que los orfanatos
fi nalidad
patrocinados reciban los recursos recaudados.
Método
Mé todo de solución
sol ución
  Leer de
detenidamente
tenidamente la oració
oraciónn pres
presentada
entada analizando el cont
contenido
enido ddee ca
cada
da uno de lo
loss
segmentos.
  Definir los tipos de relación que existen entre los segmentos mencionados.
  Establecer el ororden
den en el que deben aparecer los cconectores
onectores según su ttipo.ipo.
  Leer las alternativas y hallar la que contenga los conectores en bbase
ase al orde
ordenn es
establecido.
tablecido.

Ejercicio aplicativo: Elija la alternativa que, al insertarse en los espacios en blanco, dé


sentido coherente y preciso al texto.
Los bombardeos en el Medio Oriente continuaron _____ la comunidad internacional se había
pronunciado enérgicamente _____ cesen las hostilidades entre los países; _____, hicieron
caso omiso.
A) a pesar de que – para que –así B) con el fin de que – no obstante – es decir
C) pues – siempre que – por eso D) pese a que – a fin de que –sin embargo
E) porque – esto es – en cambio

Los ejercicios de información eliminada  son pruebas


pertinentes para evaluar la capacidad de discernimiento, la
 jerarquización de datos y la economía lingüística. Un
ejercicio de este tipo está conformado por cinco enunciados:
cuatro están cohesionados en torno a un tema específico y
uno resulta no pertinente. Para la eliminación de esta
información se recurre a los dos siguientes criterios básicos:
CEPRE-UNI HUMANIDADES 185

CICLO INTENSIVO ESCOLAR NACIONAL Material de Estudio Nº 1 

CRIT
CRITER
ERIO
IOS
S DE ELI
ELIMI
MINA
NACI
CI N DE
DE INFO
INFORM
RMAC
ACII N

REDUNDANCIA IMPERTINENCIA

que reitera que aporta una información


innecesariamente una ajena o contradictoria con el
idea ya incluida en otros tema que se desarrolla en el
enunciados. texto.

Método de solución
Método sol ución
  Leer la secuenc
secuencia
ia de oraciones que present
presentaa el ejercicio e identificar el ttema
ema central.
  Determinar, en una ssegunda
egunda lectura, la oración que resulta redundante o imp impertinente,
ertinente, de
acuerdo con el tema hallado.

Ejercici
Eje rcicio
redundanteo aplicativo: Lea las cinco oraciones, determine el tema y elimine la
o impertinente. l a oración que es
I. El emisor y el receptor son elementos de la comunicación. II. El emisor es el elemento de
la comunicación que haciendo uso del código, emite el mensaje. III. El canal es el medio físico
a través del cual se envía el mensaje. IV. El contexto es otro elemento fundamental de la
comunicación. V. El receptor es el elemento que recibe la información y también se le llama
descodificador.

Tema:__________________Oración
Tema:__________________Oración eliminada: ______ Criterio de eliminación___________

PRÁCTICA

I. DEFINICIONES - Elija la palabra que se defina con la premisa premis a planteada.


planteada.
1. a. Gran aabundancia
bundancia de alg
algo.o.
b. Abundancia suma, plenitu
plenitudd ext
extraordinaria.
raordinaria.
c. Abundancia, riqueza y sobra de bienes.
i. Exuberancia ii. Opulencia iii. Plétora
A) ib – iia – iiic B) ib – iic – iiia C) ic – iia – iiib D) ia – iic – iiib E) ic – iib – iiia

2. _________: Hábito o costumbre de mentir.


A) Mitomanía B) Patraña C) Embuste D) FalsificaciónE) Mendacidad

3._______:Acto
3.______ _:Acto por el que el fiscal o un particular ejercen ante un juez o un tribunal
la acción penal contra quienes se estiman responsables de un delito.
A) Memorando B) Sindicación C) Querella D) Imputación E) Dictamen

II. ANALOGÍAS - Elija el par análogo


análogo al par base escr
escrito
ito en mayúsc ulas.
4.
A) PAYASO:
misionero REÍR::
: convertir B) bailarina : danzar C) arlequín : remedar
D) púgil: boxear E) poeta: inspirar

CEPRE-UNI HUMANIDADES 186

CICLO INTENSIVO ESCOLAR NACIONAL Material de Estudio Nº 1 

5. SANGRE : ANIMAL ::
A) gasolina : auto B) combustible : máquina C) xilema : arbusto
D) savia : planta E) electricidad : cable

6. ESTUDIAR : EVALUACIÓN ::
A) curar: enfermedad B) aconsejar: orientación C) diagnosticar: terapia
D) ensayar: audición E) celebrar: éxito

III. PRECISIÓN LÉXICA  - Elija el término que, al sustituir la palabra subrayada, resulte
el más adecuado
adecuado para el contexto
context o plantea do.  
plant eado.

7. Las estrategias usadas por los abogados de Paolo Guerrero no le dieron la absolución que
deseaba.
A) provocaron B) propiciaron C) produjeron D) generaron E) dirigieron

8. Una nueva investigación sobre el cáncer refuerza la tesis de que este mal no es una
enfermedad específica.
A) corrobora B) garantiza C) avala D) determina E) dilucida

9. La castración química frena la producción de testosterona, pero es reversible y pasajera.


A) obstaculiza B) altera C) disminuye D) inhibe E) restringe
IV. ANTONIMIA CONTEXTUAL  - Elija el término que, al sustituir la palabra subrayada,
exprese el significado opuesto de la oración.
10. Los rescatistas continuaron la búsqueda del joven desaparecido. 
A) invalidaron B) perpetuaron C) suspendieron D) archivaron E) denegaron

11. Ante la prensa, un candidato adversario, musitó algunos insultos en contra del candidato
que ganó las elecciones municipales.
A) exclamó B) profirió C) vociferó D) pronunció E) injurió

12. Algunas actividades empresariales revelan la capacidad de las personas. 


A) ineptitud  B) ignorancia C) debilidad D) necedad E) temeridad

V. CONECTORES LÓGICO TEXTUALES  - Elija la alternativa que, al insertarse en los


espacios en bl anco, una adecuadamente
adecuadamente las ideas del texto.
13. _____ su primera novela fue un rotundo fracaso, _____ el tema desarrollado era trivial,
escribió una segunda novela, _____, esta vez, no la publicó.  
A) Aunque – es decir – en cambio B) Si – en otras palabras –  por ello
C) No obstante – o sea – además D) A pesar de que – pues – pero
E) A causa de que – y – entonces

14. _____ recién empezaba su tra trabajo


bajo en la empresa, sobresalió rápida
rápidamente.
mente. _____ se
adaptó al ambiente laboral, logró, _____, que su jefe le diera un aumento.
A) Sin embargo – Entonces  – así B) Puesto que – Asimismo – también
C) Aunque – Dado que – incluso D) Por ello – Si bien – por otra parte
E) Pero Incluso además

CEPRE-UNI HUMANIDADES 187

CICLO INTENSIVO ESCOLAR NACIONAL Material de Estudio Nº 1 

15. _____ haya


haya muchos coconflictos,
nflictos, el ma
matrimonio
trimonio debe ser uuna na unión permanen
permanente,
te, ____
implica un profundo compromiso.
compromiso. _____, cuando ssurjan
urjan problemas es crucial solu
solucionarlos.
cionarlos.
A) Así como – pues  – Es decir B) No obstante – entonces – Sin embargo
C) Aunque – porque – Aun así D) Así – solo si  –– Entonces
E) A pesar de que – ya que – Por eso

VI. INFORMACIÓN
INFORMACIÓN ELIMINADA
EL IMINADA  – Elija la oración que no es pertinente o es redundante
con el contenido
cont enido global del texto.
16. I. La anhedonia es la incapacidad de sentir dicha o gusto durante las actividades
placenteras y se produce como parte de una serie de condiciones. II. La mayoría de las
personas, en algún momento de su vida, perderá interés en las cosas que solían producirle
gran deleite o placer. III. La anhedonia lleva esta pérdida al límite: se vuelve imposible
disfrutar de las cosas que alguna vez provocaron felicidad, como la música, el
el sexo,
 sexo, la
 la comida
o una buena conversación. IV. La incapacidad de sentir placer ante situaciones que solían
ser satisfactorias podría estar motivada por un trastorno depresivo mayor o por otras
afecciones como la esquizofrenia, la psicosis y la enfermedad de Parkinson. V. La anhedonia
no es simplemente una apreciación reducida del sabor del chocolate (por ejemplo), sino
que los mecanismos de recompensa subyacentes están deteriorados.
A) I B) II C) III D) IV E) V

17. I. Un estudio de la Universidad Bamako (Malí) realizado entre 5 mil personas concluyó
que los fanáticos del reguetón son menos inteligentes y tienen un coeficiente intelectual
bajo. II. Según este estudio, a las personas que les gusta el ritmo del reguetón son, en
promedio, 20% menos inteligentes que otras que escuchan géneros como la música clásica
o el rock. III. Para llevar a cabo el estudio, las personas entrevistadas fueron sometidas a
distintas pruebas. IV. A los participantes del estudio se les midió su CI, se les aplicó test sobre
conocimientos y preferencias musicales, luego pasaron por una entrevista social. V. De las
personas entrevistadas,
entrevistadas, las que gustan del rock y de la música clásica demostraron ser mucho
más receptivos con su ambiente y mostraron una capacidad de crítica superior a los amantes
del reguetón, quienes parecían tener expresiones y respuestas
r espuestas más simples.
A) I B) II C) III D) IV E) V

VII. PLAN DE REDACCIÓN - Elija la alternativa que contenga la secuencia correcta que
deben seguir los lo s enunciados para que la e estr
structu
uctura
ra del texto sea aadecuada.
decuada.
18. ¿QUÉ ES UN ECLIPSE SOLAR PARCIAL?
I. La órbita de la Luna está ligeramente inclinada respecto a la de la Tierra, por lo que a veces
se mueve por encima o por debajo de la cara del planeta.
II. No es tan impresionante su efecto, pero igual llama el interés y puede ser visto por más
espectadores.
III. Un eclipse solar parcial ocurre de dos a cuatro veces al año y crea un efecto menos
impactante que un eclipse solar total.
IV. La Luna se interpone entre la Tierra y el Sol cada mes, pero no siempre se alinean para
producir eclipses.
V. Un eclipse solar total ocurre cuando la Luna oculta al Sol, de lo cual se proyecta una sombra
en la Tierra.
A) I-IV-III-V-II B) IV-I-V-III-II C) I-IV-II-V-III D) IV-II-V-I-III E) I-II-IV-V-III
19. MIEDO DE LOS VARONES A LAS MUJERES INTELIGENTES

CEPRE-UNI HUMANIDADES 188

CICLO INTENSIVO ESCOLAR NACIONAL Material de Estudio Nº 1 

I. Tras el estudio, los investigadores entendieron que mientras las féminas buscan hombres
inteligentes con una mente clara y aguda, ellos las preferirían menos brillantes. 
II. No se trata de una fobia, de hecho pueden llegar a ser muy buenos amigos, sin embargo
no intentarían entablar una relación sentimental con ellas.
III. Los especialistas de la Universidad de Nottingham (Inglaterra) fueron los encargados de
revelar por qué los hombres le tendrían pánico a las mujeres inteligentes. 
IV. El estudio concluyó que ellos preferirían a una mujer con un parecido al de su propia madre
para que él no tenga dificultades en ser un macho alfa.
V. La investigación también reveló que las mujeres autosuficientes e inteligentes, además de
ser solitarias, son más difíciles de conquistar para los varones.
A) III-II-I-V-IV B) I-V-IV-II-III C) I-III-II-V-IV D) I-IV-V-I-III E) III-I-II-IV-V

VIII. INCLUSIÓ
VIII. INCLUSIÓN N DE IINF
NFORMAC
ORMACIÓNIÓN - Eli
Elijaja la
l a oración
oraci ón que, al iinser
nsertars
tarse
e en el espaci
espacioo en
blanco, complete de manera
manera lógica la información global del texto.
20. I. De entre las más de 7 mil
mil patologías de baja prevalencia existentes, la hemofilia es una
enfermedad poco común
común que afecta cas
casii en su totalidad a hombres. II. La enfermedad es está

ligada al cromosoma X, de tal manera que son las mujeres (cromosomas XX) las portadoras
de la hemofilia, pero son los hombres (cromosomas XY) quienes desarrollan los síntomas. III.
 ___________. IV. Generalmente, estas presentan niveles de hemofilia más bajos que los
afectados, por lo
cura definitiva, losque no necesitarían
avances científicossometerse
y médicosahan
un tratamiento
permitido elV.desarrollo
Aunque node existe una
diferentes
tratamientos para combatirla.
A) Tan solo en casos excepcionales se han presentad
presentadoo casos de mujeres afectadas de igual
forma que los varones que la padecen.
B) Es de origen congénito y se produce por una deficiencia de proteínas que intervienen en
el sistema de coagulación de la sangre.
la sangre.  
C) Este tipo de enfermedad se manifiesta en la persona a través de hemorragias frecuentes,
que pueden ser internas o externas.
D) Es considerada también una enfermedad incapacitant
incapacitante,
e, pues en la mayoría de los casos
se producen daños articulares.
E)
E) El
 El sexo femenino es el portador de la hemofilia, pero son los hombres quienes padecen
los síntomas de la enfermedad.

21. I. ______________. II. El nombre comercial era Enovid y su indicación, insólita hasta la
fecha: prevenir el embarazo tras una relación sexual. III. Pasaron los años y lo que pronto se
conoció con el nombre genérico de la píldora -en España
España no se legalizó hasta 1978. IV. No
obstante, se convirtió en un fármaco habitual en el botiquín de millones de mujeres para
evitar la maternidad de forma reversible
reversible.. V. Más de medio siglo despu
después
és de la aprobación
de Enovid, ningún científico ha conseguido desarrollar un fármaco eficaz equivalente para el
varón.
A) Un grupo de científicos present
presentóó una nueva píldora anticonceptiva para hombres al parecer
segura cuando se usa diariamente durante un mes.
B) Científicos japoneses demuestran en ratones una posible estrategia para encontrar la
píldora anticonceptiva masculina.
C) En 1960, la FDA, organismo que regula los fármacos en EEUU, aprobó un medicamento
que marcó un hito en la historia del feminismo
D) El Enovid
aprobó 10 de  para
juniouso
depor1957, la Administración
la 
trastornos menst ruales. de Alimentos y Medicamentos (FDA)
menstruales.

CEPRE-UNI HUMANIDADES 189

CICLO INTENSIVO ESCOLAR NACIONAL Material de Estudio Nº 1 

E) En la década de 1930, los científicos descubrieron que altas dosis


de andrógenos,
de  andrógenos, estrógenos
 estrógenos o progesterona inhibían la
la ovulación.
 ovulación.  

22. I. El concepto de amor en Sartre tiene un marcado corte existencialista, como toda su
filosofía. II. Aborda este tema, así como muchos otros, sobre la base de su principio: «El
hombre está condenado a ser libre». III. Considera que somos cuerpos con una esencia
sexual y que a través de ellos nos manifestamos, nos exponemos al mundo y a los otros. IV.
 ______________.V.
 ____________ __.V. Por lo tanto, el amor no es más que el encuentro de dos libertades que
se entienden y se respetan, y nunca dejarán de ser libres.
A) Es decir, en esa exposición al mundo, la libertad del amante captura la libertad del ser
amado y la convierte en algo fascinante.
B) Por eso, en su manifestación hacia los demás, el hombre crea sus actos, sus elecciones,
y todo lo que implica su vida.
C) Además, junto a su compañera inseparable, Simone de Beauvoir,Beauvoir,   comprendió que la
libertad está por encima de todo.
D) A pesar de que, según este filósofo, el amor no es más que el encuentro de dos libertades
que se entienden y se respetan.
E) Pero en esa exposición de cada individualidad, hay que tener en cuenta la libertad esencial
de cada uno.

IX. COHCOHERE ERENCI


NCIAA Y COHESI
COHESIÓNÓN TEX
TEXTUAL
TUAL - Elij
Elija
a el orden correct
cor recto
o que deben seguir
los enun ciados para que el texto sea coherente y cohesivo .
23. I. Luego, el 8 de marzo de 1908, 15,000 mujeres salieron a las calles de Nueva York para
exigir un recorte del horario laboral, mejores salarios, el derecho al voto y el fin del trabajo
infantil. II. Este fue un período de expansión, pero muchas mujeres eran explotadas y no había
una ley que las protegiera. III. Debido a los precedentes de la lucha femenina, en 1910 en
Copenhague se celebró una conferencia internacional entre organizaciones socialistas del
mundo y se propuso la creación de una Día de la Mujer de carácter internacional. IV. En
consecuencia, el 8 de marzo de 1857, las mujeres que trabajaban en la industria textil
de Nueva York, organizaron una huelga. V. La iidea dea de un Día Internacional de la Mujer surgió
al final del siglo XIX en un mundo industrializado.
A) V-I-IV-II-III B) V-II- III-IV-I C) V-IV-II-I-III D) V-II-IV-I-III E) V-I-III-IV-II

24. I. Por eso, Esparta logró el poder de todas las ciudades-estado de la Antigua Grecia
gracias a su disciplina militar basada en una educación que no permitía la existencia de
débiles ni pusilánimes. II. La ciudad de Esparta fue fundada en el siglo IX a.C. por el pueblo
dorio que penetró en la península del Peloponeso en busca de tierras fértiles. III. A causa de
esta férrea educación y a su duro sistema ético y guerrero, los espartanos aún están
considerados entre los mejores guerreros de la Historia. IV. Además, su ejército no solo se
basaba en el sistema hoplítico, sino también desarrollaron un sistema social y educativo
dedicado por completo a la victoria y al bien de la ciudad por encima del éxito personal.V. En
los siglos siguientes, la ciudad creció y el aumento de la población hizo que los espartanos
buscaran una ampliación de su territorio a través de la guerra.
A) II-I-III-V-IV B) III-IV-I-II-V C) II-V-I-IV-III D) II-V-IV-III-I E) II-V-I-III-IV

X. COMPRENSIÓN DE LECTURA - Luego de analizar el texto, elija la alternativa que


contenga
cont enga la respuesta a cada pregunt
pregunta.
a.

CEPRE-UNI HUMANIDADES 190

CICLO INTENSIVO ESCOLAR NACIONAL Material de Estudio Nº 1 

A diferencia de lo que ocurría en la Edad Media, donde el hombre era considerado


fundamentalmente desde una perspectiva teológica, los humanistas valoraron al hombre
desde una perspectiva mundana, no divina, es decir, el hombre era visto como un ser natural
e histórico. El humanismo pone énfasis en el hombre como un ser racional capaz de practicar
el bien y encontrar la verdad. Esta corriente afirma que el hombre valora la vida activa, ansía
la gloria y el poder que obtiene con su sagacidad; por eso, interviene en la política, se
convierte en pedagogo, canciller, secretario o historiador de la corte. No obstante, no por ello
desprecia la contemplación espiritual ni la
l a reflexión.

http://www.webdianoia.com/m
http://www.webdianoia.com/moderna/renhum/
oderna/renhum/renhum.htm
renhum.htm

25. El tema central del texto es


A) las características
características ggenerales
enerales del hum
humanismo.
anismo. B) la visión del hom
hombre
bre durante el Med
Medioevo.
ioevo.
C) la noción del hombre según el humanismo. D) las actividades políticas del humanismo.
E) la perspectiva teológica del ser humano.

26. Resulta incompatible con lo expuesto en el texto, afirmar que


A) el hombre puede desempeñar diferentes cargos públicos.
B) el humanismo destaca la dignidad y el valor de los humanos.
C) la
D) el espiritualidad
humanismo desacraliza
carece de todas las concepciones
importancia del hombre.
para el humanismo.
E) el humanismo tuvo un gran impacto en la literatura y el arte.

TEXTO 2
Para el liberalismo no existe algo así como «el hombre colectivo» sino el «hombre individual».
El liberalismo dirige toda su atención al individuo y su principio ético se reduce al concepto de
libertad individual. Pero, ¿qué es la libertad? Tomando la famosa definición de I. Berlin, se
tienen dos perspectivas. La libertad negativa consistiría en la ausencia de obstáculos externos
que impidan al individuo llevar a cabo sus propias decisiones. La positiva, por otro lado,
consistiría en que la persona pueda disponer de la posibilidad de hacer con su vida cuanto
desee. En el primer caso, se incide en la limitación del poder y en la ley para evitar esas
trabas. En el segundo, el poder se utiliza para proporcionar a cada persona aquello que
necesite para realizarse.
http://www.liberalismo.org/art
http://www .liberalismo.org/articulo/1/201/filos
iculo/1/201/filosofia/etica/
ofia/etica/
27. Se infiere del texto que la libertad negativa
A) rechazaría el control de la voluntad del sujeto.
B) proporciona recursos para el desarrollo del individuo.
C) busca mejorar las condiciones de vida de llaa colectividad.
D) es necesario en situaciones de inestabilidad social.
E) altera
altera las reuniones políticas de las sociedades colectivas.

28. ¿Cuál es el enunciado compatible con el texto?


A) Los derechos que tienen las personas restringen el desempeño del Estado.
B) La libertad solo puede verse coaccionada por decisión de la propia persona.
C) El “hombre colectivo” fue el baluarte teórico de la propuesta liberal.
D) La
E) El beneficio particular
libertad positiva es imposibilita las condiciones
un recurso imposible de en
de llevar vidauna
de sociedad.
los otros.

CEPRE-UNI HUMANIDADES 191

CICLO INTENSIVO ESCOLAR NACIONAL Material de Estudio Nº 1 

SEMANA IV: Plan de redacción / Inclusión de información

Método de soluc ión


Método
•  Leer detenidamente el título y los enunciados presentados.
••   Determinar el tipo de secuencia del ejercicio.
Ordenar las oraciones de acuerdo a la secuencia encontrada.

Ejercicio
Ejercici o aplicativo:  Determine el tipo de secuencia y establezca el orden que deben
seguir los enunciados para que la estructura del texto sea adecuada.
1. CÉSAR VALLEJO
I. Durante su infancia, se dedicó a apoyar en la iglesia pues quería ser sacerdote.
II. En 1910, ingresa a la Facultad de Letras de la Universidad Nacional de Trujillo.
III. En 1913, regresa a Trujillo para retomar
r etomar sus estudios y se gradúa en 1915.
IV. César A. Vallejo nació en Santiago de Chuco, pueblo de La Libertad (Perú).
V. Sus carencias económicas lo obligan a abandonar su carrera y a mudarse.
Tipo de secuencia: ___________
_____________________
__________ Orden: ____________
________________________
______________

2. LA ANFETAMINA
I. Generó, por ejemplo, problemas en la circulación y presión.
II. Esto provocó, sin embargo, efectos negativos en el corazón.
III. Esto significa que estimula al sistema nervioso central.
IV. La anfetamina es un agente de tipo adrenérgico sintético.
V. Esta estimulación fue utilizada para combatir el cansancio.
Tipo de secuencia: ___________
_____________________
__________ Orden: ____________
_________________________
_____________

3. CIENCIAS DE LA SALUD
I. La primera vertiente se centra en el estudio y conocimiento sobre la salud-enfermedad.
II. Su objetivo no es solo proveer información, sino promover cambios favorables en la vida.
III. Se organizan en dos vertientes que se reúnen para mantener, reponer y mejorar la salud.
IV. Las ciencias de la salud son el conjunto de disciplinas dedicadas al bienestar de la
persona.
V. La otra vertiente se basa en la
l a aplicación de los conocimientos para el bienestar del
sujeto.

CEPRE-UNI HUMANIDADES 192

CICLO INTENSIVO ESCOLAR NACIONAL Material de Estudio Nº 1 

Tipo de secuencia: ___________


________________
_____ Orden: ____________
_______________________
_____________
__

Método de soluc ión


Método
  Leer las oraciones con detenimi
detenimiento
ento y plant
plantear
ear la lógica de su estruct
estructura.
ura.
  Marcar la alternativa qque
ue contenga informa
información
ción con la correcta pro
progresión
gresión temática, sin
ideas impertinentes y que, de darse el caso, concuerde con los conectores lógicos-textuales
lógicos-textuales..

Ejercicio aplicativo: Elija la opción que contenga la información necesaria para que, al
insertarse, restituya el sentido del texto.
I. La robótica es una ciencia que estudia el diseño y construcción de máquinas capaces de
desempeñar tareas. II. No se trata de cualquier tarea, sino de aquellas realizadas por el ser
humano o que requieren del uso de inteligencia. III. _____________
__________________________.
_____________. IV. Sin
embargo, en la práctica (reproducir
(reproducir tareas como an
andar,
dar, correr o coger un objeto sin rom
romperlo)
perlo)
no se ha obtenido
concretizar el deseoresultados
humano satisfactorios.
de crear seresV.semejantes
Por ello, lasa investigaciones continúan
nosotros que nos a fin del
descarguen de
trabajo.
A) En 1805, Henri Maillardert construyó una muñeca mecánica capaz de hacer dibujos.
B) En la ciencia ficción, el hombre ha imaginado a los robots visitando nuevos
nuevos mundos.
C) Actualmente, ya alcanzó un nivel muy elevado y cuenta con un correcto aparato teórico.
D) El ser humano ha logrado construir máquinas que imiten las partes del cuerpo humano.
E) Ahora, el uso de los robots industriales está concentrado en operaciones muy simples.

PRÁCTICA
I. DEFI
DEFINIC
NICION
IONES
ES - Elija la palabr
palabraa que se defin a con la
l a premisa
premi sa plant
planteada.
eada.
1. _______: Manifest
Manifestación
ación suave o decorosa de ideas cu cuya
ya recta y franca expresió
expresiónn sería
dura o malsonante.
A) Preciosismo B) Prosaísmo C) Anfibologí
Anfibología
a D) Eufemismo E) Retoricismo
2. _______: Identificación
Identificación mental y afectiva de un sujeto con el estado de ánimo de otro.

CEPRE-UNI HUMANIDADES 193

CICLO INTENSIVO ESCOLAR NACIONAL Material de Estudio Nº 1 

A) Filantropía B) Empatía C) Munificencia D) Emulación E) Equivalencia

II. ANALOG
ANALOGÍA ÍA - Elija el par análogo al p
par
ar base escrito
escrit o en mayúsc
mayúsculas.
ulas.
3. RESBALAR: CAER ::
A) aderezar: cocer B) insultar: proferir C) ofuscar: errar
D) golpear: llorar E) birlar: expoliar

4. DELEZNABLE: CONSISTENCIA ::
A) inconcuso: firmeza B) irresoluto: duda C) atribulado: congoja
D) gárrulo: concisión E) patético: algarabía

III. PRECISIÓN LÉXICA - Elija el término que, al sustituir la palabra subrayada, resulte
el más adecuado
adecuado para el contexto
cont exto pl
plantea
anteado.
do.
5. La secretaria, fastidiada por las burlas y mofas incisivas del gerente, lanzó el portafolio.
A) resentida B) indignada C) lastimada D) humillada E) exasperada

6. El trato adusto del capataz de la obra no daba miedo a ninguno de los obreros. 
A) infundía B) generaba C) producía D) arreciaba E) motivaba

IV. ANTONIMIA
exprese CONTEXTUAL
el significado opuesto de- Elija el término que, al sustituir la palabra subrayada,
la oración.
7. Es viable que la ciencia se desarrolle al margen de los intereses políticos.
A) inconducente B) inexorable C) ilógico D) imposible E) complicado

8. No es cuestionable el uso de la «píldora del día siguiente», sino cómo se la adquier


adquiere.
e.
A) permisible B) consensual C) tolerable D) lícito E) recomendable

V. CONECTORES LÓGICO TEXTUALES - Elija la alternativa que, al insertarse en los


espacios en bl anco, una adecuadamente
adecuadamente las ideas del texto.
9. _____ se deteriorase una región de la corteza cerebral, resultaría afectada la calidad de
sueño; _____, se reduciría la capacidad memorística, _____ mermaría la competencia
lingüística.
A) Siempre que – sin embargo – pues B) Solo si – en consecuencia – aunque
C) Aunque – no obstante – pero D) Aun cuando – también – además
E) Si – asimismo – incluso

10. Algunos productos manufacturados son perjudiciales para la fauna marina; _____, el
plástico contamina los mares ____ no es biodegradable, ____ puede ser ingerido por los
peces.
A) verbigracia – debido a que – entonces B) porque – y – además
C) entonces – pero – además D) es decir – incluso – ya que
E) por ejemplo – aunque – pues

VI. INFORMACIÓN EL IMINADA  – Elija la oración que no es pertinente o es redundante


INFORMACIÓN ELIMINADA
con el contenido
cont enido global del texto.
11. I. Según el DRAE, un eufemismo es una «manifestación suave o decorosa de ideas cuya
recta o franca
la palabra expresión
o expresión sería
que dura oamalsonante».
sustituye malsonante».II.
II. En olas
nombres secretos sectas, para
sagrados el eufemismo puede
evitar revelar ser
estos
a los no iniciados. III. Los eufemismos ya eran frecuentes en las sociedades primitivas en las

CEPRE-UNI HUMANIDADES 194

CICLO INTENSIVO ESCOLAR NACIONAL Material de Estudio Nº 1 

que se evitaba mencionar las palabras tabúes, sobre todo, las de carácter religioso. IV. En el
Renacimiento, se desarrollaron los eufemismos por afán estético como: suplemento del
sol (por lámpara), el consejero de las gracias  (por espejo), baño interior  (por (por vaso de
agua), el trono del pudor  (por
 (por mejillas), etc. V. Para el preciosismo francés del s. XVII, los
eufemismos debían mantener una interpretación meramente ambigua.
A) I B) II C) III D) IV E) V

12. I. El hombre mediocre es incapaz de usar su imaginación para concebir ideales que le
propongan un futuro por el cual luchar. II. El hombre mediocre
m ediocre se vuelve sumiso a toda rutina
y se adhiere a un rebaño o colectividad, cuyas acciones o motivos no cuestiona, sino que
sigue ciegamente. III. El mediocre es dócil, maleable, ignorante, un ser vegetativo, carente de
personalidad, contrario a la perfección y cómplice de los intereses creados que lo hacen
borrego del rebaño social. IV. El hombre mediocre vive según las conveniencias y no logra
aprender a amar, pues se vuelve vil, escéptico y cobarde. V. El hombre mediocre es un
individuo conformista y parasita según las oportunidades se le presenten.
A) I B) II C) III D) IV E) V

13. I. Entrando en materia sobre la adquisición del lenguaje, para Chomsky es una función
biológica normal de los seres humanos. II. Según Chomsky, el ser humano posee una
propiedad
III. Sobre elmental que le del
aprendizaje permite aprender
lenguaje, la lengua
Chomsky de lados
plantea comunidad en la que
líneas generales se desarrolla.
para abordar el
problema: enfoque empirista y enfoque racionalista. IV. Algunos conceptos por los cuales se
verifica la teoría innatista
innatista de Chomsky son ttomados
omados desde el desa
desarrollo
rrollo del habla infantil. V.
Para comprender cómo se adquiere el lenguaje, se debe estudiar separadamente un sistema
cognoscitivo, un sistema de conocimientos y el sistema de la competencia lingüística, según
Chomsky.
A) I B) II C) III D) IV E) V

VII. PLAN DE REDACCIÓN - Elija la alternativa que contenga la secuencia correcta que
deben seguir los lo s enunciados para que la eestr
structu
uctura
ra del texto sea aadecuada.
decuada.
14. EL GÉNERO HOMO
I. La crisis climática de la época ocasionó una desertificación lo que desencadenó su
extinción.
II. Los primeros homínidos bípedos fueron los integrantes del llamado género
 Australopithecus.
III. Los primeros Homo, en tanto, derivaron de otro grupo que gradualmente se volvió
carnívoro.
IV. Esta presión evolutiva obligó a algunos a especializarse en la recolección de vegetales.
V. Estos prosperaron nnotablemente
otablemente en el este de África entre 4 y 2,5 millones de años atrá
atrás.
s.
A) I-III-II-V-IV B) II-V-I-III-IV C) I-III-V-IV-II D) II-V-I-IV-III E) II-V-III-I-IV

15. EL DESARROLL
DESARROLLO O EVOLUTIVO DEL PENSAM
PENSAMIENTO
IENTO
I. Cuenta por ello, con una serie de conductas básicas, innatas e involuntarias llamadas
reflejos.
II. Los razonamientos en la niñez no se reducen al plano tangible, sino que son interiorizados.
III. Al nacer el bebé tiene problemas para reconocer sus propios límites y el medio que lo
rodea.
IV. En la siguiente etapa, el cerebro está ya capacitado para formular pensamientos
abstractos.

CEPRE-UNI HUMANIDADES 195

CICLO INTENSIVO ESCOLAR NACIONAL Material de Estudio Nº 1 

V. En la infancia se establecen relaciones de causalidad restringidas al ámbito de lo sensorial.


A) III-I-V-II-IV B) III-V-I-II-IV C) III-V-I-IV-II D) III-I-V-IV-II E) III-II-V-IV-I

16. LA NATURA
NATURALEZALEZA DE DELL CONOCIMI
CONOCIMIENTO
ENTO
I. Lo que no sabemos es lo esencial, tampoco explicaría la situación de nuestro conocimiento
conocimiento..
II. Optar por
por buscar la vverdad;
erdad; sin embargo, es una de las eleccion
elecciones
es humanas más vitales.
III. Nuestro conocimiento es limitado e incierto; no obstante existe y es relevante
r elevante para la vida.
IV. Ello revela que es ilógico creer en la omnipotencia de la razón como en la de la igno
ignorancia.
rancia.
V. Que no lo sepamos todo no implica, necesariamente
necesariamente,, que lo que sabemos es lo trivial.
A) III-II-I-V-IV B) III-I-IV-II-V C) III-II-I-IV-V D) III-V-IV-I-II E) III-V-I-IV-II

VIII. INCLUSIÓ
VIII. INCLUSIÓN N DE IINF
NFORMAC
ORMACIÓN
IÓN - Eli
Elija
ja la
l a oración
oraci ón que, al iinser
nsertars
tarse
e en el esp
espacio
acio en
blanco, complete de manera
manera lógica la información global del texto.
17. I. La sal es el condimento más antiguo y el único mineral comestible. II. Además, en la
industria alimentaria es ampliamente utilizada como conservante natural. III.
 ______________________.
 ____________ __________. IV. Según las investigaciones, este mineral haría que los
linfocitos T coadyuvantes, fundamentales para la defensa del sistema inmunitario, se
transformen en células dañinas. V. Estas células serían las responsables del desarrollo de
patologías como la esclerosis múltiple, la enfermedad de Crohn o la diabetes, entre otras.
A)
B) Es
Se recomendable
ha descubiertoconsumir
que podríasalaumentar
con yodoelpara di sminuir
disminuir
riesgo el riesgo
de padecer de padecerpatologías.
determinadas de bocio.
C) Existen muchos estudios en torno a los beneficios y perjuicios de su ingesta cotidiana.
D) Su utilización como conservante natural resulta ser muy provechoso para la industria.
E) Los linfocitos T son un tipo de célula blanca de la sangre presente en el cuerpo humano.

18. I. _______________________. II. Entre ellos, el elemento orgánico por excelencia, el


carbono, se crea en el interior de las gigantes rojas. III. Esto sucede mediante la fusión casi
simultánea de tres núcleos de helio o partículas alfa. IV. Y en la síntesis del carbono parecen
incurrir una serie de eventualidad
eventualidadeses muy difíciles de explicar. V. En síntesis, de no ser por la
existencia «casual» de un estado excitado en el núcleo del carbono 12, probablemente no
habría vida en el universo.
A) Las masas de losl os quarks y la intensidad de la interacción electromagnética son variables.
B) Los físicos, en las últimas décadas, se han interesado por investigar la síntesis del carbono.
C) El carbono y el oxígeno son bases para la vida, pero el hidrógeno no por diversas razones.
D) Los elementos químicos esenciales para la vida se sintetizan en el corazón de las estrellas.
E) En el interior de las estrellas, la síntesis del núcleo de carbono 12 procede en dos periodos.

19. I. Los senos maxilares han permitido que la nariz humana evolucione en cada país en
función del clima. II. Su evolución ha tenido el fin de mejorar su función principal: filtrar el aire
para que sea lo más respirable posible. III. Así, en climas fríos la nariz es más estrecha y
alargada, para poder calentar el aire antes de que siga su camino hacia los pulmones. IV.
 _______________________.
 ____________ ___________. V. Ya qque ue ahí el aire generalmente ya es caliente y húmedo, lo
que interesa es dirigirlo hacia los pulmones lo más rápido posible.
A) Eso explica por qué la nariz de los habitantes del norte de Europa es afilada.
B) Los senos nasales de los europeos son 36% mayores al de los norteafricanos.
C) En climas cálidos, sin embargo, la nariz suele tener una forma ancha y corta.
D) Si es
E) En lasmás estrecha
regiones conyclima
puntiaguda, existe más
seco y caliente, espacio
la nariz para suele
también los senos maxilares.
ser alargada.

CEPRE-UNI HUMANIDADES 196

CICLO INTENSIVO ESCOLAR NACIONAL Material de Estudio Nº 1 

IX. COHE
COHEREN RENCIA CIA Y C COHEOHESIÓ SIÓN N TE
TEXTU
XTUAL AL - Eli ja el or den co rrecto
rr ecto que d
deben
eben seguir
segu ir los
enunciados p ara que el texto sea coherente y cohesivo.
20. I. Los resultados indicaron que los ancianos presentan una menor capacidad de recordar
información tras dormir. II. Se ha descubierto que las personas mayores experimentan una
fase profunda del sueño menor que los jóvenes. III. Ellos reclutaron a 33 pacientes sanos,
entre jóvenes y ancianos y los sometieron a una serie de pruebas. IV. En efecto, los
investigadores de la Universidad de California (Berkeley) han podido establecer esta
asociación causa-efecto. V. En consecuencia, los ancianos, por lo general, experimentan bajo
rendimiento de la memoria.
A) II  ––IV –III  ––  –
I–V B) I  ––II  ––IV –III  ––V C) II  ––V –IV –III  ––I D) I  ––V –IV –III  ––II E) II  ––V –I  ––III  ––IV

21. I. Los protones son partículas complejas cuya carga eléctrica no se halla concentrada en
un punto. II. Sin embargo, en 2010 una colaboración internacional anunció que un nuevo tipo
de experimento proporcionaba un valor menor. III. Hasta hace poco, los físicos pensaban que
conocían bien el radio característico de esa nube: 0,88 femtómetros. IV. Así que este era el
de 0,84 fm, con 5 desviaciones estándar de diferencia con respecto a las mediciones
precedentes. V. Más bien, esta se extiende en el espacio y forma una especie de «nube».
A)   –
I–V –III  ––II  ––IV B) III  ––II  ––I  ––IV –V C) I  ––II  ––IV –V –III D) III  ––II  ––  –
I–V –IV E)   –
I–V –II  ––III  ––IV

22. I. En intervienen
regiones conclusión, enlos el análisis mostraron
desarrollo del sistemaque ocho de los II.
nervioso. genes contenidos
Además, en estas
desencadenan
cambios etológicos que permitieron a este cánido convertirse en el mejor amigo del hombre.
III. Ellos secuenciaron y compararon el genoma de 60 perros y de 12 lobos, e identificaron
3,8 millones de variantes genéticas. IV. Luego, a partir de ellas, aislaron 36 regiones de ADN
que llevaron a la selección del perro doméstico. V. Investigadores de la Universidad de
Uppsala realizaron un exhaustivo estudio sobre la evolución del perro doméstico.
A) V –III  ––II  ––I  ––IV B) V –I  ––III  ––IV –II C) V –  –
I–III  ––II  ––IV D) V –III  ––IV –I  ––II E) V –IV –I  ––III  ––II

X. COMPRENSIÓN DE LECTURA - Luego de analizar el texto, elija la alternativa que


contenga
cont enga la respuesta a cada pregunt
pregunta. a.
TEXTO 1
El positivismo reduce más la distancia del pensamiento a la humanidad, una distancia que la
propia realidad ya no tolera. Al no pretender ser más que algo provisional, meras abreviaturas
de lo fáctico que ellos subsumen, los tímidos pensamientos ven desvanecerse, junto con su
autonomía respecto a la realidad, su fuerza para penetrarla. Solo en el distanciamient
distanciamientoo de la
vida cobra vida el pensamiento y queda este verdaderamente enraizado en la realidad
empírica. Si el pensamiento se refiere a los hechos y se mueve en la crítica de los mismos,
no menos se mueve por la diferencia que establece. Este es su modo de expresar que lo que
es no es del todo como él lo expresa.
T. ADORNO, Mínima moralia.
23. Es incompatible con el texto afirmar que el pensamiento
A) es completamente análogo a la realidad. B) tiene carácter efímero y es evanescente
evanescente..
C) es un aspec
aspecto
to con
considerado
siderado ppor
or el posit
positivismo.
ivismo. D) es influenciado por la fuerza de
dell tiempo.
E) posee la capacidad de arraigarse empíricamente.

24. Si el pensamiento expresase tal cual es la realidad,


r ealidad,
A) perdería
B) se su poder
distanciaría penetrativo
completame nteen
completamente delalavida misma.
humanidad.
C) sería una fuente de conocimiento empírico de esta.

CEPRE-UNI HUMANIDADES 197

CICLO INTENSIVO ESCOLAR NACIONAL Material de Estudio Nº 1 

D) hombres criticarí
criticarían
an el me
medio
dio en el que viven
E) el positivismo habría tenido éxito en la filosofía.
fi losofía.

TEXTO 2
El existencialismo es una doctrina de la acción contraria al quietismo : para el existencialism
existencialismoo
solo hay realidad en la acción. Este pensamiento tiene dos caras: por un lado es duro para
aquellas personas descontentas con lo que son, para los que no han triunfado en la vida;
estas personas pueden engañarse diciendo que en realidad sus actos no muestran su
auténtica valía, diciendo que hay en ellos capacidades, talentos o disposiciones
desaprovechadas,, que el mundo les ha impedido dar de sí todo lo que realmente son. Pero,
desaprovechadas
por otro lado, esta doctrina es optimista pues declara que el destino de cada uno está en sus
manos y lo predispone a la acción, a no vivir de
d e sueños, de esperanzas y realizar su proyecto:
el héroe no nace héroe, se hace héroe; si se es cobarde es como consecuencia de una
decisión, no porque fisiológica o socialmente se esté predispuesto para ello; el cobarde se
hace cobarde, pero hay siempre para el cobarde una posibilidad de no ser por más
tiempo cobarde, como para el héroe la de dejar de ser héroe.
http://www.Filosofiaconte
http://www.Filosofiacontemporanea/Sartre/
mporanea/Sartre/Sartre-Existencialismo.
Sartre-Existencialismo.htm
htm

25. Según el texto se puede aseverar que el existencialismo


A) propugna
B) concilia conla la contemplación
inacción humana pasiv
pasiva
frenteaal
demundo.
la vida.
C) puede abordarse desde dos cariz antagónicos.
D) revitaliza las actividades de todas las personas.
E) predetermina el destino de la condición humana.

26. Si alguien afirmara tenazmente que el arte se hereda genéticamente,


A) sería considerado un discípulo del existencialismo.
B) estaría en las antípodas de la argumentación del autor.
C) también declararía que el destino se forja por cada uno.
D) las ideas existencialistas se verían corroboradas al fin.
E) la labor artística ya no sería un problema para el hombre.

SEMANA V: Coherencia y cohesión textual / Comprensión DE lectura


Para que un texto sea tal, es necesario que posea una
unidad de sentido y de forma; es decir, que las palabras
y oraciones estén relacionadas entre sí. Así, todo texto
debe ser coherente y cohesivo. La coherencia, por un
lado, se refiere al significado global de un texto; para
ello, cada uno de los enunciados que lo conforma debe
girar en torno a un tema central y, al mismo tiempo,
desarrollarlo. Por otro lado, la cohesión  se evidencia
en la presencia de recursos que se utilizan para enlazar
las distintas partes del texto.
CEPRE-UNI HUMANIDADES 198

CICLO INTENSIVO ESCOLAR NACIONAL Material de Estudio Nº 1 

INTERRELACIÓN
TEXTUAL DE IDEAS

Mental Lingüística

COHERENCIA COHESIÓN

Tema centr
central
al e Conectores
ideas lógicos y
principales referentes

Método de solución
Método sol ución
 Leer los enunciados propuestos e identificar el tema central
 Determinar el orden que deben seguir los enunciados medi mediante
ante la identificación de los
elementos gramaticales que relacionan las oraciones entre sí
 Leer el nuevo orden y confirmar que el texto sea coherente y los mecanismos de cohesión
estén bien empleados

Ejercicio
Ejercici aplicativo : Lee atentamente el texto y responde a las preguntas planteadas sobre
o aplicativo:
coherencia y cohesión.
TEXTO
El distrito del Rímac está en la provincia de Lima. Es reconocido por sus monumentos
coloniales; algunos de estos, lamentablemente, han sido destruidos por el vandalismo, por
ejemplo, la Alameda de los Descalzos, la cual ha sufrido el robo de casi todas sus estatuas
originales. Dichas obras de arte fueron esculpidas en fino mármol italiano. Se destacan tres,
las representaciones de los meses, como Julio, Setiembre y Diciembre entre las más bellas.
Sin embargo, hoy solo se conservan algunas de ellas.

- Preguntas sobre coherencia


¿Cuál es el tema central del texto?
 ________________________
 ____________ ________________________
________________________
________________________
________________________
______________
Escribe dos ideas acerca del tema central.
 ________________________
 ____________ ________________________
________________________
________________________
________________________
______________
 ________________________
 ____________ ________________________
________________________
________________________
________________________
______________

- Preguntas sobre cohesión


¿Cuáles son los conectores que presenta el texto? Indica la relación que establecen entre las
proposiciones. ________________________
 ____________
 ________________________ ________________________
________________________
________________________
______________
 ________________________
 ____________ ________________________
________________________
________________________
________________________
______________

CEPRE-UNI HUMANIDADES 199

CICLO INTENSIVO ESCOLAR NACIONAL Material de Estudio Nº 1 

¿Qué referentes presenta el texto? ¿A qué elementos hacen alusión? Escribe cinco.
 ________________________
 ____________ ________________________
________________________
________________________
________________________
______________
 ________________________
 ____________ ________________________
________________________
________________________
________________________
______________

Método de soluc ión


Método
  Leer el texto y subrayar las ideas o frases más importantes
  Determinar el tema y la idea principal
  Leer casa pregunta y responder según el nivel al que corresponde

Ejercicio Aplicativo: Lea atentamente el texto, luego responda cada pregunta planteada,
según los niveles de comprensión lectora.
TEXTO
Los roles que se desempeñan y el estatus que se ocupa van relacionados con una forma de
vida que con
épocas, involucra los niveles
excepción quizádederiqueza, autoridad
los pueblos y poder.que
primitivos Lastenían
sociedades en todas sus
una economía de
subsistencia, han observado estas diferencias sociales. ¿A qué se debe que alguna gente
tenga poder y autoridad sobre otros si supuestamente ante el Estado todos somos iguales?
La respuesta es sencilla: la riqueza puede durar muchos años, incluso más de lo que dura la
vida de una persona, además, se puede transmitir. Estas dos características hacen posible
que el poder se pueda heredar y dure por muchas generaciones. Con ello, la igualdad de la
era primitiva se quedó atrás y surgió el sistema de estratificación social, utilizado por las
sociedades en donde los estatus sociales están ordenados jerárquicamente sin que se
puedan mover.

1. Básicamente, el autor cent


centra
ra su explicación en
A) la equidad social de los pueblos tribales. B) la causa de los roles y estatus sociales.
C) la transmisión generacional del poder. D) la ineficacia del Estado y sus efectos.
E) las razones de la estratificación social.
2. En el texto, el sentido contextual del término TRANSMITIR es

CEPRE-UNI HUMANIDADES 200

CICLO INTENSIVO ESCOLAR NACIONAL Material de Estudio Nº 1 

A) Legar. B) entregar. C) dar. D) remitir. E) traspasar.

3. Resulta compatible con el texto sostener que la riqueza


A) debe ser repartida según el trabajo del hombre.
B) es un factor crucial para establecer la igualdad.
C) ha permitido combatir la estratificación social.
D) puede perennizar el poder y llaa jerarquía social.
E) fue un asunto muy polémico en la era primitiva.

4. De lo expuesto en el texto, es posible inferir que los roles y los estatus


A) se vinculan con el nivel de riqueza que se posee.
B) determinaron la jerarquía en los pueblos primitivos.
C) designarían los niveles de riqueza de las personas.
D) son el producto de cualquier sociedad estratificada.
E) permiten
permiten desarrollar una economía de subsistencia.

5. Si la riqueza no pudiese ser heredada,


A) la pobreza se incrementaría alarmantemente
alarmantemente..
B) el poder no sería conservado por generaciones.
C) la igualdad regiría todos los estratos sociales.
D) la noción de Estado, sin duda, desaparecería.
E) los roles sociales habrían encontrado su fin.

PRÁCTICA
I. DEFI
DEFINIC
NICIÓN
IÓN - Elija la palabr
palabra a que se defina
defin a con lla
a premi
premisa
sa plant
planteada.
eada.
1. _________: Graduar la cantidad o porción de otras cosas.
A) Dosificar B) Distribuir C) Administrar D) Calibrar E) Mesurar
2. _________: Que huye y se esconde de las gentes.
A) Misántropo B) Impío C) Retraído D) Introvertido E) Huraño

3. _________: Renta, utilidad o beneficio renovable que rinde un capital.


A) Plusvalía B) Interés C) Rédito D) Tributo E) Gravamen

II. ANALOG
ANALOGÍA ÍA - Elija el par análogo al par base escrito en mayúscul as.
4. AUDAZ: COBARDE ::
A) timorato: indeciso B) pusilánime: medroso C) acoquinado: achantado
D) resuelto: apocado E) osado: intrépido

5.TACITURNO: MELANCÓLICO ::
A) afable: antipático B) retraído: tímido C) mendaz: veraz
D) impúdico: modesto E) huraño: sociable

III. PRECISIÓN LÉXICA - Elija el término que, al sustituir la palabra subrayada, resulte
el más adecuado
adecuado para el contexto
context o plantea
plant eado.
do.
6. Amenazaron al director a fin de que renuncie a su cargo.
A) Molestaron B) Amonestaron C) Extorsionaron D) Desafiaron E) Expoliaron
7. El delincuente podría pasar largos años de su vida encerrado en la cárcel.
A) recluido B) enclaustrado C) arrestado D) confinado E) internado

CEPRE-UNI HUMANIDADES 201

CICLO INTENSIVO ESCOLAR NACIONAL Material de Estudio Nº 1 

8. Los homicidas deben pagar sus culpas en la cárcel.


A) sufrir B) purgar C) purificar D) compensar E) limpiar

IV. ANTONIMIA CONTEXTUAL - Elija el término que, al sustituir la palabra subrayada,


se el signifi cado opuesto de la oración. 
exprese
expre
9. La congresista se mostró suspicaz ante las preguntas de la prensa.
A) incauta B) cautelosa C) crédula D) prudente E) confiada
10. A medida que investigaba, la información se tornaba cada vez más abstrusa.
A) comprensible B) incuestionable C) irrefutable D) equilibrada E) seria
11. La absolución del acusado fue un adecuado fallo judicial.
A) crítica B) censura C) indulgencia D) resolución E) condena

V. CONECTORES LÓGICO TEXTUALES - Elija la alternativa que, al insertarse en los


espacios en bl anco, una adecuadamente
adecuadamente las ideas del texto.
12. Los niños perturbados son hostiles; ____, se comportan agresivamente, ____ los
problemas afectan su conducta. ____, esto no es el único que influye en su agresividad.
A) incluso – ya que –Por ello B) también –por tanto –No obstante C) es decir  –pues –Sin embargo
D) solo si – porque – Pero E) o – dado que – Por tanto
13. ____ la globalización es ventajosa____ enriquece a los pueblos con oportunidades, esta
presenta muchos riesgos, ____ produce temor al proteccionismo económico empresarial.
A) Si –
 –entonces –a fin de que B) Siempre que –incluso –para que C) Dado que –ya que –por tanto
D) Si bien – pues – por ejemplo E) Aunque – pero – por eso
Aunque
14. ____ la economía peruana ha mejorado, los sueld sueldos
os de los profesion
profesionales
ales no aumentan
aumentan;;
 ____, muchos salieron a protes
protestar,
tar, ____ ssienten
ienten que ssuu trabajo no es valorado.
A) Aunque – incluso – pues B) Dado que – entonces –pero C) Porque –sin embargo –así
D) Aun cuando – además – por eso E) A pesar de que – por ello – ya que
VI. INFORMACIÓN
INFORMACIÓN ELIMINADA
EL IMINADA  – Elija la oración que no es pertinente o es redundante
con el contenido
cont enido global del texto.
15. I. El monte Everest es la montaña más alta del mundo con una altura de 8848 metros
sobre el nivel del mar. II. Su nombre le fue adjudicado en honor de George Everest, geógraf
geógrafoo
británico, en 1865. III. Está localizada en el Himalaya, en el continente asiático, y marca la
frontera entre Nepal y China. IV. Actualmente el Everest está sufriendo deshielo y pérdida de
nieve. V. Presenta, incluso, niveles dañinos de metales pesados contaminantes como el
cadmio y arsénico.
A) I B) II C) III D) IV E) V

16. I. Los principales cultivos del Perú se hallan en Lambayeque, La Libertad, Piura, Ancash
y Apurímac. II. El algodón genera la producción
pr oducción de 400 empresas textiles ubicadas en Piura
y Ancash. III. En Lambayeque, Piura y La Libertad existen aproximadamente 400 molinos
para el procesamiento de arroz. IV. La papa es producida por 6 mil pequeñas comunidades
agrarias, la mayoría, ubicadas en la sierra. V. El maíz amiláceo y el maíz amarillo son
producidos en Cajamarca y Apurímac.
A) I B) II C) III D) IV E) V

CEPRE-UNI HUMANIDADES 202

CICLO INTENSIVO ESCOLAR NACIONAL Material de Estudio Nº 1 

VII. PLAN DE REDACCIÓN - Elija la alternativa que contenga la secuencia correcta que
deben seguir los lo s enunciados para que la e estr
structu
uctura
ra del texto sea a adecuada.
decuada.
17. EL ENFOQUE ESTRUCTURAL DE FREUD
I. El yo surge del ello pero, a diferencia de este, se basa en el principio de la realidad.
II. El súper yo es el agente moralizador constitu constituido ido por las normas morales y el yo ideal.
III. El enfoque estructural de Freud plantea la noción de un ello, un yo y un súper yo.
IV. Gracias a estas característ
características icas están relacionados funcionalmente unos con otros.
V. El ello es un agente mental que contiene a los instintos que exigen ser satisfechos.
A) III  ––  –
I–V –II  ––IV B) III  ––V –I  ––II  ––IV C) III  ––V –I  ––IV –II D) III  ––IV –V –I  ––II E) III  ––I  ––V –IV –II

18. UBICACIÓN DE LAS CUENCAS DE HIDROCARBURO


I. Actualmente, existen alrededor de 16 cuencas c uencas sedimentarias con hidrocarburos.
II. Otras cinco están ubicadas sobre el flanco oriental de la cordillera de los Andes.
III. Diez de estas cuencas
cuencas se encuen encuentran
tran en el zócalo cont continental
inental y en la cost
costa.
a.
IV. La exploración de hidrocarburos se inició en el Perú a mediados del siglo XIX.
V. La restante se ubica en la llanura amazónica, que incluye a otras subcuencas.
A) IV –  –
I–III  ––V –II B) I  ––IV –II  ––III  ––V C) IV –I  ––V –III  ––II D) I  ––IV –III  ––II  ––V E) IV –I  ––III  ––II  ––V

19. OBRAS Y PREMIOS DE VARGAS LLOSA


I. Estos fueron El abuelo  y Los jefes  que fueron publicados en El Comercio y Mercurio
Peruano.
II. En 1956, empezó con seriedad su carrera literaria con la publicación de sus primeros
relatos.
III. Seis años después escribió La ciudad y los perros y consiguió el Premio Biblioteca Breve.
IV. En el 2010, el mundo reconoce su inteligencia al entregarle el Premio Nobel de Literatura.
V. Mario Vargas Llosa es uno de los más importantes novelistas y ensayistas
contemporáneos.
A) V –II  ––I  ––IV –III B) V –II  ––I  ––III  ––IV C) V –II  ––III  ––I  ––IV D) V –I  ––II  ––IV –III E) V –IV –II  ––I  ––III

VIII. INCLUSIÓ
VIII. INCLUSIÓN N DE IINF
NFORMAC
ORMACIÓN IÓN - Eli
Elija
ja la
l a oración
oraci ón que, al iinser
nsertars
tarse
e en el espaci
espacio o en
blanco, complete de manera
manera lógica la información global del texto.
20. La hepatitis es una infección viral que produce la inflamación del hígado y puede ser de
varios tipos. II. La hepatitis A se transmite por vía digestiva, por el contacto con agua y
alimentos contaminados. III. ________________________. IV. La de tipo C se transmite a
través de transfusiones de sangre al igual que la hepatitis B. V. La hepatitis E se encuentra
en las heces y se transmite por vía digestiva al igual que la de tipo A.
A) La hepatitis G es otro tipo de hepatitis que está suscitando diferentes investigaciones.
B) La hepatitis B se transmite por contacto sexual, vía placent
placentaria
aria o sangre contaminada.
C) El virus de la hepatitis B se halla en todos los fluidos corporales de la persona infectada.
D) La de tipo B, por lo general, se manifiest
manifiestaa a través de naúse
naúseas,
as, diarreas, fat
fatiga
iga e ictericia.
E) La de tipo B bloquea el paso de la bilis que produce el hígado al descomponer las grasas.

21. I. El ser humano presenta diferent


diferentes
es actitudes frente a la realidad. II. La actitud moral, por
ejemplo, valora la conducta como correcta o incorrecta. III. _______________
_______________________.
________. IV.
La actitud religiosa se muestra cuando los sujetos piensan que la realidad está determinada
por algo extramaterial. V. La actitud estética se manifiesta en aquellas reacciones de gozo y
satisfacción ante la belleza.
A) La actitud pragmática, en cambio, valora los objetos de acuerdo a su utilidad.
B) La actitud filosófica fue valorada por los filósofos modernos y contemporáne
contemporáneos.
os.

CEPRE-UNI HUMANIDADES 203

CICLO INTENSIVO ESCOLAR NACIONAL Material de Estudio Nº 1 

C) La actitud religiosa es propia de autoridades eclesiástic


eclesiásticas
as y seguidores de Dios
Dios..
D) La actitud moral se observa al ayudar al prójimo o al hacer una obra de caridad.
E) La actitud moral y religiosa
r eligiosa se relacionan por las consecuencias de la conducta.

IX. COHE
COHEREN
RENCIA
CIA Y C
COHE
OHESIÓ
SIÓN
N TE
TEXTU
XTUAL
AL - Eli ja el or den co rrecto
rr ecto que d
deben
eben seguir
segu ir los

enunciado
enunciados s para que
22. I. Es beneficiosa el texto
porque sea coherente
el desarrollo económicoy cohesivo.
de los países industrializados se debe
propagar al resto del mundo. II. Los críticos de la globalización consideran, en cambio, que
esta generará más pobreza y desigualdad. III. Pues las naciones menos desarrolladas, al
estar en contacto con el denominado primer mundo, verán crecer sus propias economías. IV.
Ellos creen que son los l os países desarrollados los que encuentran un mayor mercado para sus
productos. V. Algunos econo economistas mistas consideran qu quee la globalización es bene beneficiosaficiosa y otros
que es perjudicial.
A) V –I  ––II  ––III  ––IV B) V –I  ––III  ––II  ––IV C) V –I  ––III  ––IV –II D) V –III  ––IV –I  ––II E) V –II  ––I  ––III  ––IV

23. I. Una vez elegido el tema, realizan una serie de experimentos para medir los resultados.
II. Ya que el campo de la motivación es tan amplio, los investigadores centran su esfuerzo en
un tema específico. III. También pueden alterar las concentraciones hormonales u otras
sustancias químicas en el cuerpo. IV. Por ejemplo, pueden efectuar operaciones para al alterar
terar
la estructura del cerebro y observar sus efectos. V. Este puede, ser por ejemplo, el hambre,
la sexualidad, la agresión, la conducta de logro, etc.
A) II  ––V –IV –III  ––I B) II  ––V –I  ––III  ––IV C) V –II  ––IV –I  ––III D) II  ––V –I  ––IV –III E) V –II  ––I  ––III  ––IV

24. I. Se encarga también de facilitar que las empresas puedan acceder al mercado en
igualdad de condiciones. II. Por último, protege al consumidor aplicando multas a las
empresas que violan los derechos de sus clientes. III. Se encarga, por ejemplo, de llevar el
registro y supervisar el cumplimiento de la norma técnica del origen del pisco. IV. Su función
es promover una cultura de leal y honesta competencia, y proteger todas las formas de
propiedad intelectual. V. En 1992, se creó el Instituto Nacional de Defensa de la Competencia
y de Protección de la Propiedad Intelectual (Indecopi).
A) V –I  ––IV –III  ––II B) V –IV –II  ––III  ––I C) V –III  ––I  ––II  ––IV D) V –IV –III  ––I  ––II E) V –IV –  –
I–III  ––II

X. COMPRENSIÓN DE LECTURA - Luego de analizar el texto, elija la alternativa que


contenga
cont enga la respuesta a cada pregunt
pregunta.
a.
TEXTO 1
La angustia es un estado afectivo de carácter penoso que se caracteriza por aparecer como
reacción ante un peligro desconocido o impresión. Suele estar acompañado por intenso
malestar psicológico y por pequeñas alteraciones en el organismo, tales como elevación
elev ación del
ritmo cardíaco, temblores, sudoración excesiva, sensación de opresión en el pecho o de falta
de aire. Es equivalente a la ansiedad extrema o miedo. Sin embargo, por ser un estado
afectivo de índole tan particular y no conocido, ha sido tema de estudio de una disciplina
científica: la psicología, que ha realizado los principales aportes para su conocimiento y lo ha
erigido como uno de sus conceptos fundamentales.
Diane Papalia, Psicología
 
25. Si una persona presenta constantes estados de angustia se puede inferir que
A) mostrará un desequilibrio emocional permanente.
B) tendrá una personalidad de tipo ansiosa y depresiva.
C) recibirá tratamiento psicológico y terapias de relajación.

CEPRE-UNI HUMANIDADES 204

CICLO INTENSIVO ESCOLAR NACIONAL Material de Estudio Nº 1 

D) estará expuesta a situaciones de peligro e impresiones.


E) será vulnerable a desarrollar problemas del corazón.

26. Una idea compatible con el texto es que la angustia


A) presenta
presenta síntomas semejantes a las fobias. B) es un temor opresivo sin una causa precisa.
C) se presenta en mayor proporción
proporción en adultos. D) es estudiada principalmente
principalmente por la psiqui
psiquiatría.
atría.
E) genera dificultades en el control de esfínteres.

TEXTO 2
La actitud crítica no «elimina» nuestra visión natural del mundo, sino que esta pura visión
natural o espontánea de nuestro conocer adquiere dimensiones y perfiles de rigor
susceptibles de muy diversas vías de justificación. En la actitud crítica no nos basta
simplemente con conocer y saber que conocemos, sino que la actitud crítica ha de comenzar
por formularse estas preguntas fundamentales: qué significa conocer, qué valor tiene el
conocimiento, qué seguridad puedo llegar a tener en la posesión de mis conocimientos
conocimientos..
S. Rábade Romeo, Estructura del conocer humano, pág. 30

27. Se infiere del texto que la actitud crítica


A) presenta una naturaleza involuntaria sobre el saber del universo.
B) implica un rol activo del sujeto con relación a su conocimiento.
C) se inicia con el cuestionamiento acerca de la función del saber.
D) supone una actitud
actitud ingenua frent
frentee a la duda del conocimie
conocimiento.
nto.
E) excluye de manera absoluta la visión espontánea del mundo.
CEPRE-UNI HUMANIDADES 205

CICLO INTENSIVO ESCOLAR NACIONAL Material de Estudio Nº 1 

También podría gustarte